You are on page 1of 377

1::on tents

Pre/ace
1 1: Blood Banking
Tl1e Importance of Certification 1 Questions
CMP. Licensure & Qualification' 58 Answer Key
59 Answers with Explanations
Preparing for & Taking the soc
Certification Examinations 85 2 : Urinalysis & Body Flulds
85 Questions
Tips for Preparing for the
122 Answer Key
!!:xaminations
123 Answers with Explanations
'?eading & References
141 3: Chemistry
141 Questions
196 Answer Key
197 Answers with Explanations

219 4 : Hematology
219 Questions
290 Answer Key
291 Answers with Explanations
31 3 5 : Immunology
313 Questions
342 Answer Key
343 Answers with Explanations

355 6 : Microbiology
355 Questions
422 Answer Key
423 Answers with Explanations

451 7: Molecular Biology


451 Questions
460 Answer Key
461 Answers with Explanations

465 8 : Laboratory Operations


465 Questions
500 Answer Key
501 Answers with Explanations

0'2018ASCP ISBN978·089189-6609 Cllnlcal Laboratory Certification Exomlnatlons iii


1: Blood Bank Blood Products

Blood Bank
The following items have been identified generally as appropriate for both entry /eve/ medical
laboratory scientists and medical laboratory technicians. //ems that are appropriate for medical
hiboretory scientists only are marked with an 'MLS ONLY."

1 Questions 58 Answers with Explanations


1 Blood Products 59 Blood Products
8 Blood Group Sy stems 61 Blood Group Systems
·t 7 Blood Group Immunology 65 Blood Group Immunology
22 Pliyslology & Pathophysiology 67 Physiology & Pathophysiology
29 Sero/ogle & Molecular Testing 71 Serologic & Molecular Testing
47 Transfusion Practice 78 Transfusion Practice

Blood Products
1. The minimum hemoglobin concentration In a fingerstick from a male blood donor is:
a 12.0 gldl (120 g/L)
b 12.5 g/dl (125 gll)
c 13.0 g/dl (130 g/ L)
d 13.5 gldl (135 g/L)
2. A cause for indefinite deferral or blood donation is:
'"
O~Y
a diabetes
b residence in an endemic malaria region
c positive test for Trypanosoma cruzi
d history of therapeutic rabies vaccine
3. Which of the following prospective donors would be acoepted lor donation?
""
au a 32-year-old woman who received a translusion in a complicated delivery 5 months
previously
b 19-year-old sailor who has been stateside tor 9 months and stopped taking his
anti-malarial medication 9 months previously
c 22•year-old college student who has a temperature of 99.2°F (37.3°C) and slates that
he feels well. but is nervous about donating
d 45-year-old woman who has just recovered from a bladder infection and is stilt taking
antibiotics
4. Which one of the following constitutes permanent deferral status of a donor?
a tattoo 5 months previously
b recent close contact with a patient with viral hepatitis
c 2 units or blood transfused 4 months previously
d conlirmed positive test for HBsAg 10 years previously
5. According to AABB standards, which of the fo61owlng donors may be accepted as a blood
....
ON,Y donor?
a traveled to an area endemic for malaria 9 months previously
b spontaneous abortion at 2 months of pregnancy, 3 months previously
c resides with a known hepatitis B patient
d received a blood translusion 22 weeks previously

02018 ASCP ISBN 978-089169-6609 Clinical Laboratory Certification Ex•mlnetlon• 1


j
1: B lood Bank Bloor1 Pro~ ~
. uUcf$ '
6_ Wh ich of the following must be included on the label of a unit of Red BIOOd Cells -
Leukocytes Reduced? I
a known leukocyte count for the unit
b phlebotomist idenlification
c unique collection facility iden tifier
d date of blood collection
7. In order to be a plateletpheresis donor, the platelet count must be at least:
a 150,000/µL
b 200,000/µL
c 250,000/µL
d 300,000/µL
8. Prior to blood donation. the intended venipuncture site must be cleaned with a scrub
solution containing:
a hypochlorile
b green soap
c 10% acetone
d povidone iodine
9. All donor blood testing must include:
a complete Rh phenotyping
b anti-CMV testing
c d irect antiglobulin tes t
d serological test for syphilis
10. During the preparation of Platelets from Whole Blood, the blood should be:
a cooled towards 6 •c
b cooled towards 20- 24 •c
c warmed to 37°C
d heated to 57°C
11. The transport temperature for Red Blood Cells Leukocytes ~educed is
a 1-6°C
b 1-1o·c
c 18-20"C
d 20-24•c
12. The transport temperature for Apheresis Platelets is
a 1-s·c
b 1-1o·c
c:· 1s-2o·c
d 20-24·c
13. The test that is currently used to detect donors who are infected with the AIDS virus is:
a anti-HBc
b anti-HIV· 1/2
c HBsAg
d ALT
14. A donor who has just donated 2 units of Apheresis Red Blood Cells will be deferred from
further blood donation for a minimum of how many weeks?
a 8
b 12
c 16
d 24 I
2 Tho Board of Certification Study Gulde 60
ISAN Q71\.JlA91B~609 C'2Ql8~S(P l
1: Blood Bank Blood Products
15. Red Blood Cells Leukocytes Reduced must be prepared by a melhod known lo reduce the
leukocyte count to
a <8.3 x 10s
b <5.0 x 106
c <5.5 x 10 10
d <3.0 x 1011
16. A unil of Red Blood Cells that expires in 32 days has just been irradiated. The expiration
date of th is unit will
a remain the same
b be reduced by 4 days
c be reduced by 14 days
d be increased by 2 days
17. A unit of Red Blood Cells expiring in 35 days is split into 5 small aliquots using a sterile
pediatric quad set and a sterile connecting device. Each aliquot must be labeled as
expiri ng in:
a 6 h ours
b 12 hours
c 5 days
d 35 days
18. When platelets are stored on a rotator set on an open bench top. the ambient air
temperature must be recorded:
a once a day
b twice a day
c every 4 hours
d every hour
19. Which of th e following is the correct storage temperature for the component listed?
a Cryoprecipitated AHf?, 4°C
b Fresh Frozen Plasma (FFP), -20°C
c Red Blood Cells, Frozen, -40°C
d Platelets, 37°C
20. Six units of Red Blood Cells are issued to the OR at 9am in a cooler, validated ta maintain
a temperature of 1-1o•c for 2 hours. Forty minutes later, the cooler containing the units of
blood is returned to lhe blood bank as surgery was cancelled. What should be done with
these units?
a discard the units as they were issued to a specific patient
b inspect units and establish that appropriate temperature has been maintained
c continue to store in cooler since surgery is rescheduled for tomorrow •
d put units back into inventory as only 40 minutes has elapsed since issue
21 . An acceptable storage temperature for Red Blood Cells, Frozen is:
l'l.S
OHtY
a -ao•c
b - 2o·c
c - 12·c
d 4•c
22. Red Blood Cells, Leukocytes Reduced must be slored at:
a 1-6°C
b 1-1o·c
c 1-2o•c
d 20-24•c

@2018ASCP ISBN 97B-Oa9169·6609 Clinical Laboratory Certification Examinations 3


Blood Products
1: Blood Bank
If the seal is entered on a unit of Red Blood Cells stored at 1-6°C, what is the maximum
23.
allowable storage period , in hours?
a 6
b 24
c 48
d 72
24. Cryoprecipitated AHF must be stored at:
a s-1o·c
b s- 18°C
c 1-s·c
d 1-1o•c
CryoJ)fecipitated AHF must be transfused within what period of time following thawing and
25.
pooling?
a 4 hours
b 8 hours
c 12 hours
d 24 tiours
Plasma Frozen Within 24 Hours After Phlebotomy (PF24) and thawed for transfusion has
26.
an expiration of
a 6 hours
b 12 hours
c 24 hours
d 5 days
27. Apheresis Platelets must be stored at:
a 1-6°C
b 1-1o·c
c 10-18°C
d 20-24•c
28. According to AABB Standards, Fresh Frozen Plasma must be infused within what period
of time following thawing?
a 24 hours
b 36 hours
c 48 hours
d 72 hours
29. s·c o r below. has a shelf life
Cryoprecipitated AH F, if maintained in the frozen state a t - 1
of:
a 42 days
b 6 months
c 12 months
d 36 months
~· Quality control of Apheresis G ranu locytes must demonstrate wh ich of th e following
OHL< granulocyte counts in 75o/o of units tested ?
a 1.0 x 1010
b 2.0 )( 1010
c 3.0 )( 1010
d 4.0 )( 1010

4 T he Board ot Certification Study Gulde 6o ISBN 978·089189-6609 CJ2018~


1: Blood Bank Blood Products \
31.
t.'LS
0'..Ll
An important determinant of platelet viability during storage is: -
a plasma potassium concentration
b plasma pH
c prothrombin time
d activated partial thromboplaslin time
32. Thawed Plasma must be stored at:
a s- 1a•c
b 1-6°c
c 1-1o•c
ct 20-i4•c
33. During storage, the concentration or 2,3-diphosphoglycerate (2,3-DPG) decreases In a
~v unit of:
a Platelets
b Fresh Frozen Plasma
c Red Blood Cells
d Cryoprecipitated AHF
34. Cryoprecipilated AHF:
a is Indicated for fibrinogen deficiencies
b should be stored at 4 •c prior to administration
c will not transmit hepatitis B virus
d is indicated for the treatment of hemophilia B
35. Which Apheresls Platelets product should be irrad iated?
MLS
CRY a autologous unit collected prior to surgery
b random stock uni! going to a patient wilh OIC
c a directed donation given by a mother for her son
d a directed donation given by an unrelated ramify friend
36. Irradiation of a unit of Red Blood Cells is done to prevent the replication of donor:
a granulocytes
b lymphocytes
c red cells
d platelets
37. Plastic bag overwraps are recommended when thawing units of FFP in 37°C water baths
because they prevent:
a the FFP bag from cracking when it contacts the warm water
b waler from slowly dialyzing across the bag membrane
c the entry ports from becoming contaminated with water
d the label from peeling off as the water circulates In the bath
38. Which of the following blood components must be prepared within 8 hours after
phlebotomy?
a Red Blood Cells
b Fres!i Frozen Plasma
c Red Blood Cells, Frozen
d Cryoprecipitated AHF
39. Which of the following infectious agents reties solely on donor questioning to avoid
ttansmission from transfused blood products?
a Trypanosoma cruzi
b Plasmodium falciparum
c Zika virus
d CMVvirus

@2018 ASCP ISBN 978--089189-6609 Cllnlcal Laboratory Certification Exam/notions 5


1: Blood Bank
Blood Products
40.
Which <?f th.e following practices at the time of blood collection helps minimize bacterial
contamination of platelet products?
a use of 18-gauge needle
b diversion pouch
c green soap scrub
d UV irradiation
41. Which of the following traveO-related responses will result in donor deferral?
Id.$
Oto.V
a residence in England for 6 months during 1990
b residence in Scotland for 1O months during 2016
c residence in Chad 1O years previously
d residence in Canada 2 years pieviously
42. Which of the following practices has been u seful in reducing the incidence of Transfusion
Related Acute Lung Injury (TRALI}?
a use of Fresh Frozen Plasma from male donors
b use of Fresh Frozen Plasma from female donors
c pathogen reduction treatmen t of Fresh Frczen Plasma
d leukocyte-reduced Fresh Frozen Plasma
43. Upon inspection, a unit of Apheresis Platelets is noted to have visible clots, but otherwise
appears normal. The technologist should:
a issue without concern
b filter to remove the clots
c centrifuge to express off the clots
d quarantine for Gram stain and cullure
44. According to AABB Standard s, a t least 90% of atl Apheresis Platelets units tested sha'.t
contain a minimum of how many platelets?
a 5.5 >< 10 10
b 6.5" 10 10
c 3.0" 10 11
d 5.0 )( 10 11

....
45•
ONLY
a
According to AABB Standards, Platelets prepared from Whole Blood shall have at least:
5.5 >< 10 10 platelets per u.!'lit in at least 90% of the units tested
b 6 .5" 1010 platelets per u7iit in 90% of the units tested
c 7.5 >< 101 0 plate:ets per unit in 100% of the units tested
d 8.5 >< 10 10 platelets per unit in 95% of the units tested
46. Which o f the following is proper proced ure for prepa ration of Platelets from Whole Blood?
a light spin followed by a hard spin
b light spin followed by 2 hard spins
c 2 light spins
d hard spin followed by a light spin
47. According to AABB Standards, what is the minimum pH required for Apheresis Platelets at
"""
°"''
the end o f the storage period?
a 6.0
b 6 .2
c 6.8
d 7.0

6 Tho Board o f Certlnc:atlon Study Gulde 6e ISBN 978-089 169-Wl9 (l2018ASCP


1: Blood Bank Blood Products
~8. According to AABB Slandards, what is lhe minimum hemoglobin level for an aulologous
donor?
a 11 .0 gldL
b 12.0 gldL
c 12.5 g/dL
d 13.0 gfdL
l.o. Upon expiration, a unil of thawed Plasma Frozen Within 24 Hours (PF24) is converted lo
!hawed Plasma. This thawed Plasma can be stored for an addilional
a 1 day
b 4 days
c 14 days
d 28 days
50. The lransfusion service is preparing aliquots from a unil of Red Blood Ceils Leukocyles
Reduced with lhe aid of a sterile connecting device for a pediatric patient When checking
the weld for one of lhese aliquots, it is noted lhat lhe weld is incomplete and leaking. This
unil is lhen resealed with an acceptable weld. Whal will !he expiralion date of lhis unit be?
a 6 hours
b 24 hours
c 3 days
d original expiration dale
51. Whal percenlage of red blood cells must be relained when preparing Red Blood Cells
Leukocytes Reduced?
a 50%
b 70%
c 85%
d 100%
52. Of the following blood components, which one should be used to prevent H LA
~. alloimmunization of the recipient?
a Red Blood Cells
b Granulocyles
c Irradiated Red Blood Cells
d Leukocyte-Reduced Red Blood Cells
53. Which of the following infectious agents must be tested for using a nucleic acid testing
(NAT) assay?
a HBV
b HTLV 1111
c syphilis
d cytomegalovirus
54. What is the primary reason lhat Infectious agents can be transmitted following blood
~. transfusion?
a pathogen reduction technology failure
b donor in the window period of early infec.Uon
c leukocyte-reducllon failure
d donor history questionnaire nol completed
SS. What isfare the minimum pretransfusion tesllng requiremenl(s) for autologous donallons
collected and lransfused by the same facility?
a ABO and Rh typing only
b ABOIRh type, antibody screen
c ABO/Rh type, antibody screen, crossmatch
d no prelransfuslon testing is required for autologous donations

Q2018 ASCP ISBN 978-08918U 609 Clinical L•bor•tory Certific•tion Examinations 7


- 1: Blood Bank Blood Group Syst
ems """'
56. In a quality assurance program. Cryoprecipitated AHF must contain a minimum or how --
'-"!:S
tr·~~v
many international units or Factor VIII?
a 60
b 70
c 80
d 90

Blood Group Systems


57. Which or the following antibodies is usually clinically insignificant?
a antl-P
b anti-P1
c anti-Pk
d anti-p
58. Refer to the following diagram:

R,r R,R,

1 2

II

1 2 3 4

Given the most probable genotypes of the parents. which of the following statements beSt
describes the most probable Rh genotypes of the 4 childre.n?
a 25% will be Ror. 25%will be R 1r, and 50% will be R 1R 1
b 50% will be R 1 r and 50% will beR1R 1
c 100% will be R 1r
d 100% will be R1R 1
59. The linked HLA genes on each chromosome constitute a(n):
a allele
b trait
c phenotype
d haplotype

so. An individual's red blood cells give the following reactions with Rh antisera:
=~li-D •nll-C anll· E antl-c anll-e Rh control
3+ 0 3+ 3+ 0
The Individual's most probable genotype is:
a DCe/DcE
b DcE/dce
c Dee/dee
d DCeldce

l 8 The Board of Certiflc,a llon Study Gulde G.


I: Blood Bank Blood Group Systems
,;1. A blood donor has the genotype: hh, AB. Using anti-A and anti-B antisera . the donor's red
cells will type as group
a A
b B
c 0
d AB
:.2. An individual has been sensitized to the k antigen and has produced anti-k. What is the
most probable Kell system genotype?
a KK
b Kk
c kk
d KoKo
63. Anti-Fy3 will fail to react with which of the following enzyme treated red cells?
MlS
°"' a Fy(a+b- )
b Fy(a- b+)
c Fy(a- b- )
d Fy(a+b+)
64. A mother has the red cell phenotype D+C+E-o-e+ with anti-c (liter of 32 at AHG) in her
serum. The father has the phenotype D+C+E-c+e+. The baby is Rh-negative and >110!
affected with hemolytic disease of the newborn. What is the baby's most probable Rh
genotype?
a r'r'
b r'r
c R1R1
d R1r
65. In an emergency situation, Rh-negative red cells are transfused into an Rh-positive person
of the genotype Cfle/CDe. The first antibody most likely to develop is:
a anti-c
b anti-d
c anti-e
d anti-E
66. Most blood group systems are inherited as:
a sex-linked dominant
b sex-linked recessive
c autosomal recessive
d autosomal codominant
67. The mating of an Xg(a+) man and an Xg(a-) woman wilD only produce:
a X9(a-) sons and Xg(a-) daughtess
b Xg(a+) sons and Xg(a+) daughters
c Xg(a-) sons and Xg(a+) daughters
d Xg(a+) sons and Xg(a- ) daughters
68. Refer to the following data:
anll-C antl-e
+ + +

Given the reactions above, which is the most probable genotype?


a R1R1
b R 1r'
c Ri/'
d R1Rz

i:.2016 ASCP ISBN 978-089189·6609 Cllnlcal Laboratory Certification Examinations 9


zq
1 : B lood Ban k Blood Group Systems
69. A patient's red cells type as follows:
antl-0 antl-C anti-E
4+0 0
Which of the following genotype would be consistent with these results?
a RoRo
b R 1r
c R1R2
d R,r
70. The red cells of a nonsecretor (se/se) will most likely type as:
a Le(a- b-)
b Le(a+b+)
c Le(a+b-)
d Le(a- b+)
71 . Which of the following phenotypes will rea:t wl!h anti-f?
a rr
b R1R1
c R~2
d R,R2
72. A patient's red blood cells gave the following reactions:
antl-0 antl-E antl-e anti-I
+ • + + + 0

The most probable genotype of this patient Is:


a R1R2
b Rzr"
c R,r
d R , R,
Anti·K Is identified in a patient's serum. If random crossmatches are performed on 10
donor units, approximately how many would be expected to be compatible?
a 1
b 3
c 7
d 9
74. A woman types as Rh-positive. She has an anti-c titer of 32 at AHG. Her baby has a
negative DAT and is not affected by hemolytic disease of the newborn. Wllat is the father's
most likely Rh phenotype?
a rr
b r" r
c R1r
d R1r
75. Which of the following red cell typings are most commonly found in the African American
donor population?
a Lu(a- b-)
b Jk(a-b-)
c Fy(a-b-)
d K-k-

10 The Bon rd of Cortlllcallon St udy Gulde 6o


ISBN 978-089189-0009 C2f) tSASGP
1: Blood Bank Blood Group Systoms
76. Four units of blood are needed for elective surgery. The patient's serum contains anti-C,
anti-e. anti-Fy• and anti-Jkb. Which of the following would be the best source of donor
blood?
a test all units in current stock
b test 100 group 0 , Rh-negative donors
c test 100 group-compatible donors
d rare donor registry
77. A donor is tested with Rh antisera with lhe following results :
a nli·D antl-C antl-E antl-c ontl· e Rh
control
+ • 0 + • 0

What is his most probable Rh genotype?


a R 1R 1
b R 1r
c Ror
d R2f
78. A family has been typed for HLA because t of the children needs a stem cell donor. Typing
results are fisied below:
father: Al.3:88.35
mother. A2,23;812.18
child #1: A1 .2;B8.12
child #2: Al,23;88 .18
child #3; A3,23;B18,?

What is the expected B antigen in child #3?


a A1
b A2
c 912
d 935
79. Which of the following is the immunodom.inant sugar responsible for the A antigen?
a fucose
b N-Acetytgalactosamine
c galactose
d N-Acetylglucosamlne
80. A patient is group O. Rh-negative with anti·D and anti-K in her serum. What percentage of
~' the general Caucasian donor population would be compatible with this patient?
a 0.5
b 2.0
c 3.0
d 6.0
81. The observed phenotypes in a particular population are:
....
CH.Y Phenotype Number of persons
Jk(a•b- ) 122
Jk(a+b•) 194
Jk(a- b•) 84

What is the gene frequency of JI<" in this population?


a 0.31
b 0.45
c 0.55
d 0.60

C0l18ASCP ISBN 97&-0891~ Clinical Labontory Certlflcatlon Ex •mlnatJons 11


Blood Group Syst
~
1 : Blo od Bank ems
82. In a random population, 16% of the people are Rh-negative (IT). What percentage of the- -
~. Rh-positive population is heterozy!;ous for r?
a 36%
b 48%
c 57%
d 66%
aJ. Which of the following is considered to be a high prevalence antigen?
a Vel
b Js•
c s
d K
84. Tho reason that group O individuals have the most amount of H antigen on their red eels
compared to other ABO phenotypes is:
a Group 0 individuals produce more precursor type I chain
b Group A, B and AB individuals are heterozygous for the H gene
c The O gene produces more transferase enzyme which produces more H antigen
d H antigen is left unchanged by the absence of A and/or B transferase enzymes
85. Which phenotype could not result from lhe maling of a Jk(a+b+) female and a Jk(a-b•)
male?
a Jk(a+b-)
b Jk(a+b+)
c Jk(a- b+)
d Jk(a-b-)
86. A palient is typed with the following results:

I P~1ient• s
anli·A
ontl·B
anti·A.B
cells with
0
0
2•
Patient's scrum w1th
A1red cells
Bred cells
Ab screen
2+
4•
0

The most probable reason for these findings is that the patient is group:
a O; confusion due to faulty group 0 antiserum
b O; with an anti-A 1
c A,; with an anti-A1
d A 1; with an anti-A
87. W hich of the following phenotypes is lhe result or homozygous inheritance of the
corresponding genes?
a Le(a+b-)
b M+N+
c Fy(a-b+)
d Jk(a+b+)
88. Given the following serologic reactions, what is the most likely A subgroup?
Anti-A Anll-9 Antl-A ,B oc1ll•
A 1 cells B cells
2•ml 0 2+-mf 1T 4+ 0
Mf•mi•ed field agglutination
a A,
b A2
c A3
d A,

12 The Board of Cel1lflcatlon Study Gulde 6e


·1: t:l IOOCI t:lank Blood Group Systems
89. The enzyme responsible for conferring H activity on the red cell membrane is alpha-:
a galactosyl transferase
b N-acetylgalactosaminyl transferase
c L-fucosyl transferaso
d N-acety1g1ucosaminyl transferase
90. Even in the absence of prior transfusion or pregnancy, individuals with lhe Bombay
phenotype (Oh) will always have naturally occurring:
a anti-Rh
b anti-K0
c anti-U
d anti-H
91. Which of the following antibodies in the Lutheran system is most likely to be lgM and
detected as a direct agglutinin?
a anti-Lu"
b anll-Lub
c anti-Lu3
d anti-Au•
92. Which of the following antibodies is neutralizabte by pooled human plasma?
....
OH.• a anti-Kn•
b anti-Ch
c anti-Yk•
d anti-Cs•
93. Antibodies from which of the following blood group systems are notorious for causing
delayed hemolytic transfusion reactions?
a Rh
b Kell
c Duffy
d Kidd
94. Ha.A antibodies are:
a naturally occurring
b induced by multiple transfusions
c directed against granutocyte antigens only
d frequently cause hemolytic transfusion reactions
95. Genes of the major hlstocompatibility complex (MHC):
a code for HLA-A, HLA-B, and HLA-C a~tigens only
b are linked to genes in the AlilO system
c are the primary genetic sex-determinants
d contribute to the coordination of cellular and humeral immunity
lsolmmunization to platelet antigen HPA-',a and the placental transfer of maternal
...,
96.
.... antibodies would be expected to cause newborn:
a erythroblastosis
b leukocytosis
c leukopenla
d thrombocylopenia

Cllnlc1/ taboratOtY Certlfic•tion ExamlnatJons 13


Cl2~18ASCP ISBN 978-069189·6E09
~----===---------------
1: Blood Bank
Blood Group System
~~~tigens would be found in !he saliva of an Individual wilh !he genotype Sese Lele~ """
97
·

a A ,H
b Leb, A, H
c lea, Leb, A, H
d Le•
98. Which of !he following genes is not in the MHC class I region?
l\.S
"'" a HLA-A
b HLA-B
c HLA-C
d HLA-DR
99. Which of the following Rh antigens has the hrghest frequency in Caucasians?
a 0
b E
c c
d e
f OO. Anti-0 and anti-Care identified in the serum of a pregnant woman, gravida 2, para 1. Ten
~' months previously she received Rh irrmune globulin (RhlG) at 28 weeks' gestation. Tests
of the patient, her husband, and the child revealed the following
anti· D anti·C antf·E anti·c antl-e
patient 0 0 0 +
father 0 • 0 + •
Child 0 + 0 + •
The most ijke/y explanation for the presence of anti-0 is that !his a ntibody is:
a actually anti-cw
b from the RhlG dose
c actuany ariti-G
d naturally occurring

...
101 . The phenomenon of an Rh-positive pe1Son whose serum contains anli-0 is best explained
..... by:
a gene deletion
b missing anligen epilopes
c trans position effect
d gene inhibition
102.
.....
o...v
When the red cells of an individual fail l e> react with anli-U, they usually fail to react with:
a anti-M
b anli-Leb
c anti-S
d anti-P1
103. Which of the following red cell antigens are found on h .
a M, N 9 1ycop onn-A?
b Le•, Leb
c S, s
d P, P 1, ~

f 4 Th• Board of Certjfic:aUon Study Guld• 6e


ISBN 978-089189-6609 C20t8ASCI'
1: Blood Bank Blood Group Systems
104. Paroxysmal cold hemoglobinuria (PCH) is associated with antibody specificity toward
which of the following?
a Kell system antigens
b Duffy system antigens
c P antigen
d I antigen
!fl5. Which of the following is a characteristic of anti·i?
a associated with warm au toimmune hemolytic anemia
b found in the serum of patients with infectious mononucleosis
c detected at lower temperatures in the serum of normal individuals
d found only in the serum or group 0 individuals
! ~6. In a case of cold agglutinin disease, the patient's serum would most likely react 4+ at
immediate spi n with:
a group A cells. B cells and 0 cells, but not his own cells
b cord cells but not his own or other adult cells
c all cells of a group 0 cell panel and his own cells
d only penicillin-treated panel cells, not his own cells
107. Cold agglutinin disease is. associated with an antibody specificity toward which of the
following?
a Fy:3
b p
c I
d Rh:1
108. Which of the following is a characteristic of anti-i?
a often associated with hemolytic disease or the newborn
b reacts best at room temperature or 4•c
c reacts best at 37°C
d is usually lgG
109. The Kell (K1) antigen is:
a absent from the red cells of neonates
b strongly immunogenic
c destroyed by enzymes
d has a frequency of 50% in the random population
110. In chronic granulomatous disease (CGD), granutocyte function is impaired. An association
~tr exists between this clinical condition and a depression of which of the following antigens?

a Rh
b p
c Kell
d Duffy
111. The antibodies of the Kidd blood group system:
a react best by the indirect antiglobulin test
b are predominantly lgM
c often cause allergic transfusion reactions
d do not generally react with antigen posi~ve, enzyme treated RBCs

02018ASCP ISSN 978·089189-8609 Cllnlcal Laboratory Cert.iflcatlon Examinations 15


tmm:Jr-------------~~~
1: Blood Bank Btooa G
ou ; ; ,
· . P Syit
112. Proteolytic enzyme treatment of red cells usually destroys Which antigen? ~
a Jk8
b E
c Fy"
d k
113. Anti-Fy8 is:
a usually a cold-reactive agglu~lnin ealed red blood cells
b more reactive when tested w 1tll enzyn:ie 1r .
c capable of causing hemolytic transfusion reacllons
d orten an autoagglulinin
114. Resistance 10 malaria is best associated with which of the following bloOd groups?
a Rh
b Iii
c p
d Duffy
115. What percent of group p donors would be compatible with a serum sample that contained
MLS
CHl.Y
anti-X and anti-Y if X antigen rs present on red cells of 5 of 20 donors, and Y antigen is
present on red cells of 1 of 10 donors?
ii 2.5
b 6.8
c 25.0
d 68.0

...116.
"""
What is linkage disequilibrium in reference to HLA haplotypes?
a occurrence of H!.A genes in the same haplotype more often than would be expected
based on the gene frequencies
b displacement of HLA genes on different chromosomes
c occurrence of HLA genes in the same haplotype fess often than would be expected
based on the gene frequencies
d recombination or HLA genes during meiosis.
117. What is the aJ?Proximale probabilily of finding compatible blood among random Rh-positile
11.$
MY units for a patient who has anti-c and anti-K? (Consider that 20% of Rh-positive don<WS
lack c and 90% lack K)
a 1%
b 10%
c 18%
d 45%

16 Tiie Board of Cert"flc


' •Uon SJudy GuJde 6e
1 : Blood Bank Blood Group Immunology
.,.
'· 1\The25-year-old Caucasian woman, gravida 3, para 2, required 2 units of Red Blood Cells .
antibody screen was positive and the results of the antibody panel are shown below:

Cell 0 c c E e K Jk• Jkb Le' Leb M N P1 AHG


1 + + 0 0 + + + + 0 + + + + 0
2 + + 0 0 + 0 + 0 0 + + 0 0 0
3 + 0 + + 0 0 + + 0 + + + + 1+
4 + + + 0 + 0 0 + 0 + + 0 + 1+
5 0 0 + 0 + 0 + + 0 + + 0 0 1+
6 0 0 + + + 0 + 0 + 0 + + 0 1+

7 0 0 + 0 + + + + + 0 + + + 1+

8 0 0 + 0 + 0 0 + 0 + 0 + + 1+
auto 0

What is the most probable genotype of this patient?


a rr
b ,.,.
c R0r
d R1R1

Blood Group Immunology


119. Which of the following is responsible for the production of blood group antibodies?
a B cells
b T cells
c NK cells
d dendritic cells
120. In a primary immune response. which imnunoglobulin class appears first?
a lgG
b lgM
c lgA
d lgE
121. What is the most common clinical incident that results in alloantibody production?
a viral infection
b solid tumor
c red cell transfusion
d autoimmune disease
122. Which lmmunoglobuiin class can cross the placenta?
a lgG
b lgM
c lgA
d lgE

02018 ASCP IS8tf 978-089189-6609 Clinical loboralory C.rtillcotlon &omtmfoiA :17


1 : Blood B ank
123. W h ich of th f . . Blood Group Immunology
· e 0 1lowing 1mmunogl b " ·
I
c ass1cal pathway? o uwns rs most efficient at activating complement via the
a lgG2
b lgG 4
c lgM
d lgA
124. Which of the following immunoglobulins is most efficient at ca using direct
h emagglutination?
a tgG
b lgM
c lgA
d lgE
125. Wh ich o f the f°'1owing structures of an immunoglobulin molecule is involved in the
activation of complement via the classical pathway?
a Fe region
b Fab region
c Hinge region
d J chain
126. lmrnunoglobulin classes are differentiated a::cording to the molecular structure of
a light chains
b heavy chains
c Fab fragment
d Fe fragment
7. M acrophages and monocytes have Fe receptors for which of the following
immunoglobulins?
a lgG
b lgM
c lgA
d l gD
128. The membrane attack complex (MAC) formed during the activation of complement via the
classical pathway consists of
a C1q, C1r. C1s
b C3a. C3b. C3d
c C 5a, C5b
d C5b through C9
129. Appro priate antigen-antibody ratios are important to avoid an e xcess of u nbound antibody.
k nown as
a dosage effec t
b pf>I effect
c pos tzone effec!
d prozone effect

~~O. ":fhich of the. following pairs of immunoglobulins Is most efficient at activating complement
..,, via the cla ssical pathway?
a lgG1 and lgG3
b lgG1 and lgG4
c lgG2 and lgG3
d lgG2 and lgG4

8 The Board of Certification St udy Guido 6e


ISBN 978-089189-6609 C>2018ASCP
1: Blood Bank Blood Group Immunology
131. In ASO hemolytic transfusion reactions, ccmplement is activated via which of the following
pathways?
a alternative
b classical
c lectin
d polyclonal
132. Many enhancement media used in the blood bank promote hemagglutination in the
presence of lgG antibodies by reducing which of the following?
a hydrophilic forces
b low ionic potential
c van der Waals forces
d zeta potential
133. The following panel results were obtained with the serum of a patient from the prenatal
~~Y clinic.

LISS
Cell D c c E e K Jk• Jk• Fy• Fy• M N P1 IS 37•c AHG
1 + + 0 0 + + + + 0 + 0 + + 0 0 0
2 + + 0 0 + 0 + 0 0 + + 0 0 3+ 2+ 2+

3 + 0 + + 0 0 + + 0 + 0 + + 0 0 0
4 + + + 0 + 0 0 + 0 + + + + 3+ 2+ 2+

5 0 0 + 0 + 0 + + 0 + + 0 0 3+ 2+ 2+

6 0 0 + + + 0 + 0 + 0 0 + 0 0 0 0
7 0 0 + 0 .. + + + + 0 + + + 3+ 2+ 2+
8 0 0 + 0 + 0 0 + 0 + 0 + + 0 0 0
auto 0 0 0

To evaluate potential risk to the fetus, what additional studies should be performed?
a test additional cells to rule.out anti-c
b treat serum with dithiothreitol and repeat panel
c Rh phenotype the patient
d perform 0 testing on father's blood
134. Blood group antigen and antibody hemagglutination reactions are influenced by which of
the following?
a temperature
b Ca2 • ions
c antigen presenting cells
d memory cells
135. Which of the following blood group antigens is the most immunogenic, or has the greatest
ability to initiate antibody production in an individual who lacks the antigen?
a Fya
b s
c Jkb
d D

C21l18ASCP ISSN 978·089169·6E09 Clinical Laboratory Cert.incatlon Examinations 19


4
1 : Bloo d Bank Blood Group lmmuno/ogy
136. W hich of the following blood group antibodies will no lo~ger react with its respective
antigens once those antigens are treated with proteolytic enzymes?
a anti-C
b anti-K
c a ntl-Fya
d anti-Jka
137. The immune response to red cell antigens with numerous epitopes results in a
heterogeneous population of antibodies referred to as
a bivalent
b epitope-specific
c monoclonal
d polyclonal
138. Some blood group antibod ies may react stronger with the red cells of Individuals w ho have
inherited 2 identical alleles for the antigen to which the antibody is directed. This is known
as
a postzone effect
b dosage effect
c prozone effect
d equivalence effect
139. lgG antibody and/or C3b sensitized red blood ce lls can be phagocyllzed by cells in the
reticuloendothelial system (RES). This ptenomenon is clinically recognized as
a Immune tolerance
b allergic sequalae
c e xtravascular hemolysis
d lntravascular hemolysis
140. Which of the following reagents is used to facilitate hemagglutination following the
sensitiza tion o f red cells with an lgG alloantibody?
a anti-human globulin serum
b low ionic strength saline
c polyethylene glycol
d 22°/o bovi ne albumin
141. The addition of antibody-sensi!Ozed red cells (Check Cells) to all negative anti-h uman
globl?lln (AHG) tests ensures that
a the test was interpreted correctly
b the test was incubated a t the correct temperature
c AHG reagent was added to each test
d patient serum was added to each test
142. In which of the following clinical situations will the direct anliglobulin test be positive?
a s ickle cel l disease
b hemolytic disease of the fetus and newborn
c posttransfusion purpura
d multiple myeloma
143. In which of the following is the Indirect antiglobulin test utilized ?
a rev erse ABO testing
b immediate spin crossmatch
c C a ntigen testing
d a ntibody detection (screening) test

Tho Board o f c e rtlOcatlon Study Gulde&. ISBN 978-069169-6609 C2018A.~


20
1: Blood Bank Blood Group lmm11nology

144. Polyspecific AHG reagents contain


a anti-lgG
b anti-C3d
c anti-lgG and anti-lgM
d anti-lgG and anti-C3d
145. A raise-negative direct antlglobulin test can be the result or
a neutralized AHG reagent
b sample from gel-separator tubes
c overcentrifugation
d dirty glassware
146. A false-positive indirect antiglobulin test can be the result of
a insufficient saline washing of red cefls
b inadequate incubation time
c overcentrifugation
d dissociation of cell bound lgG
147. The following ABO results are noted on a sample from a 90-year-old male admitted to the
ER for possible GI bleeding:
Coils testing: Serum tostJn g:
Anti-A: O A 1 cells: 0
Anti-8: O B cells; h

What might be a likely explanation for these results?


a wrong sample collected
b patient's disease
c patient's age
d subgroup of B
148. lgG-sensitized red cells that activate complement may not complete complement
~~' activation to cell lysis. In addition to lgG on the sur1ace o f these red cells, what
complement component is present?
a C1q
b C3a
c C3b
d C5b
149. A negative result using solid phase adhererce assays will demonstrate indicator red cells
as
a a red blood cell pellet in the bottom of the well
b a diffuse pattern of red blood cells throughout the well
c red blood cell clumps symmetrically located throughout the well
d a red supernatant, indicating lysis
150. A 4+ positive reactions using gel technology will appear as red blood cells
a in a pellet at the bottom of the microtubule
b dispersed throughout the gel media
c in a layer at the top of the gel media
d suspended at the mid -point of the gel media
151. One of the advantages of performing antibody screening (detection) studies using gel
technology is
a saline washing is not required
b centrifugation is not required
c special equipment is not required
d precise volumes or serum are not required

02018ASOP ISBN 978-0891~09 Cllnlcat L,aboratory Certification Examinations 21


1 : B lood an
B k
.
• · ·.1 - - -

ts have which or the following as a disadvant


·•.t ... ' HtlJQ

9~
''"Ysr010 1
noclonal blood banking reagen age7
152. Mo
3 little to no batch variation
b cost effectivenes~
c high antigen efficiency
d overspecificity .
. . suspected case of warm autoimmune hemolytic a
153. The direct antiglobuhn ter ~n ~ng monospecific reagents would be used in furthe~de!l'ia
is positive. Which of the o owi 1 ~eq
antiglobulin testing?
a anti-C3b
b anti-C3d
c anli-C4
d anti-lgM
. Low ionic strength saline (LISS) acts as an enhancement medium and facilitates nnlibO<Jy
154
uptake by
a activating complement . .
b increasing flexibility in hinge region
c removing water molecules
d reducing zeta potential

Physiology & Pathophysiology


155. Transfusion of which of the following is needed to help correct hypofibrinogenemia due kl
~v DIC?
a Whole Blood
b Fresh Frozen Plasma
c Cryoprecipitated AHF
d Platelets
156. A blood component used in the treatment of hemophilia A is:
a Factor VII I
b Fresh Frozen Plasma
c Platelets
d Whole Blood
157. Blood typing after a normal labor and delive ry showed that the mother is group A, .
IU
00.Y D-negallve and demonstrates anti·D In her serum. Her slightly jaundiced newborn is
anemic and types as group 0 , 0-negative with a 4 + DAT. Previous lab work showed .
lhat the father was group 0 , 0-positive. From the information given, which test result is
questionable?
a paternal D type
b newborn D type
c maternal ABO type
d newborn OAT
158. A u~it of Fresh F:ozen Plasma was inadvertently thawed and then immediately
IU
O!<l.Y refngerated at 4 Con Monday morning. On Tuesday evening this unit may still be
transfused as a replacement for:
a all coagulation factors
b FactorV
c Factor VIII
d Factor IX

22 The Board of CertlflcaUon Study Gulde 6o e20t·


I I ISBN 978-089189-66()9
1: Blood Bank Physiology & Pathop/1ysiology

...
159.
.....
,
A newborn demonstrates petechiae, ecchymosis and mucosal bleeding. The preferred
blood component for this infant would be:
a Red Blood Cells
b Fresh Frozen Plasma
c Platelets
d Cryoprecipitated AHF
160. Which of the following would be the best source of Platelets for transfusion in the case of
s alloimmune neonatal thrombocytopenia?
1...

°"'' a father
b mother
c pooled platelet-rich plasma
d polycythemic donor
161 . An obstetrical patient has had 3 previous pregnancies. Her first baby was healthy, the
second was jaundiced at birth and required an exchange transfusion, while the third was
stillborn. Which of the following is the most likely cause?
a ABO incompatibility
b immune deficiency disease
c congenital spherocytic anemia
d Rh incompatibility

.......,
162. A specimen of cord blood is submitted to the transfusion service for routine testing. The
following results are obtained:
antl-B : anti-0: Rh-control: direct antlglobulin test:
nega1ive 3• negative 2+

It is known that the father Is group B, with the genotype of cde!cde. Of the following 4
antibodies. which 1 is the most likely cause of the positive direct antiglobulin test?
a anti-A
b anti-0
c anti-c
d anti-C
163. ABO-hemolytic disease of the newborn:
a usually requires an exchange transfusion
b most often occurs in first born children
c frequently results in stillbirth
d Is usually seen only in the newborn of group 0 mothers
164. Which of the following antigens is most likely to be involved in hemolytic disease of the
fetus and newborn?
a Le•
b P1
c M
d Kell
165. ABO hemolytic disease of the fetus and newborn (HDFN) differs from Rh HDFN in that
a Rh HDFN is clinically more severe than ABO HDFN
b the direct antiglobulin test Is weaker In Rh HDFN than ABO
c Rh HDFN occurs In the first pregnancy
d the mother's antibody screen is positive in ABO HON

02018ASCP ISBN 978-069189-6609 C//nlcal Laboratory Certification Examinations 23


1: Blo od Ban k Pll yslo/ogy & Pott
IOplJ.,s·
, 'ot~
166. The following results were obtained:

""" antt-B ontt-D Wook D DAT
Ab screen
Infant 0 0 0 NT 4•
Ni
4• 0 0 0 NT
antJ.o
mother

NT • not tesled
Which of the following is the most probable explanation for these results?
a ABO hemolytic disease of the fetus and newbor~ .
b Rh hemolytic disease of the fetJs and newborn; infant has received intrauterine
transfusions
c Rh hemolytic disease of the felJs and newborn, infant has a false-negative Rh 1 .
d large fetomaternal hemorrhage YPing
167. A group A, Rh-positive infant of a group 0 , R~:po~ilive mother has a. weakly PQsiti11e
antiglobulin test and a moderately elevated b1hrub1n 12 hours after birth. The mosttikdireci
~~is: ~
a ABO incompatibility
b Rh incompatibility
c blood group incompatibility due to an antibody to a low frequency antigen
d neonatal jaundice not associated with blood group
168. In suspected cases of hemolytic disease of the fetus and newborn, what significant
4\S infonmation can be obtained from the baby's blood smear?
ONLY

a estimation of WBC , RBC. and platelet counts


b marked increase in immature reutrophils (shift to the left)
c differenti al lo estimate the absolute number of lymphocytes present
d determination of the presence of spherocyles
169. In treating hyperbilirubinemia in hemolytic disease of the fetus and newborn (HOFN), lig~
phototherapy is used to
a breakdown conjugated into unconjugaled billrubin
b prevent further hemolysis of fetal red cells
c breakdown unconjugaled bilirubln to a nontoxic isomer
d stimulate production of fetal liver enzymes to breakdown unconjugaled bilirubin
170. Fetal neonatal alloimmune thrombocytopenia (FNAIT) is caused by which of the follo"ing
maternal antibodies?
a lgM alloantibodies against ABO antigens
b lgG alloantibo&.es against HLA antigens
c lgM alloantibodies against glyc:iprotein V (GPV) antigens
d lgG alloantibodies against HPA antigens
171 . The purpose of performing antibody titers on serum from an Immunized, pregnant woman
MLS
OllY is to:
a determine the identity of the antibody
b identify candidates requiring additional fetal monitoring
c decide if the baby needs an intrauterine transfusion
d determine if early induction of labor is indicated
172. Which unit should be selected for exchange transfusion if the newborn is group A.
Rh-positive and the mother is group A, Rh-positive with anti-c?
a A, CDe/COe
b A, cOE/cOE
c o. cde/cde
d A, cde/cde

24 The Board of ConificaUon Study Gulde 6o ISBN 978.089189-661>9 ..J


C}.ltl.st'
1: Blood Bank

.......
P/Jyslology & Patl1op/1yslology
173. A mother is group A, with anti-D in her serum. Whal would be the preferred blood product
if an intrauterine transfusion is indicated?
a 0 , Rh-negative Red Blood Cells. Irradiated, CMV sale
b O. Rh-negative Red Blood Cells. Irradiated, CMV sale. HgbS-negative
I
c A. Rh-negative Red Blood Cells, Irradiated. CMV safe
d A. Rh-negative Red Blood Cells, Irradiated, CMV safe, HgbS-negative
174. Laboralory studies of maternal and cord blood yield the following results:
t.l$
...y Maternal blood Cord blood
0. Rh-negative 8, Rh·posltlve
antl ·E in serum OAT=2•
antl·E 1n etua1e

11 exchange transfusion is necessary. the best choice of blood is:


a B. Rh-negative, E+
b B. Rh-positive, E+
c 0 , Rh-negative, E-
d 0, Rh-positive, E-
175. A blood specimen from a pregnant woman is found to be group B. Rh-negative and the
serum contains anli·D with a liter or 512 . vVhat would be the most appropriate type or
blood to have available for a possible exch<nge transfusion for her infant?
a O. Rh-negative
b O. Rh-positive
c B. Rh-negative
d B, Rh-positive
176. Blood selected for exchange transfusion must:
a lack red blood cell antigens correspondirg lo maternal antibodies
b be <3 days old
c be the same Rh type as the baby
d be ABO compatible with the lather
177. When the main objective of an exchange transfusion Is to remove the infant's antibody·
sensitized red blood cells and to control hyperbilirubinem1a, the blood product of choice is
ABO compatible:
a Fresh Whole Blood
b Red Blood Cells (RBC) washed
c RBC suspended in Fresh Frozen Plasma
d heparinized Red Blood Cells
178. To prevent graft-vs-host disease, Red Blood Cells prepared for infants who have received
intrauterine transfusions should be:
a saline-washed
b irradiated
c frozen and deglycerolized
d group- and Rh-compatible with the moth~r
179. Which or the following is the preferred specimen for the initial compatibility testing In
l:tv exchange transfusion therapy?
a maternal serum
b eluate prepared from infant's red blood cells
c paternal serum
d infant's postexchange serum

C2018 ASCP 1561<1978-089189·6609 C//n/cal Laboratory Certification EKOm/natlons 25


1: Blood Bank Physiology & pilll)Op/) ~
0
180. The rollowing results are seen on a maternal postpartum sample: Ysio1 9)
o o control Wook D Woak 0
mother's postpartum somple: 0 0 1•"'' o <•ntroi

mf =mixed field
The most appropriate course of action is to:
a report the mother as Rh-negative
b report the mother as Rh-positive
c perform an elution on mother's RBCs
d investigate ror a letomaternat hemorrhage
In cases where antepartum RhlG has been adm!nistered and anti-D is detecte .
181. mother's sertJm at delivery, what laboratory studies may help determine ii the d in the
from the antepartum RhlG, or due to maternal alloimmunization? anti-0 is
a determine antibody titer of maternal anti-0
b repeat antibody identification studies with room temperature incubation
c perform inhibition studies on anti-0 with maternal saliva
d repeat antibody identification studies using AET treated serum
182. Refer to the following information:
Rosette fetal
screen using
untreated O+
Rh control WoakO Weak 0 control lndicat0< tells
Postpartum anll·D
0 0 0 8 rosotte$15 f'"1ds
molher 0
0 NT NT NT
ne\Vbom 4•

NT = not tested
What is the best interpretation for the laboratory data given above?
a newborn needs weak 0 testing
b mother needs further weak O testing
c mother has a larger than normal FMH
d mother has a negative rosette test

:~~,~~~:d01~ ~1 ~~~~e~:i~t~~~~ai~~d~cate
183.
0 1
abfe tomaternal hemorrhage of 35 ml of
u e go u11n would be required?
a 1
b 2
c 3
d 4
184. A
in fetomatemal hemorrhage
an Rh-negative of 35 mL of.fetal Rh-positive packed RBCs has been detected
woman How
a · many vials of Rh immune globulin should be given?
0
b 1
c 2
d 3
185. Criteria
a thdetermining
. Rh 1mmune globulin eligibility include·
mo er 1s Rh-positive ·
b .infant .1s Rh-negative
c mother has not been previo I . .
d infant has a positive direct a~~igylo1mbml~mzed to the 0 antigen
u in test

26 Tho B oartl of Certification Study Gulde 6e


ISBN976-089189·6609 e2QtS.>-<l°
1: Blood B ank Plryslology & Patl1opl1ysiology
l~6· While performi.ng rou~ine postpartum tesling for a-;; Rh immune globulin (RhlG) candidalo .
..,. a weakly pos1t1ve antibody screening test was found. Anti-0 was identified. This antibody is
most likely the result of:
a massive letoma~ernal hemorrhage occurring at the time or this delivery
b antenatal admm1strahon of Rh Immune globulin at 28 weeks· geslation
c contamination of the blood sample w~h Wharton jelly
d mother having a positive direcl antiglobLlin test
187. Rh immune globulin adminislrallon would not be Indicated in an Rh-negative woman who
has a(n):
a first trimester abortion
b husband who rs Rh-positive
c anti-0 titer of 1:4,096
d positive direct antiglobulin tesl
188. A Kleihauer-Betke stain of a postpartum blood film revealed 0.3% fetal cells. What is the
estimated volume (ml ) of the fetomaternal hemorrhage expressed as whole blood?
a 5
b 15
c 25
d 35
189. Based upon Kleihauer-Betke test results, which of the following formulas is used to
determine the volume of fetomaternal hemorrhage expressed in ml of whole blood?
a % of fetal cells present x 30
b % of fetal cells present x 50
c % of maternal oells present " 30
d % of maternal cells present x 50
190. An acid elution stain was made using a 1-hour post-delivery maternal blood sample. Out
of 2,000 cells that were counted, 30 of them appeared to contain fetal hemoglobin. It
is the policy of the medical center to add 1 vial of Rh immune globulin to the calculated
dose when the estimated volume of the hemorrhage exceeds 20 ml. of whole blood.
Calculate lhe number of vials of Rh immune globulin that would be indicated under these
circumslances.
a 2
b 3
c 4
d 5
191. The rosette test will detect a retomaternal hemorrhage (FMH) as small as:
a 10 ml.
b 15 ml
c 20ml
d 30 ml
192. Indications for an autologous hematopoieti: progenitor cell (HPC) transplant include
patients who have
a Hodgkin lymphoma with high-dose chemotherapy
b congenital hemoglobinopathies
c congenital immunodeficiency disorders
d inborn errors of metabolism

Clinical Laboretory Certification Examinarlons 27


02018ASCP ISBN 978-0891~509
r1 1y~1v1u~y o. ~ilf/ r
0 11 ,.
1 · Blood Bank . . P Ys1010
· t •• correct about tiematopo1et1c progenitor tell ( !ly
193. Which of the following statemen s ,. HPc)
transplantation? . tation requires that at least 5 out o f 10HLA 10 •
a successful allogene1c tr~ns~la~ de bone marrow. umbilical cord blood (UCS)CI rna1c~
b sources for HPC collecllon inc u • alld
mobilized peripheral blofoHd PC is preferred to apheresis collection because of f
c bone marrow collectton o ewer
side effects . d . to the patient over a period of at least 6 hours
d HPC products are infuse in . .
. ne hemolytic anem ia due to an t1-E has a hem
194. A 40-year-old man with autoT rmh~u rent will most likely be treated with : oglobin
level of 10.8 g/dl (108 g/L). is pa 1
a Whole Blood
b Red Blood Cells
c Fresh Frozen Plasma
d no transfusion
195. A patient in the immediate post bon? mar.row transplant period has a hematocrit of 21%
The red cell product of choice for this patien t w ould be. ·

a packed
b saline washed
c microaggregate filtered
d irradiated
196. Which of the following systems plays an important role in .Transfusion-Related Acute L
Injury (TRALI), transfusion-associated ~raft versus host disease (TA-GVHD), platelet Ullg
refractoriness, and Febrile Nonhemoly11c Transfu.sron Reactions (FNHTR) as well as in
hematopoietic stem and organ transplanlat1on reiecllon?
a Rh
b HLA
c Lewis
d Diego
197. Which of the following statements is true about Class II HLA antigens?
a they are found on the surface of most nucleated cells.
b Bg antigens are part of HLA Class II
c HLA-DR. HLA-DQ and HLA-DP are all Class 11
d they are only located on neurons and platelets
198. The most widely accepted QC test to measure probable Hematopoietic Progenitor Cell
(HPC) engraftment is
a clonogenic assay
b cell viability
c CD34+ cell enumeration
d manual differential
199. Pathologic oold autoantibodies differ from benign cold autoantibodies in
a antibody specificity
b immunoglobulin class
c antibody titer
d ability to bind complement
200. The DAT in a patient with WAIHA is most often positive for
a lgG only
b C3 only
c lgM only
d fgG and C3

28 Tho Boord of Certification study Guido 61


1: Blood Bani< Seroloyic & Moloculor Testing
201. Whal is the most important consideration ror a patient requiring red cell transfusion due to
severe anemia when their serum contains a warm autoantibody?
a determine specificity of autoanlibody
l
b determine the immunoglobulin class of the autoantibody
c determine the presence of underlying alloantibody(ies)
d avoid transfusion therapy
202. A drug-induced immune hemolytic anemia caused by a drug-independent antibody would
have which of the following results?
a positive DAT with lgG; positive antibody screen
b positive DAT with lgG; negative eluate
c positive DAT with C3d; negative antibody screen
d negative DAT; positive antibody screen
203. The autoantibody most often implicated in Paroxysmal Cold Hemoglobinuria (PCH) is
a cold-reactive, lgG, anti-P
b cold-reactive, lgM, anti-P
c cold-reactive, lgG, anti-I
d cold-reactive. lgM, anti-I

Serologic & Molecular Testing


204. Anti-E is identified in a panel at the antiglobulin phase. When check cells are added to the
tubes, no agglutination is seen. The most appropriate course of action would be to:
a quality control the AHG reagent and check cells and repeat the panel
b open a new vial of check cells for subsequent testing that day
c open a new vial of AHG for subsequent testing that day
d record the check cell reactions and report the antibody panel result
205. A serological calibration was completed for a new centrifuge received in the blood bank.
I.ts
°'" Time In seconds 1 5 20 25 30
is button delineated? yes yes yes yes
is supernatant dear? no yes yes yes
button easy to resuspend? yes yes yes no
strength of reaction? +m 1+ 1• 1+

Given the data above, the centrifuge time for saline tests for this machine should be:
a 15 seconds
b 20 seconds
c 25 seconds
d 30 seconds
206. The following blood typing results were noted on a patient's sample:
MLS
O.O.v Anti-A Antl-B Antl·D A 1 Cells B Cells
0 4+ 4• 4+ 0
Rh Typing Results: C-E-c+e+

What is the patient's likely ethnicity?


a Asian
b Hispanic
c Black
d White

C2018ASCP ISBN 978·0891811-6609 Cl/nlcal Laboratory Certification Examinations 29


Sero/ogle & Molecular Testing
1: Blood Bank 1
207. Which of the following represents an acceptably Identified patient for sample collection 2
and transfusion?
a hand.wrillen band with patient's name and hospital identification number is affixed lo the
patient's leg
b the addressographed hospital band is taped to the patient's bed
c an unbanded patient responds positively when his name is called
d the chart transported with the patient contains his armband not yet attached
208. Samples from the same patient were received on 2 consecutive days. Test results are
summarized below:
Day # 1 O•y#2
anti·A 4+ 0
anli-B 0 4•
anti-0 3+ 3•
0 4+
A1 cells
0
a cells 4+
0
Ab screen 0
How should the request for crossmatch be handled?
a crossmatch A, Rh-positive units with sample from day 1
b crossmatch B. Rh-positive units with sample from day 2
c crossmatch AB. Rh-positive units with both samples
d collect a new sample and repeat the tests
209. The following test results are noted for a unit of blood labeled group A, Rh-negative:
Cells tested wlt.h:
anti·A anb·B anli-D
4+ 0 3•

What should be done next?


a transfuse as a group A. Rh-negative
b transfuse as a group A, Rh-positive
c notify the collecting facility
d discard the unit
210. In what patient population may we observe the following results?
MLS
Clf<.V Anti-A Anli·B Anll· D A, cells a cells
4+ 0 4+ 0 1+

a labor and delivery patient


b 30..year old GI bleed patient
c 2-year old pre-surgical patient
d 16-year old ACL repair surgery patient
·r . .
you have to screen to fi~db • u ·1~~ 51 ~ve with anti-A, i.n their serum . Mow many units
1
A patient Is a subgroup or A (A ) Rh
211. would
a 1 "' a is oompat1ble with the patient 's anti-A,?
5
b 10
c 15
d 20
212. A patient is group A, Rh-positive but is re . .
(BMT) on Friday. After the trans lant ceiving a Gr~up ~· bone marrow transplant
demonstrate when the patient's ~ed , wllhat hemagglubnal 1on pattern will the patient
ce s are tested with anti-A?
a rouleaux
b aggregation
c polyagglutination
d mixed-field

30 Tho Board of CertiOcalion Study Gulde Se


ISBN 978-089189-6609 C2Q1$A.~
1: Blood Bank Sero/ogle & Molec11/or Testing
213. A group B. Rh-negative patient has a positive DAT. Which of the following situations would
occur?
a all major crossmatches would be incompatible
b the weak D lest and control would be poSJtive
c the antibody screening test would be posilive
d the forward and reverse ABO groupings would not ag ree
214. The following reactions were obtained:
Cells tested with : Sorum tested with:
onll·A anti-B anti-A,B A 1 cells B cells
4+ 4+ 4•

The technologist washed the patient's cells with saline , and repeated the forward typing. A
saline replacement technique was used with the reverse typing. The following results were
obtained:
Cells lasted w ith: Sorum tc1t1d with:
anll·A anti·B anti-A,B A 1 cells B cells
4• 0 4+ 0 4+

Based on these results, a likely diagnosis for the patient may be:
a acquired immunodeficiency disease
b Bruton agammaglobulinemia
c multiple myeloma
d acquired · B· antigen
215. What ABO type is found in group A 1 individuals following deacetylation of their A antigens?
....
""'• a Acquired B
b B(A)
c Amod
d Aint
216. In a Group O individual with Le and Se genes, what ABH and Lewis antigens are present
in their secretions?
a Le". Leb
b Le8 , Leb, H
c Le 8 , H
d Leb, H
211. The Rh-negative phenotype results from the complete deletion of what gene(s)?

a RHO and RHce


b RHCE
c RHO
d RHO and RHCE
218. Mixed field agglutination encountered in ABO grouping with no history of transfusion would
most likely be due to:
a Bo mbay phenotype (Ohl
b T activation
c A 3 red cells
d positive indirect antiglobulln test

cllnlc•l Labon1tory Certlflcallon E••mlnatlons 31


02018ASCP ISBN978'089189-6609
. & M o l ecular Tes ting
Sero/og1c
..
1 : Bl o o d Bank 0
autocontrol is

2•
screening cell 1 1+ 2+
screening cell II 1+ 2+
autocontrol 1+ ?
f this discrepancy ·
Whal is the most likely cause o
a A 2 with anti-A t
b cold alloantibody
c cold autoantibody
d acquired-A phenomenon r nt's ABO seru m typing?
What method may be used to resolve the pa ie
....
221.
OOl.Y
Andtl-
Room
Imm edi ate t em pe rature L ISS LISS -
Anti- Anti · Anti - A1 B bo y (RT) 31•c AHG
A o cells cells screen spin
8
Screen 1+
3+ 1+
cell 1 2+
4+ 2+ 4+
4+ 0
Screen 1+
2+ 3+ 1+
cell 2
Auto
3+ 2+
control

LISS ~ IOlh' ionic strength solution


a OTT-treat patient's red cells
b warm saline wash p atient's red cells
c enzyme treat patient's red cell s
d cold -autoadsorption
222. The following results were obtained on a patient's blood sample during routi ne AB O and
Rh testing:
Cell testing: Seru m testing:
a nti-A: 0 A, cells: 4+
anti-B: 4+ B cells: 2+
anli*D : 0
autocontrol: 0
Select the course of action to resolve th is problem:
a enzyme treat the patient's red cells and repeat the forward blood typing
b test the patient's serum with A 2 cells and the patient's red cells w ith anti-A 1 lectin
c repeat the ABO antigen grouping using 3>< wa shed sali ne-suspended cells
d perform antibody screening procedure at immediate spin and A HG using group O cells

32 The Board of Certification Study Guid e 6 e ISBN 978.0891 ~ 02018 ASCP


1 : Blood Bank Serologic & M o/ocular Testing
223. The i;>atient's results are listed below. What is the most likely specificity based on the
....
"'' reachon pattern?
antibody screen reactive \vilh all caIts 3• using solid phase
d irect anligk>buin te$l anti-lgG posi:ivo, anll·C3d negative
antibody identification studies rcaclive \Vilh all cells 3-+ using sohd phase
eluate testing reactive \Vilh all cells 3+ us1ng solid ptiase
a high incidence antibody
b autoantibody
c multiple alloantibodies
d cold reactive antibody
224. The test for weak D is performed by incubating patient's red cells with:
a different dilutions of anti-D
b anti-D antiserum
c anti-Du antiserum
d antiglobulin antiserum

..
,225. Using th e antigen typing results below, what is the pa tient's likely phenotype?
ONtY
+
0

a R1R1
C
0
E
+
c
+ +.
o I G
+

b R2R2
c R2r
d R1R2
226. The following results were obtained when testing a sample from a 20-year-old, first-time
blood donor:
Forward group; Reverse group:
anti-A anli· B A1cells B cells
0 0 0 3+

What is the most likely cause of this ABO discrepancy?


a loss o f antigen due to disease
b acquired B
c phenotype Oh "Bombay•
d weak subgroup of A
227. A mother is Rh-negative and the father Rh-positive. Their baby is Rh-negative. It may be
concluded that:
a the father is homozygous for D
b the mother is heterozygous for D
c the father is heterozygous for D
d at least 1 of the 3 Rh typings must be incorrect
228. Some blood group antibodies characteristically hemolyze appropriate antigen-positive red
cells in the presence of:
a complement
b anticoagulants
c preservatives
d penicillin

Clln/c1/ Laboratory Certfficalion Enmlnatlons 33


C2018ASCP ISBN 978-08918~
1 : Blood Bank
229. Review the following schematic diagram:
PATIENT SERUM +REAGENT GROUP ·o· CELLS
Scrologic & M olcc11/or Tasting
-
INCUBATE - READ FOR AGGLUTINATION
WASH - AOO AHG - AGGLUTINATION OBSERVED

The ne><t step would be to:


a add "check cells" as a confirmatory measure
b identify the cause of the agglutination
c perform an elution technique
d perform a direct antiglobulin test
· d ·in pretransfusion testing:
The following results were obtaine
230.
IAT
screening cell I 3+
0 3+
screening coll II
autocon1rol 0
The most probable cause of these results is:
a rouleau><
b a warm autoantibody
c a cold autoantibody
d multiple alloantibodies . f bl d
Tve
1 and crossmatched with 6 units o oo .
231. A patient is typed as group 0, Rh·posi tin both antibody screening cells and 2
Al the indirect antiglobul~n (IAT) phase~~ ~~ t~~ most likely cause of the incompatibility?
1
crossmatched units are incompatible. a is
a recipient alloantlbody
b recipient autoantibody
I c donors have positive DATs
d rouleaux
232. Refer lo the following data:
hemoglobin : 7.4 g/dL (74 g/Ll
re1iculocyte count: 22%

direct antlglobulln test Ab Screen - IAT


potyspecific 3+ SC I: 3+
tgG. 3+ SC 11: 3+
C3: 0 auto: 3+

Which clinical condition is consistent with the lab results shown above?
a cold hemagglutinin disease
b warm autoimmune hemolytic anemia
c penicillin-induced hemolytic anemia
d delayed hemolytic transfusion reaction
233. A patient received 2 units of Red Blood Cells and had a delayed transfusion reaction.
Pretransfusion antibody screening records indicate no agglutination except after the
addition or lgG-sensitized cells. Repeat testing of the pretransfusion specimen detected
an antibody at the antiglobulin phase. What is the most likely explanation for the original
results?
a red cells were overwashed
b centrifugation time was prolonged
c patient's serum was omitted from the original testing
d antiglobulin reagent was neutralized

34 The Board of Cortlrlcallon Study Guide 6e ISBN 978·089t89·6609 C20 18 Af


1: Blood Bank

4+
•• 4•
Serologic & Mo/ocular Testing
234. Based on the results below, what is the next step in determination of the patient's ABO/Rh
type?
Antl·A Anll·B A1 cotls
0
B Clll$
0
I
a interpret at AB, O-pos1tive
b interpret at AsubB. D-positive
c repeat ABO red cell typing and include a control
d add room temperature Incubation with ABO red cell typing
235. The major crossmatch will detect a(n):
a group A patient mistyped as group o
b unexpected red cell antibody in the donor unit
c Rh:n~gative.donor unit mislabeled as Rh-positive
d recipient antibody directed against antigans on the donor red cells
236. Based on these reactions, what is the patient's ABO type?
Anti·A Antl ·B U/ex f!uropaeu s Dollchos blflorus
4• 0 0 4•
a A1
b A2
c Amoc1
.d A,
237. Which of the following would most likely be responsible for an incompatible antiglobulin
crossmatch?
a recipient's red cells possess a low frequency antigen
b anli-K antibody in donor serum
c recipient's red cells are polyagglutinable
d donor red cells have a positive direct anliglobulin test
238. A patient has a variable reacti ng anti-P1 patte<'n in antibody identification studies. What
~:, test can be used to verify the specificity or anti-P 1?
a P 1 neutralization
b PEG-AHG
c cloroquine-AHG
d DTT-AHG
239. In the process or identifying an antibody, the technologist observes 2+ reactions with 3 or
10 cells at immediate spin (IS) and room temperature (RT). There were no reactions at
37•c or AHG. Whal is the most likely antibody?
a an!i-Jkb
b anti-Le•
c an ti·C
d anti-Fy•
240. During an emergency situation with no time to determine ABO group and R~ tyP? on a
current sample for transfusion, it is noted that previous reC01:ds state the patient 1s known
to be A, Rh-negative. The technologist should:
a refuse to release any blood until the patient's sample has been typed
b release A Rh-negative Red Blood Cells
c release O Rh-negative Red Blood Cells
d release O Rh-positive Red Blood Cells

02018 ASCP ISBN 978.08918!H609


Cl/nlc1l l•bor•tory Certification Exomlnlt/ons 35
1: Blood Bank
241. A 29 -year-old male is he h .
Serologi c & Molccul nr Testing
-
requi red STAT Of th f mo~r aging severely. He is AB . Rh-negative. Six u nits or blOO<I are
preferable for· 1
e O lowing types available in the blood bank. which would be most
crossmatch?
a AB , Rh-positive
b A, Rh-negative
c A, Rh-positive
d 0, Rh-negative
242. A Pc:'ti~nt is group A B, Rh-positive and has an a ntiglobulin- reacting a nli-A1 in his serum.
2
He is in the operating room bleed ing profLsely and group A 2 B Red Blood <?ells are not
available. Which of the following blood types is first choice for crossmatching?
a B, Rh-positive
b B, Rh-negative
c A 18 , Rh-positive
d 0 , Rh-negative
243. A 1 0°/o red cell su spension in s aline is used in a compatibility test. Which of the following
would most likely occur?
a false-positive resu lt due to antigen excess
b false-positive result due to the prozone phenomenon
c false-neg ative result due to the prozone phenomenon
d false-negative result due to a ntigen excess
244. A patient seru m reacts with 2 of the 3 antibody screening cells a t the AHG phase. Eight
.,,.,.
of the 10 units crossmatched were in com :>atib le at the AHG phase. All reactions are
"" markedly enhanced by enzymes. These resltl ts are most consistent with:
a anti-M
b anti-E
c anti-c
d anti-FY"
245. A patient received 4 units of blood 2 years previously and now has multiple antibodies. He
has not been transfused since that time. It would be most helpful to:
a phenotype h is cells to determine which additional alloa ntibodies may be produced
b recommend the use of directed donors, which are m ore likely to be compatible
c use proteolytic enzymes to destroy the "in vitro" ac tivity of some o f the antibodies
d freeze the patient's serum to use for a1tigen typing of compatible units
246. Autoantibodies demonstrating blood groLp specificity in warm autoimmune hemolytic
anem ia are associated more often with which b lood group system?
a Rh
b I
c p
d Duffy
247. An antibody that causes in vitro hemolysis and reacts with the red cells of 3 out of 1O
AHG-crossmatched donor units is most likely:
a anti-Lea
b anti-s
c anti -k
d anti-E

36 The Board of Cortificatlon Study Guido 6e ISBN 978-089189-6e09 02018 ASC


1: Blood Bank serologic & Mo/ocular Tasting
248. A patienl's serum reacted weakly positive (1+w) with 16 or 16 group 0 panel cells al
:;c,:,
the AHG lest phase. The autooonlrol was negative. Tests with ficin-treated panel cells
demonstrated no reactivity at the AHG phase. Which antibody is most likely responsible
for these results?
a anti-Ch
b anti-k
c anli·e
d anti.Jsb
249. An antibody identification study is performed with the 5-cell panel shown below:

Antigens
1 2 3 4 5 Tosi results
+ 0 0 + + +
"D

.. II>
:::J II 0 0 + 0 + 0

....
n Ill
tV
0
0
+


+
+
+
0
0
+
0
+
v + + + 0 0 +

auto 0

An antibody against which of the following antigens could not be excluded?


a 1
b 2
c 3
d 4
250. A 25-year-old Caucasian woman , gravida 3, para 2, required 2 units of Red Blood Cells.
Ul $
OMV The antibody screen was positive and the results o f the antibody panel are shown below :

Cell D c c E e K J k• Jkb Le• Leb M N P, AHG


1 + + 0 0 + + + + o· + + + + 0
2 + + 0 0 + 0 + 0 0 + + 0 0 0
3 • 0 + + 0 0 • 0 • + + + 1+
4 • + + 0 • 0 0 0 • + 0 • I+

5 0 0 + 0 • 0 • 0 + + 0 0 1•
6 0 0 • + • 0 • 0 • 0 • + 0 1+

7 0 0 + 0 • + • • + 0 + • • 1+

8 0 0 + 0 + 0 0 + 0 + 0 + • 1•
auto 0

Which of the following antibodies may be the cause of the positive antibody screen?
a anti-M and antl·K
b anti-c and anti-E
c anti-Jk• and anti-c
d anti-P 1 and anti-c

C'<Ot8 ASCP ISBN978.089189-6600 Cllnlcal Labonotory Certification Examln1tlona 37


1 : Bloo d Bank scrologic & Molecular Testing
,
251 . T he sample from a 45-year-old Asian
· woman, grav1da · 2 , p;:ira 2 demonstrated
. a positive
'4l
antibody screen. The results of the antibody panel are shown below.
""''
Leb M N P1 AHG
Cell 0 c c E e K Jk• Jkb Le"
+ + + + 0
+ 0
1
2
+
+
+
+
0
0
0
0
+
+
+

0
+
+ 0 0 + .. 0 0 0
1+
+ + +
3 + 0 + + 0 0 + • 0 +
+ 1+
+ 0
4 • + + 0 + 0 0 • 0 +
1+
+ 0 0
5 0 0 + 0 + 0 + • 0 +
+ + 0 1+
+ + 0 + 0 + 0
6 0 0 +

.. .. + + • .. 0 + + + 1+
7
B
0

0
0

0 +
0

0 • 0 0 • 0 + 0 + .. 1+
auto 0

Which common antibody has not been ruled out by the panel?
a anti-C
b anli-Leb
c anll-Jk8
d anti-E
252. A 5 .year-o ld with c hronic upper respiratory infection.s arr~ves in the Emergency Room with


. .. chronic anemia. The antibody screen and an~ibody 1denlifi~atoon p~nel are all stro~gly,
reactive when tested by solid phase alltomat1on. The OAT 1s negative, and the patients
phenotype is shown below. With these initial findings, what 1s suspected?
c E c e K k Fya Fyb Jka Jkb
4• 4+ 4+ 4+ 0 1+ 0 0 4+ 4+

a PCH
b Mcleod syndrome
c WAIHA
d CHO
253. A male patient's sample demonstrates a pattern most consistent with anti-0. The patient
is Rh-negative, and was transfused with Rh-positive blood emergently after a motor
vehicle accident 2 ye ars previously. The anti-0 shows variable reactivity when tested with
0-positive cells. What test would be appropriate to enhance the anti-0 reactivity and verify
specificity?
a ficin- AHG
b DTT-AHG
c trypsin-AHG
d albumin-AHG
254. In the process of identifying an a ntibody, the technologist observed 2+ reactions with 3
?f the 1.0 cells in0 a panel after the i~mediate spin phase. There was no reactivity after
incubation at 37 C and after the anti-human globulin test phase. The antibody most likely
is :
a anti-P 1
b anti-Lea
c anti-C
d anti-Fy8

38 The Board of Certification Study Gulde 6e ISBN 978--089189-6609 Cl2018ASCI


1: Blood Bank Sero/ogle & Moloc11/ar Testing
255. Tra nsfusion of Ch+ (Chido positive) red cells to a patient with anti-Ch has been reported to
cause:
a no clinically significant red cell destruction
b clinically si~nificant immune red cell destruction
c decreased 1Cr red cell survivals
d febrile transfusion reactions
256. R~sults of a serum sample tested against a panel of reagent red cells gives presumptive
LU
~v1de~ce ~ran alloantibody directed agairst a high incidence antigen. Further
"'" mvest1gat1on to confirm the specificity should include which of the following?
a serum testi ng against red cells from random donors
b serum testing against red cells known to lack high incidence antigen s
c se rum testing against enzyme treated aulologous red cells
d testing or an eluate prepared from the patient's red cells
257. Which of the following genes on chromosome 1 encodes for the 4 common antigen
combinations ce, cE, Ce and CE.
a RHD
b RHCE
c RHDandRHCE
d RHd and RHce
258. Refer to the following panel:
LU
ONLY
LISS
Cell 0 c c E e K Jk• Jk• le• Le• M N P, IS 37'C AHG
1 + + 0 0 + + + + 0 + + + + 0 0 2+
2 + + 0 0 + 0 + 0 0 + + 0 0 0 0 3+
3 + 0 + + 0 0 + + 0 + + + + 1+ 1+ 3+
4 + + + 0 + 0 0 + 0 + + 0 + 0 0 0
5 0 0 + 0 + 0 + + 0 + + 0 0 0 0 2•
6 0 0 + + + 0 + 0 0 + + 0 1+ 1+ 3+
7 0 0 + 0 + + + + • 0 + + + 0 0 2+
8 0 0 + 0 + 0 0 + 0 • 0 + + 0 0 0
auto 0 0 0

Based on the results of Ule above panel, the most likely antibodies are:
a anti-M and anti-K
b anti-E, anti-Jk8 and anti-K
c antl-Jk8 and anti-M
d anti-E and anti-Leb
259. Which characteristics are true of all 3 of the following antibodies: anti-Fy• , anti-Jk8 , and
anti-K?
a detected at IAT phase and may cause hemolytic disease of the fetus and newborn
(H OFN) and transfusion reactions
b not detected with enzyme trealed cells: may cause delayed transfusion reactions
c requires the IAT technique for detection: usually not responsible for causing HDFN
d may show dosage effect; may cause s9vere hemolytic transfusion reactions

02018ASCP ISBN 978-089189-6609 Cllnlcal Laboratory Certification Examinations 39


.....
1: Blood Bank Serologic & M o /ocula r Tes ting

~~o. Rorer lo lhe following cell panel:


C!U
En:z:ymos

Cell 0 c c E e K Jk• Jkb Le• Lo" M N P, AHG AHG

1 + + 0 0 + + + + 0 + + + • 3• 4+

0 3• 4+
2 + + 0 0 + 0 + 0 0 • + 0
0 0
3 + 0 + + 0 0 + + 0 • + +

4 ... + + 0 + 0 0 + 0 ... + 0 + 2• 3+

5 0 0 + 0 + 0 + + 0 ... • 0 a 0 0

6 0 0 + • + 0 + a + 0 + + 0 0 0

7 0 0 + 0 + • + + 0 + + ... 0 0

8 00 + 0•00 0 + 0 + • 0 0
auto 0 O

Based on lhese results. which or the following antibodies is most likely present?
a anti-C
b an li-E
c antl-D
d anti-K
261. A pregnant woman has a positive antibody screen and lhe panel results are given below:
IAS
""y LISS Enzyme
Cell o C c E e Fy• Fy'> Lo• Le• M N P, IS 31•c AHG AHG
1 • +00 + + + + 0 + 0 + + ... + 0 0 0 0
2 •+ QO +O+O ... 0 0 .. + 0 0 1+ 1+ 2+ 0
3 +0+ + 00 + + + ... 0 + + + + 0 0 1+ 0
4 + ++0+00+ 0 + 0 + • 0 + 0 0 0 0
5 0 0 + 0 + 0 + + + + 0 + + 0 0 0 0 1+ 0
6 0 0 + + + 0 + 0 0 0 + 0 + ... 0 0 0 0 0
7 00+0 + + + + 0 + + 0 + + +
8 o a • o + o 0 + .. 0 0 + 0 + +
0
1+
0
1+
0
2+
0
0
auto O O 0 0

Wha t is the association or the antibod (i ) . . .


newborn {H DFN)? Y es w ith hemolytic disease of the fe tus and
a usually fatal HDFN
b may cause HDFN
c is not associated with HDFN
d HDFN cannot be determined
262. Wh.ich o f the following tests ls most com
patient's red blood cells in vivo? manly used to detect antibodi es attach ed to a
a direct antiglobulin
b complem ent fixation
c indirect antiglobulln
d immunofluorescence

40 Tho Bo~rd Of Certification Study Gulde 6e


1: Blood Ban k Serologic & Molecular Testing
263. Anti-I in cold agglutinin disease may cause a positive direct antiglobulin tesl (DAT)
because or:
a anti-I agglutinating the cells
b C3d bound to the red cells
c T-activation
d C3c remaining on the red cells after cleavage of C3b
264. Which direct antiglobulin test results are associated with an anamnestic antibody response
;;;,~,
in a recently transfused patient?
Test result
result A
result 8
....
Polyspeeific

1+
lgG
+ml
CJ
0 0
Control

0 1+ 0
result C 2+ 2+ 0 0
cesult D 4+ 4+ 4+ 0
mr=n1ixed field
a result A
b result B
c result C
d result D
265. A patient's antibody identification panel demonstrated anti-M. The antibody was most
reactive with homozygous M+ cells compa1ed to heterozygous M+ cells. Which of the
following cells would demonstrate the strongest reaction?
a M- N+S- s+
b M+N+S+s+
c M+N- S-s+
d M+N+S-s-
266. Polyspecific reagents used in the direct antiglobulin test should have specificity for:
a lgGand lgA
b lgG and C3d
c lgM and lgA
d lgM and C3d
267. In the direct antiglobulin test, the antiglobuli n reagent is used to:
a mediate hemolysis of indicator red blood cells by providing complement
b precipitate anti erythrocyte antibodies
0

c measure antibodies in a test serum by fixing complement


d detect preexisting antibodies on erythrocytes
268. AHG (Coombs) control cells:
a can be used as a positive control for anti-C3 reagents
b can be used only for the indirect antiglobulin test
c are coated only with lgG antibody
d must be used to confirm all positive anti~lobulin reactions
269. A 56-year-old female with cold agglutinin disease has a positive direct antiglobulin test
(DAT ). When the DAT is repeated using monospecific antiglobulin sera, which of the
following is most likely to be detected?
a lgM
b lgG
c C3d
d C4a

C2018ASCP ISBN 978-089189-6609 Clinical Laboratory Certification Examinations 41


1 : Blood Bank scrologlc & Mofocul ar Testing

270. The mechanism that best explains hemolytic anemia due to penicillin is:
UlS
ON\.'!'a d rug-dependent antibodies reacting with drug-treated c.e lls
b drug-dependent antibodies reacting in the presence of drug
c drug-independeflt with autoantibody production
d nonimmunologic protein adsorption with positive DAT
. . · 1 complement pathway by:
271 . Use of EDTA plasma prevents activation of the c1ass1ca
a causing rapid decay of complement comi;onents
b chelating Mg .. ions. which prevents the assembly of C6
c chelating c a•· ions, w hich prevents asse mbly of C1
d preventing chemotaxis
272. Which of the following blood bank chemicals produce Kell null cells?
....
....y
a OTT
b ficln
c formaldehyde
d chloroquine diphosphate
· • ... Its in the deletion of both GYPA and GYPB. Which of
Inheritance of the rare tvr· ge~e resu d on red cells in the presence of a WI<
the following blood group antigens is not expresse
gene?
a Kx
b En•
c f
d G
274. The drug cephalosporin can cause a positive direct antiglobuli~ test due to m.odifica_tion
of the RBC membrane by the drug which is independent of antibody production. T his
mechanism related to Ille drug cephaloporin is best d escribed a s:
a drug -dependent
b complement related drug-dependent
c drug-autoanlibody
d nonimmunologic protein adsorption
275. During prenatal studies, a woma n is noted to have a pos itive a ntibody screen and anti-Kp•
is identified. What percentage o f units will be compatible for this patient if t ra ns fusion is
necessary?
a <2%
b 50%
c 8 5%
d >98°/o
276. Crossmatch results at the antiglobulin phase were negative. W hen 1 drop of check cells
was added, no agglutination w as seen. T he most like ly e xp lanation is th at the:
a red cells were overwashed
b centrifuge speed was set too high
c residual pa tient serum inactivated the AHG reagent
d labo ra torian did not add enough check cells
277. Which of the following might cause a false-negative indirect antiglobulln test (IAT )?
a over-reading
b lgG-coated screening cells
c addition of an extra d rop o f serum
d too heavy a cell suspe nsion

i2 The Board o r Certification Study Gulde Ge ISBN 978-089189-6609 02018ASCP


1: Blood Bank Serologic & Mol oc11lar Testing
278. The purpose or testing with anli·A,B is to detect:
a anli-A 1
b anti·A2
c subgroups of A
d subgroups of O
279. A group 0, Rh-negative pregnant female has anti-Val In her serum. II needed, how might
blood be provided for her infant?
a maternal donation
b paternal donation
c random ABO-identical unit
d random group 0, Rh-negative unit
280. Which of the following antigens gives enhanced reactions with Its corresponding antibody
following treatment of the red cells with proteolytic enzymes?
a Fy"
b E
c s
d M
281. In a prenatal workup, the following results were obtained:
Forward Group: Reverse Group:
anti-A anti·B anti- 0 Rh conlrol A1 cell$ B cells
4• 2+ 4• 0 0 3+
DAT: negauve
antibody screen: negative

ABO disCfepancy was thought to be due lo an antibody directed against a component or


the typing sara. Which test would resolve this discrepancy?
a A 1 lectin
b wash patient's RBCs and repeat testing
c anti-A, anti·B and extend incubation of the reverse group
d repeat reverse group using A2 cells
282. Refer lo the following panel:

LISS
Cell D c c E e K Jk• Jkb Fy• Fy'> IS 37"C AHG
1 + + 0 0 + + + + + + 0 0 2+
2 + + 0 0 + 0 + 0 + + 0 0 2+
3 + 0 + + 0 0 0 + + + 0 1+ 3+
4 + + 0 0 + 0 0 + 0 + 0 0 0
5 0 0 + 0 + 0 + + + + 0 0 2•
6 0 0 + + • 0 • 0 + 0 0 1+ 3+
7 0 0 + 0 + + 0 + + 0 0 0 2+
a 0 0 + 0 + 0 0 + 0 + 0 0 0
auto 0 0 0

Based on the results of the above panel, which technique would be most helpful in
determining antibody specificity?
a proteolytic enzyme treatment
b urine neutralization
c autoadsorption
d saliva inhibition

C2018ASCP ISBN 978-08918g.6609 Cllnlcol L•boratory Cert.ificatlon &a"'lnatlons 43


1 : Blood Bank serologic & M o/ocular Tosung l
283· A patient's serum sample was reactive with all cells except the autocontro l when tested by
PEG-AHG. The patient's phenotype was confirmed as C- E<:c:e+; K-k+ , Kp(a- b+).
J s(a- b+); Fy(a-b+); Jk(a-b- )· M+N+S+s+. Phenotypically s1m1lar cell s ~ere tested
and found to be nonreactive. in what poi;ulalion of donors are we most likely to find a
compatible donor for this patient?
a African
b Middle Eastern
c South American
d Tahitian
284. To confirm a serum antibody specificity identified as anti-P,, a neutralization study was
performed and the following results obtained:
P1+ RBCs
so rum + P 1 subs lance: negative
scrum + saline: negative

What conclusion can be made from these results?


a anti-P 1 is coofirmed
b anti-P 1 is ruled out .
c a second antibody is suspected due to the results of the negattve contro l
d anti-P 1 cannot be confirmed due to the results of the negative control
285. Plasma neutralizaticm is best used to verify which of the following antibodies?
......
.,..., a an ti-Lub
b anti-M
c anti-Ch/Rg
d anti-V
286. To confirm the specificity of anti-Leb, an inhibition study using Lewis substance w as
performed with the following reS1Jl!s:
Le(b+} cells
tubos with patie nt serum + Lewis substanco: 0
tubes with patient serum + saline control: +

What conclusion can be made from these results?


a second antibody is suspected due to the positive control
b anti-Leb is confirmed because the tubes with Lewis substance are negative
c anti-Leb is not confirmed because the tubes with Lewis substance are negative
d anti-Leb cannot be confirmed because the saline positive is control

....
287.
()M.Y
Which of the foUowi.ng is the correct illterpretation of this saliva neutralization testing?
lndlcotor calls
Sample A B 0
saliva plus anti-A: + 0 0
saliva plus anti-B: 0 + 0
saliva plus anti-H: 0 0 0
a group A secretor
b group B secretor
c group AB secreter
d group 0 secreter

44 Tho Board of Certification Study Gulde 60


ISBN978-089189·6609 "201~A~(
1: Blood Bank Serologic & Molecular Testing

...
288. A person's saliva. incubated w~th the following antibodies and tested with the appropriate
.....A2. 0, and B indicator cells, gives the following test results:
Antibody specificity Test re.silts
nnh-A reactive
ontt·B inhibited
Mli-H inhibited

T11e person's red cells ABO phenotype is:


a A
b AB
c B
d 0
289. An antibody screen performed using solid phase lechnology revealed a diffuse layer or red
blood cells on the bottom or the well. These results indicate:
a a positive reaction
b a negative reaction
c scrum was not added
d red cells have a positive direct antiglobu'in test
290. Reagent antibody screening cells may not detect all antibodies. Which of the following
antibodies is most likely to go undetected?
a anti-Co•
b anti-S
c anti-Cw
d anti-Xg•
291. Which of the following genes on chromosome 1 is made up of 2 exons, leading to the
expression of the Duffy glycoprotein and its antigens?
a OAF
b FYAB
c FY
d ACKR1
292. While performing an antibody screen, a test reaction is suspected to be rouleaux. A saline
replacement test is performed and the reaction remains. What is the best interpretation?
a original reaction of rouleaux Is confirmec
b replacement test is invalid and should be repeated
c original reaction was due to true agglutir.ation
d antibody screen is negative

...
293. A 10-year-old girl was hospitalized because her urine had a distinct red color. The patient
OU had recently recovered from an upper respiratory Infection and appeared very pale and
lethargic. Tests were performed with the following results:
hemoglobin: 5 g/dL (SO gill
re1.ieutocyte count 15%
OAT: weak reactivity with poly·specific and anti-C3d; anti-l9G was negatlvo
antibody screen: negative
Donath-Landsteiner test: positive: P- cens s hovted no hemolysis

The patient probably has:


a paroxysmal cold hemoglobinuria (PCH)
b paroxysmal nocturnal hemogloblnuria (FNH)
c warm autoimmune hemolytic anemia
d hereditary ery1hroblaslic mullinuclearity with a positive acidified serum test (HEMPAS)

02018ASCP ISBN 97S-089189.Q609 Cllnlcat Laboratory Cet1iflcatlon Examinations 45


1 : Blood Ban k Sero/ogle & M olocu/;Jr Testing~
294. Which of \he following is useful for re moving lgG from red blood coils with a positive DAT
to p erfo rm a phenotype?
a b romelin
b chloroquine
c LISS
d OTT
· o of the antibodies is identified as
295. A patient's serum contains a mixture of antibodi e~- ~e present. Whal technique(s)
an\i -0. A nti-Jke, anti-Fy8 and possibly anothe_r ant1bo Y are
may be h elpful to identify the other ant1bod·1(1es)?
a e nzyme panel; select cell panel
b thiol reagents . . r rme
c lowering the pH and increasing the incdu~a. iono~bination with selective adsorption
d using albumin as an enhancement me 1a '" c
296. A samp.'e gives the following results:
Cells w ith: Sorum with:
2+
antl·A 3+ A 1 cells
0
antl·B 4+ e cells
W hich lectin should be used first to resolve this discrepancy?
a UIBX europBBUS
b Arschis hypogaea
c Dolichos biflorus
d Vicia graminea
A • ear-old female is admilled with anemia of undetermined origin. Bloo~ samples are
26
297. receiJed with a crossmatch request for 1 units of Red Blood Cells. The pa~1ent is group
A. Rh-negative and ha~ no history of transfusion or pregnancy. The following results were
obtained in pretransfu s1on testing:
37'C IAT
IS
0 3+
screening cell I 0
0 3+
screening cell II 0
0 3+
a utocontrol 0
0 0 3+
donor unit

The next step to continue this investigation would be:


a do an antibody identification panel
b use the saline replacement technique
c use the pre-warm tech nique
d perform a warm autoadsorption
298. A patient's serum was reactive 2+ irl the antiglobulin phase of testing with all cells on a
111.$
OIC.V
routine pane9 including their own. Transfusion was perfo rmed 6 m o nths previously. The
optimal adsorption method to remove the autoantibody Is:
a autoadsorption using the patient's ZZAP-treated red cells
b autoadsorption using the patient's LISS-t reated red cells
c adsorption using enzyme treated red cells from a norma l donor
d adsorption using m ethyldopa-treated red cells
299. In a cold autoadsorption procedure, pretreatment of the patient's red c ells w ith which of
~v the following reagen ts is helpful?
a ficln
b phosphate-buffered saline at pH 9 .0
c low ionic strength saline (LISS)
d albumin

46 The Board of Certlnc atlon St udy Gulde 6e ISBN 978-089189-6609 IC'2018Af


1: Blood Bank Transfusion Practice

300. The process of separation of antibody from its antigen is known as:
a diffusion
b adsorption
c neutralization
d etulion
301. Which of lhe following is most helpful 10 confirm a weak ABO subgroup?
....
o"' a adsorption-elution
b neutralization
c testing wilh A 1 leclin
d use of anli·A.B

Transfusion Practice
302. Deglycerolized Red Blood Cells are used lo transfuse patients with
a an antibody lo a high-incidence red cell antigen
b Anli-c and anli-K alloantibodies
c chro nic anemia
d increased risk of CMV infection
303. Cryoprecipitated AHF Ira nsfusion is recommended as a treatment for patients with
a type 1 vWF disorder
b factor V coagulation deficiency
c factor VIII coagulation deficiency
d DIC with hypofibrinogenemia
304. The primary indication for granulocyte transfusion is:
a prophylactic treatment for infection
b additional supportive therapy in those patients who are responsive to antibiotic therapy
c cl inical situations where bone marrow recovery is not anticipated
d severe neutropenla with an infection that Is nonresponsive to antibiotic therapy
305. A 42-year-old male of average body mass has a history of chronic anemia requiring
transfusion support. Two units o6 Red Blood Cells are transfused. If the pretransfusion
hemoglobin was 7 .0 g/dl (70 g/L}, the expected posttransfusion hemoglobin concentration
should be:
a 8.0 g/dl (80 g/L)
b g_o g/dl (go g/L)
c 10.0 g/dl (100 g/L)
d 11 .0 g/dl (110 g/L}
306. How many units of Red Blood Cells are required to raise the hematocrit of a 70 kg
nonbleeding man from 24% to 30%?
a 1
b 2
c 3
d 4

C2018 ASCP ISBN 978-089189-6609 Cllnleaf Laboratory Certifi~atlon Examinatlons 47


1 : Blood Bank Transfusion Pr<ict;c-:-W

~...~7 · '. 0 r. which o r the following translusion candidates would CMV-sa fe blood be mo st likely
ONI.• 1nd1cated?
a renal dialysis patients
b sickle cell patient
c bone marrow and hematopoietic cell transplant recipients
d CMV-seropositive patients
308 . Although ABO compatibility is preferred. ABO incompatible product may be administ-:ired
.....
Otlll' when transfusing:
a Single-Donor Plasma
b Cryoprecipitated AHF
c Fresh Frozen Plasma
d Granulocytes
309. Transfu sion of plateletpheresis prod ucts from HLA-compatible donors is the preferred
treatme nt fo r:
a recently diagnosed cases of TTP with severe thrombocytopenia .
b acute leukemia in relapse with neutropenia. thrombocytopenia and sepsis
c immune thrombocytopenic purpura
d severely thrombocytopenic patients. known to be refractory to random donor platelets
310. w ashed Red Blood Cells are indicated in Y.'hich of the following situations?
.....
""" a an lgA-deficient patient with a history of transfusion-associated ana phylaxis
b a pregnant wo man with a history o r hemolytic disease of the newborn
c a patient with a positive DAT and red cell autoa ntibody
d a newborn with a hematocrit of <30%
311. Which o r the following is consistent with standard blood bank procedure governing the
infusion of fresh frozen plasma?
a only blood group-specific plasma may be administered
b group 0 may be admin istered to recipients of all blood g roups
c g roup AB may be adm:nistered to AB recipients only
d group A may be administered to both A and O recipients
312. A patient who is g roup AB, Rh-negative needs 2 units of Fresh Frozen Plasma. Which of
the following units of plasma would be mos t acceptable for transfusion?
a group 0 , Rh-negative
b group A, Rh-negative
c group B, Rh-positive
d group A B. Rh-positive
313. ~hat inc~ement of platelets in the typical 70 -kg human is expected to result fro m each
single unut of Platelets transfused to a non-HLA-sensitized recipient?
a 3,000-5,00/µL
b 5,000 - 10,000/µL
c 20,000-25,000/µL
d 25,000-30,000/µL
;n4. Platelet tra ns fus ions are of m ost value in treating:
CM.v a hemolytic transfusion reaction
b posttransfusion purpu ra
c functional plalelet abnonmalities
d immune thrombocytopenia

48 The Boord of Cortlncatlon Study Gulde 6e ISBN 978-089189-6609 Q::O\~,\~'l'


1: Blood Bank Tr ansfusion Prncticc

315. A common strategy lo reduce alloimmunizallon in chronically transfused patients wit~


Sickle Cell Disease (SCD) is to provide red oell units matched ror which or the following
antigens?
a FY''· Fyb. and Oi 3
b M. N, and S
c Jk•, Jkb and s
d C, E, and K
316. Guidelines for emergency release of blood require that
a ABO group and Rh typing of recipient be performed berore the unit is released
b only g roup 0 Whole Blood be issued
c only Rh-negative blood is used for Rh -negative patients
d the physician signs a document authorizing the emergency release
31 7. Fresh Frozen Plasma from a group A. Rh-positive donor may be safely transfused to a
patient who is group;
a A . Rh-negative
b B, Rh-negative
c AB, Rh-positive
d AB, Rh-negative
318. A patient admitted to the trauma unit requires emergency release of Fresh Frozen Plasma
(FFP). Which of the following blood groups :if FFP should be issued?
a A
b B
c AB
d 0
319. Fresh Frozen Plasma:
a contains all labile coagulative factors except cryoprecipitated AHF
b has a higher risk of transmitting hepatitis than does Whole Blood
c should be transfused within 24 hours of thawing
d need not be ABO-compatible

..
320. Four units of group A platelets were transfused to a group AB patient as group AB platelets
ORY were not available. The patient's pretransfusion platelet count was 10,000/µL
and the posttransfusion count was 16.000/µL. From this information. the laboratorian
would most likely conclude that the patient
a needs group AB platelets to be more effective
b clinical data does not suggest a need for platelet transfusion
c has developed antibodies to the transfused platelets
d should receive irradiated platelets
321. Hypotension , nausea. flushing, fever and chills are symptoms of which of the following
transfusion reactions?
a allergic
b circulatory overload
c hemolytic
d anaphylactic

02ol8ASCP ISBN 978·089189·6609 Cllnlcal Laboratory Cerlificatlon Examinations 49


1: Blood Bank Tr.~nsfuslon JJrac11ce

~,~2 · A patient has become refractory to platelet transfusion . Which of the following are probable
o...v causes?
a
transfusion or Rh-inco mpatible platelets
b
decrea sed pH of the platelets
c
deve lopm ent of an alloantibody with anti-D specificity
development of antibodies to HLA a ntigen
d
323. A patient has symptoms indicating a possitle hemolytic transfusion reaction. What should
be done immediately?
a stop the transfusion and discard the unit
b contact the patient's doctor to ask if the transfusion should be stopped
c stop the transfusion and call the patient's doctor to report the reac~ion
d have patient blood s amples sent to the lab to investigate the reaction
324. Posttransfusion purpura is usually caused by:
1.U
""" a a nti-A
b white cell antibodies
c anti-HPA- 1a
d platelet wash-<>ut
325. An unexplained fa ll in hemoglobin and mild jaundice in a patient tra nsfused with Red
Blood Cells 1 week previously would most likely indicate:
a paroxysmal nocturnal hemoglobinuria
b posttransfusion hepatitis infection
c presence of HLA antibodies
d delayed hemolytic transfusion reaction
326. tn a delayed transfusion reaction, the causative antibody is generally too weak to be
detected in routine compatibility testing and antibody screening tests, but is typically
detectable at what point after transfusion?
a 3- 6 hours
b 2 days-2 weeks
c 60-90 days
d after 120 days
327. Which of the following is a potential complication of massive tran sfusions?
a citrate toxicity with hypercalcemia
b heparin-induced thrombocytopenia
c ~ypolhermla due to 1- s • c storage temperature of red cells
d iron overload from donor red cells leaking intracellular iron
328. Severe intravascular hemolysis Os most likely caused by antibodies of hi h bl d
system? w c oo group
a ABO
b Rh
c Kell
d Duffy
329. Which of the following blood group system~ 1
hemolytic transfusion reactions? • s m ost commonly as sociated with delayed
a Lewis
b Kidd
c Lu
d I

50 Th e Board of Certification Study Guide 6e


ISBN 978·0891811-6609 Q2018ASCP
1: Blood Bank Tr.1nsf11s/on Practice

330. Aller receiving a unit of Red Blood Cells, a palient immediately developed !lushing.
nervousness, fever spike of 102°F (38.9°C), shaking, chills and back pain. The plasma .
hemoglobin was elevated and there was hemoglobinuria. Laboratory investigation of this
adverse reaction would most likely show:
a an error in ABO grouping
b an error in Rh typing
c presence of anti-Fy• antibody In patient's serum
d presence of Gram-negative bacteria in blood bag
3.31, A trauma patient who has just received 1Ounits of blood may develop:
a anemia
b polycythemia
c leukocytosis
d thrombocytopenia

.......
332. Five days after transfusion. a patient becomes mildly jaundiced and experiences a drop
in hemoglobin and hematoorit with no apparent hemorrhage. Below are the results of the
transfusion reaction workup:

anll-A antl-B onti-0 A.1 Celts B cells Ab screen DAT


p atient
pretransfuslon neg 4+ 3• 4• neg neg neg
patient
posttransfusion neg 4+ 3+ 4+ neg 1+ I+
donor #1 neg neg 3• 4+ 4+ neg
donor#2 neg 4+ 3• 4+ nGg neg

In order to reach a conclusion, the medical lab scientist should:


a retype the pre- and posttransfusion patient samples and donor #1
b request an EDTA tube be drawn on the patient and repeat the OAT
c repeat the pretransfusion antibody screen on the patient's sample
d perform an elution on the posttransfusion sample and identify the antibody in the eluate
and the serum
333. The most appropriate laboratory test for early detection or acute posttransfusion hemolysis
is:
a visual inspection for free plasma hemoglobin
b plasma haptoglobin concentration
c examination for hematuria
d serum bilirubln concentration
334. During initial investigation of a suspected hemolytic transfusion reaction, it was observed
that Blood Bank paperwork and patient sample and bOood component labels were correct,
the posttransfuslon reaction plasma was yellow in color as was the pretra nsfusion sample.
and the direct antiglobulin test was negative. Repeat ABO typing on the posttransfusion
sample confirmed the pretransfusion results. What is the next step in this investigation?
a repeat compatibility testing on the suspecied unil(s)
b perform plasma hemoglobin and haptoglobin determinations
c use enhancement media to repeat the antibody screen
d no further serological testing is necessary

Olol8ASCP ISBN 978-089189·6609 Cllnlcal Laboratory Certification Examinations 51


1 : B lood Bank
335. Wh'ch
1 o f lhe following transfusion reactions Is characterize
· d by high fever. shock. -
Tn:i n s fu slo n Practice •

hemogloblnuria, DIC and renal failure?


3 bacteri11I contamination
b circulatory overload
c febrile
d anaphylactic
·
336. H emoglobmuna, · · d bl d' g are symptoms of which or the
hypotens1on and generalize ee m
following transfusion reactions?
a allergic
b circulatory overload
c hemolytic
d anaphylactic
337. Patients who are chronically transfused with red cell components can deve lop
a iron overload
b low ferritin
c hypokalemla
d hypertensive reaction
338. A patient's record shows a previous anti-Jkb, but the current antibody screen is negative.
What further testing should be done before transfusion?
a phenotype the patient's red cells for the Jkb antigen
b perform a cell panel on the patient's serum
c crossmatch type specific units and release only compatible units for transfusion
d give J k(b-), crossmatch compatible blood
339. Which of the following is associated with a risk of developing transfusion-associated graft
versus host disease (TA-GVHO)?
a patients receiving leukocyte reduced blood compone nts
b directed donation from first-degree family member
c patients transfused with irradiated blood components
d autologous blood donation prior to surgery
340. A patient is readmitted to the hospital w ith a hemoglobin level of 7 g/dl (70 g/L) 3 weeks
after receiving 2 units of red cells. The initial serological tests are:
ABO/Rh: A+
anttbody screen: negative
OAT: 1+ mixed field

Which test should be performed next?


a antibody identification panel o n the patient's serum
b repeat the ABO type on the d onor units
c perform an elution and identify the antibody in the eluate
d crossmatch the post reaction serum with the 2 donor units
341. In a delayed hemolytic transfusion reaction. the direct antiglobulin test on the post-
transfusion sample can be:
a negative
b mixed-field positive
c positive due lo complement
d negative when the antibody screen is negative

52 Tho B oard al Certifloatlan Study Gulde 6o


ISBN 97S-089189-6609 C2018ASCf
1: Blood Bank Transfusion Prac tice
342. To prevent donor lymphocytes from cngraftment in the bone marrow of an
immunosuppressed patient, all transfusion products must be
a washed
b leukocyte-reduced
c treated with UV light
d irradiated
343. For a patient who has suffered an acute herrolytic transfusion reaction, the primary
treatment goal should be to:
a prevent alloimmunization
b diminish chills and fever
c prevent hemoglobinemia
d reverse hypotenslon and minimize renal carnage

.......,
344. Nine days after being transfused with an HLA-matched platelet transfusion , a patient
develops a fever, watery diarrhea. skin rash and demonstrates increased liver enzymes.
This patient may have transfusion-associated:
a allergic urticaria reaction
b hepatitis C
c graft vs host disease
d septicemia with endocarditis
345. The preferred replacement fluid to maintain normal oncotic pressure and intravascular
~v fluid levels for patients who have therapeutic plasma exchange (TPE) for thrombotic
thrombocytopenic purpura is:
a IVlg
b plasma
c crystalloid solutions
d isotonic saline
346. Which of the following disease states is trealed with therapeutic plasma exchange {TPE)?
.....
""' a gout
b myasthenia gravis
c intrahepatic cholestasis
d Crohn disease
347. A 50-year-old patient with acute lymphocytic leukemia has symptoms of dyspnea, visual
~' abnormalities, and headache. The blast count is greater than 100,000/µl. What type of
apheresis is indicated to treat this patient?
a extracorporeal photopheresis (ECP) to remove immature lymphocytes
b selective absorption apheresis to remove immature lymphocytes
c cytapheresis to remove immature lymphocytes . .
d therapeutic plasma exchange (TPE) to remove antibodies
348. Which of the following transfusion reactions occurs after infusion of only a few milliliters or
blood and gives no history of fever?
a febrile
b circulatory overload
c anaphylactic
d hemolytic

~181\SCP ISSN 978-0S918UOO!I Cl/nlcal Laboratoty Ceroncatlon Examinations 53


1 : Blood Bank ----..........
Transfusion Practic., 1
349. F ever and chill
s are symptoms of which o f the following transfusion reactions? -
a citrate toxicity
b circulatory overtoad
c allergic
d febril e
3so. Hives and itching are symptoms of which of the following transfusion reac tions?
a febrile
b allergic
c circulatory overload
d bacterial
351. A temperature rise of 1•c or more occurring in association with a tra nsfusion, w ith no
abnormal results in the transfusion reaction investig ation, usually indicates which of the
following reactions?
a febrile
b circulatory overload
c hemolytic
d anaphylactic
352. A 65-year-old woman experienced shaking, chills, and a fever of 102°F (38.9°C)
approximately 40 minutes following the transfusion of a second unit of Red Blood Cells.
T he most likely explanation for the patient's symptoms is:
a transfusion of bacterially contaminated blood
b congestive heart failure
c anaphylactic transfusion reaction
d febrile transfusion reaction
353. Prior to initiating a blood tra ns fusion. the transfusionist and another q ualified individual
mu st
a match the blood component to the recipient using 2 independent identifiers
b verify that the recipient's vital signs are within normal limits
c review the patient's medical record to verify the need fo r a tran sfusion
d order a baseline hemoglobin and hematocrit before the tra nsfusion is s ta rted
354. Use of only male donors as a source of olasma intended for transfusion is advocated to
~v reduce whic h type of reaction?
a allergic
b TRALI
c hemolytic
d TACO (circulatory overload)
355. Which of the following must be verified in the transfusion service prior to the issue of bloo
products?
a a ntibody detection test result
b date and time when the patient's bk>od sample was drawn
c name of the transfusionist administering the blood product
d expiration date and, if applicable, expiration time of the blood product

54 Tho Board of Certlncatlon Study Gulde 60


ISBN 978-089189-66()9 C2018AS
1: Blood Bank Transfusion Practice
356. Symptoms of ?yspnea. hypoxemia, and pulmonary edema within 6 hours of tran srusion is
most likely whrch type of reaction?
a anaphylaclic
b hemolytic
c febrile
d TRALI

357. In an emergency situation. what type of blood should be given to a remale patient of child·
bearing age 11the ABO group and Rh type are unknown?
a Group 0 , Whole Blood should be given
b Group O. Rh-negative red cells should be given
c Group 0, Rh-positive red cells should be given
d Group 0, Washed Red Blood Cells should be given
358. During the issue of an autologous unit of Whole Blood, the supernatant plasma is
observed to be dark red in color. What would be the best course of action?
a the unit may be issued only for autologou> use
b remove the plasma and issue the unit as Red Blood Cells
c issue the unit only as washed Red Blood Cells
d quarantine the unit for further testing
359. Coughing, hypoxiemia and difficult breathing are symptoms of which of the following
transfusion reactions?
a febrile
b allE*gic
c circulatory overload
d hemolytic
360. Which of the following statement regarding red cell transfusion to infants less than 4
months old is correct?
a only phenotype identical units should be issued
b fresher units (generally <7 days) should be issued
c irradiated blood components are contraindicated
d crossmatching with a current sample is required for each transfusion
361. Congestive heart failure, severe headache and/or peripheral edema occurring soon after
transfusion is indicative of which type of transfusion reaction?
a hemolytic
b febrile
c anaphylactic
d circulatory overload
362. Which of the following must be performed on a patient before and after receiving a blood
transfusion?
a blood pressure, pulse, respiration rate and temperature
b blood pressure, pulse, temperature and urine occult blood
c blood pressure, temperature, respiration rate and .hematocrit
d blood pressure, temperature, pulse and hematocnt

Cllnlcal Laboratory Certification Examin::ttions 55


t2l18ASCP ISSN 978-089189-6609
1 : Bloo d Bank
Transfusion Pr;ict/
I
363. A Patient became h . . c&
Blood Cells Sh h YPO1ens1~e and.went into shock after receiving 50 ml o f a unit of Red
lransfus·10 · e. ad a sh?kmg chill and her temperature rose to 104.8 . F (40.4 •c ). f\
addil'o nt reaction investigation wa s initialed but no abnormal results were seen. 'Nhat
1 na1 esting should be performed?
a Gram s tain and culture of the donor unit
b ly mphocytotoxicity tests for leukoagglutinins
c plasma lgA level
d elution and antibody identification
364. T he most frequent transfusion-assocla\ej di sease complication of blood transfusions is:
a cytomegalovirus (CMV)
b syphilis
c hepatitis
d A IDS
365. The purpose of Irradiation or blood components is to:
a prevent postlransfusion purpura
b prevent graft-vs-host (GVH) disease
c sterilize components
d prevent noncardiogenic pulmonary edema
366. Which or the following patient groups is at risk of developing graft-vs-host disease?
IU
""'' a full term infants
b patients with history of febrile tran sfusion reactions
c patients with a positive direct anliglotulin test
d recipients of blood donated by immediate family members
An intraoperalive strategy fo~ patient blood management is
..367.
...
...., a a nemia assessment and treatment
b autologous blood donation
c acute normovolemic hemodilution
d limiting phlebotomy-related blood loss
368. Therapeutic plasmapheresis is performed in order to:
......
ONLY
a harvest granulocytes
b harvest platelets
c treat patients with polycythemia
d treat patients with plasma abnormaliti es
369. Plasma exchange is recommended in the treatment of patients w ith macroglobulinemia in
~v order to remove:
a antigen
b excess lgM
c excess lgG
d abnormal platelets
370. The most important step in the safe administration o f blood is to:
a perform compatibility test;ng accurately
b get an accurate patient history
c exclude dlsqualiified donors
d accurately identify the donor unit and recipient

56 The Board o f Certincetlon S l udy Gulde 6e


ISSN 978-089189-6E09 02018~
1: Blood Bank Transfusion Practice
371 . Transfusion-associated HTLV 1111 incidence is low due lo the rollowing laboralory lesting
a nucleic acid testing for HTLV I/II
b HTLV I core antibody testing by ELISA
c HTLV II surface antigen testing by chemiluminescent immunoassay (ChLIA)
d lgG antibody testing for HTLV I/II by ELISA or ChLIA
372. Which viral diseases have a lower incidence of transfusion-associated infections due to
nucleic acid testing (NAT)?
a HIV-1. HCV, WNV
b HIV-2, HBV, CMV
c HTLV 1111, HCV, WNV
d CMV, WNV, HCV

Cl/n/ca/ Labor.ttoty Cortiflcallon Examinations 57


C2018ASCP ISBN 97B-089189-6609
1: Blood Bank Tran s fusion Praci· ,
•co
1. c SS. a 109. b 163. d 217. c 271. c 325. d
2. c 56. c 110. c 16• . d 218. c 272. a 326. b
3. c 111. a 165. a 219. c 273. b 327. c
S7. b
4. d c 166. c 220. c 274. d 328. a
58. b 112.
5. b 113. c 167. a 221. d 275. d 329. b
59. d 276. c
6. c 114. d 168. d 222. d 330. a
60. d 277. d
115. d 169. c 223. b 331. d
7. a 61. c 278. c 332. d
8. d 62. a 116. a 170. d 224. b
117. c 171. b 22S. c 279. a 333. a
9. d 63. c 280. b 334. d
10. b 64. a 118. d 172. a 226. d
119. a 173. b 227. c 281. b 335. a
11. b 65. a 282. a 336. c
12. d 66. d 120. b 174. d 228. a
121. c 175. a 229. b 283. d 337. a
13. b 67. c 284. d 338. d
14. c 68. d 122. a 176. a 230. b
123. c 177. c 231. a 285. c 339. b
15. b 69. a
124. b 178. b 232. b 286. b 340. c
16. b 70. c
17. d 71. a 125. a 179. a 233. c 287. d 341. b
126. b 180. d 234. c 288. c 342. d
18. c 72. a 289. a 343. d
73. d 127. a 181. a 23S. d
19. b 290. c 344. c
20. b 74. c 128. d 182. c 236. a
c 129. d 183. b 237. d 291. d 345. b
21. a 75.
d 130. a 184. d 238. a 292. c 346. b
22. a 76.
77. b 131. b 185. c 239. b 293. a 347. c
23. b
78. d 132. d 186. b 240. c 294. b 348. c
24. b 349. d
79. b 133. b 187. c 241. b 29S. a
25. a
80. d 134. a 188. b 242. a 296. c 3SO. b
26. c
81. c 13S. d 189. b 243. d 297. d 351. a
27. d
82. b 136. c 190. c 244. c 298. a 3S2. d
28. a
83. a 137. d 191. a 245. a 299. a 353. a
29. c
30. a 84. d 138. b 192. a 246. a 300. d 354. b
31. b as. d 139. c 193. b 247. a 301. a 35S. d
32. b 86. c 140. a 194. d 248. a 302. a 356. d
33. c 87. c 141. c 195. d 249. a 303. d 357. b
34. a 88. c 142. b 196. b 250. b 304. d 358. d
35. c 89. c 143. d 197. c 251. d 30S. b 3S9. c
36. b 90. d 144. d 198. c 252. b 306. b 360. b
37. c 91. a 14S. a 199. c 253. a 307. c 361. d
38. b 92. b 146. c 200. d 254. b 308. b 362. a
39. b 93. d 147. c 201. c 2SS. a 309. d 363. a
40. b 94. b 148. c 202. a 256. b 310. a 364. c
41. a 95. d 149. a 203. a 257. b 311. d 365. b
42. a 96. d 150. c 204. a 2S8. b 312. d 366. d
43. d 97. c 151. a 205. b 259. a 313. b 367. c
44. c 98. d 1S2. d 206. c 260. a 314. c 368. d
45. a 99. d 153. b 207. a 261. b 315. d 369. b
46. a 100. c 154. d 208. d 262. a 316. d 370. d
47. b 101. b 15S. c 209. c 263. b 317. a 371. d
48. a 102. c 1S6. a 210. c 264. a 318. c 372. a
49. b 103. a 1S7. b 211. a 265. c 319. c
50. b 104. c 158. d 212. d 266. b 320. c
51 . c 105. b 159. c 213. b 267. d 321. c
52. d 106. c 160. b 214. c 268. c 322. d
53. a 107. c 161. d 215. a 269. c 323. c
54. b 108. b 162. c 216. b 270. a 324. c

58 Tho Board of Cortlncatlon Studv Gul de 6e ISRN R7Ml89189·6609 02018.t.sCI' ,


2: Urinalysis & Body Fluids
Url11alysls: Prcrmalytical Examl11atio11

Urinalysis & Body Fluids


n1e followi11g items llave b een identified generally as appropriate for both entry level medical
laboratory scientists and m edica l laboratory technicians. Items that are appropriate for medical
laboratory scientists only are marked with an "MLS ONLY."

85 Quostions
85 Urinalysis: Preanalytica/ Examination 122 Answers with Explanations
87 Urinaly sis: Physical Exam/notion 123 Urinalysis: PreanalytiC<JI Exominatlon
124 Urinalysis: Physical Examination
90 Unnalysis: Chemical Examination
125 Urinalysis: Chemical Examination
94 Urinalysis: Microscopic Examination
127 Unnalysis: Microscopic Examination
104 Urin;,lysis: Complete Examination
108 Urine Physiology 131 Urinalysis: Complete Examination
132 Urine P/1ysiotogy
110 Other Body Fluids
133 Other Body Fluids

Urinalysis: Preanalytica/ Examination


1. After receiving a 24-hour urine for quantitative total protein analysis, the laboratorian must
first:
a subculture the urine for bacteria
b add the appropriate presetvative
c screen for albumin using a dipstick
d measure the total volume
2. A falsely low result for urobllinogen may occur if the urine specimen is:
a exposed to light
b adjusted to a neutral pH
c oooled to room temperature
d oollected in a nonsterlle container
3. A d ean-catch urine is submitted to the laboratory for routine uri.nalysis and cul~ure. The
routine urinalysis is done first, and 3 hours later, the specimen 1s sent to the m1crob1ology
department for culture. The specimen should:
a be centrifuged, and the supernatant cultured
b be rejected due to the time delay
c not be cultured if no bacteria are seen
d be processed for culture only if the nitrate is positive
4. Which of the following urine results is most ikely to be affected by prolonged exposure to
light?
a pH
b protein
c ketones
d bllirubin
5. Urine samples should be examined within 1 hour or voiding because:
a RBCs, leukocytes and casts agglutinate on standing for several hours at room

b urob1hnogen increases an d b'I'


temperature 11rubin decreases •
after
r th prolonged
· e exposure to light
c bacterial contamination will cbauste ~lklalnin~z::fu7a~ me~~b~~ism
d ketones will increase due to ac ena a

0'2o18 ASCP ISBN 978-089189~609 C/lnlcat l.Jlboratory Certification Exam/notions 85


-..ii

2 : Urinalys is & Body F l uids Urinalysis: Preana ly tical Examinar;011

6. The following results were obtained on a urine specimen at 8:00 am:


Mt S
ONlV pH: 5. 5
protein: 2+
glucose: 3+
ketones: 3+
blood: negative
bilirubin: positive
nitrite: positive
If this urine specimen was stored uncapped at s•c without preservation and retested
at 2:00 pm, which of the following test results would be cha nged due to these storage
conditions?
a glucose
b ketones
c protein
d nitrite
7. A urine specimen comes to the laboratory 7 hours after it is obtained. It is acceptable for
culture only if the specimen has been stored:
a at room temperature
b at4-7°C
c frozen
d with a preservative additive
8. Which of the following would be affected by allowing a urine specimen to remain al room
temperature for 3 hours before a nalysis?
a occult blood
b specific gravity
c pH
d protein
9. A 24-hour urine from a m an w ho had no evidence of kidney impairment was sent lo the
......
O~ILY
laboratory for hormone determination. The volum e was 600 ml, but there was some
q uestion as to l he completeness of the 24-hour collection. The next step would be to:
a perform the hormone determination, since 600 m l is a normal 24-h our urine vo lume
b check the creatinine level; if it is <1 g , do the procedure
c report the hormone determ ination in mg/dL in case the specim en was incomplete
d check the creatinine level; if it is > 1 g, do the procedure
10. Failure to observe RBC casts in a urine specimen can be caused by:
a staining the specimen
b centrifug ing an unmixed specimen
c mixing the sediment after decantation
d ex amining the sediment first under low ::iower
11. eG~R ~alculated by the MDRD-IDMS traceab le stud y equatio n take s into account the
I.II.$
<lHtY patient s age, race, gender and serum concentration o f which of the follow ing?
a urea
b ammonia
c creatinine
d cystatin C
12. The creatinine clearance is reported in:
a mg/dL
b mg/24 hours
c mUmin
d mU24 hours

86 The Board of C ertification S tudy Guld e 6 e


ISBN 978-089189-6609 Co2()18ASCI'
-- 2: Urinalysis & Body Fluids
13.
Urinalysis: Pllysical Ex aminatio n
An increased microalbumin concentration !rem a random urine collection is predictive or:
a diabetes mellitus
b nephropathy
c hypertension
d nephrotic syndrome

Urinalysis: Physical Examination


14. A patient with uncontrolled diabetes mellitus will most likely have:
a pale urine with a high specific gravity
b concentrated urine with a high specific gravity
c pale urine with a low specific gravity
d dark urine with a high specific gravity
15. While performing an analysis of a baby's urine. the laboratorian notices the specimen has
a •mousy" odor. Of the following substances that may be excreted in urine, the one that
most characteristically produces this odor is:
a phenytpyruvic acid
b acetone
c coliform bacilli
d porphyrin

,,..16., An ammonia-like odor is characteristically associated with urine from patients who:
""' a are diabetic
b have hepatitis
c have an infection with Proteus sp
d have a yeast i~fection
17. Urine that develops a port wine color after standing may contain:
a melanin
b porphyrins
c bilirubin
d urobilinogen
18. Acid urine that contains hemoglobin will darken on standing due to the formation of:
.....
oou a myoglobin
b sulfhemoglobin
c methemoglobin
d red blood cells
19. Urine from a 50-year-old man was noted to tum dark red on standing. This change is
caused by the presence of:
a glucose
b porphyrins
c urochrome
d creatinine
20. The clarity of a urine sample should be determined:
a using glass tubes only; never plastic
b following thorough mixing of the specimen
c after addition of sulfosalicy1ic acid
d after the specimen cools to room temperature

t?lt~ASCP ISBN 978-089189-6609 Clinical l.abor•tory Cert1"cation EKaminallons 87


2: Urinalysis & Body F luids
21 ·
Urinalysis : Phys/cal
Milky urine from a 24 -year-old woman would most likely contain:
Examinat;o~ i
--...._
a spermatozoa
b many white blood cells
c red blood cells
d bilirubin
22. A brown-black urine would most likely contain:
a bile pigme nt
b porphyrins
c melanin
d blood cells
23. The yellow color of urine is primarily due to:
a urochrome pigment
b methemoglobin
c bilirubin
d homogenistic acid
24. Red urine may be due to:
a bilirubin
b excess urobilin
c myoglobin
d homogenistic acid
25. A urine specimen collected on an apparently healthy 25-year-old man shortly after he
ULS
Of/LY finished eating lunch was c loudy but showed normal results on a multiple reagent test strip
analysis. The most likely cause of the turbidity is:
a fat
b wh ite blood cells
c urates
d phosphates
26. In which of the following metabolic diseases will urine tu rn dark brown to black upon
....$
ONl.Y
standing?
a phenylketonuria
b alkaptonuria
c maple syrup disease
d aminoaciduria
27. Urine osmolality is related to:
a pH
b filtration
c specific gravity
d volume
28. Urine specific gravity is an index of the ability of the kidney to:
a filter the plasma
b concentrate the urine
c alter the hydrogen ion concentration
d reabsorb sodium ions

88 The Board of Certification Study Guide Se ISBN 978-08918%609 ~2o 1 eA5'


2: Urinalysis & Body Fluids Urlnalysls: Physical Ex11111in11tion
29. Osmolality is a measure of:
a dissolved particles. including ions
b undissociated molecules onty
c total sall concentration
d molecule size
JO.A pati?nl urine sample has an increased protein and a high specific gravity. Which or the
~v following would be a more accurate measure or urine concentration?
a osmolality
b ketones
c refractive index
d pH
31. To prepare a solution appropriate for quality control of the refractometer, the laboratorlan
should use:
a urea with a specific gravity or 1.040
b water with a specific gravity of 1.005
c sodium chloride with a specific gravity of 1.022
d calcium chloride with an osmolarity of 460
32. A urine's specific gravity is directly proportional to its:
a turbidity
b dissolved solids
c sall content
d sugar content
33. lsosthenuria is associated with a specific gravity which is usually:
....
cuv a variable between 1.001 and 1.008
b variable between 1.015 and 1.022
c fixed around 1.010
d fixed around 1.020
34. The fluid leaving the glomerulus normally has a specific gravity of:
11.S
.,., a 1.001
b 1.010
c 1.020
d 1.030

...
35. A deficiency in arginine vasopressin (antidiur~tic hormone [ADH]) is associated with a:
"'" a urine specific gravity around 1.031
b low urine specific gravity
c high urine specific gravity
d variable urine specific gravity
36. When using a refractometer to measure urine concentration, the laboratorian must correct
for which of the following in their calculations?
a temperature
b pH
c glucose
d volume
37. Calibration of rerractometers Is performed by measuring the specific gravity or distilled
::t:v water and:
a protein
b glucose
c sodium chloride
d urea

Cllnlcal Laboratory Cerliflcation Examf,,etlons 89


Qlo1a .0.SCP ISBN 978-08918~6609
2: Urinalysis & Body Fluids Urin alysis: C /1cmlcal E xam/111111011 "1
38. Tho method of choice for pe1iormi ng a specinc gravity mee1surement of urine following
administration of x-ray contrast dye Is:
a reagent test strip
b refractometer
c urinomcter
d densitometer
39. Which of the following urinary parameters are measured duri ng the course of
~~v concentration and dilution tests to assess renal tubular function?
a urea. nitrogen and creatinine
b osmolality and specific gravity
c sodium and chloride
d sodium and osmolality
40. Refractive index is a comparison of:
1.ttS
""'' a llght velocity in solutions to light velocitr In solids
b light velocity in air to lignt velocity in so utions
c light scattering by air to light scattering by solutions
d light scattering by particles in solution

Urinalysis: Chemical Examination


41. Which o f the following can give a false-negative urine protein reading?
MlS
°"'v a contamination with vaginal discharge
b heavy mucus
c presence o f blood
d very dilute urine
42. The pH of a urine specimen measures the:
...s
OM.v a free sodium ions
b free hydrogen ions
c total acid excretion
d volatile acids
43. Upon standing at room temperature, a urine pH typically:
a decreases
b increases
c remains the same
d changes depending on bacterial concentration
44. Urine reagent test strips should be stored in a(n):
a refrigerator (4-7°C)
b incubator (37°C)
c cool dry area
d open jar exposed to air
45.
IA.S The principle of the reagent test strip for urine protein depends on:
Ot.tY
a an enzyme reaction
b protein error of indicators
c copper reduction
d the tolu idine reaction
~2 : Urinalysis & Body Fluids Urinalysis: C/Jemlcal Exmn/nat/011

46. The protein section of the urine reagent test strip is most sensitive to:
:i albumin
b mucoprotein
c Bence Jones protein
d globulin

47. Routine screening of urine samples ror glycosuria is performed primarily to detect:
~. a glucose
b galactose
c bilirubin
d ketones
43. Which of the following reagents is used to react with ketones In the urine?
a sodium nitroprusside
b acetoacetic acid
c acetone
d beta-hydroxybutyric acid
49. A test pad of a urine reagent strip is impregnated with only sodium nitroprusside. This pad
"" will react with:
""' a acetoacetic (diacetic) acid
b leukocyte esterase
c beta-hydroxybutyric acid
d ferric chloride
so. A reagent test strip pad impregnated with stabilized. dia2oti2ed 2,4-dichloroaniline will
yield a positive reaction with:
a bilirubin
b hemoglobin
c ketones
d urobilinogen
51. Which of the following substances may interfere with the reagent test strip pad for
,.. leukocyte esterase and yield a false-negative result?
"'·' a granulocytic leukocytes
b ketones >10.0 mg/dl
c protein >500 mg/dl
d glucose <3 g/dl
52. Excess urine on the reagent test strip can turn a nonmal pH result into a falsely acidic pH
~. when which of the following reagents runs into the pH pad?
a tetrabromphenol blue
b citrate buffer
c glucose oxidase
d alkaline copper sulfate
53. When employing the urine reagent test strip method, a false-positive protein result may
occur in the presence of:
a large amounts of glucose
b x-ray contrast media
c Bence Jones protein
d highly alkaline urine

Clinical Labomtory Certification Examinations 91


Urinalysis: Clremlcat Exnm/na
2 : Urinalysis & Body Fluids
110 ~~
54. A 17-year-old girl decided to go on a slarvation diet. After 1 week or starving herself.~
substance would most likely be round In her unne? I
a protein
b ketones
c glucose
d blood
55. A 2-yea r-old child had a posilive urine ketone. This would most likely be caused by:
a vomiting
b anemia
c hypoglycemia
d biliary tract obstruction
56. A patient's urinalysis revealed a positive bllfrubin and a decreased urobilinogen lever.
•••
ONlY
These results are associated with:
a hemolytic disease
b biliary obstruction
c hepatic disease
d urinary tract Infection
57. A urine specimen with an elevated urobilinogen and a negative bllirubin may indicate:
Ul$
""'v a obstruction or the biliary tract
b viral hepatitis
c hemolytic jaundice
d cirrhosis
58. Microscopic analysis of a urine specimen yields a moderate amount of red blood cells in
spite of a negative result for occult blooc using a reagent strip. The laboratorian should
determine if this patient has taken :
a vitamin C
b a diuretic
c high blood pressure medicine
d antibiotics
59. The purpose for routinely screening diabetes mellitus patients for microalbuminuria is to
~v monitor the development of:
a urinary tract infection
b renal disease
c yeast infections
d diabetes insipidus
60.
....
O'c.T
The principle of the reagent strip test for microalbuminuria is:
a a diazo reaction
b the protein error of indicators
c an immunochemical reaction
d the release of hydrogen ions to an indicator

....61.
Cl'&Y
The reason that an album·n·1 .crea t'1nine
· ratio
· can be performed on a random specimen ·1s:
a creatinine corrects for over or under body hydration
b a first ~orning specimen may be too concentrated
c albumin ~rr~cts for over or under body hydration
d the reaction 1s sensitive to any level of albumin

92 Th" Board of Certiflcallon Study Gulde 6e


ISBN 978-089189-6609 C20t8ASCI'
p '
2: Urin alysis & Body Fluids Urlllalysis: Cllemlcal Exomi11otio11
62. To prepare the reagent used in confirmatory protein tesling . a laboratorian would:
a dissolve 3 g sulfos;ilicylic acid in 100 ml of water
b dissolve 5 g trichloroacelic acid in 100 ml or water
c combine 3 ml of hydrochloric acid and 9i ml or water
d combine 5 ml of glacial acetic acid and 95 ml of water
63. A positive result for bilirubin on a reagent test strip should be followed up by:
a notifying the physician
b requesting a new specimen
c performing an lctotest®
d performing a urobilinogen
64. A clear. red-brown urine specimen resulted in a positive reaction for blood on the reagent
test strip, but no RBCs were seen on microscopic examination. These resulls most likely
indicate the presence of:
a pyridium
b ascorbic acid
c porphyrins
d myoglobin
65. A urine tested with Clin itest® exhibits a pass-through reaction and is diluted by adding 2
drops of urine to 10 drops water. This is a dilution of:
a 1:4
b 1:5
c 1:6
d 1:8
66. While performing a routine urinalysis. the laboratorian notes a 2+ protein resull. They
should:
a request another specimen
b confirm with the acid precipitation test
c test for Bence Jones protein
d report the result obtained without further testing
67. The confirmatory test for a positive protein result by the reagent strip method uses:
a Ehrlich reagent
b a diazo reaction
c sullosalicylic acid
d a copper reduction tablet
68. A urine specimen is ana lyzed for glucose by a glucose oxidase reagent test strip and a
copper reduction test. If both results are positive, which of the following interpretations is
correct?
a galactose is present
b glucose is present
c lactose is not present
d sucrose is not present
69. A woman in her ninth month of pregnancy has a negative urine glucose result with the
~r reagent test strip, but a positive reaction with the copper reduction method. The reducing
substance most likely responsible for these results Is:
a maltose
b galactose
c glucose
d lactose

C'4Jl~ASCP ISBN 978-089189-6609 Clinical bbonitory Certification Exomln•tlons 93


2 : Urina ly s is & Body Fluids
:;;;--7-~-:--:----.:...__
Urinalysis: Microscopic Exa 111 1
_ .:.:..::_________-:----:---::-::-::--::-:-:::::=~--..::ar,o
;, .J
70. A~ urinalysis performed on a 2-week-old infant with d iarrhea shows a negative reaction ~
with the glucose oxidase reagent test s trip. A copper reduction tablet test should bo
performed to check the urine sample ror the presence of:
:i glucose
b galactose
c bilirubin
d ketones
71. When using the sulfosalicylic acid test. false-positive protein results may occur in the
l.'l$
presence of:
°"" a ketones
b alkali
c g lucose
d radiographic contrast media

....72. Which of the following is the primary reagent In the copper reduction tablet?
'"'" a
b
sodium carbonate
copper sulfate
c glucose oxidase
d polymerized dlazonium salt

Urinalysis: Microscopic Examination


73. In most compound light microscopes, the ocular lens has a magnification of:
a 10><
b 40><
c 50><
d 100><
74. The best way to lower the light intensity of the microscope is to:
a lower the condenser
b adjust the aperture diaphragm
c lower the rheostat
d raise the condenser
75. The advantage to using phase microscopy in urinalysis is to:
a provide higher magnification
b enhance constituents w ith a low refractive index
c allow constituents to stain more clearly
d provide a larger field of view
76. The presence of leukocytes in urine is known as:
a chyluria
b hematuria
c leukocytosis
d pyuria
77. Oval fat bodies are:
a squamous epithelial cells that contain lipids
b renal tubular epithelial cells that contain lipids
c free-floating fat droplets
d white blood cells with phagocytized lipids

94 The Board o f Certification Study Gulde Se


ISBN 978-089189-6609 fl2018ASCI'
2: Urinalysis & Body F luids Url11alysis: Microscopic Examination
78. A microscopi~ examination of urine sediment reveals ghost cells. These red blood cells
may be seen 1r1 unno with a:
a > 2% glucose concentrations
b specific gr;:ivity < 1.007
c large amounts of ketone bodies
d neutral pH
79. Glitter cells are a microscopic finding of:
a red blood cells in hypertonic urine
b red blood cells in hypotonic urine
c white blood cells in hypertonic urine
d white blood cells in hypotonic urine
so. What cell is most commonly associated with vaginal contamination?
a white
b transition al
c squamous
d glitter
81. A reagent test strip pad for blood has been reported positive. Microscopic examination
fails to yield red blood cells. This patient's condition can be called:
a hematuria
b hemogloblnuria
c oliguria
d hemosiderinuria
82. Ghost red blood cells are seen in urine that is:
a acidic and dilute
b alkaline and dilute
c acidic and concentrated
d alkaline and concentrated
83. An eosinophil count may be requested on urine from a patient with suspected:
IA.S
""" a acute glomerulonephritis
b cystitis
c renal lithiasis
d acute interstitial nephritis
84. Clue cells are a form ot.
a squamous epithelial cell
b urothelial cell
c white blood cell
d renal tubular epithelial cell
85. Which of the following cells is most likely to be seen in the urine sediment following a
catheterization procedure?
a squamous epithelial
b transitional epithelial
c white blood
d renal tubular epithelial

Clinical Laboratory Certification Examinations 95


2: Uri nalysi s & Body Fluids Urinalysis: Microscopic Examlna 110~l
86 · A oa11onl <1dm1llc d following an acc1dont involv111q m11sslve crush 1n1unes has lhe follow~
unnalysh; results g
rntcroscoplc findings
col6r redb~n
5 10
renal tubular eptlheha~
c1,., ly c:.lo••
, 011
renal tubu1ar ce I casts 1. ;i
spoc.f.c grnv1ty
pH 60
protein 1~

blood large
glucosr neg alive
ketonem negative
nilr1te negative
leukocytes negAtlve
b11irubln: nogatlve
u•oblllnogon· negatlve

The discrepancy between the ..large" result for blood on the reagent test strip and the
absence of RBCs on microscopy may be explained by.
a failure 10 mix lhe specimen before cerlnfuging
b mistaking R BCs for RTE cells
c contaminaling ox idizing detergents in lhe container
d the presence of myoglobin in the urine specimen
W hal is the most likely diagnosis given this microscopic finding?
....
87.
001.Y

a glomerulonephritis
b pyelonephrilis
c nephrotic syndrome
d cystilis

96 The Board of Certification St udy Gulde 6•


ISBN 978-089189-6609 ~·
2: Uri nalysis & Body Fluids
Urinalysis· Microscopic Examination
88 ldenhfy the formed •fem mt this photomrcrograph.
....
.....
11

• • ••
• • • ••
• •• • •
• • • •
.. • •
•• • •
• • ••

•• • • •
a RBC
b wee
c epi thelial cell
d yeast
89. All casts are typically composed of:
a albumin
b globulrn
c immunoglobulins G and M
d uromodulin
90. Hyaline casts are usually found:
a in the center of the coverslip
b under subdued light
c under very bright light
d In the supernatant
91. Which of the following casts rs most likely to l>e found in healthy people?
a hyaline
b red blood cell
c waxy
d white blood cell
92. Whrch of the following casts is most indicative of end stage renal disease?
a hemoglobin
b granular
c cellular
d waxy

...
93. A laboratonan performed a STAT microscopic urinalysis and reported the followrng .
""-' wee 10-13/HPF
RBC 2-6/HPF
hyai.ne casts 5-7/LPF
b•cteria 1+

The centrifuge tube was not discarded and the urine sediment was reevaluated
microscopically 5 hours after the above results were reported. A second laboratorian
reported the same results, except 2+ bacteria and no hyallne casts were found The most
probable explanation for the second technologist's findings Is:
a sediment was not agitated before preparing the microscope slide
b casts dissolved due to decrease rn urine pH
c casts dissolved due to increase in urine pH
d casts were never present in this specimen

C21ll9ASCP ISBN978-1169189-6609 Cllnlc•I L•oorarory Certification Examln•tlons 97


2: U r i nal y s i s & B o dy Fluids Urinalysis: Micros copic Exnminotio~
94 . Which of tho lollowing aids 1n differentiating a spherical transitional cell lrom a round rona1
tubulnr cell?
a Spherical transitional cell is larger
b eccentrically-placed nucleus in the renal tubular cell
c eccentrically-placed nucleus in the sphcriC<JI transitional cell
d round renal tubular cell is larger
95. The urine microscopic constituents that best differentiate between cystitis and
pyelonephritis are:
a w ees
b bacteria
c Rees
d wee casts
96. Epithelial cell casts are m ost indicative or:
..llS
O>ILY a glom erulonephriti s
b nephrotic syndrome
c tubular necrosis
d pyelonephritis
97. Granular casts found in the urine of a football player admitted to the hospital w ith a broken
leg occurring during the game can be the result of:
a excessive bruising
b strenuous exercise
c excess power drink inge stion
d bone fracture
98. Which o f the following casts most frequently appears to have a brittle co nsistency?
a hyallne
b granular
c waxy
d fatty
99. To distinguish between a clump o f W BCs and a W BC cast, it is i mportant to observe:
a the presence of free-floating W BCs
b a positive leukocyte reaction
c a positive nitrite reaction
d the presence of a cast matrix
100. Spherical transitional ep ithelial cells may b e confused w ith:
a ova l fat bodies
b renal tubular epith elial cells
c glitter cell s
d lym pho cytes
101.
Prior to reporting a red blood cell cast, it is Important to observe:
a free-noaling RBCs
b hyaline casts
c granular casts
d increased w hite blood cells

SS Tho Board of Cartlnc;ation St11rlv ,,., ,... _ n .


2: Urinalysis & Body Fluids
-
102. In a specimen with a large amount of bT
11
b.
Urinalysis: Microscopic Examination
would be most noticeably bile-stalncd? ru in. which of the following sediment consliluenls
a squamous epithelial cells
b while blood cell casls
c cyslinc crystals
d renal tubular epithelial cell casts
l03 The prese11ce of red blood cell casls
· Corre1ales best with which of the following conditions?
;::.:, a glomerulonephrilis
b pyelonephrilis
c nephrotic syndrome
d cystitis

104. A while precipitate in a urine specimen wilh a pH of 7.5 would most probably be caused
by:
a amorphous urates
b WBCs
c amorphous phosphates
d bacteria
105. Which of lhe following is an abnormal crystal described as a hexagonal plate?
a cysline
b tyrosine
c leuclne
d cholesterol
106. The primary component of most urinary calOJli is:
UIS
""' a calcium
b uric acid
c leucine
d cystine
107. After warming, a cloudy urine dears. This is due to the presence of:
a urates
b phosphates
c WBCs
d bacteria
108. Tiny, colorless. dumbbell-shaped crystals were found in an alkaline urine sediment. They
most likely are:
a calcium oxalate
b calcium carbonate
c calcium phosphate
d amorphous phosphate
109. Which of the following crystals may be found in acidic urine?
a calcium carbonate
b calcium oxalate
c calcium phosphate
d triple phosphate

t'21>18ASCP ISBN 9784:1891~609


Clinical Laboratory Ce.rtificatlon l!Kaminatlons 99
2: Urinalysis & Body Fluids :1
Uri1111/ysls: Microscopic Examina110
~ 10. Using polanzed light microscopy, wl11c.h of t"le following urinary elements are birefnngoni?
""'' a cholesterol
b triglycerides
c fatty acids
d neutral fats
111. Which of the following crystals appear as fine, silky needles?
a cholesterol
b leuc1ne
c hemosiderin
d tyrosine
112. Which of the following crystals is seen in an amber urine with a positive bilirubin?
a ammonium biurate
b cysllne
c tyrosine
d uric acid
113. Following ingestio n of ethylene glycol (anti'reeze) numerous crystals are found in th•
urine. The shape or these c rystals is:
a flat with notched corners
b oval/dumbbell
c coffin-lid
d rosettes/rhomboid
114. Cholesterol crystals will most likely be observed in urine that contains:
a 3+ glucose
b 4+ protein
c WBC casts
d triple phospha te crystals
115. The finding of a large amount of uric acid crystals in a urine specimen from a 6-month-old
"""
CIM.Y boy:
a may actually be d iaper fibers
b could indicate Lesch-Nyhan syndrome
c should not be reported
d may indicate improper feeding

....
116. The following crystal is found in an:

1
0


I
'

~ :~die pH and is considered nonpathologic


~ a~~:~n~~~:dn~ '~o~s7~~~:~e~ 3~~~ro~t~ologic
a aline pH and is conside red pathologic

100 Th B
e Oerd Of CertlncaUon Study Guide 6e
........-
2: Urinalysis & Body Fluids Urf111Jlysls: Microscopic Examlnat/011
117. Identify this crystal:
..

'
..
I
a uric acid
b calcium phosphate
c calcium carbonate
d triple phosphate acid

.......,
118. Alkaline urine showed this microscopic finding .

The laboratorian should:


a dilute with saline
b request a new sample
c culture ror bacteria
d dissolve with acetic acid
119. Polarized light can often be used to differentiate between:
a unc acid Cf)'Stals and cystine crystals
b hyaline and waxy casts
c squamous and transitional epithelial cells
d red blood cells and white blood cells
120. Which of the following contaminants has a dimpled center and will polarize?
a starch
b oil droplets
c air bubbles
d pollen grains

C~ASCP ISBN 978-089189-6609 Clinical Labo,.tory Certification Examln•tlon• 101


........
2: Urina lysis & Body Fluids Ut /1wly sls Mlct 011c; o11/1. l '"""'"' "' 011
121. The r>rcsanco of this elom,..nf n 11ru1~ 11ut1r:.ttt"i U l~ 11rcse11ca of, --.....

0
0 0 0

oe
Q
© (

0
a a UTI
b powder
c carbohydrate deficiency
d high urine amylase
122. Identify the formed element 1n this photomicrograph

a cloth fiber
b hyaline cast
c granular cast
d waxy cast
123. A laboratonan 1s having trouble differentiating between red blood cells, oil droplets and
yeas1 cells on a urine microscopy. Acetic acid should be added to the sediment to.
a lyse the yeast cells
b lyse the red blood cells
c dissolve the oil droplets
d crenate the re d b lood cells
124. A urine specimen is tested and has the follo wing results:
'"
°"1Y reagent strip
glucose 3+
microscopic findings
>100 WBCs/hpl
p1ote1n: 1+ many yeast cells
This is indicative of:
a diabetes mellitus
b contamination
c pyelonephntis
d diabeles insipidus

102 The Board of Certification Study Guide 6e


ISBN 978-089189·66-09 C20 16A!
-----
.. 2: Urina lysis & Body Fluids Urinalysis . Microscopic Examination
,
12s. When identifying urinary crystals. which reagent test strip result is most Important?
a protein
b pH
c specific gr,1vity
d nitrite
126. Bacteria are considered significant in the urine sediment when the:
a nitnte 1s positive
b protein is positive
c specimen is cloudy
d leukocytes are present

121. Which of the following exhibits rapid motility in urine sediment?


a spermatozoa
b Trichomonas vagina/is
c Garcfnerella vagina/is
d Enterobius vermicularis
128. Which or the following positive chemical reacllons is most closely associated with the
presence of yeast in the urine sediment?
a nltrile
b protein
c glucose
d blood
129. Which of the following reagent test strip pads would most likely yield a positive result
consistent with this urine microscopic finding?

..
•. ~·

" / '
a ketones
b glucose
c specific gravity
d nitrite
130. A 21 -year-old woman had glucose in her urine, but a normal fasting blood glucose. These
findings are most consistent with:
a renal glycosuria
b diabetes insipidus
c diabetes mellitus
d alkaline tide

WR IS8N9 9189-6609
2 = Urin a lysi s & Body F l u ids Urinal ysis: Comp ioto E x am/nor·.....,
'Ol'J I
Urinalysis: Comple te Examination
131. A 59-yea r-o ld man is evaluate d for back pain. Urin e s tudies (urlnelysis by multiple reagent
~' s trip) include:
urina lysis microscopic findings
specific gravity; 1.0 17 rare epithelial calls
pH: 6 .5 u rine p ro te in e l e ctrophor esis
protein: nogativo monoelonttl spike in gamma globulin re gion
glucose: negative
blood: negative

Which of the following statements best explains these results?


a urine protein is falsely negative due to the specific gr~vity ..
b urine protein is falsely negative because the method 1s not sens1t1ve for Bence Jones
protein . .. .
c m icroscopic examin ation is fa lsely negative du e to the spec1f1c gravity
d eleclrophoresis is incorrect and should be repeated
132. The results of a urinalysis on a firs t morning specimen are:
spoc iric gravily: 1.024 microscopic fin dings
pH: 8.5 uric acid c rystals
protein: negative
glucose: negative

The next step is to repeat the:


a microscopic examina tio n
b p rotein and g lucose
c specific gravity
d p H and microscopic examination
133. The following urinalysis results were obtained from an 18-year-old woman in labor:
pH : 6 .5
protein: 30 mg/dl
glucose: 250 mgidl
ketones: negative
billrubin: small (color slightly abnormal)
blood: negative
nilrite: negative
urobilinogen: 0.1 EUfdl
specific gravity: 1 .025
copper reduction test: 1.0 gfdl

W h ich.of the following is th e most lik ely explan a tion for the patient's positive copper
reduction test?
a only g lucose is p resent
b only lactose Is p res ent
c glucose and possibl\f:' ? l her reducing s ubsta n ces/sug a rs are present
d results a re false-positive due to the presence of p rotein

1 04 The Boa rd of Certlncation Study Gulde 6o


ISBN 978-089169-6609 C:
. z: urinalysis & Body Fluids
- Th following urine resull . Urinalysis: Complete Examination
t34. e s were obtained on a 25
"'' -year-old remale:
,,._, pH: 7.0
color: yo flow mlcroscopfc rindlngs
appearanc,o: cloudy bacteria: mAny
ptotefo: l• wee casts: 0-3/tpl
WBC/hpl: 30-40
glucose: negallvo
blood: small
specific gravity: 1.015

These results are most compatible with:


a glomerulonephritis
b renal calculus
c cystitis
d pyelonephritis
135. A urinalysis performed on a 27 -year-old wom 3 n y1·elds th e ro11owing
· results:
....
O<.' specific gravity: 1.008
microscopic findings
pH: 5.0 WBC/hpl: 10-15
protorin: 2+
RBCfhpl: 30· 55
glucose: negative
casts/lpr: hyaline, 5. 7; Rec. 2· 5:
ketones: negative
granular. 2-3
bilirubfn: negative uric actd crystars: moderate
blood; 3+
nitrite: negative
leukocytes: positive
uroblllnogen: 0.1 EU/dl

These findings are most consistent with :


a yeast infection
b pyelonephritis
c bacterial cystitis
d glomerulonephritis
136. A 62-year-old patient with hypertipoproteinemia has a large amount or protein in his urine.
Microscopic analysis yields moderate to mar.y fatty, waxy. granular and cellular casts.
Many oval fat bodies are also noted. This is most consistent with:
a nephrotic syndrome
b viral infection
c acute pyelonephritis
d acute glomeru!onephritis

C/in/cal Labonitory Catllflcatlon Examinations 105


~

2: Urinalys is & Body Fl uids U1/11a fys l s: Co 111plofo cx;1111111,, 110 ,.,
137. A patient has 2 separale urinalysis reporls. which co111nl11 the followiny uot;:i: --...._
Tes t Report A Roport O
spcclllc ornvily 1,004 1.017
pH 5.5 7.0
prololn ncgntlvo 1 ..
glucose ncg:itivo r\egall•10
bJood npgaUvo small
microscopy rare epithelial ceHs
1- 2 granular cast/Ip!
1-3 hyaline cas111pr
modcral e cpilhelial cells

Wh ich of the following sla tements best explains these results?


a protein, glucose and microscopy of A are false-negatives because or the specific !Jravity
b protein and glucose are false-positives in B due to the specific gravity
c microscopic of A is false-negative because o f th e pH
d microscopic of B is false-positive because of the pH
138. A 4-year-old girl develops edema following a recent immunization. Laboratory studies
r.u
O'ii:.''
reveal:
serum albumin: 1.8 g/dl (18 g/L)
serum cholesterol: 450 mg/dl (11 .66 mmol/L)
serum urea nitrogen: 20 mg/dl (7.14 mmoVL)
urinalysis: proleln 4+; hyaline. granular and rauy casts

These findings are most compatible with:


a acute poststreptococcal glomerulonephritis
b minimal change disease
c acute pyelonephritis
d diabetes mellitus
139. A specimen with a negative nitrite reaction and a positive leukocyte esterase reaction that
has WBCs, WBC casts, but no bacteria in the sediment will be seen In ca ses of:
a cystitis
b pyelonephritis
c acute interstitial nephritis
d acute glomerulonephritis
140. Urinalysis results on a female patient who brings a u rine specimen to the physician's office
~. for her annual physical are:
color: yellow micros cop ic findings
cl arity: cloudy squamous epllhelial cells: moderate
specific gravity: 1.020 WBC/hpr: 0- 2
pH; 7.0 bacteria: heavy
protein: trace
glucose: negativo
ketones: negative
blood: negative
bilirubin: negative
urobilinogen: 0.2 mg/d L
nitrite: positive
leukocytes: negative

What act ion shou ld be taken?


a place the p~tient on a broad spectrum antibiotic for 7 da s
b ask the patient to collect another specimen t th ffl Y
c tell the patient she will be called wh a e o ce
d have the patient return in a week w'ethn the culture and sensitivity reports are back
1 a new specimen

106 Tho Board of Cortlflcatlon Study Guido 6e


ISBN 978·089189-6609 Cl2018ASCP
2: Urinalysis & Body Fluids
Urinalysis: Complete Examination
141. A palient with lupus erythematosus has th f 11 . . .
e o owing unnalysos results:
"'
""' colot: red
m icroscopic findings
clarity: cloudy
specific gravily: 1.011 WBC/hpf: 5 -10
RBC/hpf: 40-50
pH: 6.0
casls/lpf: hyaline. 2·4: RSC, 3· 5
protein: 3+
glucose: negative
ketones: negative
blood: large
bilirubin: negative
urobilinogen: 1.0 mg/dL
nitrite: negative
leukocytes: trace

These results would be associate<! with:


a chronic glomeru lonephritis
b chronic pyelonephritis
c acute interstitial nephritis
d acute tubular necrosis
142. The sediment of a urine specimen with a reagent strip glucose of 250 mg/dl (13.8 mmol/L}
and a pH of 5.5 is ideal for the presence of:
a cystine crystals
b Trichomonas vagina/is
c Candida albicans
d ammonium biurate crystals
143. A patient with severe back pain has the follo'Ning urinalysis results:
color: dark yellow microscopic findings
clarity: hazy RBC/hpf: 10·20
specific gravily: 1.030 squamous epilhelial cells: moderale
pH: 6.0 calcium oxa1a1e cryslals: moderate
protein: 1tace
glucose: negative
kelones: neg alive
blood: small
bilirubin: negative
urobilinogen: 0.4 mg/dL
nnrite: negative
leukocytes: negative
In addition to the presence or blood, what other reagent test strip result relates to the
patient's symptoms and likely diagnosis with renal calculi?
a specific gravity
b pH
c protein
d urobilinogen

Cllnlcal Laborwtory certification Examinations 107


tl71)la ASCP ISBN Q78-()811189-Wl9
2: U rina lysis & Body F l uids
Urine Physiology
·rn tely·
144. The normnl renal threshold for glucose in the adult Is approx• a ·
-
Urlrro Pllys/o/og~ •

a 50 mg/dl (2.8 mmol/L)


b 100 mgldl (5.5 mmolll)
c 160 mg/dl (8.8 mmol/L)
d 300 mg/dl {16.5 mmolll) 3
145. The volume of urine excreted in a 24-hour period by an adu lt patient w as oo ml. This
condillon would be termed:
a anuria
b oliguria
c polyuria
d dysuria
146. A patient has glucosuria. hyperglycemia and polyuria. These findings are most consistent
with:
a renal glucosuria
b diabetes mellitus
c emotional stress
d eating a heavy meal
147. The normal glomerular filtration rate is:
a 1mllmin
b 120 ml/min
c 660 ml/min
d 1,200 ml/min
148. Normal urine primarily consists of:
a water, protein and sodium
b water, urea and protein
c water, urea and sodium chloride
d water, urea and bilfrubln
149. An abdominal fluid is submitted from surgery. Th e physician wants to determine if this fluid
could be urine. The laboratorian should:
a perform a culture
b smell the fluid
c test for urea and creatinlne
d test for protein, glucose and pH
150. Arginine vasopressin (antldluretlc hormone [ADHJ) regulates the reabsorption of:
1.11.S
ONl.Y a water
b glucose
c potassium
d calcium
151. Which of the following components would be present in the filtrate as a result of glomerulai
damage?
a glucose
b amino acids
c nonprotein nitrogen compounds
d large molecular weight proteins

108 The Board of Certiflc.a tlon Study Gulde 6e


ISBN 978·089189-6609 ©2018.ASCI
2: Urinalysi s ~ Body Fluids
Urine Physiology
1s2. Polyuria is usually correlated with:
a acute glomerulonephritis
b diabetes mellitus
c hepatitis
d lubular damage

153. Cessation of urine flow is defined as:


a azotemia
b dysuria
c diuresis
d anuria

154. The reason for performing a Clinites~ on a newbom's urine is to check for:
a fructose
b galactose
c glucose
d lactose
155. Ketones in urine are due to:
a complete utilization of fatty acids
b incomplete fat metabolism
c high carbohydrate diels
d renal tubular dysfunction
156. Reagent test strip pads for ketones primarily measure:
a acetone
b acetoacetic acid
c cholesterol
d beta-hydroxybutyric acid
157. Bilirubinuria may be associated with:
a strenuous exercise
b increased destruction of platelets
c viral hepatitis
d hemoly1ic anemia
158. Myoglobinuria is most likely to be noted in urine specimens from patients with which of the
~' following disorders?
a hemoly1ic anemia
b lower urinary tract infection
c myocardial infarction
d paroxysmal nocturnal hemoglobinuria

...
159. A patient with renal tubular acidosis would most likely excrete a urine with a:
..., a lowpH
b high pH
c neutral pH
d variable pH
160. Glycosuria may be due to:
a hypoglycemia
b increased renal threshold
c renal tubular dysfunction
d increased glomerular filtration rate

t?rll!llSCP IS8N978-08918!).6609 Clinical labontrory CertificaUon Examinations 109


2: Uri nalysis & Body F luids
;;::-::::-~.:..._~~~~.::....~:....:...:.:...::_~~~~~~~~~~~~~~~~~~~-- ~
Otltor Body Flu/ l
161 . T ho <1ro<1 of tho nephron th at is impermeable lo water Is the: ---._
s
1·..
0~1-V
a pro xima l convoluted tubule
b d escending loop of Henle
c ascending loop of Henle
d d istal convoluted tubule
162. The urinary tract structures respo nsible for renal concentration are the:
I/LI;
0'1.Y
a renal pelvis
b cortical nephrons
c renal papillae
d )uxtamedullary nephrons
163. The most accurate test to determine renal concentration is:
a osmolarity
b glomerular filtration rate
c specific gravity
d tubular reabsorption rate
164. Failure of the nephron to produce ammonia from glutamine will result in urine witt: a:
""'
o"'v a low specific gravity
b positive nitrite
c high pH
d positive protein

Other Body Fluids


165. To avoid falsely elevated cerebrospinal fluid cell counts :
a use an aliquot from the first tube collected
b use only those specimens showing no turbidity
c centrifuge all specimens before counting
d select an aliquot from the last tube collected
166. A turbid cerebrospinal fluid is most commonly caused by increased:
"""
°"'' a
b
white blood cells
protein
c g lucose
d bacterial organisms

....167.
ONIY
The normal concentration of prote ins in cerebrospinal Ouid, relative to serum protein, is:
a < 1 mgldL
b 5-10 mgldL
c 25-35 mgldL
d 50-60 mg/dL
168. T<? prepare the reagen t used for mucin dot determination of synovial fluid water is mixed
with: '
a hydrochloric acid
b sodium hydroxide
c sodium chloride
d glacial acetic acid

11 0 The Board of Certification Study Gulde 6e


ll>RN 978-089189-6609 ¢20l!i\.S(1
: Urinalysis & Body Fluids
-,,..,
2 Otllcr Body Fluids

159. In addition to the sperm count in a fertility study. analysis of seminal fluid should also
"'' include:
a time of liquefaction. estimation of motility morphol
b motility•.morp.ho~ogy, test for al.kaline pho'sphataseogy
c time of hqucfdct~on, test for acid phosphatase , qualitative test ror hemoglobin
d time of hquefaction, quahtatlve test for henwglobin and motility

170. The following lab values were obtained on a body fluid sample:
,,..,
"'' protein: 3 gldL (30 g/L)
albumin: 2 .1 g/dL (21 OIL)
hyaluronato: 0.4 gldl (4 g/L)
glucose: 80 mgldl (4.4 mmol/L)
lactate: 10 mg/dl (1.1 mmol/L)

The sample is:


a pleural fluid
b synovial fluid
c urine
d cerebrosplnal fluid
171. A physician attempts to aspirate a knee joint and obtains 0.1 ml of slightly bloody fluid .
.u Addition of acetic acid results in turbidity and a clot. This indicates that:
ONtY
a the fluid is synovial fluid
b plasma was obtained
c red blood cells caused a false-positive reaction
d the specimen is not adequate
172. Synovial fluid is analyzed with a polariZing microscope. Strongly birefringent needles are
,.. seen. This most likely indicates:
"" a monosodium urate crystals
b calcium pyrophosphate crystals
c corticosteroid crystals
d talc crystals
173. A sperm count is diluted 1 :20 and 50 sperm are counted in 2 secondary squares of the
~. Neubauer counting chamber. The sperm count is:

a 5,000/ml
b 50,000/ml
c 500,000/ml
d 5,000,000/ml
174. The principal mucin in synovial fluid is:
lU
"" a hyaluronate
b albumin
c: orosomucoid
d pepsin
175. The synovlal fluid easily forms small drops from the aspirating syringe. This viscosity is:
""
"" a normal
b increased
c associated with inflammation
d associated with hypothyroidism

Clinical L•bor.ttoty CertificaUon Examln•tlons 111


J
~
2: Urinalysis & Body Flu ids Other Body Ftu·
;:;;-~~--:~~~~~.:.....~~~~~~~~~~~~~~~~~-:-~~~~~....:...--:'Us
176. Pleural transudates differ from pleural exudates In thot transudates have: ._
lts
"'" a Ouid:serum protein ratio >0.5
b pleural Ouid:serum cholesterol >0.3
c Ouid:serum LO ratio >0.6
d a WBC count <1000/µL
177. Pleural flu id from a patient with congestive heart failure would be expected to:
"''l.$
ON\Y
a contain bacteria
b have a high protein content
c be purulent
d appear clear and pale yellow
178. Monosodium urate (M SU ) and calcium pyrophosph~t~ dehydrate (CPPO) crystals can be
....
..... distinguished by using a red compensator in a polarizing f!11croscope. When the crystal is
aligned with the slow vibration or the compensator. which is true?
a M SU are blue
b MSU are yellow
c CPPO are blue
d corticosteroids are yellow
179. False-positive results can occur for fecal occult blood due to the ingestion o r
a ascorbic acid
b horseradish
c acetaminophen
d blueberries
180. The chromogen for the fecal occult blood test is:
a gum guaiac
b NADH
c o-toluidine
d p-aminocinnamaldehyd e
181. A build up o f fluid in a body cavity is called a(an):
a effusion
b transudate
c exudate
d metastasis
182. A fluid sample was collected by thoracentesis. A serum sample was coll ected immediately
afterward. !he LO flu id to serum ratio was 0 .9 . There were 5,000 WBC/µL, with 75%
PMNs. Which of the following describes this fluid?
a pleural effusion exudate
b pericardia! effusion exudate
c pleural effusion transudate
d pericardia! effusion transudate
183. Ascites is collected by:
a thoracentesis
b lumbar puncture
c amniocentesis
d paracentesis

112 Tho Board of Certification Study Gulde 6e


ISBN 978.()89189-6609 C201B¢
> :z: urinalysis & Body Fluids Oiiier Body Fluids
184. Amniotic fluid may b? tested for the concentra:ion of lamellar bodies. This test is
performed to determine:
a fetal lung maturity (FLM)
b hemolytic disease or the newborn (HON)
c neural tube defects
d congenital birth defects

18s. Amniocentesis should be performed to:


a screen for Down syndrome
b to confir~ a ~igh maternal serum alpha-retoprotein (MSAFP)
c to test b1hrubm levels for an Rh-positive mother
d test folic acid levels in fetal blood
l86. A sweat chloride >60 mEq/L (60 mmol/L) Is indicative of:
a multiple sclerosis
b muscular dystrophy
c respiratory distress syndrome
d cystic fibrosis
187. The most common genetic defect associated with cystic fibrosis is called:
a CFTR delta-F508
b trisomy 2 1
c Philadelphia chromosome
d fragile X
188. The presence of oligocional bands in the CSF but not in the serum is associated with:
....
...y a spina bifida
b hydrocephalus
c Reye syndrome
d multiple sclerosis
189. Normal CSF contains all of the following proteins except:
....
ctlY a translerrin
b transthyretin
c albumin
d fibrinogen
190. A CSF was collected from a 5-year-old with a fever, and 3 tubes were transported to the
lab. Tube 1 had 50.000 RBC/µl and 48 WBCiµL. Tube 3 had 10 RBC/µ L and 0 WBC/µl.
What is the most likely explanation for the dis~repancy?
a tube 3 was QNS
b bacterial meningitis
c subarachnoid hemorrhage
d traumatic tap
191. An Increased lgG index on a CSF specimen 11dicates:
"'
- a antibody response to bacteria
b synthesis of lgG in the CNS
c brain tumor
d breach of the blood brain barrier
192. The appearance of normal CSF Is:
a pale yellow and clear
b ootortess and clear
c opalescent
d xanthochromic

Cl013ASCP ISBN 97a-089189-6609 Clinical Laboratoty Certification Exeminalion& 113


2: Urina lys i s & Body Fluids Oiiier Body ~
::::--7"".=-::--::-~~~-=-..:. .: :....:...:...:.:..:..:::.::_~~~~~~~~~~-:::-~-.-::::-=--:-:'."'."::-:-:::-""7""~F...:.:.:
'"•i•aas• 1
193. A CSF was hazy and the WBC wos too high to perform undiluted. The technologist took'
SO µL of sample and added 500 µL of saline. The cell count on the d iluted sample was
WBC per µL. This should be multiplied by: 200
a 10
b 11
c 1/10
d 1111
194. The finding of hemosiderin laden macrophages in a CSF sample indicates:
a bacterial infection
b viral Infection
c previous hemorrhage
d traumatic tap
195. Which CSF results are most consistent with bacterial meningitis?
CSF sample Glucose Protein Lactate
A 20 mg/dL (1. 1 mmol/L) 50 mg/dl (500 mg/L) increased
B 75 mgldl (4.1 mmol/L) 20 "'9fdl (200 mg/L) increased
c 20 mgtoL (1. 1 mmo4/L) 45 mg/<IL (450 mg/L) deaeased
0 75 mgldL (4.1 mmol/L) 120 "'9/<IL (1,200 mg/L) decreased

a sampleA
b sample B
c sample C
d sample D
196. Which of the following is the best indicator of Reye synd ro me for CSF (hepatic
encephalopathy)?
a glutamine
b ammonia
c ALT
d bilirubin
197. The tau isofonn of transferrin is a carbohydrate deficient protein found only in:
a CSF
b sweat
c amn iotic fluid
d semen
198.
Which marker can be used to identify if a body flu id contains semen?
a acid phosphatase
b alkaline phosphatase
c fructose
d hyaluronic acid
199,
Which stain is used to measure sperm viability?
a eosin nigrosin
b Wright
c toluidine blue
d Papanicolaou

114 The Board of Cortificatlon Study Guido


68
z: Urinalysis & Body Fluids
O ther Bocly Flu/els
200. The dimensions of a standard Neubaue h
r emc-cytometer are·
a 3 mm x 3 mm x 0. 1 mm ·
b 1 mm x 1 mm x 10 mm
c 3 mm x 10 mm x 1 mm
d 1 mm x 1 mm x 0.3 mm
201. Rapid forward progression of sperm is rated as:
a 1.0
b 2.0
c 3.0
d 4.0

202. Laboratory characteristics of malabsorpti


.u include: on s~ndrome due to pancreatic insufficiency
' '" a increased fecal fat
b fecal leukocytes
c positive Clinitest®
d fecal occult blood
203. Pilocarpine iontophoresls refers to the specific process of:
a inducing sweat
b separating proteins in CSF
c measuring ions in sweat
d measuring pilocarpine in CSF

204. During sweat collection. a consideration that can result in a falsely high result is:
\l.!I
'" " a high ambient temperature
b evaporation
c preparation of area with type 1 water
d high sweat rate

205. Qualitative methods used as screening tests for cystic fibrosis include:
a coulometric Cl measurement
b Cl selective electrodes
c sweat conductivity
d pilocarpine iontophoresis
206. Which pair does not match with respect to amniotic ftuld ?
a colorless-norma l
b dark red-brown- fetal dealh
c dark green-hemolytic disease of the newborn
d blood-streaked- traumatic collection
207. Amniotic ftuid is evaluated using a Liley graph and change in absorbance at 450 nm. What
is being evaluated, and why?
a bilirubin, which increases in HON
b AFP, which increases In splna bifida
c hCG, which increases In Down syndrome
d lamellar bodies, which increase with fetal lung maturity
208. Which assay for fetal lung maturity using amniotic fluid may be performed using the
~i, platelet channel of an automated hematology analyzer?
a phosphatidylglycerol
b US ratio
c lamellar body count
d bilirubin

0"418ASQ> ISBN 978-089189-6609 Clln/cal Laboratory Certification Examinations 115


Other B~
:~:.:..:.:.'..::~~~~~~~~---:-:--:-~;;-:-:;::;:-;::-;;::;;:i~;;;:;:-----~Ff111~.
2: Urinalysis & Body Fluids
. . . .Ii d · body Ouids by their ability to: --...__·t
l
209. Tnglycendes (chyle) can be 1dent11e in '-

a polarize light
b stain with Sudan 111
c sediment upon standing
d glitter
210. Peritoneal lavage is used to:
a detect intra-abdominal bleeding in blunt injury
b dialyze patients with end stage renal disease (ESRD)
c replace ascites with saline
d perform therapeutic thoracentesis
211. Acetylcholinesterase activity may be measured on amniotic fluid when a positive alpha.
fetoprotein result is obtained to evaluate for:
a respiratory distress syndrome
b hemolytic disease of the newborn
c fetal lung maturity
d neural tube defects
212. W hich semen result would be considered abnormal?
a sample pours in droplets after 60 minutes
b >50°/o are motile within 1 hour of collection
c pH 7 .5
d motility grade of 1.0
213. Increased CSF lactate is found in:
a bacterial meningitis
b Reye encephalopathy
c spina bifida
d multiple sclerosis
214. Decreased CSF protein can be found in:
a meningitis
b hemorrhage
c multiple sclerosis
d CSF leakage
215. What calculation is used to determine if there is a breach in the blood-brain barrier?
a lgG index
b CSF/serum albumin index
c fluid/serum LO ratio
d albumin gradient
216. Results from a cloudy, yellow pleural fluid collected from a 56-year-old male are listed
below:
wee count 1550/µL (predominate neutrophils)
Fluid/serum blllrubin: 0.6
Fluid/serum 101$1 protein: 0 .9
Fluid/serum LO: 0 .7

These results indicate the fluid is most likely:


a a transudate
b an exudate
c chylous
d pseudochylous

116 lho Boa,.d o f Cartificalion Study Guido 6o lS8N978-0891~9 Q20IS¢


p
: Urinalysis & Body Fluids Oiiier Body Fluids
.,
2
ii7. Which of the following would be evidence of a sucx:essful vasectomy?
3
seminal pH > 7 .8
b tess than 1 ?%
of sperm with forward progression
c azoosperm1a
d head-to-head agglutination

218. uamniotic fluid is to be collected for fetal lung maturity testing, amniocentesis should be
performed at:
a 1-5 weeks' gestation
b 6· 10 weeks' gestation
c 14· 18 weeks' gestation
d 30·42 weeks' gestation

219. An acholic or pale/clay-colored stool is characteristic of:


3 a posthepatic biliary obstruction
b malabsorption syndrome
c steatorrhea
d maldigestion

220. The opening p_ressure for a CSF collection is low, so only 1 ml of fluid is collected into
a labeled, steri le tube and sen t to the laboratort- Which or the following tests should be
performed first?
a manual cell count
b Gram stain and culture
c glucose
d total protein
221. Results from a wee differential pe1furmed on CSF from an adull patient suspected of
having meningitis are listed below:
neutrophils: 3%
lymphocytes: 62%
monocytes: 23%
eoslnophils: 12'4

These results are highly suggestive of:


a bacterial meningitis
b viral meningitis
c fungal meningitis
d tubercular meningitis
222. A vaginal swab is received in the laboratory for testing and immediately placed in sterile
physiological saline to prepare a suspension. A wet mount is prepared and the laboratorian
reports the presence or numerous clue cells. This most likely indicates the patient has:
a bacterial vaginosis
b candidiasis
c trichomoniasis
d atrophic vaginits
223. Results from a milky peritoneal fluid demonstra:e an elevated triglyceride content and the
presence of chylomicrons. These results indicate the fluid is most lik~y:
a a transudate
b an exudate
c pseudochylous
d chylous

L CllOl&ASCP ISBN 978-089189-6609 Ctlnlcal Labor1tory Certification Examinntlons 117


2: Urinalys is & Body Fluids
~~~~....::....~~~~~=-~~~~~~~~~~~~---:~~::-~:;---;:-~~~~~'...:""00//ttfis-
Bod~1
Other
224. Which of the following may be measured in CSF to evaluate the effectiveness o f treal;;;----
for multiple sclerosis? ent
a bilirubin
b lactate
c myelin basic protein
d glutamine
225. Amniotic fluid is collected from a 24-year-o ld female a t 33 we_eks' gestation and sent to th
laboratory for analysis. The specimen arrives in a clear. plastic container that has been e
exposed to light for over an hour. Which of the following analyses may be affected by this
error in specimen transport?
a lamellar body count
b alpha-fetoprotein
c ti450 determination
d foam stability index
226. Synovial fluid is typically collected using a sterile needle and syringe and then transferred
to collection tubes for testing. Which of the fol lowing anticoagulants wou ld be appropriate
to use for the aliquot sent for a manual cell count and crystal evaluation?
a liquid EDTA
b sodium polyanethol sulfonate
c sodium fluoride
d lithium heparin
227. Which of the following sample preparaticns is appropriate to use for evaluation of sperm
agglutination?
a dried smear stained with Papanicolaou stain
b fixed smear stained with eosin-nigrosin stain
c smear stained with Wright stain
d wet preparation
228. A xanthochromic CSF specimen is cen trifuged resulting in a pink-colored supernatant.
This indicates the presence of:
a red blood cells
b oxyhemoglobin
c bilirubin
d methemoglobin
229. A wet preparation is made from a fresh fecal specimen and stained with Wright stain. The
laboratorian reports the presence of 3 -5 WBC per high power field noting that the cells are
neutrophils. These results indicate the patient most likely has:
a a posthepatic biliary obstruction
b a noninflammatory condition such as malabsorption
c an inflammatory condition such as ulcerative colitis
d malabsorption syndrome
230. pH determinations are clinically relevant for which of the following?
a cerebrospinal fluid
b pericardia! fluid
c pleural fluid
d synovial fluid

118 Tho Board of Certification Study Guido 6e


ISBN 978--039169-66()9 Q2018ASCI'
p 2
: Urinalysis & Body Fluids Other Body Fluids
--; A green-colored .amniotic fluid is received in the laboratory for testing. The color of this
23 · specimen most likely ind1c<1tes tile presence of:
a bilirubin
b meconium
c blood
d hemoglobin

23 2. A white blood cell differential .is performed on CSF from an adult patient suspected of
having memng1t1s. The following results were reported:
neutrophils: 89%
lymphocytes: 7%
monocytes: 3°/o
eosinophils: <to/o

These results are highly suggestive of:


a bacterial meningitis
b viral meningitis
c fungal mening itis
d tubercular meningitis

233. synovial fluid from a ?8-year-old male reveals rhombic crystals with weak positive
birefringence when viewed using polarizing microscopy. These crystals can be identified
as:
a cholesterol
b monosodium urate
c calcium pyrophosphate dihydrate
d hydroxyapatite
234. Evaluation of sperm morphology is performed by staining an air-dried smear with Wright,
Giemsa, or Papanicolaou stain and evaluating 200 sperm using:
a a 4x objective (40• magnification)
b a 1ox objective (100x magnification)
c a 40x objective (400x magnification)
d oil immersion and a 100x objective (1000x magnification)
235. An undiluted CSF specimen is loaded onto a Neubauer hemocytometer and the following
results are recorded after counting alt nine 1.0 mm2 quadrants on both sides:
WBC. side 1: 100
WBC. side 2: S5

The laboratorian should:


a report the average W6C/µL from side 1 and 2
b report the difference in WBC/µL from side 1 and side 2
c report the sum of WBC/µL from side 1 and 2
d clean and reload the hemocytometer
236. An amniocentesis is performed on a 32-year-old female at 17 weeks' gestation. The
specimen is received in the laboratory and the clinician wants to confirm the specimen
is amniotic fluid , not urine. Wh ich of the following tests would be the most helpful in
distinguishing these 2 fluids?
a glucose, protein, urea, and creatinine
b glucose, bilirubin, urea, and AST
c protein, creatinine, AST, and ALT
d protein, bilirubin, osmolality, and creatinine

OlOlaASCP ISBN 978-089189-6609 CPnlcal Labotatory Certification Examinations 119


2: Urina lysis & Body Fluids
• .
Oiiier Body ~
Ffu1u, . . .
23 7. The presence of rice bodies in a synovial fluid is strongly associated with: ~
a gouty ar1hntls
b rheumatoid ar1hritis
c infection with Staphylococcus oureus
d traumarrc collection
238. A positive amine or "whiff' test on a vaginal secretion most likely indicates:
a candidiasis
b trichomoniasis
c proliferation of Gardnerella vagina/is
d proliferation of Lactobacillus acldoph"Jus
239. The presence of which of the following may result in a CSF specimen with an oily
appea rance?
a radiographic oonlrast media
b neutral triglycerides
c >500 WBC/µL
d 30 mgfdl total protein
240. Which o f the following studies is performed by aspirating seminal fluid with a Pasteur
pipette and observing the formation or droplets as it is allowed to fall under only the
In fluence of gravity?
a concentration
b motility
c vitality
d viscosity
241. An US ratio of 2.4 on amniotic fluid collected at 34 weeks' gestation indicates:
a increased risk of neu ra l tube defects
b increased rtsk for RDS
c fe tal lung m aturity
d fetal lung Immaturity
242. To qua litatively aid in differentiating malabsorption and maldigestion, 2 slides are made
from the stool specimen, pretreated with ethanol (slide 1) or a cetic acid (slide 2), and
stained with :
a eosin-nigrosin
b oil red O
c safranin
d melhylene blue
243. M easurement of which of the following tumor markers may be useful in evaluati ng pleural
or peritoneal effusions for malignancy?
a CEA
b AFP
c hCG
d CA 125
244. Increased presence of fetal fibroneclion {fFN) in cervicovag inal secretions from a 24-year·
old female at 35 weeks' gestation indicates a(an):
a increased risk for hemolytic disease of the newborn
b decreased risk for hemolytic disease of the newborn
c increased risk fo r preterm delivery
d decreased risk fo r preterm delive ry

120 Tho Board of Certification Study Guide 6o ISBN 978-069159-6669 020lS¢


d
2: Urinalysis & B ody Fluids
- · f · Otl1er Body Fluids
245• :;~c~~ ~~~~~~~r~~f CSF results would strongly suggest damage to the blood-brain-
a total protein of 40 mg/dl
b CSF/serum albumin index of 17
c lgG index or 0.4
d CSF/plasma glucose of 0.6

246. Which of the following results would be co ·d


. . . nsi ered abnormal for seminal fluid?
a liquefaction time >60 minutes
b pH of 7.6
c 25% sperm with normal morphology
d 65% sperm with rapid linear progression
247. A phosphatidylglycerol immunochemical lid t
amniotic fluid collected at 30 week • s . e est. shows no visible agglutination for an
s gestation. This should be reported as:
a negative
b low positive
c high positive
d invalid
248. A paired fasting plasma specimen was collected at the same time an arthrocentesis was
performed. The difference in glucose concentrations between the 2 fiuids was reported as
55 mg/dl. This result indicates a(an):
a noninflammatory condition
b inflammatory condition
c septic condition
d hemorrhagic condition
249. A 1O ml suspension, in water, is made from a bloody stool sample collected from a
neonate. The specimen is centrifuged and the resulting pink supernatant transferred in
equal volumes to 2 tubes. The first tube serves as a reference while the second tube is
alkalinized with 1 ml of 0.25 M sod ium hydroxide. The second tube changes to a yellow
color within 2 minutes. Th is reaction indicates the presence of:
a fetal hemoglobin (HgbF )
b maternal hemoglobin (HgbA)
c fetal white blood cells
d maternal while blood cells
250. In synovial fluid . the most characteristic microscopic finding in gout is:
a calcium pyrophosphate crystals
b cartilage debris
c monosodium urate crystals
d hemosiderin-laden macrophages
251. In synovial fluid, the most characteristic finding in pseudogout is:
a calcium pyrophosphate dihydrate crystals
b cartilage debris
c monosodium urate crystals
d hemosiderin-laden macrophages

Cl/n/cal L•boratory certificati on Examination• 121


2 : Urinalysis & Body Fluids
,
O t11cr Body Ft :i
UlcJ~
d a 157. c 209. b

2. a
53. d 105.
158. c 21 0. a
54. b 106. a
3. b 107. a 159. b 211 . d
55. a 212.
4. d 56. b 108. b 160. c d
5. c 109. b 161. c 213. a
57. c 214. d
6. b 58. a 110. a 162. d
7. 111. d 163. a 215. b
b 59. b
c 164. c 216. b
8. c 60. c 112.
217. c
9. d 61 . a 113. b 165. d
114. b 166. a 218. d
10. b 62. a
11 . c 63. c 115. b 167. a 219. a
116. b 168. d 220. b
12. c 64. d 221 . c
13. b 65. c 117. a 169. a
14. a 66. b 118. d 170. b 222. a
119. a 171. a 223. d
15. a 67. c 224. c
16. c 68. b 120 . a 172. a
69. d 121. b 173. d 225. c
17. b 226.
18. c 70. b 122. a 174. a ::!

71. d 123. b 175. c 227. cl


19. b d 228. b
72. b 124. a 176.
20. b d 229. c
73. a 125. b 177.
21. b b 230. c
74. c 126. d 178.
22. c 179. b 231. a
23. a 75. b 127. b
76. d 128. c 180. a 232. a
24. c 181. a 233. c
25. d 77. b 129. d
78. b 130. a 182. a 234. d
26. b 235. d
79. d 131. b 183. d
27. c 184. a 236. a
28. b 80. c 132. d
81. b 133. c 185. b 237. b
29. a 186. d 238. c
30. a 82. b 134. d
31. c 83. d 135. d 187. a 239. a
84. a 136. a 188. d 240. d
32. b
33. c 85. b 137. a 189. d 241 . c
86. d 138. b 190. d 242. b
34. b
35. b 87. d 139. c 191. b 243. a
36. c 88. a 140. b 192. b 244. c
37. c 89. d 141. a 193. b 245. b
38. a 90. b 142. c 194. c 246. a
39. b 91 . a 143. a 195. a 247. d
40. b 92. d 144. c 196. a 248. c
41. d 93. c 145. b 197. a 249. b
42. b 94. b 146. b 198. a 250. c
43. b 95. d 147. b 199. a 251. a
44. c 96. c 148. c 200. a
45. b 97. b 149. c 201. d
46. a 98. c 150. a 202. a
47. a 99. d 151. d 203. a
48. a 100. b 152. b 204. b
49. a 101. a 153. d 205. c
50. a 102. d 154. b 206. c
51. c 103. a 155. b 207. a
52. b 104. c 156. b 208. c

122 The Board of Cortificatlon Study Guide 6e ISBN 978-089189·6609 C2()IB~;Gi'


p
3; c hemist ry Carb o /ryclro tos

Chemistry
The following, ite'!15 liave beo'! identified generally as appropriate for bol/1 entry level medical
Jaboratory sc'.entists and medical labora~ory technicians. Items that are approp riate for m edical
loboratoiY sc1ent1sts only are marl<ed with an 'M LS ONL y:·
- Questions 196 Answers with Explanations
141
1,1 carbO/Jydrates 197 Carbohydrates
144 Acid-Base Balance 197 Acid-Bose Balance
1, 7 Eleetrolytes 198 Electrolytes
151 Proteins & Other Nitrogen-Containing 198 Proteins & Other Nitrogen-Containing
compounds Compou11ds
161 Heme Derivatives 200 Hemo Derivatives
166 Enzymes 201 Enzymes
171 Lipids & Lipoproteins 204 Lipids & Upoprotelns
174 Endocrinology & Tumor Markers 205 Endocrinology & Tumor Markers
180 TOM & Toxicology 209 TOM & Toxicology
182 Quallty Assessment 21 0 Quality Assessment
184 LabOratory Mathematics 211 LobOratory Mathematics
188 Instrumentation 213 lnstrumontaUon

Carbohydrates
1. Following overnight fasting, hypoglycemia in non-diabelic adults is defi ned as a glucose of:
a s70 mg/dL (S3.9 mmol/L)
b s60 mg/dL (S3.3 mmol/L)
c s55 mg/dl (s 3.0 mmol/L)
d s45 mg/d l (S2.5 mmol/L)
2. The following results are from a 21-year-old patient with a back injury who appears
otherwise healthy:
whole blood glucose: 77 mg/dL (4.2 mmol/L)
serum glucose: 88 mgldL (4 8 mmol/L)
CSF glucose: 56 mgldL (3.1 mmol/L)
The best interpretation of lheso results is that:
a the whole blood and serum values are expected but the CSF value is elevated
b the whole blood glucose value should be higher than the serum value
c all values are consistent with a normal healthy Individual
d the serum and whole blood values should be identical
3. The preparation of a patient for standard glucose tolerance testing should include:
a a high carbohydrate diet for 3 days
b a low carbohydrate diet for 3 days
c fasting for 48 hours prior to testing
d bed rest for 3 days
4. If a fasting glucose was 90 mg/dl, which of the following 2-h~ur postprandial glucose
results would most closoly represent normal glucose metabolism?
a 55 mg/dl (3 .0 mmol/L)
b 100 mg/dl (5.5 mmol/L)
c 180 mg/dl (9.9 mmol/L)
d 260 mg/dl (14.3 mmol/L)

CNnlcal Llboratory Certification Examinations 141


3 : C h emistry Car~
Ydraro,
s. A healthy person with a blood glucose of 80 m g/dl (4.4 mmol/L) wou Id have o --.....;:
simultaneously determ ined cereb rospinal Ouid glucose value of :
a 25 mg/dl (1.4 mmol/l)
b 50 mg/dl (2.3 mmol/l}
c 100 mg/dl (5.5 mmol/L)
d 150 mg/dl (8.3 mmol/L)
6. A 25-year-old man became nauseated and vomited 90 minu tes after receiving a stand
7 5 g carbohydrate dose for an oral glucose tolerance test. The best course of action isal~
a g ive the patient a glass or orange juice and continue the test
b start the test over immediately w ith a 50 g carbohydrate dose
c draw blood for glucose and disconlin_ue test . .
d place the patient in a recumbent posit on , reassure him and continue the test
7. Cerebrospinal Ouid for glucose assay should be:
a refrigerated
b analyzed immediately
c heated to 56°C
d stored at room temperature after cen trifugation
8. W hich of the following 2-hour postprandial g lucose values demonstrates unequivocal
hyperglycemia d iagnostic for diabetes mellitus?
a 160 mg/dl (8.8 mmol/L)
b 170 mg/dL (9.4 m mol/L}
c 180 mg/dL (9.9 m molll}
d 200 mg/dl ( 11.0 mmol/L}
9. Serum levels that define hypoglycemia ir. pre-term or low birth weight infants are:
a the same as adults
b lower than adults
c the same as a normal full-term infant
d higher than a normal full-term infant
10. A 4 5-year-old woman has a fasting serum glucose concentration of 95 mg/dl (5.2 mmol/l)
and a 2-hour postprandial glucose concentration of 105 mg/ dl (5.8 mmol/ L}. The
statement which best describes th is patient's fasmng serum glucose concentration is:
a normal; reflecting glycogen breakdown by the liver
b normal; re flecting glycogen breakdown by skeletal muscle
c abnormal; indicating diabetes mellilus
d abnormal; indicating hypoglycemia
11. Pregnant women with symptoms of thirst, frequent urina tion o r unexplained weight toss
should have which of the following tests performed ?
a tolbutamide test
b lactose tolerance test
c epinephrine tolerance test
d glucose tolerance test
12. In the fasting state, the arterial and capillary blood g lucose concentration v aries from the
venous glucose concentration by approximately how many mg/dl (mmol/L)?
a 1 mg/dl (0.05 mmol/L) higher
b 5 mg/dl (0.27 m mol/L) higher
c 10 mg/dL (0.55 mmol/L) lower
d 15 mg/dl (0.82 mmolll) lower

142 T h• Boar d of Certification Study Gul de &e


ISBN 978-089189-6609 C)20I&¢
hem•»" Y
3: C Carbollydrates
.---:;fhe conversion of glucose or other hexoses int0 1
13. . ac1ate or pyruvate is called:
3
glycogenes•s.
b gtycogenolys1s .
c g1uconeogenes1s
.
d 91yco1ys1s
Whieh one of the following values obtained d
14. of diabe1es mellitus? unng a glucose tolerance 1est are diagnostic

3 2-hour specimen = 150 mgld L (8.3 mmol/L)


b fasting plasma glucose =126 mgldL (6.9 mmol/L)
c fasting plas~a glucose = 110 mg/dL (6.1 mmoUL)
d 2-hour specimen = 180 mg/dL (9.9 mmol/L)
Hemoglobin A1 c represents:
1s.
a vallne subslil~lion for glulamine at the Slh position of 1he beta chain of heme lobin
b ketone formation due to hydrolysis of the alpha and beta C-ter · f · g
abnormal hemoglobin minus arming an
o!
c glycosy1ation va!ine in the polypeptide N-terminus of normal adult hemoglobin
d glucose reduction m the presence of oxygen al the N-terminus of the polypeptide
chains of hemoglobin

1s. Monitoring long-term glucose control in patients with adult onset diabetes mellitus can best
be accomplished by measuring:
a weekly fasting 7 am serum glucose
b glucose tolerance testing
c 2-hour postprandial serum glucose
d HgbA,c
17, A patient with type I, insulin-dependent diabetes mellitus has the following results:
Test Patient Rererence Range
fasting blood glucose 150 mgldl (8.3 mmoVL) 70- 110 mg/dl (3.9·6.1 mmol/t)
H gbA1c· 8.5% 4.0-6 0%
fructosamine: 2.5mmoUL 2.0-2.9 mmollL

After reviewing these test results, the technologist concluded that the patient is in a:
a "steady state" of metabolic control
b state of flux, progressively worsening metabolic control
c improving state of metabolic control as indicated by fructosamine
d state of flux as indicted by the fasting glucose level
18. Total glycosylated hemoglobin levels in a hemolysate reflect the:
a average blood glucose levels of the past 2-3 months
b average blood glucose levels for the past week
c blood glucose level at the time the sample is drawn
d HgbA1c level at the time the sample is drawn
19. Which of the following glycosylated hemoglobins is recommended by the ADA guidelines
for testing diabetic patients?
a HgbA 1a
b HgbA2
c HgbA1b
d HgbA1c
20. A Patient with hemolytic anemia will:
a show a decrease in glycated Hgb value
b show an Increase in glycated Hgb value
c show little or no change in glycated Hgb value
d demonstrate an elevated HgbA,
Clinical Laboratory Cerllflcation Examlnanons 143
t'lois"SCP ISBN 97&-089189-6609
3: C h e mist ry lillli
Acid-801so B I

Acid·Base Balance
26. The expected blood gas results for a patient in chronic renal failure would match the
pattern of:
a metabolic acidosis
b respiratory acidosis
c metabolic alkalosis
d respiratory alkalosis
27. Severe diarrhea causes:
a metabolic acidosis
b metabolic alkalosis
c respiratory acidosis
d respiratory alkalosis

144 Tho Board of Certllica tlon S tudy Gulde 6e ISBN 978-08918U609 oiotsASCi'
' 3: chemistry
n. The following blood gas results were obtained:
pH: 7.16
Acid-Base Balanco

p0,: 86mm Hg
pCOi: 60 mm Hg
Ot saturation: 92%
HC03 : 21 mEq/l (21 mmol/L)
rco2: 23 mEq/l (23 mmol/L)
base excess: -8.0 mEq/L (-8 ommolll)

The patient's resulls are compatible with which of th f .


e ollowing?
a fever
b uremia
c emphysema
d dehydration

29. ro8d~~~ t~!:t contribute to a pC02 electrode requiring 60- 120 seconds to reach equilibrium

a diffusion characteristics of the membrane


b actual blood p0 2
c type of calibrating standard (ie, liquid or humidified gas)
d potential of the polarizing mercury cell
30. Ari emphysema patient suffering from fluid accumulation in the alveolar spaces is likely to
be 1n what metabolic state?
a respiratory acidosis
b respiratory alkalosis
c metabolic acidosis
d metabolic alkalosis
31. At blood pH 7.40, w hat is the ratio of bicarbonate to carbonic acid?
a 15:1
b 20:1
c 25:1
d 30:1
32. The reference range for the pH of arterial blood measured at 37°C is:
a 7.28-7.34
b 7.33- 7.37
c 7.35-7.45
d 7.45-7 .50
ll. A 68-year-old man arrives in the emergency roon with a glucose level of 722 mgldL
(39.7 mmoUL) and serum acetone of 4+ undiluted. An arterial blood gas from this patient is
likely to be:
a lowpH
b high pH
c low p0 2
d high p02
34. A patient is admitted to the emergency room in a state of metabolic alkalosis. Which of the
following would be consistent with this diag nosis?
a high TC02 , increased HC03
b low TC02 , increased HC03
c high TC02, decreased H2C03
d low TC0 2 , decreased H 2C03

Cllnlc•I Lol>onitory Certification Examln•tlons 1 45


3: Chemistry Acid-B<>se e ~
::-::-~:--~~~~--:--:--:-:---:---::=--::-::::-:::;::-::::::-;::::;-=--:-:-=-:-:;:-;:-:-:=-~-;:-::~:-;:--:-"":'."""::..:.~a~
35. A person suspected of having metabolic alkalosis would have wh ic h of the following nc&
laboratory findings?
a C0 2 content and pC0 2 elevated, pH decreased
b C02 content decreased and pH elevated
c co 2 content. pC0 2 and pH decreased
d C02 content and pH elevated
36. Metabolic acidosis is described as a(n):
a increase in C02 content and pC02 with a decreased pH
b decrease in C0 2 content with an increased pH
c increase in C02 with an increased pH
d decrease in C02 content and pC02 with a decreased pH
37. Respiratory acidosis is described as a(n):
a increase in C02 content and pC02 with a decreased pH
b decrease in C02 content with an increased pH
c increase in C02 content with an increased pH
d decrease in C02 content and pC02 with a decreased pH
38. A common cause of respiratory alkalosis is:
a vomiting
b starvation
c asthma
d hyperventilation
39. Acidosis and alkalosis are best defined as fluctuations in blood pH and C02 content due to
changes in:
a Bohr effect
b 0 2 content
c bicarbonale buffer
d carbonic anhydrase
40. A blood gas sample was sent to the lab on ice, and a bubble was present in the syringe.
The blood had been exposed to room air for at least 30 minutes. The following change in
blood gases w ill occur:
a C02 content increased/pC02 decreased
b C02 content and p02 increased/pH increased
c C02 content and pC02 decreased/pH decreased
d p0 2 increased/HC0 3 decreased
41. The following laboratory results were obtained:
Serum e lectrolytes
sodium: 136 mEq/L (136 mmol/L)
potassium: 4.4 mEq/l (4.4 mmol/L)
chloride: 92 mEq/L (92 mmol/L)
bicarbonate: 40 mEq/L (40 mmol/L)
Arterial blood
pH: 7.32
pC02: 79 mm Hg

T hese results are most compatible with:


a respiratory alkalosis
b respiratory acidosis
c metabolic alkalosis
d metabolic acidosis

146 The Board of Certification Study Guld e Se ...on1aAscP


ISBN 978-089189-6609 v •" ....ol
~chemistry E/oc trolytes
;..--; iect the test which evaluates renal tubular run ll
, 2• 5e c on.
3 1VP
b creatini~e dearancc
c 05 molanty_ . . .
d microscopic urinalysis
A patient had the following serum results:
,J. NA":
0
14 .mEq/L (140.mmol/L)
K': 4.0 mEqlL (4.0 mmol/L)
gtuoose: 95.mgldL (5.2 1nmol/L)
BUN: 10.mgldL (3.57 mmot/L)

Which osmolality is consistent with these results?


a 188
b 204
c 270
d 390
'-'· The degree to which the kidney concentrates the glomerular filtrate can be determined by:
a urine creatine
b serum creatinine
c creatinine clearance
d urine lo serum osmolality ratio

4s. Osmolal gap is the difference between:


a the ideal and real osmolalily values
b calculated and measured osmolality values
c plasma and water osmolality values
d molality and molarity at 4°C

Electrolytes
'5. The most important buffer pair in plasma is the:
a phosphatelbipllosphate pair
b hemoglobinfimidazole pair
c bicarbonate/carbonic acid pair
d sulfate/bisulfate pair
47. Quantltation of Na• and K• by ion-selective electrode is the standa rd method because:
a dilution is required for flame photometry
b there is no Kpoprotein interference
c of advances in electrochemislry
d of the absence of an internal standard
48. What battery of tests is most useful In evaluating an anion gap of 22 mEq/L (22 mmol/L)?
a Ca2•, Mg2+, P0-4 and pH
b BUN, creatinine, salicylate and mel!lanol
c AST, ALT, LO and amylase
d glucose, CK, myoglobin and cryoglobulin

.__
OI01aAScl> ISSN97&-089189-6609 cnnical Labor•tory Certification Examinations 147
3 : C h emis try
49. A patient with myetoproliferative disorder has the following values:
Hgb: 13 OltlL (130 mmoUL)
H c1· 38%
WBC: 30 • 10>1µL (JO x 109/ L;
plntolols: 1000 • 103/µL (1000 • 10!/L)
sorum Na· : 140 m Eq/L (140 mmol/L)
serum K ' : 7 mEqlL (7 mmol/L)

The serum K• should be confirmed by:


a repeat testing of the original serum
b testing freshly d rawn serum
c testing heparinized plasma
d atomic absorption spectrometry
50. Most of the carbon dioxide present in blood is in the form of:
a dissolved C0 2
b carbo nate
c bicarbonate ion
d ca rbonic acid
51. Serum ·•anion gap· is increased in patients with :
a renal tubular acidosis
b diabetic alkalosis
c metabolic acidosis due to diarrhea
d lactic acidosis
52. The a'lion gap is useful for q uality contrcl of laboratory results for:
a amino acids and proteins
b blood gas analyses
c sodium , potassium, chloride, a nd total C02
d calcium, phosphorus and magnesium
53. The buffering capacity of blood is maintained by a reve rsible exchange process between
bicarbonate and:
a sodium
b potassium
c calcium
d chloride
54. In respiratory acidosis, a compensatory mechanism is th e increase in:
a respiration rate
b ammonia formation
c blood pC02
d plasma bicarbonate concentration
55. Which of the following electrolytes is the chief plasma catioo whose main function is
maintaining osmotic pressure?
a chloride
b calcium
c potassium
d sodium
56. A potassium level of 6.8 mEqlL (6.8 mmol/L) Is obtained. Before reporting the results, the
first step the technologist should take is to:
a check the serum for hemolysis
b rerun the test
c check the age of the pa tient
d do nothing, simply report out the result
14 8 The Board of Cer1ificollon Study Gulde 6e IS8N978·089 189.0009 e4()tSASCP
: chemistry
3
The solute that contributes the most t0 th Electrolytes
51. e total s
a glucose erum osmolality is:
b sodium
c chloride
d urea
ss. A sweat c hll)oride resbult of 55 mEq/L (55 mmol/L) d
(52 mmo11 were o tained on a patient who h an . a sweat sodium or 52 mEqlL
best 1nterpretat1on of these results is: as a history or respiratory problems. The
a normal
b normal sodium and an abnormal chi ·
c abnormal results onde te~t should be repeated
d intermediate results

59. Which of the following is true about direct . .


. ion selective electrodes for electrolytes?
a whole blo?d specimens are acceptable
b elevated hp1ds. cause falsely decreased resuts
c elevated proteins cause falsely decreased results
d elevated platelets cause falsely increased results
60. Sodium determination by indirect ion selective electrod · f
e 1s a1se1y decreased by:
a elevated chlwide levels
b elevated lipid levels
c decreased protein levels
d decreased albumin levels

61. A.physician reques.ted that electrolytes on a multiple myeloma patient specimen be run by
direct ISE and not indirect ISE because:
a excess protein binds Na in indirect ISE
b Na is falsely increased by indirect ISE
c Na is falsely decreased by indirect ISE
d excess protein reacts w ith diluent in indirect ISE
62. Which percentage of total serum calcium is nondiffusible protein bound?
a 80-90%
b 51-60%
c 40-50%
d 10-30%
63. The best method for ionized calcium involves the use of:
a valinomycin incorporated into a semipermeable membrane that allows for change in
current
b ion selective electrode that detects change in potential when Ca 2• binds reversibly to
the membrane
c 8-hydroxyquinoline selective membrane that binds with Ca 2• to prevent other ions
which may change potential
d biuret reaction that removes protein prior to binding with arsenazo ions to cause change
in voltage
64. The regulation of calcium and phosphorous metabolism is accomplished by which of the
following glands?
a thyroid
b parathyroid
c adrenal glands
d pituitary

t2oi8ASCp ISBN 978-089189-6609 Clinical Laboratory Corliflcallon Examinations 149


3: Chernistry
65. A patient has the following test results:
increased scrunl cntciunl levels
decreased ser,1nl phosphate fevets
Increased fevols of parathyroid horil'looo

This patient most likely has:


a hyperparathyroidism
b hypoparathyroidism
c nephrosis
d steatorrhea
66. A hospita lized patient is experiencing increased neuromuscular irritability (tetany). Which
o f the followin g tests should be ordered immediately?
a calcium
b phosphate
c BU N
d g lucose
67. Which of th e following is most likely to be ordered in addition to ionizedcalcium to
d etermine th e cause of tetany?
a magnesium
b phosphate
c sodium.
d vitamin D
68. A reciprocal relationship exists between:
a sod ium and potassium
b calcium and phosphate
c chloride and C02
d calcium and magnesium
69. Fasting serum phosphate concentration is controlled primarily by the :
a pancreas
b skeleton
c parathyroid glands
d sma ll intestine
70. A low concentration of serum phosphorus is commonly found in:
a patients who are receiving carbohydrate hyperalimentation
b chronic renal disease
c hypoparathyroidism
d patients with pituitary tumors
71. The fo llowing serum laboratory results were obtained:
Albumin: d ecreased
Calcium: decreased
Creatinine : incieased
Phosphorus: increased
Magnesium: increased

T hese results are most compatible with:


a multiple myeloma
b p ri mary hyperparathyroidism
c c hronic renal failure
d secondary hyperparathyro idism

1 50 Tho Soard of Certlflcallon Study Gulde 60 ISBN 978-089189-6609


~ Proteins & Otller Nitrogen-Containing Compounds
3: c11e . •
/:-;& Otlier Nitrogen-Contammg Compounds
n,orein
,,
lb . . .
function of serum a umin m the peripheral b'ood is to·
~a . .
72- ·ntain colloidal osmotic pressure
1
a marease antibody production
b crease fibrinogen forma1·ion
.1nC
cd in d .
maintain blOO VISCOSI y
't

a pleural effusion caused by Streptococcus pne1Jmoniae, the protein value of the pleural
~~ ~uid as compared to the serum value would probatly be:
"''
3
decreased by 2
b decreased by y,
c increased by y,
d equal
The first step in analyzing a 24-hour urine specime1 for quantitative urine protein is:
71.
subculture the urine for bacteria
3
b add the appropriate preservative
c screen for albumin using a dipstick
d measure the total volume
IS. When performing a manual protein analysis on a xanthochromic spinal fluid, the technician
.., should:
"" a perform the test as usual
b make a patient blank
c centrifuge the specimen
d dilute the specimen with deionized water
76. The direction in which the proteins migrate (ie, toward anode or cathode) during
"' electrophoretic separation o f serum proteins, ·at pH 8.6, is determined by:
""' a the ionization of the amine groups, yielding a net positive charge
b the ionization of the carboxyl groups. yielding a net negative charge
c albumin acting as a zwitterion
d the density of the gel layer

..,71. The protein that has the highest dye-binding capacity is:
"'' a albumin
b alpha globulin
c beta globulin
d gamma globul in

C:.)1g"5C
P ISSN 978-089189-6609 Cllnlcal Laboratory Cortlffcatlon EJiamlnatlons 151
3: Chemis try Prote ins & Otl1or Nitrogen- Containing Compo "'ii
:;:~-;::;--;--;---:--:-:--:::---:---:::--:--::-~~~~~~~~~~~~~~~~~~~~....::unds
78. Refer to the following illustration: --......
The serum protein electrophoresis pattern shown below was obtained on cellulose ac
a t pH 8.6. eta1e

'

- -
(+) ~-___:-:::__..:..._
----~~ :-)

Identify the serum protein fraction on the left of the illustration.


a gamma globulin
b albumin
c alpha-1 globulin
d alpha-2 globulin
79. The biuret reaction for the analysis of serum p rotein depends on the number of:
Mt.a
a free amino groups
""'' b free carboxyl groups
c peptide bonds
d tyrosine residues
80. In protein electrophoresis, using agarose gel medium and pH 8.6 buffer, an unexpected
• <S protein band migrates between the beta and gamm a region could most likely be due to:
OM.Y .
a oligoclona l proteins from multiple sclerosis
b fibrinogen from incomplete clotting
c bis-albumin
d alpha-1 antitrypsin
81. The relative migration rate o f proteins on agarose gel is based on:
MUI
ONl.'f a molecular weight and concentration
b surface charge and molecular size
c charge-to-mass ratio
d molecular sieving
82. The order or migration in serum protein electrophoresis at pH 8 .6. beginning with the
MLS fastest migration is as follows:
""" a albumin, alpha-1 globulin, alpha-2 globulin, beta globulin. gamma globulin
b alpha- 1 globulin. alpha-2 globulin. beta globulin, gamma globulin, albumin
c albumin, alpha-2 globulin, alpha-1 globulin, beta globulin. gamma globulin
d gamma globulin, beta globulin, alpha-2 globulin, alpha-1 globulin, albumin
83. Wh ich of the following am ino acids is associated with sulfhydryl group?
JAL&
Cf<.Y 3 cysteine
b glycine
c serine
d tyrosine

152 The Board of Certificatjon Study Gulde 6e ISBN978-08918~609 e21J!SAScP ~


r 3:
ChernlSll Y
syrup urine disease is characteriz db
tJrotein s & Olli NI
. er trouon-Co11t:ri11l11g Compounds
Maple .d ? e Y an rncre;ise · hi
5'· amino acr s in w ch or lhe following urinary
-·~· 3
phenylalanine
'~A
tyfOSh~
b valine. leucine and isoleucine
~ cystine and cysterne
creased serum albumin concentrations are see . .
SS· 1n . d n rn which or lhe following conditions?
,., a nephrot1c s~n rome
'"" b acute hepatitis .
chronic inflammation
cd dehydrat•ion
" Th8 following data was obtained from a cellulose acetat . .
""' e protein electrophoresis scan:
elb<Jrrin area: 75 units
gamma globulin area: 30 uni1s
1otal area: 180 units
101a1 protein: 6.5 gidL (65 glL)

The gamma globulin content in g/dl is:


a 1.1 g/dl (11 g/L)
b 2.7 g/dl (27 g/L)
c 3.8 g/dl (38 g/L)
d 4.9 g/dl (49 g/L)

87• Apatient is admitt~ withbbili.aryf cir~hosis. If a serum protein electrophoresis is performed,


1 u11n ractrons will be mJst often be elevated?
... which of the following go
"" a alpha·1
b alpha-2
c bela
d gamma
88. Which of the following serum protein fractions is most likely to be elevated in patients with
.., nephrotic syndrome?
QtJ

a alpha· 1 globulin
b albumin
c alpha-2 globulin and beta globulin
d beta globulin and gamma globulin

Cl!nical Loboralory Certification Examlnallons 153


~IBASCP ISSN 97S.08918!Ml609
3: Chemistry Proteins & Other Nitrogen -Containing Com ~
;;;;-~;::;-::~:-::::--;;:~;:;;::-:-~::-;;;:-:::::::;-:=:-~~~~~~~~~~~~~~~~~~-...:P:,:oun~
89. Refer to the following illustration: --.__:
"-'--$
"""

'- ....
/,,-..~ -~~~-....
(+) ---'-~""------""-- (-)
Patient valuos Reference values
total protein 7.3 g ldl (73 gil l 6 .0-8.) g/dl (60·80 g lL)
albumin 4 .2 g/dl (42 gl L) 3 .6-5.2 g/dl (36-52 gll)
alpha-1 0.0 g/dl (0 g /L) 0 .1 ·O 4 g/dl (1 ·4 g/L)
a lpha·2 0 .9 g/dl (9 gll ) 0.4·1.J g/dL (4 ·10 g/L)
beta 0.8 gldl (8 gll) 0.5·1 .2 gldl (5 - 12 g/L)
gamma 1.4 gldl (14 gll) 0.6 · 1.6 gldl (6 · 16 9/L)

This electrophoresis pattern is consistent with :


a cirrhosis
b monoclonal gammopalhy
c polyclonal gammopathy (eg, chronic Inflammation)
d alpha-1 antitrypsin defi ciency; severe emphysema
90. Refer to the following illustration:
.....
ON.V

Patient values Reference values


total protein 8.9 gldl (89 g /L) 6 0·6.0 g/dl (60-80 g/ L)
albumin 4 .8 g/ dl (48 g/ L) 3.6-5.2 gldL (36-52 g/L)
alpha·1 0.3 gldl (3 g/L) 0 .1-0.4 gldl (1 · 4 g /L)
a lpha-2 0.7 9/dL (7 g/L) 0 4- 1.0 gldL. (4-10 g/L)
beta 0.6 gldl (8 g/L) 0 .5· •.2 gldl (5- 12 g/L)
gamma 2.3 gldl (23 g/L) 0 .6· ".6 gldL (6· 16 gll)

The serum protein electrophoresis pattem is consistent with:


a cirrhosis
b acute innammation
c monoclonal gammopathy
d polyclonal gammopathy (eg, chronic inflammatio n)

154 The Board of CortJncatlon Study Guide Ge ISBN 97&-089189-E609 t120l&ASGI'


~ . Chemistry Pro toi11s & Otlror
3· ., fer to the following pa ttern: · Nltrogon -Co11tai11ing Compoumls

!:· "er--------------------
""'

Patient values
Reference volucs
total protoin 6.1 g/dL (61 g/L)
6.0 · 8.0 g/dL (60-80 g/L)
albumin 2 .3 gldl (23 gl L) 3.6 · 5.2 9/d t (36·52 git)
81pM·1 0.2 g/dL (2 g/L) 0.1· 0.4 gldt (1-4 git )
alpha-2 0 .5 gldl (5 glL) 0.4• 1.0 g/dl <• · 10 git )
be la 1.2 gldL (12 g/ L) 0 .5· 1.2 g/dl (5 · 12 git )
gamma 1.9 gldt (19 g/l) 0.6 · 1.6 gldl (6·16 gll )
This pattern is consistent with:
a cirrhosis
b acute inAammation
c polyclonal gammopathy (eg, chronic inflammation)
d alpha-1 antitrypsin deficiency; severe emphysema
92. A characteristic of the Bence Jones protein that is used to distinguish it from other urinary
...,proteins is Its solubility:
(11l1
a in ammonium sulfate
b in sulfuric acid
c at 4o-so•c
d al 1oo·c
...
93. The electrophoretic pattern of plasma sample as compared to a serum sample shows a:
au a broad prealbumin peak
b sharp fibrinogen peak
c diffuse pattern because of the presence of anticoagulants
d decreased globulin fraction
St At a pH of 8 .6 the gamma globulins move toward the cathode . despite the fact that they
~ are negatively charged. What is this phenomenon called?

a reverse migration
b molecular sieve
c endosmosis
d migratory inhibition factor

Cllnlcal Laboratory Certirtcatlon Examinations 155


~lSAS(;p ISBN978-089189-6609
3: Chemistry Proteins & Oilier Nitrogen-Containing Compo ;
~~~~~~~~~~~~~--.-~~~~~~~~~~~~~~~~~~~~-..::_:._~nels
95. Rerer to the following illustration: ---.....
UtS
ON1.Y

Patient values Refetencc values

total pro tein 7.8 gldL (78 g lL) 6 .0-8.0 g/dL (60 -80 g/L)
a lbumin 3.0 g/ dL (30 glL) 3 .6-5.2 g/dL (36 -52 glL)

alpha-1 0.4 g/dL (4 g/L 0.1 · 0.4 g/dL (1 -4 g/L)


alpha-2 1.8 g/dL (18 g/L) 0.4-1.0 g ldL (4 -1 0g/L)
beta 0.5 gldL (5 g/L) 0.5·1.2 g/dL (5- 12 g/L)
gamma 1.1 g/dL (11 g/L) 0 .6 -1 .6 gldl (6- 16 g/L)

The serum protein electrophoresis pattern is consistent with:


a c irrhosis
b acute inflammation
c polyclonal gammopathy (eg, chronic inflammation)
d alpha-1 antitrypsin deficiency; severe emphysema
96. Refer to the following illustration:
MLS
OM.Y

Pa tient va lues Referenc.e values


to tal protein 8.5 g/dl (85 g/L) 6 .0-8.0 g ldL (60- 80 glL)
albumin 4.3 g/dl (43 g/L) 3.6 · 52 g/dL (36-52 g/L)
alpha -1 0 .3 g/dL (3 g/L) 0.1 · 0 .4 g/dL (1 ·4 gll)
alpha -2 0.7 g/dL (7 g/L) 0.4- 1.0 g/dl (4 -1 0 g/L)
beta 0.9 g/dl (9 g/L) 0.5- 1.2 g/dL (5· 12 g/L)
gamma 2.3 g/dl (23 g/L) 0 .6·1.6 gldL (6 · 16 g/L)
The above serum protein electrophoresis pattern is consistent with:
a nephrotic syndrome
b monoclonal gammopathy
c polyclonal gammopathy (eg, chronic inflammation)
d alpha-1 a ntitrypsin deficiency; severe emphysema

....97.
(lNLY
Analysis of CSF for oligoclonal bands is used to screen for which of the following disease
states?
a multiple myeloma
b multiple sclerosis
c myasthenia gravis
d van Willebrand disease
~
3: C
hernistry Proteins & ou1er N1trogen-Cont;i/11ing

Compo1111cls
~ h identification ol Bence Jones protein is best .
,, T e . . aceomphshed by:
,.. sutrosalicyhc acid lesl
::.;, 3 arine reagent strips
b ~mmunofixation electrophoresis
~ :mmunoelectrophoresis
T tal iron-binding capacity measures the serum iron t . .
99. ,o • ransporting capacity or:
a tiemoglobm .
b ceJ\Jloplasmm
c translerrin
d rerritin
toii The first step In the quantllation of serum iron is:
3
direct reaction with appropriate chromogen
b iron saturation ?f t~anslemn
c rree iron prec1p1tahon
d separation of iron from transferrin

tll. Apatient's blood was drawn at 8 am for a serum iron determination. The result was 85 µg/dl
.., (15.2 µmolll). A repeal specimen was drawn at 8 pm; the serum was stored at 4•c and
"'' runthe next morning. The result was 40 µgldl (7 .2 µmolll). These results are most likely
due to:
a iron deficiency anemia
b improper storage of the specimen
c possible liver damage
d the time or day the second specimen was drawn
t02. An elevated serum iron with normal iron binding capacity is most likely associated with:
"'"" a iron deficiency anemia
b renal damage
c chronic anemia
d septicemia
tll. Decreased serum iron associated with increased TISC is compatible with which of the
:;:, following disease states?
a anemia of chronic infection
b iron deficiency anemia
c clvonic liver disease
d nephrosis
104. A patient has the following results:
M.$
cu Patient values Reference values
seNm iron 250 µg/dL (44.8 pmoVL) 60- 150 pgldL (10.7-26.9 pmol/L)
TIBC 250 µg/dL (62.7 p moUL) 300-350 pg/<IL (53.7 -62.7 pmol/L)

The best conclusion is that this patient has:


a normal iron status
b iron deficiency anemia
c chronic disease
d iron hemochromatosls
tos. To assure an accurate ammonia level result, the specimen should be:
a Incubated at 37•c prior to testing
b spun and separated immediately, tested as routine
c spun, separated, iced, and tested immediately
d stored at room temperature until tested

Cl:il!ASCp ISSN 978-0891~ Cllnlcal LaboratotY Cerllflc•llon Examinations 157 j


b
3: C h emist ry Proteins & Ot11or Nltrogcn-Conf<Jinfng Co~1>01111
106. Erroneous nmmonia levels can be eliminoled by oil or the following except: "t
a assuring wator and reagen ts are ammonia-free
b separating plasma from cells and pcrrorming test analysis as soon as possible
c drawing the specimen in a prechilled lube and immersing the tube in ice
d storing the specimen protected from hght until the analysis is done
107. A critically ill palient becomes comatose. The physician believes the coma is due to
~'N.'v
hepatic fa ilure. The assay most help ful in this diagnosis is:
a ammonia
b ALT
c AST
d GGT
108. A serum sample demonstrates an elevated result when tested with the Jaffe reaction. Th'
indicates: •s
a prolonged hypothermia
b renal functional impairment
c p regna ncy
d archythmia
109. In order to prepare 100 ml of 15 mg/dL BU N (5.35 m mol/L) working standard from a stoef\
.....
K$ standard containing 500 mgfdl (178.5 mmol/L) of urea nitrogen. the amount of s!ock
solution that should be used is:
a 3mL
b 5mL
c 33mL
d 75 ml
110. A patient with g lomerulonephritis is mosl likely to present w ith the following serum results:
t.11.S
•"'• a creatlnine decreased
b calcium increased
c phosphorous decreased
d BUN increased
111 . The principle excretory form of nitrogen is/are:
.o•LY
... a amino acids
b creatlnine
c urea
d u ric acid
112. In the Jaffe reaction, c reatinine reacts with :
....°"''
. a alkaline sulfasalazine solution to produ ce a n orange-yellow complex
b potassium iodide to form a reddish-purple complex
c sodium nitroferricyanide to yield a reddish-brown color
d alkaline picrate solution to yield an orange-red complex
113. Creatinine c learance is used to estimate the:
a tubular secretion of creatinine
b glomerular secr!Mlon of creatfnfne
c renal glomerular and tubular mass
d g lomerular filtratio n rate

1 58 The Board o f Certification Study Gulde &c ISBN 978-089189-6609 oiots.tSd'


d
r 3:
Chemistry
..
Proteins & 0th .
er Nitrogen-Containing Compo11nds
,;....--;-;: 1ood creahmne value of 5.0 mgldl (442.0 mol . ,
ti'- ~f~he following blood values? IJ IL) is most likely to be found with which
osmolality: 292 mOsm!kg
~ uric acid: 8 mg/dL (475.8 µmol/L)
urea nitrogen: 80 mg/dl (28.56 mmoVL)
~ ammonia: 80 µg/dl (44 µmol/L)
1i cnnical problems encountered during the cone r . . .
f15. eused doubt as to whether the specimen was a~i~n or .an a.n:mollc ~luid specimen
~' carocedures would best establish that the Ouid is am"'~tit~ in on~1n. Which 1 ol the following
P n10 1c in origin?
3
measurement of absorbance at 450 nm
b C(eatinine measurement
c 1ectthin/sphingomyelin ratio
d numan amniotic placen tal lactogen (HPL)

ttl Which of the following represents the end prOduct of purine metabolism in humans?
~ a AMP and GMP
b ONA and RNA
c allantoin
d uric acid
ltl. Whict:1 of the following substances is the biologicaily active precursor of a fat soluble
c vitamin?
"" a biotin
b retinal
c rolic acid
d ascorbic acid
11s. The troponin complex consists or:
...
"'' a troponin T, calcium and tropomyosin
b troponin C, troponin I and troponin T
c troponin I, aclin, and tropomyosin
d troponin C, myoglobin, and actin

...tt9. The presence of C-reactive protein in the blood is an indication of:


"" a a recent streptococcal infection
b recovery from a pneumococcal infection
c an inflammatory process
d a state of hypersensitivity
tlQ. Otigoctonal bands are present on electrophoresis of concentrated CSF and also on
::t, concurrently tested serum of the same patient. The proper interpretation is:
a diagnostic for primary CNS tumor
b diagnostic for multiple sclerosis
c CNS involvement by acute leukemia
d need further testing to rule out multiple sclerosis

Cllnlc•I Laboralol}' C•rtmcatlon EJt•mlnations 159


_3;:~C~l~1e;: -1n:--;--is--:-tr~y'..____________________:_P:r~o:te:/:n:s:_:&~O:..:.t1~1 ~c~r~N~i-tr_o~g~e-n__-c__o_n_ra_1_·n_i_n=y-C-=.o~m~p:'.:'.o~
1 llf1cJ
121 · Which or the following is an example of a peptide bond? ~
A c

R,J!J
0U]H 0
11
HO - C - C a
I I II
-c-OH
I
C=O

.....
I

HBHO
B 0
COCH
I 11 I Q I
R - C- c .. N C - C - OH ~
I I I I
NH3 H R, CH>
I
COOH

a A
b B
c c
d D
122. 90% of the copper present in the blood is bound to:
a transferrin
b ceruloplasmin
c aflbumin
d cryoglobulin
123. Serum and urine copper levels are assayed on a hospital patient w ith the following results:
MUI
ONlY Patient values Reference va lues
serum Cu: 20 1-19/dL (3.1 1-1mol/L) 70- 140 IJQ/dl (11 .0 -22.0 ..,mol/L)
urine Cu: 83 1-19/dl (13.0 µmol/L) <40 µg/dL (<63 µmol/L)

This is most consistent with:


a normal copper levels
b Wilms tumor
c Wilson disease
d Addison disease
124. Which of the following determinations is useful in prenatal diagnosis of open neural tube
~v defects?
a amniotic fluid alpha-retoprotein
b amniotic fluid estriol
c maternal serum estradiol
d maternal serum estrone

1 60 Tho Board of Cortiflcallon S1udy Guido 6e


ISBN 978-089 189-0609 cioisAsd'
3: c hemistry
Hema Darl valivos
- · eetow are the results or n protein eloctrop11orcsls:
125-
...
"'"
Fraction Rel 0/o
I 4,5
2 64.5
3 3.6
4 6.5
5 12.6
- 6 7.9

These results are consistent with a(n):


a normal serum protein pattern
b normal CSF protein pattern
c abnormal serum protein pattern
d abnormal CSF protein pattern

Heme Derivatives
126. Serum concentrations of vitamin B 12 are elevated in:
~' a pernicious anemia in relapse
b patients on chronic hemodialysis
c chronic granulocytic leukemia
d Hodgkin disease

...121. AbsOrplion
cu
of vitamin 812 requires the presence of:
a intrinsic factor
b gastrin
c secretin
d rolic acid
128. The principle or the occult blood test depends Lpon the:
"'
'"'' a coagulase ability of blood
b oxidative power of atmospheric oxygen
c hydrogen peroxide in hemoglobin
d peroxidase-like activity or hemoglobin

....129. A breakdown product of hemoglobin is:


""" a lipoprotein
b bilirubin
c hematoxylin
d Bence Jon es protein
130. Hemoglobin S can be separated from hemoglobin D by:
a electrophoresis on a different medium and a:idlc pH
b hemoglobin A2 quantilation
c electrophoresis at higher voltage
d Kleihauer-Belke acid elutio n

Ctlnlcal Laborarory Certlflcarlon Exam /no tions 161


3: Chemis try H om o 0 0 , . i!ii/li
:-:-:------.:...----------------:---:--::--:-.:::::-:::~~--:-....::.:.:.'~ilU~
131. On electrophoresis at alkaline pH. which of the following is Ille slowest migrating ~
hemoglobin?
a HgbA
b HgbS
c HgbC
d HgbF

....o"'v
132. The hemoglobin that is resistant to a lkali (KOH) denaturation is:
a A
b A2
c c
d F
133. The following bilirubin results are obtained on a patient:
day 1: 4.3 mg/dL ( 73 .5 µmo11q
day 2: 4.6 mgldL (78.7 µmol/ L)
day 3: 4 .5 mg/dL (77.0 µmol/L)
day 4 : 2 .2 m g/dl (37 6 µmol/L)
days: 4.4 mg/dl (75.2 µmol/L)
day 6 : 4.5 mgldl (77.0 µmol/L)

Given that the controls were within range each da y, what is a probable explanation for the
result on day 4?
a no explanation necessary
b serum , no t plasma. was used for teslin.g

' c specimen had prolonged exposure to light


d specime n was hemolyzed
134. Urobilinogen is formed in the:
a kidney
b spleen
c liver
d intestine
135. In billrubin determinations, the purpose of adding a concentrated caffeine solution or
:::v me thyl alcohol is to:
a allow indirect bilirubin to react with color reagent
b d issolve conjugated bilirubin
c precipita te protein
d prevent any change in pH
136. If the to tal bilirubln is 3.1 mgtdl (53.0 µmoffl.) and the conjugated bilirubin is 2.0 mg/dL
~r (34 .2 µmoffl), the unconjugated bil irubin is:
a 0.5 mg/dl (8.6 µmol/L)
b 1. 1 mg/dl (18.8 µmoffl)
c 2.2 mg/dl (37.6 µmol/L)
d 5. 1 mg/dl (87.2 µmol/L)
137. The principle of the tablet test for bilirubin in urine or feces is:
.....
""v a the reaction between bile and 2,4-dichloronitrobenzene to a yellow color
b the liberation of oxygen by bile to oxid ize orthotolidine to a blue-purple color
c chem ical coupling of bile w ith a diazonium salt to form a brown color
d chemica l coupling or bilirubin w ith a dlazonium salt to form a purple color

1 62 The Bon rd of Cortlflcollon Study Guido 6e ISBN 978-089189-6609 OZolSASCP


d
r 3: G11en11°" 1 Heme Dorivativos ---,
_.-: 1ieparinized plasma samples were assa d r ..
136. Tenanalyzer 1, and by the Evelyn-Malloy mei: or bihrubln by Jendrassik-Grof method
on dy The results were all 10-20% higher fro : on analyzer 2. in a method comparison
stu · analyzer 2 most likely because:
the caffelne-benzoate stabilizer in the 2nd
a as found in analyzer 1 ana1yzer prevented ratsely decreased results
b fibrinogen
1 in the plasma samples caused f t 1
Grof method a se Y decreased results in the Jendrassik-
alcohol reagents in the second method ca . . .
c background turbidity use precrpitallon of proteins and increased
d hemolysis caused greater interference in method 1 than in method
2
139, Serial bmrubin determinations performed by Jendrassik-Gror method are charted below.
Day Collected Assoyed Result
1 7 am Sam
am 14.0 mgldl (239.4 ~molll)
7
9.0 mgldL (153.9 ~mall\.)
6
2 am pm
6
3 8 pm 15.0 mgldL (256.S µmollL)
The best explanation for the results is:
a sample on day 2 has mild hemolysis and hemoglobin deterioration
b sample on day 2 has exposure to light
c sample on day 2 shows normal day to day variation
d reagent deterioration is evident on day 3
140. In the liver, bilirubin is converted lo:
...
"'" a urobilinogen
b urobilln
c bilirubin-albumin complex
d bilirubin diglucuronide
141. In which of the following disease states is conjugated bilirubin a major serum component?
IU
ew.v a biliary obstruction
b hemolysis
c neonatal jaundice
d erythroblastosis fetalis
142. Kernicterus is an abnormal accumulation of bilirubin in:
a heart tissue
b brain tissue
c liver tissue
d kidney tissue
143. In which of the following conditions does decreased activity of glucuronyl transferase
~. result in increased unconjugated bilirubin and kemicterus in neonates?

a Gilbert disease
b Rotor syndrome
c Dubin-Johnson syndrome
d Crigler-Najjar syndrome

Cllnlcal Laboratory Certification Exan1lnations 163


3: Chemis try Heme o~
1; r1v-0
11
144. A 21-year-old man with nausea. vomiting, and jaundice has the following laboratory~
;;:,fv findings:
Tast Patient values Reference range
to tal serun1 bilinJbin 8.5 mg/dl (145.4 µmoUL) 0· 1.0 mg/dl (0.0 ·17.1 µmol/L)
conjugated serum bilirubin 6.1 mg/dl (104.3 µmol/L) 0·0.5 mg/dl (0.0-8.6 µmol/L)
urine urobilinogen increased
fecal urobilinogen decreased
urine bilirubin Positive
AST 300 U/L 0· 50 U/L
alkaline phosphatase 170 U/L 0· 150 UIL

These can best b e e xplained as representing:


a unconjugated hyperbilirubinemia, probably due to hemolysis
b unconjugated hyper bilirubinemia, pro:Jably due to toxic liver damage
c conjugated hyperbilirubinemia, probably due to hepatocellular d isease
d conj ugated hyperbilirubinemia, probably due to hepatocellular obstruction
145. B iochemical profile:
L~LS
OM.V
Test Patient values Reference range
total protein 7.3 g/dL (73 g/L) 6.0 -8.0 gldl (60 -80 g/L)
albumin 4.1 g/dL(41 g/L) 3.5-5.0 g/dl (35 · 50 g/L)
calcium 9.6 mg/dl (2.4 mmolll ) 8.5- 10.5 mg/dL (2 .1· 2.6 mmol/l )
phosphorus 3.3 mg/dl (1 .06 mmol/L) 2.5-4.5 mg/dL (0.80-1.45 mmoliL)
glucose 95 mg/dl (5.2 mmoVL) 65· 110 mg/dL (3.6. 6. 1 mmoUL)
BUN 16 mg/dL (5.71 mmoVL) 10·20 mg/dL (3.57·7.14 mmol/L)
uric acid 6.0 mg/dL (356.9 µmol/L) 2.5-8.0 mgldL (148.7-475.8 µmoVl)
creatinine 1.2 mg/dL (106. f µmol/L) 0.7-1.4 mg/dL (61.9 -123.8 µmoVL)
total bilirubin 3.7 mg/dl (63.3 µmol/L) 0.2 -0.9 mg/dl (3.4 -15.4 µmol/L)
alkaline phosphatase 275 UIL 30· 60 U/L
lactate dehydrogenase 185 U/L 100- 225 U/L
AST 75 U/L 10-40 U/L

The results the biochemical profile a re most consistent with:


a viral hepatitis
b hemolytic anemia
c common bile d uct stone
d chronic active hepatitis
146. A stool specimen that appears b lack and tarlike should be tested for the presence of:
a occult blood
b fecal fat
c trypsin
d excess mucus

1 64 The Board of Certification Study Guide Ge ISBN 978-089189-6609 e2Qf&AScP


3: chemistry
1'7. Whal substance gives feces Its normal color? Ncnw Dorlvativcs l
~' a uroerylhrin
b urochrome
c urobilin
d urobilinogen
A condition in which erythrocyte protoporph . , .
148· . . ynn is increased is:
;if, a acute intermittent porphyria
b iron deficiency anemia
c porphyria cutanea tarda
d acute porphyric attack

149, Which of the following elevates carboxyhemoglobin?


:::,1, a nitrite poisoning
b exposure to carbon monoxide
c sulfa drug toxicity
d sickle cell anemia

150. The reason carbon monoxide is so toxic is because it:


~fcv a is a protoplasmic poison
b combines with cytochrome oxidase
c has 2?0 limes the affini_ty of oxygen for hemcglobin binding siles
d sensitizes the myocardium
151. Detection of carriers of hereditary coproporphyria should include analysis of:
....
..,,, a 24-hour urine f or porphobilinogen
b fresh morning urine for delta-amlnolevulinic acid
c erythrocyte protoporphyrin
d 24-hour urine for porphyrln
152. A fresh urine sample is received for analysis for "porphyrins" or "porphyria" without further
ou information or specifications. Initial analysis shculd include:
"" a porphyrin screen and quantitative total porphyrin
b quantitative total porphyrin and porphobilinogen screen
c porphyrin and porphobilinogen screen
d porphobilinogen screen and ion-exchange analysis for porphobilinogen
153. Which of the following enzymes of heme biosyrthesis is inhibited by lead?
"'
"" a aminolevulinate synthase
b porphobilinogen synthase
c uroporphyrinogen synthase
d bilirubin synthetase

...""'154. Serum haptoglobin:


a is decreased in patients with tissue injury and neoplasia
b is increased in patients with prosthetic heart valves .
c can be separated into distinct phenotypes by starch-gel electrophoresis
d binds heme

Cllnfoal Laboratory Certllioatlon Examinations 165


ClOIBASCP ISBN971\.M9189-6609
_3_:_C~h_e_n_1_is_t_ry_:__~~~~~~~~~~~~~~~~~~~~~~~~~~E-n_~~Y'.:_'~o;
Enzymes ""'
155. The most specific enzyme test ror acute pancrealitis is:
a acid phosphatase
b trypsin
c amylase
d lipase
156. Which of the following enzymes are used in the diagnosis of acute pancrea lilis?
a amylase (AMS)
b aspartale aminotransferase (AST)
c gamma-glutamyl transferase (GGT )
d lactate dehydrogenase (l.D)
157. Which of the following enzymes catalyzes the conversion of starch lo glucose and
maltose?
a malate dehydrogenase (MD)
b amylase (AMS)
c creatine kinase (CK)
d isocilric dehydrogenase (ICD)
158. Which of the following sets of resul ts would be consistent with macroamylasemia?
......
""" a normal serum amylase and elevated urine amylase values
b increased serum amylase and normal urine amylase values
c increased serum and urine amylase values
d normal serum and urine amylase values
159. In acute pancreatitis, the following results wou ld be expected:
14!
ONlv a LD isoenzymes 1 elevates higher than 2 within 72 hours
b lipase elevates within 4 -8 hours of an attack, peaks at 24 hours and normalizes in 48
hours
c amylase and lactate rise successively in the first 24 hours and stay elevated 4 days
d amylase and lipase elevate in a few hours, but amylase normalizes within 3 days
160. Aspartale aminotransferase (AST) is characteristically elevated in diseases of the:
a liver
b kidney
c intestine
d pancreas
161. Amino transferase enzymes catalyze the:
a exchange of amino groups and sulfhydryl groups between alpha-amino and sulrur-
containing acids
b exchang~ of ami.no an? keto groups between alpha-amino and alpha-keto acids
c hydrolysis of amino acids and keto acids
d reversible transfer of hydrogen from amino acids to coenzyme
162. Aspa~ate aminotransferase (AST) and alanine aminotransferase (ALT) are both elevated
In which of the following diseases?
a muscular dystrophy
b viral hepatitis
c pulmonary emboli
d infectious mononucleosis

166 Tho Boord o f Cortiflcallon Study Guido se


ISBN 978-069189-6609 ('2018¢
r 3; chern1~u y
--lnjerpretation or and sources of interference . Enzymos
163· hat a significant source of . 'rn seru.m enzyme analysis should consider
t IS OUnd In RBCs.
a AST
b ALT
c GGT
d CK
Malic dehydrogenase is added to the as art .
t&I· ~~taiyze the conversion of:
... .,..
P ate ammotransaminase (AST) reaction to
""' a alpha-ketoglutarate to aspartate
b alpha-ketoglutarate to malate
c aspartate to oxalacetate
d oxalacetate to matate
t&S· Given the following results:
w.s alkaline phosphatase: slight incrcast
,,.., asparlato amino transrerase: marked inctease
alanine amino transferase: rnarked inc1ease
gamma-glutamyl transferose: slight increase

This is most consistent with:


a acute hepatitis
b chronic hepatitis
c obstructive jaundice
d liver hemangioma
166. Which or the foll~wing clinical disorders is associated with the greatest elevation or lactate
dehydrogenase 1soenzyme 1?
a pneumonia
b glomerulonephritis
c pancreatitis
d pernicious anemia
161. The enzyme, which exists chiefly in skeletal muscle, heart, and brain, is grossly elevated
in active muscular dystrophy, and rises early in myocardial infarction is:
a lipase
b transaminase
c lactate dehydrogenase
d creatine kinase
t68. The enzyme present in almost all tissues that may be separated by electrophoresis into 5
components Is:
a lipase
b transaminase
c creatlne kinase
d lactate dehydrogenase
169. A common cause or a falsely increased LO, fraction or lactic dehydrogenase is:
a specimen hemotysis
b liver disease
c: congestive heart failure
d drug toxicity
170. The presence 01 which of the following isoenzymes indicates acute myocardial damage?

a CKMM
b CKMB
c: CKBB
d none
Cllnlc•I L,abontory Certification Examinations 167
~lll&;p !Se/l97~1~
~
3: Chetn ist ry Enly~
q
. ~~
171 . In which of the following conditions would a normal level o r creatine kinase bo found?'---
a acute m yocardia l infarct
b hepatitis
c progressive muscular dystrophy
d intramuscular injection
172. Of the following diseases. the one most o ften associated with elevations of lactate
dehydrogenase isoenzymes 4 and 5 on eiectrophoresis is:
a liver disease
b hemolytic anemia
c myocardial infarction
d pulmonary edema
173. When myocardial Infarction occurs, the first enzyme to become elevated is:
a CK
b LO
c AST
d ALT
174. A scanning of a CK isoenzyme fractiona tion revealed 2 peaks: a s.low cath?dic peak
(CKMM ) and an intermediate peak (CKM B). A possible interpretation for this pattern is:
a bra in tumor
b muscular dystrophy
c myocardial infarction
d viral hepatitis
175. An electrophoretic separation of lactate dehydrogenase isoenzymes that demonstrates an
•U elevation in L0-1 and L0-2 in a '"flipped" pa ttern is consistent with :
ONLV
a myocardial infarction
b viral hepatitis
c pancreatitis
d renal failure

.176.
..
"'"v
Increased total serum LO activity is:
a sensitive and specific lo acute myocardial infarction
b specific to acute hepatic disease such as viral hepatitis
c sensitive to pancreatic obstruction but not pancreatilis
d sensitive but not specific to neoplastic disease
177. A 10-year-old chffd was admitted to pediatrics with an initial diagnos is or skeletal muscle
disease. The best confirmatory tests would be:
a creatine kinase and isocitrate dehydrogenase
b gamma-glutamyl transferase and alkaline phosphatase
c aldolase and creatine kinase
d lactate dehydrogenase and malate dehydrogenase
178. In the immunoinhibition phase of the CKMB procedure:
a M subunit is inactivated
b B subunit is inactivated
c MB Is Inactivated
d BB is inactivated

168 The eoord of Certlflcntlon Study Gulde Ge ISBN 978-0891S<).6609 C!2()tSASC!'


~
~try Enzymes
3. _.-
--. lhh•e
ence of increased CKMB acllvity
~ pres patient suffering from:
c .
on a Kelectrophoresis pattern is most likely
119 round in a •
acole mus~ular stress following strenuous exercise
9 11gnant hver disease
3
~ ~yocardial in_f~rclion
d severe head 1n1ury
Refer to the following Illustration:
160-
LOS
LD2
LD1
LD3
LD4

(+) -=---- ----.0.. (-)


Which of the following is the most likely interprelation of the LO isoenzyme scan
a1ustrated above?
a myocardial infarction
b megaloblastic anemia
c acute pancreatitis
d viral hepatitis
tat. Increased serum lactic dehydrogenase activity due to elevation of fast fraction (1 and 2)
on electrophoretic separation is caused by:
a nephrotic syndrome
b hemolytic anemia
c pancrealitis
d hepatic damage
182- A serum sample drawn in the emergency room from a 42-year-old man yielded the
following laboratory results:
Patient Roference range
CK: 185 U/l 15· t60 U/l
AST: 123 U/L 0· 48 U/L
CKMB: 6 U/L 2· 12 Ull

Which of the following conditions might account for these values?


a crush injury to the thigh
b cerebrovascular accident
c pulmonary infarction
d early acute hepatitis

~ Olo!!A5C1> ISBN 978-089189-6S09 C/Jn/c1I Laboratory Cmmcation Examinations 169


3: Chemistry

---
Enzy~
183. Given the following results:
"'•Os""'
a lkaline phosphal ase: markad increase
asporl ate amino transrerase: slight Increase
alanina a mino transforasc: slig ht jn crcase
gamma ..glutamyl transferase: mark&d increase

Th is is most consistent with:


a acute hepatitis
b osteitis fibrosa
c chronic hepatitis
d obstructive jaundice
184. Given the following results:
alkaline phosphalase: slight incre ase
asparl ate a minotransferase: sfight increase
alanine aminotransferase: slight increase
gamma~glutamyf transrerase: slight increase

This is most consistent with:


a acu te hepatitis
b chronic hepatitis
c obstructive jaundice
d liver hemangioma
185. What specimen preparation is used to perform the alkaline phosphatase isoenzyme
determination?
a serum is divided into 2 aliquots, one is frozen and the other is refrigerated
b serum is divided into 2 aliquots, one is heated at 56°C and the other is unheated
c no preparation is necessary since the assay uses EDTA plasma
d protein-free filtrate is prepared first
186. Regan lsoenzyme has the same p roperties as alkaline phosphatase that originates in the:
a skeleton
b kidney
c intestine
d placenta
187. The most heat labile fraction of alkaline i;hosphatase is obtained from:
a fiver
b bone
c intestine
d placenta
188. The most sensitive enzyma tic indicator fe r liver damage from e thanol intake is:
a alanine aminotransferase (ALT)
b aspartate aminotransferase (AST)
c gamma-glutamyl transferase (GGT)
d alkaline phosphatase
189. fsoenzyme assays are perfonmed to improve:
a precision
b accuracy
c sensitivity
d specificity

170 Tho Board of Certification Study Guido 60


ISBN 978-089189.U09 C201BASCf
• chem istry
3, · f Lip/els & Llpopro teins
. The protein portion o an enzyme complex Is taled lhe:
190
3
apoenzyme
b coenzyrne
c holoenzyme
d proenzyrne
Which of the following chemical determinar
191. presence of seminal Huid? ions may be of help in establishing the

3 tactic dehydrogenase (LO)


b isoci1rate dehydrogenase (!CO)
c acid phosphatase
d alkaline phosphatase
2. Which of the following enzyme substrates is the . . .
!,.9 phosphatase for quantitative endpoint reactions?m0 st specific for prostabc acid
'''" a p-nilrophenylphosphale
b thyrnolphthalein monophosphate
c beta-naphthol-phosphate
d beta-glycerophosphate

193. Lactate dehydrogenase, malate dehydrogenase, isocitrate dehydrogenase and glucose 6


... phosphate dehydrogenase all: •
(14.1
a are liver enzymes
b are cardiac enzymes
c catalyze oxidation-reduction reactions
d are class Ill enzymes

Lipids & Lipoproteins


194. High levels of which lipoprotein class are associated with decreased risk or accelerated
~' atherosclerosis?

a chylomicrons
b VLOL
c LDL
d HDL
195. The most consistent analytical error involved in the routine determination or HOL-
~' cholesterol is caused by:
a incomplete precipitation of LOL-cholesterol
b coprecipitatlon or HDL- and LDL-cholesterol
c inaccurate protein estimation of HOL-cholesterol
d a small concentration or apoB-containing lipoproteins after precipitation
l96. If the LDl.-cholesterol is to be calculated by the Friedewald formula, what are the 2
~ measurements that need to be carried out by the same chemical procedure?
a total cholesterol and HOL-cholesterol
b total cholesterol and triglyceride
c triglyceride and chylomicrons
d apolipoprotein A and apollpoprotein B

Olot&"5(;p ISSN 978~189-6609 Ci'lnlcal Laboratory Ctrllflcatlon Ex.Jmlnt1tlons 171


3: Cl1er11istry Lipids &
-197.-----=-----::;--;-:-;-;:;:--:::::-::::::::-::-::::;--::===-:-.::--;--:------
Lipa~
- -.:...:.:.::0111$
Tho chemical composition of HDL·cholcsterol corresponds to: --...__
Trlglycorldo Choles terol Protein
a 60% 15% 10%
b 10% 45% 25%
c 5% 15% 50%
d 85% 5% 2%
198. In familial hypercholesterolemia, the hallmark finding is an elevation of:
.....
"'"'' a low-density lipoproteins
b chylomicrons
c high-density lipoproteins
d apolipoprotein A 1
199. Premature a therosclerosis can occur when which of the following becomes elevated?
MlS
o•LV a chylomicrons
b prostaglandins
c low-density Hpoproteins
d high-density lipoproteins
200. Transportation of 60 · 75% of the plasma cholesterol is performed by:
...s
""" a chylomicrons
b very low-density lipoproteins
c low-density lipoproteins
d high-density lipoproteins
201. Which of the following diseases results from a familial absence of high density lipoprotein?
....
""'-' a Krabbe disease
b Gaucher disease
c Tangier disease
d Tay-Sachs disease
202. A 1-year-old girl with a hyperlipoprotein emla and llpoprotein lipase deficiency has the
following lipid p rofile:
choleslerol: 300 mgldl (7. 77 mmol/L)
LOL: increased
HDL: decreased
triglyceri des: 1200 mg/ dl (13.56 mmol/L)
chylomlctons: present

A serum specimen from this patient that was refrigerated overnight would most likely be:
a clear
b cloudy
c creamy layer over cloudy serum
d creamy layer over clear serum
203. Which of the following lipid results would be expected to be falsely elevated on a serum
J.!lS
Otil 'I' specimen from a non-fasting patient?
a cholesterol
b triglyceride
c HDL
d LDL

172 Tho Board o f Cortlflcallon Study Gulde 60 ISBN 978-089189·6609 C2QIS ASCP
heniistry
3: C Lipids & Lipoproteins
__.-: g.nionth·Old boy from Israel has gradual! 1 ••
1
zot A izures. He has an increased amount of ayP~st ~ ~b!hly lo sit up, and develops
~' s~ neurons, and he lacks the en:z:yme hexosa 0~P.dohp1d <;ailed GM 2-ganglioside in
~ndings suggest: mini ase A in his leukocytes. These
Neirnann-Pick disease
: Tay.Sachs dis~ase
c p11enylkelonuna
d Hurler syndrome
In amniotic nuid, the procedure used to determine fetal 1ung matunly
z.,..
•< . .1s:
..,
"' a teclthin/sphingomyelin
..
b creaumne
ratio

c measurement of absorbance at 450 nm


d alpha-fetoprolein
• Refer to the following illustration:
206
... AlnftCtic Fl<JiO
.,
"'' c
0 Phoq>h~ld An3fysi! ;
''
~ I
~

c
I
'•
2l
c I
I

0
u
,
-.. -----... -
I
0 ;

·;:
=i ,,
15 20 25 30 35 40 45
Gestation (weeks)

The class of phospholipid surfactants represented by the dolled line on the amniotic fluid
analysis shown above is thought to originate in \l.tiat fetal organ system?
a cardiovascular
b pulmonary
c hepatic
d placental
207. Afasting serum sample from an asymptomatic 4:;.year-old woman is examined visually
~ and chemically with the following results:
Initial appearance of se rum: milky
appearance or serum after overnight refrigeration: cream layer ewer turbid serum
triglyceride levol: 2,000 mgldL (22.6 mmol/L.1
chclesterol level: 550 mgldL (14.25 mmol/L)

This sample contains predominantly:


a chylomicrons, alone
b chylomicrons and very low-density lipoproteins (VLOL)
c very low-density lipoproteins {VLOL) and low-density lipoproteins (LOL)
d high-density llpoproteins (HOL)
203. Chylomicrons are present in which of the following dyslipidemias?
a familial hypercholesterolemia
b hYJlCrtriglyceridemia
c deficiency in lipoprotein lipase activity
d familial hypoalphalipoproteinemia

tio1aAScp ISBN 978-0891~


Clinical Laborafol)' Cerlificofjon Examinations 173

b
.
3 : Ch em1stry
Endocrinology & Tum 0~ ar1t
0
,

Jl) ;Sb
low-den sity lipoprotoins (Vlo=--- ;
209. Tho !unction or the major lipid components o r th e ve ry
.....
"s transport:
a cholesterol from peripheral cells to the liver
b cholesterol and phospholipids to peripheral cells
c exogenous triglycerides
d endogenous triglycerides
210. Turbidity in serum suggests elevation of:
a cholesterol
b total protein
c chylomicrons
d albumin

Endocrinology & Tumor Markers


211 . TSH is produced by the:
a hypothalamus
b pituitary g land
c adrenal cortex
d thyroid
212. A patient has the followi ng thyroid profile:
total T,: deeteased
free T4 : decreased
thyroid peroxidase antibody: positive
TSH: decreased

Th is patient most probably has:


a hyperthyroidism
b hypothyroidism
c a normal thyroid
d Graves disease
21 3. A45-year-old woman complains of fatigue, heat intolerance and hair loss. Total and free
T 4 are abnormally low. If the TSH showed marked elevation, this would be consistent with:
a Graves disease
b an adenoma of the thyroid
c thyrotoxicosis
d primary hypothyroidism
214. The majority of thyroxine (T4 ) is converted into the more biologically active hormone:
a thyroglobulin
b thyroid-stimulating hormone (TSH)
c triiodothyronine (T 3 )
d thyrotropin-releasing hormone
215. A 2-year-old c~ild w ith a decreased serum T 4 is described as being somewhat dwa.rfed.
stocky, ove rweight, and having coarse features. Of the following the most informative
additional laboratory test would be the serum: '
a thyroxine-binding globulin (TBG)
b thyroid-stimulating hormone (TSH)
c triiodothyronine (T3 )
d cholesterol

174 The eoard o f Certification S tudy G uida 6e


~try End ocrinology & Tumor /llarkors
;_.-;; ys for free T4 measure hormone not bound to thyrox· b. d' lb .
t6 Assa . b. d·ng globulin and· me- m mg prea umm,
2 . thyroxine· in ' .
~
_.,
. h
hyrotropin-releas111g ormone
a t •
b a11>Urn1n
c freeT3 . .
d thyroid-stimulating hormone
The recommended initial th~roid function test for either a healthy, asymptomatic patient or
211. patient with symptoms which may be related to a thyroid disorder is:
8
free thyroxine (free T 4)
~ thyroid-stimulating hormone (TSH)
c total thyroxine (T •>
d triiodOlhyronine (T3)
, The screening test for congenital hypothyroidism is based upon:
216
3 free T4 level. in the newborn .
b thyroid-b1nd1ng globuhn level 1n the newborn
c thyroid-releasing hormone level in the newborn
d total thyroxine (T4) level in the newborn

219. Which one of the lo!lowing sets o f results is consistent with primary hypothyroidism, (eg,
Hashimoto thyro1d1hs):
Result TSH T, (free thyroxW!e) Antimicrosomal antibody
result A decreased decreased positive
result B increased increased posi1ive
result C normal decreased negative
re51Jf10 ineteased decrease<! poslUve
a resultA
b result B
c result C
d result D
220. A 68-year-otd female patient tells her physician of being "cold all the time· and recent
weight gain, with no change in diet. The doctor orders a TSH level, and the laboratory
reports a value o f 8.7 µUf ml (8.7 IU/L) (reference range= 0.5-5.0 µUfml [0.5-5.0 IU/L]).
This patient most likely has:
a primary hypothyroid ism
b Graves disease
c a TSH-secreting tumor
d primary hyperthyroidism
22t. Which of the following is secre ted by the placenta and osed lor the early detection of
pregnancy?
a follicle-stimulating hormone (FS H)
b human chorionic gonadotropin (HCG)
c luteinizing hormone (LH)
d progesterone
222. During pregnancy, the form o f estrogen that predominates and may be useful in prenatal
~ screening is:
a estradiol
b estriol
c estrone
d pregnanediol

c tnical Laboratory Certlrtcaffon Exami naffons 175


3: Cherni s try Endocrinoloyy & Tumor M:trho
~~~~~~:....~~~~~~~~~~~~~.,.-~~~~~~~~~~~~~~..:.:~ """""'
223.The hormone that triggers ovulation is:
a follicle·slimuloting hom1ono (FS H)
b lutein izing hormone (LH)
c thyroid-stimulating hormone (TSH)
d human chorlon lc gonadotropin (HCG)

....224.
~\Y
Refer to the following graph:

t.tonths

The HCG levels shown in the above graph most pro bably represent:
a hydatidiform mole following miscarriage at 4 month s
b normal pregnancy
c development of hydatidiform mole
d miscarriage at 2 months with retained placenta
225. In amniotic fluid. the procedure used to detect hemolytic disease of th e newborn is:
a measurement of absorbance at 450 nm
b creatinine
c lecithin/sphingomyelin ratio
d estriol
226. During a normal pregnancy, quantitative human chorion ic gonadotropin (HCG) levels peak
how many weeks after the last mens trual period?
a 2-4
b 8-10
c 14· 16
d 18-20
227. "Laborato ry A" measures maternal serum alpha-fetoprotein (MSAFP) at 16- 18 weeks'
....
..... gestation as a screen for fetal disorders. The 16-week M SAFP median for Lab A is
32 µg/L. A 37-year-old woman has an MSAFP level of 34 µg/l at her 16th week. This
result is consistent with:
a a normal MSAFP level for 16 weeks' gestation
b possible neural tube defect, including spina bifida
c possible multiple birth (ie, twin s)
d possible trisomy disorder, including Down syndrome
228. Which of the following steroids is an adrenal cortical hormone?
a angiotensinogen
b aldosterone
c epinephrine
d growth hormone

176 Tho Board of Certlncatlon Study Guide 60


r- . Cherrusu y 1
Enriocri
3· comnion substrate is used in the bio 110
ogy & Tumor Markors
•"'hat d t ? synthesis or d
219· ;~drogens an es rogens . a renal steroids, including
cortisol
a catech<>lamines
~ pr0gesterone
d cholesterol
..,,0 biologically most active, naturally occurrin d
230. "~ . 9 an rogen Is:
•' androstened1one
.,., ab · 1
corttSO
c epiandrosterone
d testosterone
p1asma for corti~ol determinations were collected at 7 . .
Jl1. 10 pm that evening. The cortisol level of the morning am, after waking the patient, and at
sample. This is consistent w ith: sample was higher than the evening
a a normal finding
b Cushing syndrome
c Addison disease
d hypopituitarism
z)2. Night blindness is associated with deficiency of wnich of the followmg vitamins?

aA
b c
c niacin
d thiamine
lll. Beriberi is associated with deficiency of vitamin:
a A
bC
c niacin
d thiamine
23'. Scurvy Is associated with deficiency of which of the following vitamins?
a A
b C
c niacin
d thiamine
235. Rickets is associated with deficiency of which of the following vitamins?
a B1
bC
c niacin
d D
?l6. Pellagra is associated with deficiency of which of the following vitamins?
a A
b 81
c thiamine
d niacin
137
· The major action of angiotensin II is:
~ ~ncreased pituitary secretion of vasopressin
increased vasoconstriclion
c Increased parathormone secretion by the parathyroid
d decreased adrenal secretion of aldosterone

Cl/n.cal Laboralo')' Certincatlon e..mlnallons 177


3: C h emishy Endocrin oloyy & T11 111 or ~
:;-:;:;--:;:;:-::---.~~~~-::-~~-:--:-~~~:--:--~--:--.;---;---:-;-:--:----;:--:;:::--:---:---:-~~Urlfe
238. The urinary excrelion p roduct measured as an Indicator of epinephrine produclion is~
" dopamine
b dlhydroxyphenylalanine (DOPA)
c homovanillic acid
d va nillylmandelic acid (VMA)
239. Test methodology for moniloring parathyroid glandular function in diagnosis o f hyp 0 . 0
hyperparalhyroidism should be focused on assessing: '
a Intact PTHrP molecule
b the C-terminal PTH
c Intact PTH amino acids 1 -84
d N-terminal PTH amino acids 1-7
240. The most common form (95% ) of congenital adrenal hyperplasia is 21 -hydroxylase
~v deficiency, which is detected by elevated plasma:

a cortisol
b aldosterone
c 17-0H-progeslerone
d 11-deoxycortisol
241 . A diagnosis of primary adrenal insufficiency requires demonstration of:
a decreased urinary 17-keto- and 17-hydroxysteroids
b decreased cortisol production
c impaired response lo ACTH stimulation
d increased urinary cortisol excretion after metyrapone
242. The screen for adrenal cortica l hyperfunction w ith the greatest sensitivity and specificity is:
a 24-hour urine free cortiso l
b plasma cortisol
c urinary 17-hydroxycorticosteroids
d plasma corticosterone
243. A patient has signs and symptoms suggestive of acromegaly. The diagnosis would be
::.Sv confirmed if the patient had which of the following?
a an elevated serum phosphate concentration
b a decreased serum growth hormone releasing factor concentration
c no decrease in serum growth hormone concentration 90 minutes after oral glucose
administration
d an Increased serum somatostatin concentration
244. Estrogen and progesterone receptor assays are useful in identifying patients who are
~' likely to benefit from endocrine therapy to treat which of the following?
a ovarian cancer
b breast cancer
c endometriosis
d amenorrhea
245. Wh ich of the following sample collections would give an accurate assessment of potential
excess cortisol production (hypercortisolism)?
a collect a plasma sample as a baseline. and another 1-hour after administration of
metyrapone
b collect a plasma sample at 8 am only
c collect a 24-hour urine free cortisol
d collect a plasma sample at 8 am and al 8 am the next day

178 Tho Board of Cortlflcat.lon S t udy Gulde 6e


ISBN 978-0891 69~9 ('20l 6.As0'
,.....- ., ,,,istry
8
3: Ci• I rt' I' E11rlocri11oloyy & Tumor Markers
::.---::;:; is primary 1ypoco. 1so ism (Addison disea .
216. ~~rtisolism (or pituitary origin)? se) differentiated from secondary
;:, adrenal corticotropic hormone (ACTH) is d _ . .
3 secondary ecre.sed tn Primary and elevated in
b adrenalcorticotropic hormone (ACTH) is elev led . .
secondary a '" Primary and decreased in
iow aldosterone and hypoglycemia present .
thith ~econdary hyp?cortisolism
cd normal cortisol levels and blood pressure w w
1 ll"mary hypocort1sollsm
Aldoslerone is released by the adrenal cortex upo 1. .
211· n s 1mulat1on by:
a renin .
b angiotens1nogen
c angiotens1n I
d angiotensin II
4.! In developing the reference for a new EIA for CEA the range f th 1 1r
;, · was broader than that pu~lished by the vendor. Centrals are a~~ep~a~::'::~thp~~~~~~n
.., coefficient of variation. This may be explained by:
a positive interference by another tumor marker
b population skewed to a younger age
c improper temperature control during assay
d inclusion of nonsmokers and smokers in the study population

2,~. Clinical assays for tumor markers are most important for:
a screening for the presence of cancer
b monitoring the course of a known cancer
c confirming the absence of disease
d identifying patients at risk for cancer
2so. Detection of which of the following substances is most useful to monitor the course of a
patient with testicular cancer?
a alpha·fetoprotein
b carclnoembryonic antigen
c prolactin
d testosterone
251. Increased concentrations of alpha-fetoprotein (AFP) in adults are most characteristically
associated with:
a hepatocellular carcinoma
b alcoholic cirrhosis
c chronic active hepatitis
d multiple myeloma
252. Carcinoembryonic antigen (CEA) is most likely to be produced in a malignancy involving
the:
a brain
b testes
c bone
d colon
lSl, Which of the following is useful in the detection and management of carcinoma or the
Prostate?
~ total prostate-specific antigen
prostatic acid phosphatase
c human chorionic gonadotropin
d alpha-fetoprotein

Clinlcal Laboratory Certification Examinations 179


3: Chemis try TOM & Tox/co~
254. Which of the following statements most correctly describes the utility of clinical labora~
assays for tumor markers? ory
a tumor markers are useful to screen zsymptomatic patients for tumors
b tumor markers are highly specific
c tumor markers indicate the likelihood of an Individual developing a tumor
d tumor markers are useful in tracking the efficacy o r treatment
255. Cancer antigen 125 (CA 125) is a tumor marker associated with:
a breast carcinoma
b colon cancer
c lung cancer
d ovarian and endometrial carcinoma
256. In addition to carcinoma of th e prostate, elevated prostate-specific antigen (PSA) can
occur due to:
a aspirin therapy
b exogenous s teroid use
c benign prostatic hyperplasia
d statin therapy (cholesterol lowering drug)

TDM & Toxicology


257. Blood specimens for d igoxln assays should be obtained between 8 hours or more after
~v drug administration because:
a tissue and serum levels need to reach equilibrium
b serum d igoxin concentration will be falsely low prior to 6 hours
c all of the digoxin is in the cellular fraction prior to 6 hours
d digoxin protein-binding Interactions are minimal prior to 6 hours
258. A drug has a half-life of 6 hours. If a dose is given every 6 hours, a steady-state drug level
would usually be achieved in:
a 3-5 hours
b 10-12 hours
c 24-42 hours
d 48-50 hours
259. Free therapeutic drug levels are usually higher when senum protein concentrations are
~v below normal. In which of the following conditions would this most likely occur?
a acute inOammation
b nephrotlc syndrome
c pregnancy
d multiple myeloma
260. Which of the following factors is not relevant to therapeutic d rug monitoring (TOM) of the
:=-:v aminoglycosides, antibiotics and vancomycin?
a intestinal absorption
b nephrotoxicity
c ototoxicity
d renal function

180 Tho Board of Cortlflcatlon Study Gulde 6e ISBN 978-089189-E009 ~ 18 ~


,,.... 3: Chem istry . . , . TDM & Toxicology
:...--:;:;:;: drug procainam1de 1s prescribed to treat .
261· Tl:r metabolile of procainamide Is often mea~~r~c.arrhythmla. Whal biologically active
11v . r simultaneously?
phenobarb1tol
3
quinidine
~ N-ace~yl procainamide (NAPA)
d ii<loca1ne
cocaine is metabolized lo:
262· .
a carbamazepine
b codeine
c hydrooodone .
d benzoylecgonone
Tile metabolite 11-nor-tetrahydrocannabinol-9-COOH .
263· •5 days after a single use of: can be detected by immunoassay
3
a melhamphelamine
b cocaine
c benzodiazepine
d marijuana

264. A 3.year-old child wa~ evaluated for abdominal ~ain and anorexia by a physician . A CBC
revealed a hemoglobin of 9.8 g/dl (98 g/L) and tasophilic stippling of the RBCs. The
dQClor should order further tests to check for poisoning from:
a arsenic
b iron
c mercury
d lead
, 265. Zinc protoporphyrin or free erythrocy1e proloporphyrin measuremenls are useful to assess
.._, blood concentrations of:
"'
a lead
b mercury
c arsenic
d beryllium
266. A carbonate salt used to control manic-depressi~e disorders is:
a digoxin
b acetaminophen
c lithium
d phenytoin
267. An antiepileptic (or anticonvulsant) used to control seizure disorders is:
a digoxin
b acetaminophen
c lithium
d phenytoin
268. A drug that relaxes the smooth muscles of the bronchial passages is:
a acetaminophen
b lithium
c phenytoin
d theophylline

C/inlo•I Laborotory C•rtiflcation Examinations 181


3: C hc1n lstry
::::-""7"~~~_::._~~~~~~~~~~~~~~~~~~-::-::-;:-:-:::;--;:;-;:::=-:=-::----·'~•n1n
011;11~
269. /I. Ci'lrdinl qlyr o"idf' thnt is ""cd 1n 1t1 p trr>ntn11•nt o f congenit;il lie,.rt fnil~•ro ."nrl ~
:"lrrhyt1>1111;ls hy incrf'n,;1119 tho foicn nnd volocily o f myocard1fll contr:i ctor1n •'l
a d•pl)>w
b ?r f'tRm ln;'phrn
c hlhni m
d phc-nyloin
270. A sahcylale level is performed 10 detect to xicity ciwsed by ingestion o f exce ss:
n acetaminophen
b aspirin
c ibuprofen
d pseudoephedrine
271 . Lithium therapy is widely used in the treatmen t of:
a hypertension
b hyperactivity
c aggression
d manic-depressive (bipola r) disorder
272. An oclivc metabolite of amitriptyline is:

"'~' a nonriptyline
b protriptyline
c butriptyline
d norbutriptyline
273. Phenobarbital is a metabolite of:
IALS
o•.v a primidone
b phenytoin
c amobarbital
d secobarbilal
274. Testi ng for the diagnosis of lead poisoning s hould include:
a erythrocyte protoporphyrin (EPP)
b urine delta -aminolevulinic acid
c whole blood lead
d zinc protoporphyrin (ZPP)

Quality Assessment
275. Blood received in the laboratory for blood gas analysis must meet which or the following
requirements?
a on Ice, thin fibrin strands only, n o air bubbles
b on Ice , no clots, fewer than 4 air bubbles
c on ice, no clots. no air bubbl es
d room temperature, no clots, no air bubbles
276. After a difficult venipuncture requiring prolonged applic ation of the tourniquot, the serulll
~~' K• w as found to be 6.8 m Eq/L (G.8 mrnol/L). Tho best course of action is to:
a repeat the test using the same specimen
b adjust the value based on the current sorum Na •
c repeat the test using freshly drawn serum
d cancel the test

182 Tho Bon rd of Cortlncotlon Study Guido Go ISBN 978·089189-GG09 C2C)tBf<SCP


r .
3.
c11er11tstry
.
serum from a patient with metastatic carcin Quality Assessment
0
211. clot and stored at room temperature The r 111a. or the prostate was se t d
"' · o 11owing res It para e 1rom tho
;«' l'•tlont value U S were obtained:
CB.. 10.8 mg/dL (2.7 mmovl) Reference range
LO 420 U/L 8.8·10.3 mg/dL (2_2• 2•6 mmoUL
3Gid p11<>spha1aso 0.1 Ull 50· 150 U/l )
0-5.5 Ull
The technician should repeat the:
3
LO using diluted serum
b acid phosphatase with freshly drawn se
c LD wilh fresh serum rum
d tests using plasma

218 A lipemic serum is separated and frozen at _ 20 .C


., · later, prior lo performing an assay for triglycerid rohr assay.at a later date. One week
,., es, t c specimen should be·
3 warmed to 37•c and mixed thoroughly ·
b warmed to 1 s•c
and centrifuged
c transferred to a glycerated test tube
d discarded and a new specimen obtained
279• The different water content of erythrocytes and la
"' in whole blood a function of the: P sma makes true glucose concentrations
""' a hematocrit
b leukocyte count
c erythrocyte count
d erythrocyte indices

280. In a specimen collected for plasma glucose analysis, sodium fluoride:


a serves as a coenzyme of hexokinase
b prevents reactivity of non glucose reducing substances
c precipitates p roteins
d inhibits glycolysis

m. As part of a hypertipidemia screening program, the following results were obtained on a


25-year-old woman 6 hours after eating:
triglyceri<kJs· 260 mg/dL (2.86 mmoVL)
cholesterol: 120 mg/dl (3.12 mmoUL)

Which of the following is the best interpretation ol these results?


a both results are normal. and not affected by the recent meal
b cholesterol is normal, but triglycerides are elevated, which may be attributed to the
recent meal
c both results are elevated, indicating a metabolic problem in addition to the nonfasting
state
d bolh results are below normal despite the recenl meal, indicaling a melabolic problem
212· Blood was collected in a serum separator tube on a patient who has been fasting since
~idnight. The time of collection was 7 am. The laboratory test which should be recollected
1s:
a lriglycerides
b iron
c LO
d SOdium

~/~Jo: Cllnlclll Laboratory Certification Ex3m/nations 183


'~ ISBll 978-069189-6609
3: Chernistry Laboratory Mat~
283. Arterial blood that is collected in a heparinized syringe but exposed to room air auc,
most consistent with the changes in which of the following specimens? "' uld ~
0

Spocimcn p02 pea, pH


A elevated decreased elevated
0 decreased elevated decreased
c unchanged elevated unchanged
D decreased decreased decreased
a specimen A
b specimen B
c specimen C
d specimen O
284. Specimens for blood gas determination should be drawn into a syrin ge containing:
MLS
°'"v a no preservative
b heparin
c EDTA
d oxalate
285. Unless blood gas measurements are made immediately after sampling. in vitro glycolysis
of the blood causes a:
a rise in pH and pC02
b fall in pH and a rise in p02
c rise in pH and a fall in p0 2
d fall in pH and a rise in pC02
286. Which of the following serum constituents is unstable if a blood specimen is left standing
at room temperature for 8 hours before processing?
a cholesterol
b triglyceride
c creatinine
d glucose
287. An arterial blood specimen submitted for blood gas analysis was obtained at 8:30 am but
was not received in the laboratory until 11 am. The technologist should:
a perform the test immediately upon receipt
b perform the test only if the specimen was submitted in ice water
c request a v e nous blood specimen
d request a new arterial specimen be obtained
288. In monitoring g lomerular function, which of the following tests has the highest sensilivity?
KS
Ot«.v a urine sodium
b BUN/creatinine ratio
c creatinine clearance
d urea clearance

Laboratory Mathematics
289. If the pKa is 6 .1 , the co2 content is 25 mM/l, the salt equals the total C02 conte~t minu~
=
~. the carbonic acid; the carbonic acid equals 0.03 x pC0 2 and pC0 2 40 mm Hg, it maY
concluded that:
a pH = 6.1 + log((40-0.03)/(0.03))
b =
pH 6.1 + log[(25-0.0 3)/(0.03))
c pH =6.1 + log((25-1.2)/(1.2))
d pH = 6.1 + log[(1 .2)/(1.2- 25))

184 Tho Board of Contncatlon Study Gulde 6<t ISBN 978-089189-6609 C'2()!SA-<d'
--'1111
~c11ernisll'Y Laboratory Mathem atics
24.hour urin~ specim_en (total volume= 1, 136 ml) is submitted to the laborato for
3

290. ~uantit~tive unn~dErotein. Calculate the amount of protein excreted per day, if thii'total
pr0tein rs 52 mg .
a 591 mg
b 487 mg
c 22omg
d 282 mg
The totlowing results were obtained:
291. .. •
IJ(ine creattnme.
90
mg/dl (7956 µmol/L)
serum eteatinine: 0.90 mgidl (79.6 µmoUL)
patie11t's lotal bOdy surface: 1.73 ml (average= l .73 ml)
toter urine volume In 24 hours: 1500 ml

GiVen the above data, the patient's creatinine clearance, in mUmin, is:
a 104
b 124
c 144
d 150
in A 45-year-old male of average height and weight was admitted to the hospital for renal
· function studies. He had the following lab results:
ume c1ealinine: 120 mg/dl (10680 µmol/L)
se1um creatinine 1.5 mg/dL (132 6 µmol/l)
total urine volume In 24 hours: 1800 ml

calculate the creatinine clearance for this patient in mUmln.


a 100
b 144
c 156
d 225
293. One international unit of enzyme activity is the amount of enzyme that will, under specified
"' reaction conditions of substrate concentration , pH and temperature, cause utilization of
"" substrate at the rate of:
a 1 mollmin
b 1 mmol/min
c 1 µmot/min
d 1 nmoVmin

..°''
294. The bicarbonate and carbonic acid ratio is calculated from an equation by:
a Siggaard-Andersen
b Gibbs-Donnan
c Natelson
d Henderson-Hasselbatch
~s. 125
1has a physical half-life of 60.0 days. A sample tested today had activity of 10,000
0u CP1'.VmL. How many days from today will the court be 1250 CPM/ml?
a 60
b 180
c 240
d 1250

~1!AScp Cllnl<al Laborotory Certlflc•lion Eramlnotion$ 185


15eH 97B-QS9189~9
3: Che1nlstry Laboratory M a t11 oma1;c$
q
296. In sriectropl1otornetric determina tion, whl:h o f the following is the formula for calcu1a1i;;g'
~v the absorbance of a solution?
o (absorptivity x light path)/concentration
b {absorptivity "concentralion)llight path
c absorptivity x light path x concentration
d (light path >< concentration)/absorptivity
297. Which of the following is the formula for calculating absorbanoe given the percent
transmillance (0/oT) of a solution?
a 1 - log{o/oT)
b log(%T)+2
c 2 >< log(% T)
d 2 - log(o/oT)
298. Which of the following is the Henderson-Hasselbalch equation?
a pK 0 = pH+ log((acid]/ [salt])
b pK 8 = pH + log([sa lt]/(acid])
c =
pH pK 0 + log([acid)J[salt])
d pH = pK 8 + log([salt]/[acid))
299. The creatinine clearance (ml/min) is equal to:
a urinary creatinine (mg/L)/(volume of urine (ml/min) " plasma crealinine (mg/L)]
b [urinary creatinine (mg/L) >< volum e (mUmin)] + plasma crealinine (mg/L))
c urinary creatinine (mg/l ) "" [volume of urine (ml/hour) x plasma crealinine (mgll)]
d [urinary crea tinine (mg/L) >< volume (ml/hour)] +plasma creatinine (mg/l)
300. An adult diabetic with renal complications has the following results :
MLS
ONLV sO<lium: 133 mEq/L (133 mmol/L)
9lucoso: 4 87 mg/dL (26.8 mmol/L)
BUN: 84 mg/dL (30 .0 mmol/L)
creallnine: 5 mg/dL (442 .0 µmol/L)

On the basis of these results, the calculated serum osmolality is:


a 266 mOsm/kg
b 290 mOsm/kg
c 323 mOsm/kg
d 709 mOsm/kg

186 The Boord o f CortlRcatlon Study Gu lde 6e ISBN 978-0!!91~-6609 02()!&..SCI'


.
Laboratory Mal/1omatics

1
v

0 1
[SJ

me above figure shows the reciprocal of the measured velocity of an enzyme reaction
plotted against the reciprocal of the substrate concentration.
rrue statements about this figure include:
a the intercept. of t_
h e Une on the abscissa (x-axis) can be used to calculate the V max
b the straight line indicates that the enzyme reaction proceeds according to zero order
kinetics
c the intercept on the abscissa (x-axis) can be used to calculate the Michaelis-Menten
constant
d the fact that the substrate concentration is plotted on both sides of the zero point
indicates that the reaction is reversible
io2. The following results were obtained in a creatinine clearance evaluation:
:,S, urine concentration: 84 mg/dl
urine volume: 1,440 mL/24 hr
serum concentration: 1.4 mg/dL
body surface area: 1.60 mZ (average = 1.73 mZ)

The creatinine clearance in mUmin is:


a 6
b 22
c 60
d 65
3U. In the International System of Units, serum urea is expressed in millimoles per liter
~~ (mmol/L).
urea: NHi CONHz
atomic weight: N = 14 , C = 12, 0 = 16, H = 1

A serum urea n itrogen concentration of 28 mg/dl would be equivalent to wh at


concentration of u rea in SI units?
a 4.7 mmol/l
b 5.0 mmol/l
c 10.0 mmol/l
d 20.0 mmol/l
~· Calculate the blood pH given a pco2 of 60 mm Hg and a bicarbonate of 18 mmol/l :
"" a 6 .89
b 7.00
c 7.1 0
d 7 30

Cr"<!JtiA5C1> Clinical Laboratory Certification Exam inations 187


k ISBN 978-089189-6609
""'11111
3: Chemistry l11s tr11n1on1a11ofl
305 T I s the calcu lated Osm/kg. Norrn:;;-.......
· he osn101 gap is defined as measured Osm/kg m nu a1ly,
:;;;~. the osmol gap is less than:
a 10
b 20
c 40
d 60
3 . · b t 24 mEq/L a nd th e ca rbon ic a cid
06. N o rma lly the bica rbo nate concentratio n is a o u . th equa tio n
=
concentration is a bout 1.2; pK 6.1, log 20 1.3. U sing e =
pH =pK + log(sall)/[acid),
calculate the pH.
a 7.26
b 7.36
c 7.40
d 7 .42

Instrumentation
307. Stray light can be detected in a spectrophotometer by utilizing a:
°""'
"" a mercury vapor lamp
b holmium o xide glass
c potassium d ichromate solution
d sharp cutoff filter
308. In the atomic absorption method for calc ium, la nthanu m is used:
.....
""'v a as an internal sta ndard
b to bind calcium
c to eliminate protein Interference
d to prevent phosphate interference
309. Which of the following methods is susce pt ble to the solvent displacing effect that results in
falsely decreased electrolyte values?
a Ind irect ion-selective electrodes
b d irect ion-selective electrodes
c alkaline electrophoretlc separation of Ions
d fluorescence
310. Upon development of a thin-laye r chromat:igram for d rug analysis all d rug spots (including
MLS
ON.Y the s tandards) had migrated with the solvent front. The m'ost pro bable ca use for this would
be:
a environmental temperature too warm
b incorrect aqueous to nonaqueous solvent mixture
c too much sample applied
d chromatogram dried too quickly
311. To det~ct barbitural~ abuse when analyzing urine speclmens, immunoassay is the method
of choice for screening. The method of choice fo r confirmation is:
a nephelometry
b thin-layer chroma tography
c gas chromatography/mass spectrometr/
d ultraviolet absorption spectroscopy

188 T he Bo ard o f Certification Stud~ Gulde 6 o


ISBN 978·089 169.U09 0201aASCI'
. c11emislr)' . . Instrumentation
3. e phase hrgh·pert:ormance hquid chroma! . . . . .. .
1. ~e...ers lie drug monitoring. The term rever ~fTaphy is berng rncreasrngly ulrhzed rn
J1 mefllpeu se P ase rmphes Iha! the column eluant is:
1
~· umped up the column .
a P,ore polar than the stallonary phase
b n~vays nonpolar
~ ~ss polar than the stationary phase
wnen s~P:~11::u~~~r;r~:;:: by cellulose acetate electrophoresis, using veronal buffer
Jti
8
at pH · •
faster 111an albumin
~ slower than gamma globulin
raster than gamma globulin
~ faster than alpha-2 globulin
• Hemoglobin S can be separated from hemoglobin D by which of the following methods?
314
a dtrate<lb> agar gel electrophoresis at pH 5.9
b thin-layer chromatography
c alkali denaturation
d agarose gel electrophoresis at Ph 8.4
JIS. Whal is the proper pH for the buffered solution used 10 perform serum protein
electrophoresis?
a 5.6
b 7.6
c 8.6
d 9.6
JI&. The buffer pH most effective at allowing amphoteric proteins lo migrate toward the
cathode in an electrophoretic system would be:
a 4.5
b 7.5
c 8.6
d 9.5
317. On electrophoresis, transient bisalbumin emia or a grossly widened albumin zone is
:.. associated with:
a dirty applicators
b presence of therapeutic d rugs in serum sample
c endosmosls
d prestaining with tracer dye
lll Which of the following serum proteins migrate w ith the beta.globulins on cellulose acetate
~ at pH 8.6?
aceruloplasmin
b hemoglobin
c haptoglobin
d CJ component of complement
319· An electrode has a silver/silver chloride anode and a platinum wire cathode. It is
suspended in KCI solution and separated from Iha blood to be an.alyzed by a sel~lively
llemleabte membrane. Such an electrode is used to measure which of the followrng?
a pH
b pC02
c P02
d HC03

-.. · ., • . . ,.,.,.,.,Ml' Certlflcation Examinations 189


3: C h e1nistry lfls triun""'
auo
q
320. Hydrogen ion concenlralfon (pH) in blood Is usually determined by means or Which~
rollowing electrodes? e
a silver
b glass
c platinum
d platinum-lactate
321. An automated method for measuring chloride which generates silver ions in the reaction
is:
a coulometry
b mass spectroscopy
c chromatography
d polarography
322. Goulometry is often used to measure:
a chloride in sweat
b the pH in saliva
c bicarbonate in urine
d ammonia in plasma
323. In a pH meter reference electrodes may include:
a silver-silver chloride
b quinhydrone
c hydroxide
d hydrogen
324. Amperometry is the principle of the:
......
oNLY a pC0 2 electrode
b p0 2 electrode
c pH electrode
d Ionized calcium eleclrodo
325. Most automated blood gas analyzers directly measure:
a pH. HC03 and 0/o 0 2 saturation
b pH, pC02 and p02
c HC03. pC02 and p02
d pH, p02 and % 0 2 saturation
326. Blood pC0 2 may be measured by:
a direct colorimetric measurement of dissolved C02
b a self-contained potentiom etric electrode
c measu rement of C02-sa turated hemoglobin
d measurement of C02 consumed at the cathode
327. Valinomycin enhances the selectivity of the electrode used to quantitate:
a sod ium
b chloride
c potassium
d calcium
328. Which blood gas electrode is composed of a semi-permeable m embrane, a silver/silver
ch loride reference electrode and glass electrode?
a p02
b % 0 2 Sat
c pC02
d HC03

190 Tho Board of Certification Study Guido &e


,. . chem1s11 1
3
· iosl chemical methods for determining t0 1 lnstrumonliJlion
1'J l< ? a1Protein til"
l · reactiOnS · u •ze which of lhe following
molybdenum ~Jue
3
b ferri-ferrocyamde
c resorcinol-HCI
d biuret
,.., eromcresol purple al a pH of 5.2 is used in a colori .
,,.. . metnc method to measure·
3 albumin ·
b gJobUlin
c Bence Jones protein
d immunoprotein

331. Magnesium carbonate is added in an iron binding capac·1y d . . •


r e1erm1nation rn order to:
a allow color to develop
b precipitate protein
c bind with hemoglobin iron
d remove excess unbound iron

332• The most specific method for the assay or glucose in all body fluids utilizes:
a hexokinase
b glucose oxidase
c glucose-6-phosphatase
d glucose dehydrogenase
333, Which of the following would be an example of a glucose-specific colorimetric melhod?
"' a alkaline ferricyanide
""
b glucose oxidase
c hexokinase
d o-loluidine
334. Increased concentrations or ascorbic acid inhibit chromogen production in which of the
~ following commonly used glucose methods?
a rerricyanide
b ortho-toluidine
c glucose oxidase (peroxidase}
d hexokinase
335. In the hexokinase method for glucose determination, the actual end product measured is
~ the:

a amount of hydrogen peroxide produced


b NADH produced from the reduction of NAO
c amount of glucose combined with bromcresol purple
d condensation of glucose with an aromatic amine
336. Which of the following calcium procedures utilizes lanthanum chloride to eliminate
rnterfering substances?
a o-cresolphlhalein complexone
b precipitation with chloranilic acid
c chelation with EDTA
d atomic absorption spectrophotometry

. .. _, __ ,
1
.horatory C•rfffioatlon Enmlnations 191
3: C h ernistry --.........._
337 lns truni-
0
~
· nB,erotreb unconjugated b ilirubin can react with Ehrlich diazo reagent, which or the f " 10110,.
us e added? Ollow;rig
a acetone
b e ther
c distilled water
d caffeine
338.
Th e most w idely used methods for bilirubin measurement are those based on the·
a Jaffe reaction
b Schales a nd Schales method
c 8-hydroxyquinoline reaction
d Jendrassik-Gror method
339. In the Malloy and Evelyn method for the determination or bilirubin. the reagent that is
reacted with bilirubin to form a purple azobllirubin is :
a dilute sulfuric acid
b d iazonium sulfate
c sulfobromophthatein
d diazotized sulfanilic acid
340. In the Jendrassik-Grof method ror the determination o f serum bilirubin concentration
quanti!ation is obtained by measuring the green color or: ·
a azobilirubin
b bilirubin g lucuronide
c urobilin
d urobilinogen
341. In the Jendrasslk-Grof reaction for tolal bilirubin, alkaline tartrate is added to:
a form diazo bilirubin, a reddish c hromogen
b ellmlna!e many speclrophotometric interferences
c act as an accelerator
d react with delta-bilirubin
342. In the assay of lactate d ehydrogen ase. which of the fo llowing products is actually
measured?
a NAOH
b ATP
c lactic acid
d pyruvic acid
343. In the assay of lactate dehydrogenase {LO), the reaction is dependent upon which or the
following coenzyme systems?
a NAD/NAOH
b ATP/ADP
c Fe2•tFe3•
d Cu/Cu2•

192 The Board o f Certlllcalion Study Guide 6e


ISBN 978-089189-6609 e2018¢
'E
E .80 '- '-
~ .70
;- .60
~ .50
£ .40
.30 .30
<(

1 2 3 4 5 6 7 8
Minutes

This illustration represents the change in absorbance at 340 nm over a period of 8 minutes
ill an assay for lactate dehydrogenase.
True statements about this figure include:
3 the reaction follows zero order kinetics between s and a minutes
b the reaction is proceeding from lactate to pyruvate
c nonlinearity after 6 minutes Is due to substrate exhaustion
d the change in absorbance is due to reduction or NAO to NADH

345. In competitive inhibition of an enzyme reaction, the:


~ a inhibitor binds to the enzyme at the same site as does the substrate
b Inhibitor often has a chemical structure different to that of the substrate
c activity of the reaction can be decreased by increasing the concentration of the
substrate
d activity of the reaction can be increased by decreasing the temperature
3-15. The most common cause of rapid nonlinearity following the timed lag phase in an enzyme
..,
"' klnetic assay is resolved by:
a addition of more coenzyme such as NADH to the reaction following the timed lag
phase.
b preventing extraneous protein from binding to the E-S complex by making a protein free
filtrate.
c decreasing the sample volume to dilute the enzyme so that
substrate remains in excess during the reaction.
d eliminating metallic ions that may be making structural changes and inhibition to the
enzyme active site.
347. The International Federation for Clinical Chemistry {tFCC) recommends the use of
methods such as the Bessey-Lowry-Brock method for determining alkaline phosphatase
activ~y. The substrate used in this type of method is:
a monophosphate
b phenylphosphate
c disodium phenylphosphate
d para-nilrophenylphosphate

Clinloal uboratory C•niflc•tlon Examinations 193


3: Chen1 i s t ry -
lnstrum~nt . """"'111111
348. The illustr
.,,. . - b o 1ow reprosents a Lineweaver-Burk plot o f 1/v vs 1/ [S] in an enzy
n 1ion auo,,
<Nev ro<ichon and the following assumptions should be made: rne

1
v

0 1
{SJ

The enzyme concentration was the samg for reactions A and B


The subsl rale concenlra tion was in exces s ror reactions A and B
Reaction A occurred under Ideal conditions
Which of the following statements aboul reaction B is true?
a it illustrates no ncompetitive inhibition
b it illustrales compelitive inhibition
c it illustrales neither competilive nor noncompetitive inhibition
d ii could be the result of heavy melal contamination
349. Which of Iha following is a g lycolytic enzyme l hat c atalyzes the cleavag e of fructose-1 6-
diphosphate to glyceraldehyde-3-phosphale and dihydroxyacetone phosphate? '
a aldolase
b phosphofructokinase
c pyruvate kinase
d glucose-6-phosphate dehydrogenase (G6PD)
350. The substance that Is measured to estimale the serum concentration of triglycerides by
m ost methods is:
a phospholipids
b g ly<;erol
c fatty acids
d pre-beta lipoprotein
351. Which of the following methods for quantitation of high-density lipoprotein is most suited
for c linical laboratory use?
a Gomori procedure
b homogeneous
c column chromatography
d agarose gel e lectrophoresis
352. A chemiluminescent EIA:
a measures absorption of light
b is less sensitive than radioisotopic reactions
c is monitored by the use of a gamma counter
d is quantitated by the amount of light produced by the reaction

194 The Board ol Certification Study Guido 6e


ISBN 97&-0891~
~ ernlsll'Y . . /11s trr11nc11 t.11/0 11
3: Cll olalitY of a urine or serum specimen is measured by a change in the:
r11e osm .
~ zin9 point .
a tree·mentation point
b 5~ int
1

c 01id~ic pressure
d osl'flO .
• n of the following applies to cryoscopic osmometry?
·"
)>".
wn1c erature al equ1 ·1·b .
1 num .is a function
. of the number of particles in solution
a temPerature plateau for a solution is horizontal
b 1emP·ng point of a sample is absolute
d 1n111a1
c fre~z freezing of a sample produces an immediate solid state
1

.. . .
of transketolase act1v1ty 1n blood is used to detect deficiency of: :l
lSS-, p.SS3Y
,., •
thiamine ·.1
"" : folie acod .
c ascorbic. acid
d ribOflavin •
niotic fluid, the procedure used to detect Rh isosensitization is: ~
- ~ am ~
., h man amniotic placental lactogen (HPL) •3
"" ab alpha·fetoprote1
u .n ~
1

measurement of absorbance at 450 nm


c ..
d creat1mne

I
~
~llASCP IS&l978--069189·6609 Clinical Laboratory Certlflcauon Examinations 195
t11s tru11·1cn tation

261. c 313. c -
3: Chemistry 209. d 314. a
157. b 262. d
1. c 53. d 105. c 210. c 263. d 315. c
106. d
158. b 211 . b 316. a
2. c 54. d 159. d 264. d
55. d 107. a 21 2. b 265. a 317. b
3. a 160. a 213. d 318. cl
4. b 56. i) 108. b 161. b 266. c
109. a 214. c 267. d 319. c
5. b 57. b 162. b 215. b 320. b
58. d 110. d 163. a 268. d
6. c 216. b 321. a
59. a 111 . c 164. d 269. a
7. b 112. d 217. b 322. a
8. d 60. b 165. a 270. b
113. d 218. d 211. d 323. a
9. b 61. c 166 d 219. d 324. b
62. c 114. c 167. d 272. a
10. a 115. b 220. a 325. b
63. b 168. d 273. a
11. d 116. d 221. b 326. b
64. b 169. a 274. c
12. b 117. b 222. b 275. c 327. c
13. d 65. a 110. b 223. b 328. c
118. b 276. c
14. b 66. a 111. b 224. b
67. a 119. c 172. a 277. b 329. d
15. c 120. d 225. a 278. a 330. a
16. d 68. b 173. a 226. b
121. b 279. a 331. d
17. c 69. c 174. c 227. a
122. b 280. d 332. a
18. a
70. a 175. a 228. b
123. c 281. b 333. b
19. d 71. c 176. d
229. d
72. a
124. a
111. c 282. a 334. c
20. a 125. b 230. d 335. b
73. b 178. a 283. a
21. d
74. d 126. c 179. c
231. a 284. b 336. d
22. a 127. a 232. a 337. d
23. a
75. b 180. d 285. d
128. d 233. d 338. d
76. b 181. b 286. d
24. d 129. b 234. b 339. d
25. a 77. a 182. a 287. d
130. a 235. d 340. a
26. a
78. b 183. d 288. c
c 131. c 236. d 341. b
27. a 79.
132. d
184. b
237. b 289. c
28. c so. b
133. c
185. b
238. d 290. a 342. a
81. c 186. d 291 . a 343. a
29. a 134. d 239. c 344. c
30. a 82. a 187. b 292. a
135. a 240. c 345. a
31. b 83. a 136. b 188. c 293. c
84. c 189. d
241 . c 346. c
32. c 137. d 242. a 294. d
33. a 85. d 190. a 295. b 347. d
86. a 138. c 243. c
34. a 191. c 296. c 348. b
139. b 244. b
35. d 87. d 192. b 297. d 349. a
140. d 245. c
36. d 88. c 193. c 298. d 350. b
89. d 141 . a 246. b
37. a 194. d 299. b 351. b
90. c 142. b 247. d
38. d 195. d 300. c 352. d
91. a 143. d 248. d
39. c 144. d 196. a 301 . c 353. a
40. d 92. d 197. c 249. b 354. a
93. b 145. c 250. a 302. d
41. b 146. a 198. a 303. c 355. a
42. c 94. c 199. c 251 . a 356. c
95. b 147. c 252. d 304. c
43. c 200. c
44. d 96. c 148. b 253. a 305. a
149. b 201. c
45. b 97. b 202. d 254. d 306. c
98. c 150. c 255. d 307. d
46. c 203. b
99. c 151. b 308. d
47. c 204. b 256. c
48. b 100. d 152. c 257. a 309. a
101. d 153. b 205. a 310. b
49. c 206. b 258. c
so. c 102. c 154. c 311. c
207. b 259. b
51. d 103. b 155. d 312. b
156. a 208. c 260. a
52. c 104. d

ISBN 976-089189-6609 C2018 p.._cCf


196 The Board of Certification Study Guido 6e
1110 1o~g~y~----:-------------=E:.:.ry:.t::.lt::.ro:c:::'.y:. :.1::.:cs::::..:.P:..:.l~y1_:s:_:lo:.:.lo::.:y:::.y
ffematology
. ns have been. identified generally as appropriate for both entry level medical
• g1te1
fb[IO•¥"' . tists and medical laboratory tectinlclans. Items that are appropriate for mcclicat
ffl!,,,,.fOl't seientSIS on/ are marked wil/1 an 'MLS ONLY.·
13r, .-1 seien 1
tJIJ~
· ns 290 Answers wifh Explanations
aues/IO .I 291 ErythfOC';tcs: Physiology
119 111rocrtcs: PhySJO ogy
zt9 Err res: Disease Stotes 291 Eryrl1mcyfes: OiS(10S() SllltC:,
221 EJYl;:;tes: Laboratory Determinations 294 Eryt11rocytfls: Laboroto1y Dclorm'oMions
133 EfYI ytes· Physiology 296 Leul<ocyte~ · Pllysiclogy
z,3 LeukOC es: Disease States 297 LCukocy1os. Dise<1sc S12lcs
216 Leuk~es: Laboratory Determinations 300 Loukocy1es. t::ilioratc.ry Dote11ninations
151 Leuk , '~Y Laboratory Operations 301 llomatofogy labom:ory Operations l
3 Hema.a,.,,,
26 relets· Physiology 301 PIJtelets: Physiology
168 Pia
214 Pfa/e"'
.,s.:
1 Laboratory
.
Detennlnations 304 P/Jtelets: Laboratory Determhwtions
HefnOslBSIS 306 Hemostasis
211

Ety111roCytes: Physiology

.. I.
The rollowing are compounds formed in the synthesis of heme:

.., 1 coproporphyrinogen
2 porphobilinogen
3 uroporphyrinogen
4 protoporphyrinogen
Which of the following responses lists these compounds in the order in which they are
rormed?
a 4, 3, 1, 2
b 2. 3. 1, 4
c 4, 2, 3, 1
d 2, 1, 3, 4
t The majority of the iron in an adult is found as a constituent of:
a hemoglobin
b hemosiderin
c myoglobin
d lranslerrin
1
The main function of the hexose monophosphate shunt in the erythrocyte is to:
~ regu!ate the level of 2,3-DPG
c ~l'OVide reduced glutathione to prevent hemoglobin oxidation
d revent the re<!uction of heme iron
Provide energy for membrane maintenance

Cllnl~aJ Laboratory Cor6fica6on Examinations 219


4 : H e1natology fErytlirocytos: Phy
S 1O/o
l/y
4. Rorer to the following illuslration:

Which curve represents the production of alpha polypeptide ch ains of hemoglobin?


a A
b B
c c
d D
5. Refer to the following illustration:

.~ A .,.B
°"....
"' 50
40 :,.········-····~.•, 'f- 0
~~ 30· f I\, .. C
a ID 20
~0. 1 0 \!( E
;:;- 0.1.f:~;:::::::;::::~-=--"-~··~~~:1-~_J
0. -6 -3 birth 3 6
'ff. (Monlhs)

Which curve represents lhe production of beta polypeptide chains of hemoglobin?


a B
b c
c E
d D
6. Refer to the following illustration:

en
c: A B
~ so ......·-······· I
u 40 1' ··...,, ~D
~<=30
.!:::' Q) :
! \ ••
a.ID 20 \ _,.c
8_5.10 \~ E
~ olf::~::::::;:::::i_.-~-~~~:1-[_J
~ -6 -3 birth 3 6
(Months)

Which curve represents the production of gamma polypeptide chains of hemoglobin?


a A
b B
cc
d D

220 The Boa rd of Cer1lficatio n Study G uido 60


ISBN 978-0891~ 02f)tS.4SCI' J
P atol oQY
4· HerTl . Erythrocytes· Physiology
• der for hemoglobin lo combine reversibly with . · ·
In or . , oxygen, the iron must be·
1. coniplexed with haptoglob1n ·
a freely circulating in t~e cytoplasm
b attached to transfernn
~ in the ferrous state
hich of the following disease stales are leardr
s. ~:racteristically seen? op cells and abnormal platelets most
1

hemolytic anemia
~ multiple my~loma
G6PD deficiency
~ primary myelofibrosis
The characteristic erythrocyte found in perniciou~ anemia Is:
9.
• :::, a microeylic
b spherocytlc_
I .,
c hypochrom1c
d macrocytic II I
1
.
' 10.
In the normal adult, the spleen acts as a site for: I,
a storage of red blood cells
b production of red blood cells
c synthesis of erythropoietin
d removal of imperfect and aging cells
,_,11. After the removal of red blood cells from the circulation hemoglobin is broken down into:
a iron, porphyrin, and amino acids
b iron, protoporphyrin, and globin
c heme, protoporphyrin, and amino acids
d heme, hemosiderin, and globin
," 12. Heinz bodies are:
a readily identified with polycl1rome stains
b remnants of RNA
c closely associated with spherocytes
d denatured hemoglobin inclusions that are readily removed by the spleen
13. Hemolysis in paroxysmal nocturnal hemoglobinuria (PNH) is:
ltS
"'' a temperature-dependent
b complement-independent
c antibody-mediated
/ d caused by a red cell membrane defect
/14. Cells for the transport of 0 2 and C02 are:
a erythrocytes
b granulocytes
c lymphocytes
d thrombocytes
15
/ • Erythropoletin acts to:
a s~orten the replication time of the granulocytes
b stimulate RNA synthesis of erythroid cells
c increase colony-stimulating factors produced by the B lymphocytes
d decrease the release of marrow reticulocytes

rUnlr1tl I ,_ bOflJ ,Orv


Certification EJtamlnations 221
4 · H ema to logy
. Eryt/1rocytfl'I~
;·~~~ ~
16. What cell shape 1s most commonly assocl<i ted with an increased MCHC? Y
a leardrop coils
b target cells
c spherocyles
d sickle cells

...17., Which of the following 1s most closely associated with idiopathic hemochromatosos?
""' a iron overload in tissue
b larget cells
c basophilic stippling
d ringed sideroblasts
18. Which description best fits the Donath-Landsteiner antibody?
a lgM cold agglutinin
b biphasic lgM hemolysin
c lgG biphasic hemolysin
d lgG warm agglu linin
19. Which of the following represent residual nuclear fragments?
a Pappenheimer bodies
b Cabot rings
c Heinz bodies
d target cells

20. Which of the following RBC inclusions are seen in sideroblastlc anemia and conlarn high
a mounts or iron?
a Cabot rings
b Howell -Jolly bodies
c Heinz bodies
d Pappenheimer bodies
21 . The nuclealed cell in this image may be seen in the peripheral blood of a normal newborn
and is classified as a(an):

a basophilic normoblast
b polychromatophilic normoblast
c orthochromat1c normoblast
d megaloblAslic normoblast

222 Tho Bon rd of Certlfic.allon Study Gulde Ge


ISBN 978·089189·6609 Cl20 l8ASCI'
c1
,...- Hemat ology
4:
-Tne abnormfll erylhrocyte on the conte 1 or thl
Lryt111ocyte.~· P/lyslology
.'.::"'7"""----.:..._.:_:__:__.:_:_
22 following processes? s lm,igo (11rrow) may resull from which of tho


a deficiencies of cellular membrane proteins
b absence of plasma lipoproleins
c defects in the cellular lipid btlayer
d deficiencies of cellular enzymes
2l What is the composition of the inclusion seen m this RBC?

a DNA
b RNA
c iron
d denatured hemoglobin
24. Which or lhe following ions is bound to hemoglotin in melhemoglobin?
a Ca2•
b fe3•
c fe3•
d Mg2•

O'loisASCp ISBN 978·089189·6009 Cllnfcnl Lat>orat·o ry Certification Examlnat;ons 223


_4 : _____
He 111atology Erythrocytas:
::.:___________ _ _ _____ __ _ _ __ _._:::_C:_:Stares Dis~
Erythrocytes: Disease States ----....
A patient with polycylhemia vera who is treated by phlebotomy is most likely 10 dev lo
....""y
25.
deficiency of: e Pa
a iron
b vitamin B 12
c folic acid
d erylhropoietin

....""'v
26. The direct anliglobulin lest ca n help d islin;iuish:
a inherited from acquired spherocytosis
b inlravascular from extravascular hemolysis
c heterozygous from homozygous lhalassemia
d sickle cell trait from sickle cell disease
27. The anemia of chronic infection is characterized by:
,,; t:.' ltS
""'v a decreased iron stores in the reticuloendotheli al system
b decreased serum iron levels
c macrocytic erythrocytes
d increased serum iron binding capacity
28. Factors common ly involved in producing anemia in patients with chronic renal disease
""'
OK.Y
include:
a marrow hypoplasia
b inadequate erythropoiesis
c vitamin 8 12 deficiency
d increased erylhropoietin production
29. A 20-year-old woman with sickle cell anemia whose usual hemoglobin concentration
t.llS
OHl.Y
is 8 g/dl (80 g/L) develops fever, increased weakness and malaise. The hemoglobin
./ concentration is 4 g/dl (40 g/L) and the reliculocyte count is 0.1 %. The m ost likely
explanation for her clinical picture is:
a increased hemolysis due to hypersplenism
b aplastic crisis
c thrombotic crisis
d occult blood loss
30. The hypoproliferative red cell population in the bone marrow of uremic patients is caused
MLS
ONLY
by:
a infiltration of bone marrow by toxic waste products
b decreased levels of circulating erylhropoielin
c defective globin synthesis
d overcrowding of bone marrow space by increased myeloid precursors
31 . Which of the following characteristics are common to hereditary spherocytosis, hereditary
../
elliptocytosis. hereditary stomatocytosis, and paroxysmal nocturnal hemoglobinuria?
a autosomal dominant inheritance
b red cell membrane defects
c positive direct anliglobulln test
d measured platelet count

224 Tho Board of Certilicatlon S tudy Guido 6e ISBN 978-08918~09 0'20l8 ¢


,..- H rnatology
4: e .
...-: 89.year-old Caucasian female was transl
Erytllrocytes: Disoaso States
31· Anatment or chronic urinary tract infection wi:;;red 10. Iha. hospital from a nursing facility for
..., fol
II'' trelOWI·ng laboratory data·• proteinuna. The patient presented with the
wee: 10.0 • 103111L 110.o . 109,L)
RSC: 3 .1 • 10'/µL (3.1 x 1012/L)
Hgb: 7.2 g/dL (72 g/L)
Het: 24%
MCV: 78 µm3 (78 fl)
MCH: 23 pg
MCHC: 31%
serum Iron: 29 µgldL (5.2 JJffiOllL)
nee: 160 µgldL (28.6 µmol/L)
serum ferritin: 100 nglmL (100 µg/L)

These data are most consistent with which of the following conditions?
a iron deficiency anemia
b anemia of chronic inflammation
c hemochromatosis
d acute blood loss

33. A patient is admitted with a history of ch~onic b.leeding secondary to peptic ulcer.
Hematology workup reveals a .severe m1crocyt1c, hypochromic anemia. Iron studies were
requested. Which of the following would be expected in this case?
Scrum iron TIBC Storage hon
resull A decreased incteased increased
result e increased decteased increased
resull C decreased ineteased decreased
resullD increased normal decreased
a result A
b result B
c result C
d result D

1y 34:' Which of the following is most closely associated with iron deficiency anemia?
a iron overload in tissue
b macrocytes
c basophilic stippling
d chronic blood loss
35. Which one of the following hypochromic anemias is usually associated with a normal free
~' erythrocyte protoporphyrin level?
a anemia of chronic disease
b iron deficiency
c lead poisoning
d thalassemia minor
./36:" Evidence indicates that the genetic defect in thctassemia usually results in:
a the production of abnormal globin chains . . .
b a quantitative deficiency in RNA resulting in decreased globm chain production
c a structural change in the heme portion of the hemoglobin
d an abnormality in the alpha- or beta-chain binding or affirl'Ay

~18 Clinfc11l l..aboratory Cortlflcation Exan1/natlons 225


ASCP ISSN978-089189-6609
~
4 : H ematolog y Erytllrocytcs: Disease S~
::-~-:--:::-:-~~~7:-:-:--:-:-:-:::=-::::=:-::==-::::-::::;:=:::::-;-:::;::::;-:;:-:::::-:-:-:-:=::-=-:-::-~-.::..:._::'.'ares
37. A 20-year-old African-America_n man has perip.h eral bloo~ changes suggesting --.....
"'
0
th alossemia minor. The quantitative hemo9 lob111 A2 level is normal. but the hemoglob·
"' level is 5% (normal <2%). This is most consistent w ith: •n F
a alpha thalassemia minor
b beta thalassemia m inor
c delta-beta thalassemia minor
d hereditary persistence of fetal hemoglobin
38. Anemia secondary to u remia characteristically is:
°""
•'l$
a microcytic, hypochromic
b hemolytic
c normocytic, normochromic
d macrocytic
39. Which of the following sets of laboratory findings is consistent w ith hemolytic anemia?
lllS
oiu a normal or slightly increased erythrocyte survival; normal reliculocyte count
b decreased e rythrocyte survival; increased catabolism of heme
c decreased serum lactate dehydrogenase activity; normal catabolism of heme
d normal concentration of haptoglobin; marked hemoglobinuria
40. A n enzyme deficiency associated with a moderate to severe hemolytic anemia after the
patient is exposed to certain drugs and c haracterized by red cell inclusions formed by
denatured hemoglobin iS:
a lactate dehydrogenase deficiency
b G6PD deficiency
c pyruvate kinase deficiency
d hexokinase deficiency
41 . Patie nts with G6PD deficiency are least likely to have hemolytic episodes in which of the
following situations?
a following the administration of oxidizing drugs
b following the ingestion of fava beans
c during infections
d spontaneously
42. A patient has a congenital nonspherocytic hemolytic anemia. After exposure to
anti-malarial drugs the patient e xperiences a severe hemolytic episode. This episode is
characterized by red cell inclusions caused by hemoglobin denaturation. Which of the
following conditions is most consistent with these findings?
a G6PD deficiency
b thalassemia major
c pyruvate kinase deficiency ·
d paroxy~mal noctu rnal hemoglobinuria
' 43. All of the findings li'sted below may be seen in autoimmune hemolytic anemias. The one
Ml.$
considered to be the most characteristic is. r\ ;
°"" a increased reticulocyte count
b leukopenla and thrombocytopenia
c · peripheral spherocytosis
d positive direct antiglobulin test
44. Pe ripheral blood smears from patients with untreated pernicious anemia are characterized
by:
a pancytopenia and macrocytosis
b leukocytosis and elliptocytosis
c le ukocytosis and ovalocytosls
d pan'cytopenia and mlcrocytosis

226 Tho Board of Certlncatlon Study Gulde 6" ISBN 978-089189-6609 e20l BASCP
4: tte••·...-·-· - ...,,
LabO<atory tes~s perfor~ed on a Patient Ind' Eryt11rocytes: Disease Stoi cs
i's. the following disorders is most likely? ica1e macrocylosis and poncylopenia. Which of
anemia of chronic disease
~ vitamin 8 12 deficiency
c iron deficiency
d acute hemorrhage
A patient has the following laboratory data:
j6.
RSC: 2.35 x 10 8/µL (2.35 x IO"iLJ
wee· 3.o x 1031µL (3.o. 10o1Ll
Pit 95.0 • 103/µL (95.0 x IO'IL)
Hgb: 9.5 gldL (95 glL)
HCI'. 27%
MCV. 115 µm3 (115 IL)
MCHC: 35"•
MCH: 40 pg

Which of the following tests would contribute Iowa d th d. .


r e 1agnos1s?
a reticulocyte count
b platelet factor 3
c serum 812 and folate
d leukocyte alkaline phosphatase
47. The characteristic morphologic feature in folic acid deficiency is:
a macrocytosis
b target cells
c basophilic stippling
d rouleaux formation
48. A50-year-old patient was found to have the following lab results:
~\S
,,._, Hgb: 7.0 g/dl (70 g/L)
Het 20%
RBC: · 2.0 x 106/µL (2.0 x 10>2/L)

It was determined that the patient was suffering from pernicious anemia. Which of the
following sets of results most likely was obtained from the same patient?
WBCs Platelets Rellculocytes
result A 17,500 350,000 5.2%
result B 7,500 80,000 4.1%
cesult C 5.000 425.000 2.9%
result D 3,500 80,000 0.8%
a resultA
b result 8
c result C
d result O
49. The most likely cause of the macrocytosis that often accompanies primary myelofibrosis
is:
a folic acid deficiency
b increased reticulocyte count
c inadequate 8 12 absorption
d pyridoxine deficiem;;y

Cli•leal LabOrata<y Corlifleatlon Examination• 227


Erythrocytes: Disease S ·~
4: Hem ato logy . . . fo1011
d velopment in the bone marrow indicates which one~
50. Meg;iloblastic asynchronous e the
......
..... following?
a proliferation of erythrocyte precursors
b impaired synlhes1s of ONA . .
c inadequate production of erythropo1etin
d deficiency of G6PD
51. Which of the following are found in association with megaloblastic anemia?
~. a neutropenia and thrombocytopenia
b decreased LO activity
c increased erythrocyte folate levels
d decreased plasma bilirubin levels
52. Which of the following represents characteristic features of iron metabolism in pat•ents
with anemia of chronic disease?
Serum iron Tr•nslerrln saturati on TIBC
result A normal normal normal
result B increased Increased normal or slightly increased
result C normal ma rkedly increased normal
result O decreased docrease:t normal or decreased
a result A
b result B
c result C
d result D
53. A characteristic morphologic feature in hemoglobin C disease is:
a macrocytosis
b spherocytosis
c rouleaux formation
d target cells
54. Thalassemias are characterized by:
'-·
a stwctural a~nori:nal ities in the hemoglobin molecule
b absence of iron 1n hemoglobin
c decreased rate of heme synthesis
d decreased rate of globin synthesis

.55...
Ol«Y
A patient has the following blood values:
RBC: 6.5 • 108/µL (6.5 x 10'2fL)
Hgb: 13.0 gldL (130 gfL)
Hct: 39.0%
MCV: 65 µm3 (65 IL)
MCH: 21 .5 pg
MCHC: 33%

These results are compatible w ith:


a Iron deficiency
b pregnancy
c thalassemia minor
d beta thalassemia major
56.
Laboratory findings in hereditary spherocytosis Include·
a decreased wees .
b de~ireased RBC band 3 protein
c ret1culocytopenia
d positive direct antiglobu lin lost

228 The Boord of Certmcatlon S tudy Gulde Se


ISBN978-06ll18g.6609 Q20t 8ASCP
~em'atology . Erythrocytes: Dise,we Stoles ""
-\:Vhich of the following typos or polycylhemia Is a severely burned palient most likely to
51· have?
olycythemia vera
" polycythemia, secondary to hypoxia
b p I lh . .
c relative po ~cy em1.a associated with dehydralion
d polycyt1>en11a associated with renal disease

Giant, vacuolated, multin~cleated erythroid precursors are present in which of the


.! following?
"'" chronic myelocytic leukemia
~ primary myelofibrosis
c erythroleukem1a

:::,
d acute myelocytic leukemia

59. Which of the following is a significant feature of dyserylhropoiesis?


3 persistently increased M:E ratio
I
b megaloblastoid erythropoiesis
c marked thrombocytosis
d decreased ferrilin levels

60. Tile M:E ratio in erythroleukemia is usually:


-
...
><$
, 3 normal
b high
c row
d variable
61. The characieristic morphologic feature in lead poisoning is:
a macrocytosis
b target cells (codocytes)
c basophilic stippling
d roufeaux formation
62. A40-year-old man had an erythrocyte count of 2.5 x 10&/µL (2.5 x 1012/L), hematocrit of
()!.tr 22% and a retlculocyte coun t of 2.0%. What is the patient's absolute reliculocyte count?
,.,
a 5.0 x 103/µ L (5.0 x 109/L)
b 50.0 x 103/µI (50.0 x 109/L
c 500 x 103f,L (500 x 109/L)
d 5000 x 10 /µI (5000 x 109/L)
63. Which of the following is characteristic of polycylhemia vera?
'"
"'
11
a elevated urine erythropoietin levels
b increased oxygen affinity of hemoglobin
c decreased hematocrit
d decreased or absenl bone marrow iron stores
/ 64. The while cell feature most characleristic of pernicious anemia is:
a eosinophilia
b loxic granulatlon
c hypersegmentation
d reactive lymphocytes
65
, · Which parameter is most consistently abnormal in cases of hereditary ·spherocytosis?
a RBC count
b MCV
c Hemoglobin
d MCHC

0<1>1eAScP
ISBt-1 978-089189-6609 Cllnlcal Laboratory Cartlflcatlon Examinations 229
4 : Hema tology Erythrocytes: Dis~
1
66. What protein is commonly de fective in hereditary elliptocytosis? ~ 1e~
a ankyrin
b spectrin
c band 4.1
d elliptocin
67. What is the most common mechanism resulting in hereditary stoma tocytosis?
a a bnormal Na/K permeabi lity
b deficient cytoskeletal structural proteins
c inability to repair oxidative stress damage
d ATP depletion due to glycolytic enzyme defi ciency
68. What accounts for the majority of cases of sideroblastic anemia?
a clonal stem cell defect
b medications
c alcohol
d irradiation
69. To what class or d isorders does Fanconi anem ia belong?
I/LS
oN<Y a hypoproliferative
b myeloprolifera tive
c myelodysplastic
d mitochondrial
70. Two common causes of acquired pure red cell aplasia are:
·~s
ooLY a cytomegalovirus and parvovirus infection
b thymoma and parvovirus infection
c parvovirus infection and squamous cell carcinoma
d squamous cell carcinoma and thym oma
71. Whieh of the following tumors are associated with erythrocytosis due to excessive
~~v erythropoietin production?
a renal cell carcinoma
b sarcoma
c basal cell carcinoma
d squamous cell carci noma of the lung
72. Which of the following features of G6PD deficiency are typically present on a Wright
, Giemsa stained peripheral blood smear?
a Cabot rings
b microcytosis
c bite cells
d Heinz bodies
73. Which abnormal RBC morphology is associated with pyruvate kinase deficiency?
(/
a acanthocytes
b dacryocytes
c echinocytes
d drepanocytes
74. Which of the following hemoglobinopathies is associated with rod shaped crystals?
/
a HgbS
b HgbC
c HgbSC
d HgbD

230 T he Board of Certification Study Guido Ge ISBN 978-0891~609


~ atolO!JY Eryt11rocy1,,,;: Dlsonso Stntos
146111
4: w11lcl1 ol the lollowing statements about hemoglobins D anti G Is true?
75. are clinically abnormal
~· a thCY bOth migrate with HgbS on alkaline gel
b th&~ are bOth caused by mutat~ons in the beta-globin gene
~ :~=Y cannot be separated on citrate gels
rnis nemoglobinopathy results lrom a lusion product or the delta and beta gene:
1a.
•• a HgbD
'"" HgbG
b
HgbLepore .
~ HgbConstant Spring
Which of the lollowing is consistent with the diagnosis or hoterozygous beta-thalassemia?
11.
3
increased red blood cell count
b nigh MCV
c decreased HgbA2
d decreased iron stores
IS. Jiereditary. persistence of fetal hemoglobin (HPFH) is due to a loss or expression of this
I
~ globin chain:
I
a alpha I
b beta I
c gamma
d delta
Ji. Whal is the specificity of cold autoagglutinin disease?
a anli·i
b anli·H
c anti-Pr
d anti·I
so. What is the most common presentation of paroxysmal cold hemoglobinuria?
"'cu a older people with Raynaud syndrome
b children follow.ing a viral illness
c neonates with congenital syphilis
d alcoholics with advanced cirrhosis
. 81. Which of the following is the most common cause of anemia in hospitalized patients?
"!/

a inadequate iron intake


b inadequate lolale intake
c hemolytic anemia
d anemia or chronic disease
81. In a patient with an increased red cell mass into the 99th percentile and serum
erythropoietin level below reference range tor normal, which or the following criteria
confirms a diagnosis or polycythemia vera?
a ~ marrow panmyelosis
b 1nv(16) mutation
c JAK2V617F mutation
d BCR-ABL 1 translocation

.......__
Olot!AScp
ISBN978.Jl8918~g Clinic•/ Laboratory Certification Examinations 231
4 : H e matology Erythrocytes:
.
Dis~ ta 198
83. A medical technologist is examining a penpheral smenr and notices 7 largo segment ---..
neutrophils with between 5 and 7 lobes Everything else about the CBC is other,,.ise e<J
nonnal. This observed morphologic change might develop months ahead of which or th
following changes· e
a an increase 1n MCV. MCH and ROW
b a decrease in M CV. MC H . R OW
c an increase 1n metamyelocytes and bands
d a bone m arrow showing aplasia
84. Au101mmune hemolytic anemia is often a complicatJon of;
a PV
b CML
c C LL
d HCL
85. Which red cell morphology may be expected in a treated polycy1hemia vera palie' ,,
a microcytic cells
b teardrop cells
c helmet cells
d oval macrocytes
86. The red blood cell s in this image are represen tative o r an anemia tha t is:


0
a microcytic , hy pochromic
b nonmegaloblastic macrocytic
c normocytic, no rmochromic
d myelodysplast1c

232 The Board of Certln c at lon St udy Gulde 8e


1$BN 978.009189-6609 C121)18ASCP
4: Hem a to logy F::ryt111ocy1cs: Laboratory Determinations
. 1111
57.- r nos age o f tho blood smear from a newborn 111061 likely represents.

a severe G6PD deficiency


b hereditary spherocytosis
c un1realed megaloblastic anemia
d HON due to ABO incompatibility
88. Hemoglobin H disease results from:
::t, a absence or 3 of 4 alpha genes
b absence or 2 of 4 alpha genes
c absence or 1 of 1 alpha genes
d absence of all 4 alpha genes

Erythrocytes: Laboratory Determinations


89. Which or the following tech nical factors will cause a normal (low) erythrocyte
""
cw sedimentation rate?
a gross hemolysis
b small fibrin clots in \he sample
c increased room temperature
d tolling o f the lube
_,90. Which o f the RBC ondioes is a measure or the amount of hemoglobin in individual red
blood cells?
a MCHC
b MCV
c Hct
d MCH
9
1. The ROW-CV and ROW-SD performed by automated hematology analyzers are
calculations that pro vide:
a an Index of the distribution of RBC volumes
b a calculated mean RBC hemoglobin concentration
c a calculated mean cell hemoglobin
d the mean RBC volume

Cllnlcttl L.aborntory C•rt.incation Examinations 233


4 : H ematology Eryt/Jrocytes: Laboratory Doterminattons
~
92 . The erythrocyte sedimentation rato (ES R) can be falsely olovated by: -....;
a tilting tho tube
b refrigerated blood
c air bubbles in the column
d specimen being too o ld
93. A Wright-stained peripheral smear reveals the following:
"'·
OM\' orythrocy1cs enlarged 1/S" to 2• normal size
Schuffner dots
paras tes with 1rregu1ar ·spread-out• trophozoites. golden-brown pigment
12 ..24 merozo1tes
\Vida range of stagos

This 1s consistent w ith Ptasmodium:


a falciparum
b malariae
c ovate
d vivax
94. Wh ich o f the following is the formula to calculate an absolute cell count?
a number or cells cou nted/total coun t
b total counVnumber o f cells counted
c 1 Ox total count
d % of cells counted x total count
95. U sing a supra vital stain. the polychromatic red blood cells below would probably be:

a rubricytes (polychromatophilic normoblast)


b reticulocytes
c sickle cells
d target cell s
96. The mean value or a reticulocyte count on specimens of cord blood from healthy, full-term
newborns is approximately:
a O.So/o
b 2.0o/o
c 5.0%
d 8.0%
97. A red blood cell about 5 µm In diameter that stains bright red and shows no central pallor
is a
a spherocyte
b leptocyte
c m1crocyte
d macrocyte

234 The Board o f CertilicaUon Study Guido 6e ISSN 97&-08918~ Cl20t8ASC<'


4
: Hemat ology Erythrocyte's· Laboratory Doterml11otlons
Tho following results wore obtained on a Patient's blood
98 Hgb 115gdL (11Sg1u
HCI 34%
MCV 89 µm> (89 fl)
WCH 26pg
WCHC 29

Examination of a Wright·sta1ned smear or lhe same sample would most likely show:
a macrocytic. norrnochromic erythrocytes
b mlcrocytic, hypochromlc erythrocytes
c normocytic, hypochromic erythrocytes
d normocyllc. normochrom1c erythrocytes
99. Evidence of active red cell regeneration may be indicated on a blood smear by
a basophilic stippling nucleated red blOOd eels and p01ychromas a
b hypochrom1a, macrocytes and nucleated red blood cells 1
c hypochromia, basophilic shppltng and nucleated red blood cells
d Howell-Jolly bodies, Cabot nngs and basophilic stippling
too. The smear represented below displays:

a congenital ovalocytos1s
b hemoglobin C disease
c poor RBC fixation
d delay m smear preparation .
tot . The presence of excessive rouleaux formation on a blood smear Is often accompanied by
an increased:
a reticulocyte count
b Sedimentation rate
c hematocrit
d erythrocyte count .
102. The characteristic peripheral b4ood morphologic feature in mulltple myeloma is.
a cytotoxic T cells
b rouleaux formatton
c spherocytosis
d macrocytosis
t03. The M:E ratio in polycythemia vera Is usually:
a normal
b high
c low
d variable

Ctinlca, L obo ,.lory Certification Exan1lnatlon~ 235


4 · H emato logy
.
Erythrocytes: Laboratory Dot~ ~~
104. Many microspherocytes, schistocytes a.nd sph?r~ytes with budding cytoplasm can~
seen on peripheral blood smears of patients with
a hereditary spherocytosis
b disseminated intravascular coagulation (DIC)
c acquired autoimmune hemolytic anemia
d extensive bums
105. The values below were obtained using an automated hematology analyzer and .,., 0, 0
performed on a blood sample from a 25-year-old man:
Patient Normal
WBC 5 1•103/µL 50-10.0•103/µL
(5.1 • 109/L) (5.0- 10 O• 109/L)
RBC 2 94 • 106/µL 4,6-6.2' 106/µL
(2.94 •1 0'2JL) (4.6-62•10 12/L)
Hgb 13.8 gidl(138 gll) 14 - 18 g/dl (140-180 g/L)
Hct 354% 40-54%
MCV 128 µml (128 fl) 82-90 µml (82·90 fl)
MCH 467 pg 27-31 pg
MCHC 40% 32-36°,

These results are most consistent w ith wh ch of the following?


a megaloblastic anemia
b hereditary spherocytosis
c a high ti ter of cold agglutinins
d an elevated reticulocyte count
106. A 56-year-old man was admitted to the hospital for treatment of a b leeding ulcer. The
~v following laboratory data were obtained:
RBC 4.2 • 106/µL (4.2 • 1012/L)
WBC 5.0 • 103/ µL (5.0 • 106/L)
H c1: 30%
Hgb; 8 5 g/dl (85 g/L)
serum iron 40 µg/dl (7.2 µmol/L)
TIBC. 4 60 1>g/dl (82.3 µmoVL)
serum fernt1n 12 ng/ml (12 µg/L)

Examination of the bone marrow revealed the absence of iron stores. This data is most
consistent with which of the following conditions?
a iron deficiency anemia
b anemia o f chronic disease
c hemochromatosis
d acute blood loss

236 T ho Bonrd o f Cortlllcatlon S tudy Gulde &e ISBN 978-089189-6609 C0)!SASCP


d
4: Hematology Erythrocyl es Laboratory 0Ptermlnsllons
lo1.A 40-year-old Caucasian male Wfls adm1lled lo Iha hospital ror lrenlmenl or onemln.
,., lassitude. weight loss, and loss of libido. The pa11onl presented with tho lollow1ng
,., laboratory data
wac. 5 8 • 10l/µL (5.8 • 10'IL)
RBC 3 7 • 106 /µL (3 7 • 1012/l)
Hgb 10 0 g•dL (100 91L)
Hct: 32%
MCV 86 µml (86 IL)
MCH 26pg
MCHC 32!o
serum fro1i. 220 µg/dL (39 ~ µmol/L)

I
TIBC 300 µgldl (53 1 µmoVL)
serum femiin 2.800 "9'ml (2.800 µg/l)

Examination of the bone marrow revealed el)lhroid hyperplasia with a shift to the left of
erythroid precursors. Prussian blue staining revealed markedly elevated iron stores noted
With occasional sideroblasts seen. This data is most consistent w1lh which of tho following I
conditions?
a iron deficiency anemia
b anemia of chronic disease
c hemochromalosis
d acute blood loss

1os. A common source of interference in lhe cyanmothemoglobin molhod is'.


a hemolysis
b very high WBC count
c cold agglutinins
d clumped platelets
109. A patient with beta-thalassemia charactenslically has a(n):
~. a elevated A2 hemoglobin
b low lelal hemoglobin
c high serum iron
d normal red cell volume
110. With this blood picture, an addihonal test indicated is:

a alkali denaturation
b alkaline phosphatase stain
c peroxidase stain
d hemoglobin electrophoresis

Clln/ca/ uboratory Certification Examln•tiol1• 237

b
4 : H e m a to logy Erytfirocytes: Laboratory Dotorm;, . 111111111
. '"l•ons
111. A screening procedure for detecting hemoglobin Fis the: --.._
a nuorescent spot test
b dithionite solubility test
c Kleihauer-Betke test
d heat instability test
112. The most appropriate screening test for hemoglobin S is:
a Klelhauer- Betke
b dithionite solubility
c heat instability
d fluorescent spot

....113.
CIO.Y
Which of the following is characteristic of hemoglobin H?
a it is a tetramer of gamma chains
b it Is relatively stable
c electrophoretically. it represents a "fast" hemoglobin
d it has a lower oxygen affinity tha n hemoglobin A
114. In most cases of hereditary persistence of fetal hemoglobin (HPFH):
1.ILS
""'• a hemoglobin F is unevenly distributed throughout the ery1hrocytes
b the black heterozygote has 75o/o hemoglobin F
c beta and g amma chain synthesis is decreased
d gamma chain production equals alpha chain production
11 s. When using the turbidity (solubility) m ethod for detecting the presence of hemoglobin s.
·~•
CNlY
an incorrect interpretation may be made when there is a(n):
a increased reticulocyte count
b glucose concentration >150 mg/dl (8.3 mmol/L)
c blood specimen >2 hours old
d decreased hematocrit
116 . Refer to the following pallern:
...-'

-· • • +
I I pattern A

-• • ••-·•
I I pattern B
I I pattern C
I pattern O

I control
origin A, s F A
Hemoglobin eleetrophoresl1 patterns flt pH 8.4
(oeUulosa acetate strip,

Wh ich pattern is consistent with homozygous beta-thalassemia?


a pattemA
b pattern B
c pattern C
d pattern D

238 Th o Board of Ce rtitlcallon St udy Gulde 6e ISBN 978-0891119•6609 CQQ l&ASCI'


~niatology
4
Erythrocytes: Laboratory Dctcrm/n;11/ons
'
....--;, fer to the fol Iowing
' 1'II ustration:
117. "_e_--- -----,-----.

..• •-·••-· •
+
/ I I pattern A
I pattern B
I patternC
• pattern o

I control
oilgon A, s F A

Hemoglobrn electrophoresis patterns at pH 8.4


(collulose ac:etate strip)

Which electrophoresis pattern is consistent with sickle cell trait?


3 patternA
b pattern B
c pattern C
d pattern D
A native of Thailand has a normal hemoglobin 19vel. Hemoglobin electrophoresis on
118
· cellulose acetate shows 70% he moglobin A and approximately 30% of a hemoglobin with
::.:, the mobility of hemoglobin A1. This is most consistent with hemoglobin:
a C trait
b E trait
c O trait
d D trait
~

./119. The laboratory findings on a patient a re as follows :


MCV: 55 pm3 (55 fl )
MCHC: 25%
MCH: 17 pg

A stained blood film of this patient would most likely reveal a red cell picture that is:
a microcylic, hypochromic
b macrocytic, hypochromic
c normocytlc, normochromic
d microcytic, normochromic
u 120. A patient has the following laboratory results:
RBC: 2.00 x 106/µL (2.00 x 1012/l)
Hct: 24%
Hgb: 6.8 gfdl (66 g/l)
retlculocytes: 0.8%

The mean corpuscular volume (MCV) of the patient is:


a 35 µm3 (35 fl)
b 83 µm3 (83 fl)
c 120 µm3 (120 fl)
d 150 µm3 (150 fL)

·~l8ASCP ISBN 978-089 189-6609 Cllnlcol Loborotory Certification Examinations 239


4: H ern at ology Erytflrocytes: Labor;1tory Dotcrm;
11
111111111

~;--~:=:::-:-:-:--=-=----:~-.-.-.---:-.-.-.-:---.---:~-:---::=;::-:::-:-:::-:-=:--::----._:.:.:.::"llons
121. A patient has a high cold agglutinin tiler. Aulomated hematology onalyzer resulls rev~
an elevated MCV, MCH and MCHC as well as a decreased RBC. Individual erylhroc• ~
appear normal o n a stained smear, but agglulinales are noled. T he appropriale cour~ es ·
action would be to: e or
a perform the RBC, Hgb, and Hct determinations using manual methods
b perform the RBC determination by a ma nual method; use the automa ted results for th
Hgb and Hct e
c repeat the determinations using a microsample of diluted blood
d repeat the determinations using a prewarmed microsample of diluted blood
122. Which of the following is the formula for calculating the mea n corpuscular hemoglobin
(MCH)?
a HcU(RBC x 1000)
b Hgb/Hct
c RBC/Hct
d (Hgb x 10)/RBC
123. What is the MCH if the Hct is 20%, the RBC is 2.4 x 106/µL (2.4 x 101 2/L) and the Hob is
5 g/dl (50 g/L)? ~

a 21 pg
b 23 pg
c 25 pg
d 84 pg
·_,,124. Which of the following is the formula for calcu lating the MCHC?
a (f1gb x 100)/Hct
b Hgb/RBC
c RBC/Hct
d (Hct x 1000)/RBC
1.)25. What is the MCHC if the Hct is 20%. the RBC is 2.4 x 1os/µ L (2.4 x 101 2/L) and the Hgb is
5 g/dL (50 g/L)?
a 21o/o
b 25%
c 30%
d 34%
126. Which of the following is the formula for calculating the mean corpuscular volume (MCV)?
../
a (Hgb x 1O)/RBC
b Hgb/Hct
c (Hct x 10)/RBC
d RBC/Hct
127. Given the following data:
Hgb: 8 g/dL (80 g/L)
Hct: 28%
RBC: 3.6 x 106/µL (3.6 x 1012/L)
The MCVis:
a 28 µm3 (28 fl)
b 35 µm3 (35 IL)
c 40 µm3 (40 IL)
d 77 µm3 (77 IL)

240 The Board of Certification Study Gulde 6e ISBN 973-089189·6609 C>:!OlSASCP


<1
~ 4: He rnatology .
E
ryt11rocytcs: Labor.ilory Dctcrm/11al1ons
.
--::WI 31is lhe MCV 11 lhe hemalocril is 20% lh R .
t2S. he~ioglobin is 6 gldl (60 g/l)? • e BC is 1.5 " 106/µL (1 .5 " 1012/L) and lhe

~ 68 IL
b 75 fl
c 115 fL
d 133 IL
.,.. A confirmatory lest for paroxysmal nocturnal hemoglob' . .
t<>· inuna 1s:
a heat instability test
b sucrose hemolysis
c flow cytomelric immunophenolyping
d dithionite solubility

130. The Prussian blue staining of peripheral blood dentifies:


a Howell-Jolly bodies
b siderotic granules
c reticulocytes
d basophilic stippling

131. Supravital staining is important for reticulocytes since the cells musl be living In order to
/ stain the:
a remaining RNA in the cell
b iron before it precipitates
c cell membrane before it dries out
d denatured hemoglobin in the cell
132. Which of the following is used for staining reticulocytes?
a Giemsa stain
b Wright slain
c new methylene blue
d Prussian blue

,133. Which of the following stains is used to demonstrate iron, ferritin and hemosiderin?
a myeloperoxidase
b methylene blue
c specific esterase
} Prussian blue
)3( Which of the following stains can be used to differentiate siderolic granules
(Pappenheimer bodies) from basophilic stippling?
a Wright
b Prussian blue
c crystal violet
d myeloperoxidase

Oli>ISAS()p ISSN 97a.oe918~ c:Jlnlcal Laboratol)I Certification Ex~mlnations 241


4 : Hematology Erytlrrocytes: Laboratory Determinations "'11111

135. In an uncomplicated case of severe iro~ deficiency anemia, which o f the following sets--..
represents the typical pattern of results.
Marrow Marrow
Serum Serum % sidero- iron Serum
iron TIBC % Saturation blasts stores ferritin HgbA2
.L t .t. .L i T t
A
B .L .L .l. .t. .t. .t. .t.
t .1. .t. .l .1. .t.
c .1.
0 .t. .t. t t j i t

increased = i decreased ; !
a A
b B
c c
d 0
136. Which of the following test results are consistent with a diagnosis of paroxysmal nocturnal
::;;.;'v hemoglobinuria (PNH)?
a decreased conversion of NADH to NAO
b increased production of globulin
c decreased hemolysis in acidified serum
d diminished COSS on hematopoietic cells
137. Which of the following conditions is most often associated with the V617F mutation of
JAK2?
a ch ronic myelogenous leukemia
b essential thrombocy1osis
c chronic idiopathic myelofibrosis
d polycythemia vera
138. Which of the following values is calculated from the red blood cell indices in an automated
·_..,- hematology analzyer?
a red blood cell count (RBC)
b hematocrit
c mean corpuscular volume (MCV)
d red cell d istribution width (ROW)
139. The cyanmethemoglobin method of measu·ing hemoglobin cannot detect this form of
hemoglobin:
a methemoglobin
b carboxyhemoglobin
c deoxyhemoglobin
d sulfhemoglobin
140. In the hemoglobin solubility (dithionate) test, which type of hemoglobin causes turbidity
(positive reaction)?
a HgbO
b HgbE
c HgbS
d HgbA

242 6e
The Board of Certification Study Guido ISBN 978-08918!}.6009 ©2018 Af,CP
_....
4: Hematology Leukocytes: Physiology
;'1. The presence of hemoglobin H may be demonstrated by:
'" : Prussian blue stain
""'~ "'
b Wright stain .
c Giemsa stain , " •
d brill iant cresyl blue · · ·. ' '

142. The hemoglobin v~riant ".'hich is seen frequently in the Soulh East Asian population.
... demonstrates a m1crocytic blood smear, and migrates with HgbC at pH 8.6 is:
""" a HgbBarts
b HgbF
c . HgbE
d HgbH

143. A hematology analyzer counts red blood cells by which method?


a impedance
b chromogenlc
c photomelric
d turbid imetric

144, A delta check is a method that:


a determines the mean and variance of an instrument
b monitors the testing system for precision
c moni1ors palient samples day to day
d is determined by each laboratory facility

. 145. The ROW is elevated on a CBC report. The smear would show:
/ a hypochromia
b anisocytosis
c poikilocytosis
d macrocytosis

Leukocytes: Physiology
1 6. The light-colored zone adjacent to the nucleus in a plasma cell is 1he:
) 3 ribosome
b chromatin
c mitochondria
d Golgi area

~18ASCP ISBtl 978-069189-6609 Cllnlcal Laboro1toty Ctttification Examlnatlons 243


4 : Hen1 a tology p~Leukocytes:
_ _:_:::..:..:..~=~~-----------:---:--.:-:--:=:::-;:::;::::-:::::7:--.:....:.:.01011
147. Inclusions in the cytoplasm of neutrophlls as shown In the figure below are known a~

a Auer bodies
b Howell-Jolly bodies
c H einz bodies
d Dtlhle bodies
148. An increased amount of cytoplasmic basophilia in a blood cell indicates:
a increased cytoplasmic maturation
b decreased cytoplasmic maturation
c reduction in size of the cell
d decreased nuclear maturation
149. The term "shift lo the left" refers to:
a a microscope adjustment
b imma ture cell forms in the peripheral blood
c a trend on a Levy-Jennings chart
d a calibration adjustment on an instrument
150. A term that means varying degrees of leukocytosis with a shift to the left and occasional
nucleated red cells in the peripheral blootl is:
a polycythemia vera
b erythroleukemia
c leukoerythroblastosis
d megaloblastoid
151. Cells that produce antibodies and lymphokines are:
a erythrocytes
b granulocytes
c lymphocytes
d thrombocytes
152. Specific {secondary) granules of the neutrophilic granutocyte:
a appear first a t the myelocyte stage
b contain esterases
c are formed on the mitochondria
d are derived from azurophil {primary) granules
153. In normal adult bone marrow. the most common granulocyte is the:
a basophll
b myeloblast
c eosinophil
d metarnyelocyle

244 The Board of CertincatJon Study Gulde 6e


4: Hema•v•v~y •
Elevation of the total granulocyte Leukocytes: Pliysiology i
1s4. count above 7 o • 1 3
/ a relative ly~phocytosis ' 0 /µL (7.0 • 109/L) is termed:
b 1eukocytos1s
c relative neulrophillc leukocytosis
d absolute neutrophillc leukocytosis
. 155. Elevation of the total white cell count ab
ove 11 .0 • 103/µL (12 >< 109/l ). .
./ a relative lymphocytosis is termed.
b absolute 1ymphocy1osis
c leukocytosis
d relative neutrophilic leukocytosis

156. Elevation of the lymphocyte percentage abo,~ , .


• ¥ 45010 Is termed:
/ 3 relative lymphocytosis
b absolute lymphocytosis
c 1eukocytosis
d absolute neutrophilic leukocytos1s

157. Terminal deoxynucleotidyl transferase (TdT) is a marker found on:


, a hairy cells
b myeloblasts
c monoblasts
d lymphoblasts
/
155( Pluripotent hematopoietic stem cells are capable of producing:
'/ a daughter cells of only one cell line
b only T lymphocytes and B lymphocytes
c erythropoietin, thrombopoietin, and leukopoietin
d lymphoid and myeloid stem cells
159. The Philadelphia chromosome is formed by a translocation between:
a chromosome 22 and chromosome 9
b chromosome 21 and chromosome 9
c chromosome 21 and chromosome 6
d chromosome 22 and chromosome 6
.160. Phagocy1osis is a function of:
a erythrocytes
b granulocytes
c lymphocytes
d thrombocytes
161. The mechansim causing catecholamine·induced neutrophilia includes:
a a shift in granulocytes from the marginating pool to the circulating pool
b an increased exit of granulocytes from the circulation
c a decrease exit of granulocytes from the bone marrow
d granulocyte return from the tissues lo the circulating pool
t62. Which cells are involved in immediate hypersensitivity reactions?
....
_., ., a eosinophils
b basophils
c plasma oells
d reactive lymphocytes

Cllnlcal Laboratory Coroflc1tlon Examinations 245


C®t8ASCP ISBN 978.()89189-6609
4: Hematology L e11l1ocytos : Diseo
se St;ir
~

Leukocytes: Disease States


166. A 14-year-old boy is seen in the ER complaining of a sore throat, swollen glands and
fatigue. The CBC results are:
wee: 16 .0 x 103/µL (16 0 x 100/L)
RBC: 4.37 x 1o•tµL (4 37 x 101 2/L)
Hgb: 12 .8 g ldL (128 g/L)
Hct: 38.4%
Pit: 160 x 103/µL (180 x 109/L)
Dllferentlal
absolute neu1rophils: 3.9 • 10•1L
absolule lymphs: G.O • 10•1L
absolure monos: 0 .5 • 109/L
absolule atypical lymphs; 3.2 x 109/L
Wha t is the most likely diagnosis?
a acute lymphocytic leuke m ia
b chronic lymphocytic leukemia
c viral hepatitis
d infectious mononucleosis
167. The M:E ratio in chronic myelocytic leukemia is usually:
,
a normal
b high
c low
d variable

.168.
...
Otft.V
In the World Health Organization (WHO) classlficatlon, myelomonocytic leukemia would
be acute myeloid leukemia (AML):
a with myelodysplastic-related changes
b with recurrent cytogenelic changes
c not otherwise specified
d therapy-related

246 Tho Board of Certification Study Gulde 6e ISBN 978·089 1 8~609 02(J l&ASCP
,...- . i-ternatology
4· Ioun d 1
.11 erythroleukemia includ . Lo11kocytos: Dis ease S l ates
~Abnormalities
1- . e.
rapid ONA synt11es1s
a marrow fibrosis
b megaloblastoid development
~ increased eryth rocyte survival
Neutropenia is usually associated with:
110.
a bacterial infections
b viral infections
c inflammatory processes
d myeloproliferalive neoplasms

111. Auer rods are most likely present in which of the following?
a chronic myelocytic leukemia
b primary myelofibrosis
c erythroleukemia
d acute rnyelocytic leukemia

172. The following results were obtained on a 45-year-old man complaining of chills and fever:
/ wee: 23.o • 10 3/pL (23.o. 10'/L)
Philadelphia chromosome: negative
BCR!ABL fusion gene: negative
Differential
Segs: 60%
Bands: 21%
Lymphs: 11%
Monos: 3o/o
Metamyelos: 2%
Myelos: 3%
Toxic granulation, DOhte bodies and vacuoles

These results are consistent with:


a neutrophilic leukernoid reaction
b polycythemia vera
c chronic myelocytic leukemia
d. leukoerythroblastosis in myelofibrosis
/
173; In an uncomplicated case of infectious mononucleosis, which of the following cells are
Y affected?
a erythrocytes
b lymphocytes
c monocytes
d thrombocytes
.~· The reactive lymphocyte seen in the peripheral blood smear of patients with infectious
mononucleosis is probably derived from which of the following?
a T lymphocytes
b B lymphocytes
c rnonocytes
d mast cells
'v~S. The disease most frequently present in patients with reactive lymphocytosis and
"'' persistently negative heterophile antibody tests is:
a toxoplasmosis
b cytomegalovirus (CMV) infection
c herpes virus infection
d viral hepatitis

~ISASCP ISSN 978-08918g.66Q9 CNnlcal Laboratory Cettllleatlon Examinations 247


4: Hemat o logy L eulfoc ytcs: Diseases ~
17i;;--n::;;:;r:;:--;:;:::::-;::;:;::-:::;;-::~;:;;::;:;::-;;-~;;:;;;;;;-;;;;:;~i;;-t;;:;;--;:~~;:;;;:-~-;;i;:;;:::;:;-::;-::;-:-::-~::.:.::....::'._'.'.
1
"tos
...
176. Dwarf or micro megakaryocytes may be found In the peripheral blood o f patients With:-.....
""'v a pernicious anemia
b polycythemia
c primary myelofibrosis
d chronic lymphocytic leukemia
177. Which of the following is associated with pseudo-Pelger-Huet anomaly?
a aplastlc anemia
b iron deficiency anemia
c myelogenous leukemia
d Chediak-Higashi syndrome
178. Auer rods are cha racterized as:
- 1.41.S
o"'y a fu sed primary granules
b DNA precipitates
c denatured hemoglobin
d large cytopla smic granules
179. Increased numbers of basophils are often seen in:
a acute infections
b chronic myelocytic leukemia
c chronic lymphocytic leukemia
d eryth roblastosis fetalis (hemolytic disease of the newborn)
180. A hyperoellular marrow with an M :E ratio :if 6: 1 is most commonly due to:
!JlS
""'y a lymph oid hyperplasia
b granulocytic hyperplasia
c normoblastic hyperplasia
d myelold hypoplasia

....
181 . The following results were obtained:
/ ONLY wee: 5.o • 103 /µL (5.0 • 109/L)
RBC: 1.7 • 10•/µ L (1.7 • 1012/L)
MCV: 84.0 µml (84 fl)
Pi t: 89.0 • 10'/µl (89 • 109/l)
P hiladelphia chromosome: negalive
BCRIABL fusion gene: negative
Olfferentlal
Segs: 16%
Bands: 22%
Lymphs: 28%
Monos: 16o/o
Eos: 1%
Basos: 1o/o
Metamyelos: 4°/o
Myelos: 301o
Promyelos: 4%
Blaots: 5%
1 megakaryoblast 30 nucleated erythrocytes; teardrops; schistocytes.; polychromasia:
giant. bizarre platelets noted

These results are consistent with:


a essential thrombocythemia
b polycythemia vera
c chronic myelocytic leukemia
d primary myelofibrosis

248 Tho Board of Certification Study Gulde Ge ISBN 978·08918~09 ('20IBASCI'


d
0 gy l 1111k ocy tos: Diso:i!:e s 1a1a1:
101 as admillr.d Into the hos111t 1 · _
old 111~n w ·. a with ~cute leukemia. Laboratory findings
,,.,~r· 11 0w1119. ·
r·.111e 10
1\ i,c1ev 050 stoln:
0 Inst cello no~Uvo
~
,ii• .,,p ~•"''kl
~~ ~-
.. 01klyl lian.rcrnso (TdT): Btost cell• P •'t'
•h•
'!;•
.
~ •'Jn"'·n1111u
ft'
daO oogtobulin. Blast tells negative
Bl
SSJf'"eo' esl cells negative
C 02: .
~h13C
hromosome: PosiUvo
p •
...,..,.to..
r•· 11<8L ltsS
ion gene: os11ivo
6cR re most consistent with :
51111s a .
1fl8se re eiogenous leukem~a .
sO'te mY phocytic leukemia in lymphoblaslic transformation
• c11rooic ~ lymphocytic leuk~mia
D 1 ce' _acut elogenous leukemia in lymphoblastic transformation
' roo1c mY . .
d di .old man who had bee!' diagnosed as having leukemia 2 years previously
, 30-year 'tted because of cervical lymphadenopathy. Laboratory findings included the
~ r readmi
~ ..,,,.
#· ~"''"" 39.6 • 10 3/µL (39 6 • 109/l)
· wee: 3.25 • 1001µL (3.25 • 1oi21L) I
~SC: 9.4 g/dl (94 g/L)
tlgb=
tlCl
28.2o/•
86.7 µml (86.7 IL)
I
,~v: 29.0 pg
MCli' 33.4%
MCtlC: 53 x 10l/µ L (53 • 109/L)
Pl•: d iphia chromosome: pos1llve
phia e 't'
SCR!ABL fusion gene: posi ive
Differen tial
pofYs:
4o/o
Lymplls:
Monos : 2%
eos: 3%
sasos: 48%
M)'E!IOS: 13%
Promyetos: 2%
Metamyelos: 8%
Blasts: 13%
NRBCs: 11%
Bone marrow: 95% cellularlty, 50% blasl cans (some with myetoperoxidase positivily)

These results are most consistent with :


a acule myeloid leukemia
b erylhroleukemia
c chronic myelogenous leukemia (CML}
d CML in blast transformation
I;(. Bixhemical abnormalities characteristic of polycythemia vera include:
a increased serum B 12 binding capacity
b hypouricemia
c hypohistaminemia
d increased erythropoietin

ISSN 9711-089189-E609 Cllnlcal Laboratory Certification Examlnalions 249


4 : Hernatology L e11kocytos: Disoas ""
:;;;.---~:;;;;:::::::~~;:--~-:--:-:=--::-::-::~---::-:-:-:--:-:-:-:-:-::-:::::-=:::::::-;--;::;:::::;-:--:~.....:..~:::~o.:_s1ares
185. A differential cell count of 50-90% myeloblasts In a peripl1eral blood smear is typical~
which or the following? or
a chronic myelocytic leukemia
b primary myelofibrosis
c erythroleukemia
d acute myelocytic leukemia
186. The M:E ratio in acute myelocylic leukemia is usually:
a normal
b high
c low
d variable

....187. Which
0
'"
of the following is most closely associated wilh acute promyelocytic leukemia?
a ringed sideroblasts
b disseminated intravascular coagulation
c micromegakaryocytes
d Philadelphia chromosome
188. W hich of the following is most closely associated with chron ic m yelomonocytic leukemia?
a Philadelphia chromosome
b disseminated intravascular coagulation
c micromegakaryocytes
d lysozymuria
189. The absence of intermedia te maturing cells between the blast and mature neutrophil
commonly seen in acute myelocytic leukemia and myelodysplastic syndromes is called:
a subleukemia
b aleukemic leukemia
c leukemic h iatus
d leukemoid reaction
190. Which of the following is most closely associated with chronic myelogenous leukemia?
a ringed sideroblasts
b disseminated intravascular coagulation
c micromegakaryocytes
d BCRIABL fusion gene
191. The bone marrow in the terminal stage of erythroleukemia is often indistinguishable from
MLs
o.-.LY
that seen in:
a myeloid m etaplasia
b polycythemia vera
c acute myelocytic leukemia
d aplastic anemia
192. A block in the differentiation or maturation of, and an accumulation of immature
hematopoietic progenitors is a hallmark of:
a ch ronic lymphocytic leukemia
b myelodysplastic syndromes
c polycythem ia vera
d acute myelo cytic leukemia
· 193. A ll stages of neutrophils are most likely to be seen in the peripheral blood or a patient with:
a chronic myelocytic leukemia
b myelofibrosis with myeloid metaplasia
c erythroleukemia
d acute myelocytic leukemia

250 Th e B oard or Certification Stud y Guido 6e ISBN 976-089189·6609 ~taASCP J


tf C ll I O ' '"' '.._. ~ 1
4: . .. Loukocyles: Disease States
- Which of the following cond1t1011s is a iny 1 ,. •
194. . e Opro 1lerative neoplasm?
· a refractory anemia
b seeondary eryl11rocytosis
c ni yelomonocytic leukemia
d essential thrombocythemia
The following results were obtained on a SS
195· blurred vision: ·year-old man complaining of headaches and
WBC: 19 0 • 103/µL (19 Ox 10•1q
RBC 7.2 • 101/µL (7 2 x 10"/\.)
Pit: 1,056 • 10>/µl (1056 x 10•/L)
uric acid: 13.0 mg/dL (0.76 ITlmolf\.)
o, saturation: 93%
Re<I cell volume: 3.9 11 ml (normal= 1,600)
Differential
S&gS: 84%
Bands: 10"4
LymphS: 3%
Monos: 2 'Yt>
Eos: 1%

These results are consistent with :


a neutrophilic leukemoid reaction
b polycylhemia vera
c chronic myelocytic leukemia
d primary myelofibrosis
196. A patient has a tumor that concentrates erythropoietin. He is most likely to have which of
, the following types of polycythemia?
a polycythemia vera
b polycythemia, second ary to hypoxia
c benign familial potycythemia
d polycythemia associated with renal disease
197. Which of the following types of potycythemia is most often associated with lung disease?
./
a polycythemia vera
b polycythemia, seconda6)' to hypoxia
c relative polycythemia associated with dehydration
d potycythemia associated with renal disease
198. A patient diagnosed with potycythemia vera 5 years previously now has a decreased
' hemoglobin and microcytic, hypochromic red cells. What is the most probable cause for
the current peripheral blood findings?
a phlebotomy
b myetofibrosis
c preleukemia
d aplastic anemia

OlOlSASCP ISBN978-0891~09 Clln/cal l.oborotory Cor1lflcatlon Exami nations 251


4 : H ema t o logy Leukocytes: Disease S ~
. . ~~
199. A patient has been treated for polycythem1a vera ror several years. His blood smear-;;--
s~-= ~
Oval n1acrocytes
Ho well-Jolly bodies
H ype1segmented neulrophils
Large, agranular p latelets

The most probable cause of this blood picture is:


a iron deficiency
b alcoholism
c dietary 8 12 deficiency
d chemotherapy
200. In comparison to m alignant lymphocytes, reactive lymphocytes:
/
a have a denser nuclear chromatin
b are known to be T cells
c have more cytoplasm and more mitochondria
d are morphologically more variable throughout the smear
201., Which of the following 2 malignancies represent different clinical manifestations of the
YLS
oocv same d'1sease.?
a ch ronic lymphocytic leukemia and adult T cell leukemia
b hairy cell leukemia and Hodgkin lymphoma
c chronic lymphocytic leukemia and small lymphocytic lymphoma
d Seza:y syndrome and prolymphocytic leukemia
. 202 . Increased levels of TdT activity are indicative of:
.,/'l&.S
O RY a Burkitt lymphoma
b acute promyelocytic leukemia
c acute lymphocytic leukemia
d eosinophifia
203. Which of the following is true of acute lymphoblastic leukemia (ALL)?
a occurs most commonly in children 1-2 years o f age
b patient is asymptomatic
c primitive lymphoid-appearing cells accumulate in bone marrow
d children under 1 year of age have a good prognosis
204. The most common form of childhood leukemia is:
v
a acute lymp hocytic
b acute granulocytic
c acute monocytic
d chronic granulocytic
205. Chronic lymphocytic leukemia is defined as a(n):
a malignancy of the thymus
b accumulation of prolymphocytes
c accumulation of hairy cells in the spleen
d accumulation of monoclonal B cells
206. Hairy cell leukemia is a(an)'.
MLS
ONLY a acute myelocytic leukemia
b chronic leukemia of myelocytic origin
c chronic leukemia of lymphocytic origin
d acute leukemia of monocytic origin

,.
252 The Board of Certification Study Guide 6e ISBN 978·089189-6609 e2QlSASCf'
...........--oJogY
~,.· "e~~ h .. .
Leukocytes: Disease States
wnieh of the following is a c aractenstic usually associated with hairy cell leukemia?
t01· utrophilla
a 11e nonucle<1r cells with rutned edges
b 111~jtive for CDS
c !'°creased resistance to infection
d Ill
hologic variants of plasma cells include·
tl)S· fvlOIP .
nanie cells
~ cabOt oells
bite oells
~ Gaucher cells
Which of the following bone marrow findings favor the diagnosis of multiple myeloma?
t(I!.
•' presence of Reed Sternberg cells
"'' ~ sheaths of immature plasma cells
prese11ce of occasional name cell
~ presence of plasmacytic satellitosis
Which of the following have a B cell origin?
lfO.
-~ a sezary syndrome
b 1arge granular lymphocytosis
c Sternberg sarcoma
d Waldenstr6m macroglobulinemia
llf. Which of the following cells is most likely identified in lesions or mycosis fungoides?
~' a T lymphoeytes
b B lymphocytes
c monocytes
d mast cells
m. Of the following, the disease most closely associated with cytoplasmic granule fusion is:
a Chediak-Higashi syndrome
b Pelger-Hutlt anomaly
c May-Hegglin anomaly
d Alder-Reilly anomaly
ltl. Which of the following anomalies is an autosomal dominant disorder characterized by
I / irregularly-sized inclusions in polymorphonuclear neutrophils, abnormal giant platelets and
often thrombocytopenia?
a Pelger-Hutlt
b Chediak-Higashi
c Alder-Reilly
d May-Hegglin
21 4. ~f the following, the disease most closely associated with granulocyte hyposegmentation
- IS:

a May-Hegglin anomaly
b Pel~er-Hul!t anomaly
c Ched1ak-Higashi syndrome
d Gaucher disease
iis. Which of the following is associated with Chediak-Higashi syndrome?
: ~mbrane defect of lysosomes
~hie bodies and giant platelets
~ ·lobed neutrophils
mucopo1ysaccharidosis

r.untral I ttboratorv Cerl/flcat;on Examinations 253


...
Lc11kocytos: Diso;i 50 St11
4 : Hernatology !es
216. Which or Iha following is associated wilh Alder-Reilly Inclusions? -
a rnembrm1e defec t of lysosonies
b OOhle bodies and gianl plalelels
c 2-lobed neulrophils
d rnucopolysaccharidosis
217. Which of lhe following is assoc iated with May-Hegglin anomaly?
a membrane defect of lysosomes
b OOhle bodies and giant platelets
c c hronic myelogenous leukemia
d mucopolysaccharidosis
218. A differential was performed on an asymptomatic patient. The d ifferential included
60% neutrophils, 55 o f which had 2 lobes and 5 had 3 l~bes . There. were no o ther
abnormalities. This is consistent with which of the following anomalies?
a Pelger-Huet
b May-Hegglin
c Alder-Reilly
d Chediak-Higashi
219. The cytoplasmic abnormality of the white blood cell of Alder-Reilly anomaly is found In the:
a endoplasmic reticulum
b lysosomes
c mitochondria
d ribosomes
220. Patients with chronic granulomatous disease s'u ffer from frequent pyogenic infections due
• ~. to the inability of:
a lymphocytes to produce bacterial antibodies
b eosinophils to degranulate in the p$esence of bacteria
c neutrophils to kill phagocytized bacteria
d basophils to release histamine in the presence of bacteria
221. Which of the following can cause neutropenia?
a medications
b lymphoid neoplasms
c chronic myelogen.ous leukemia
d polycythemla vera
222. Which of the following immunohistochemical patterns is most consistent with CLUSLL?
a CD5-ICD23-
b CD5-/CD23+
c CDS+ICD23-
d CDS+/ CD23+
223. Which CLUSLL marker is associated wit h poor p rog nosis?
MLS
°"'' a CD138
b ZAP-70
c FMC7
d TdT
~/

__i 24. Whkich ';'f the following morphologic characteristics Is consistently associated with hairy cell
1eu em1a 7
a small cells
b dumped nuclear chromatin
c flocculent dark blue cytoplasm
d uneven cytoplasmic margin s

254 The Board of C artllication Sludy Guido 6<t


ISBN 978·069189-6609 e;!018ASCP
d
. Hel11atolo9Y Leukocytos: Dlsoaso States
4. ieast what percenlaQe of prolymphocytes must be present for a diagnosis or
•s.
2• pro1ymphocytrc leukemia?
Al
~· a ,.10%
.,_,
b ,.25%
c ,.55%
d ,.75%
Which of the following translocations is most comrronly associated with Burkitt
~ · lymphoma?
6

~· a t(8;14)
b 1(11;18)
c !(15:17)
d t(18;22)
Whal is the most common presentation of precursor T·ALL?
221.
anterior mediastinal mass
~ bQne marrow involvemenVcytopenias
c cervical lymphadenopathy
d dermal lymphatic infiltrates
• Which of the following genetic alterations is associated with a favorable prognosis in pre-
228
,.,
"' 8-All?
a t(1 ;19)
b t(4;11)
c t(9;22)
d t(12;21)
229, What is the most common immunoglobulin present in plasma cell myeloma?
a lgG
b lgA
c light chains
d lgD
ZIO.,,Which of the following is associated with mycosis fungoides?
1•1(
V..ir a sezary cells
b hairy cells
c prolymphocytes
d large granular lymphocytes

..231., Which of the following is a characteristic of Hodgkin lymphoma?


""' a bimodal age distribution
b high incidence of peripheral blood infiltration
c unpredictable lymph node involvement
d uniformly fatal
~2· Which of the following is suggestive of a myeloproliferative disorder?
a basopenia
b granulocytosis
c lack of toxic granulation
d cytopenias
•2
3
~ Which of the following is characteristic of the myelodysplastic syndromes?
a neutrophilia
~ unilin7age or multilineage dyspoiesis
effective hematopoiesls
d Propensity to develop lymphoma

Cl/nlcal Laboratory Cetlificarion Ex•mlnations 255


4· Hematology

Leukocytes: o~ aros
234. Which of the rollowing Is a WHO classification or myelodysplastic syndrome? ---
a chronic myelomonocylic leukemia (CMML)
b refractory thrombocytopenia .
c refractory cytopenia with excessive blasts (RAEB)
d refractory neutrophilia
235. Which of the following cytogenetic abnormalities is associated with a more stable clinical
u tS course in myelodysplastic syndromes (MDSs)?
O"-l.Y
a monosomy7
b del5q
c loss ofY
d del20q
236. Which of the following disorders is a category in the WHO classification of myelodysptasu
syndromes/myeloproliferative neoplasms? c
a acute myelomonocytlc leukemia (CMML)
b chronic myelogenous leukemia (CML)
c juvenile myelomonocytic leukemia (JMML)
d refractory anemia with excessive blasts (RAEB)
237. Which of the following features define chron ic myelogenous leukemia?
a JAK2 mutation
b myelocyte "bulge·
c basoP.hilia, eoslnophil ia, and thrombocytosis
d f t(g ;22)
238. Which of the following are main groups o f acute myeloid leukemia (AML) in the WHO
.... classification system?
a AML with myeloproliferative related changes
b AML with lymphoid differentiation
c AML with biphenotypic morphology
d AML, NOS
239. What type of morphologically abnormal cells proliferate in acute myelogenous leukemia
with inv(16)?
a lymphoc ytes
b erythrocytes
c platelets
d eosinophils
240. What defines acute myelogenous leukemia with minimal differentiation?
a >20°/o blasts positive for Sudan black B
b positivity for myeloperoxidase or nonspecific esterase in at least S0°k of blasts
c positivity for myeloperoxidase or nonspecific esterase in at least 100% of blasts
d >20% blasts lacking cytochem ical evidence of myeloid differentiation
241. Which of the following best describes the bone marrow criteria for the diagnosis of acute
myelomonocytic leukemia(AMML)?
a 20°/o blasts, 20°/o monocytes, 20°/o neutrophilic precursors
b 20% blasts, 50°/o monocytes, 50% neutrophilic precursors
c 20°/o blasts, go•;. monocytes, 10°k neutrophilic precursors
d 30°/o blasts, 10% monocytes, go 0/o neutrophilic precursors

256 The Board of Certlncation Study Guido Ge


Ill Lc11hocy tos: Ln/1 omtory Dctorml11ation 5
4: Hematology
iii. w11a1 is the most common presentation or acuio orythroid leukemia?
~ >80% of nucleated marrow cells are erythrold precursors
j,>50% of nuclented marrow cell s are erythrold precursors and >20% of nonorythroid
cells are myelobl<Jsts
c >50% of r1ucteated marrow cells are erythroid precursors and <20% or nonerythroid
cells are myeloblasts
d 80% blasts with 20% erythroid precursors, >20% myeloblasts

243. Which of the following is associated with acute megakaryoblastic leukemia?


a Down syndrome
b systemic mastocytosis
c isochromosome 17p
d <5% of blasts being megakaryoblastic

2«. Which of th~ following cell surface markers is a!lsociated with a more aggressive subtype
,., of CLUSLL.
""' a CD13 l
b CD21
c CD38 II
d CD125
I
245. The leukemic phase of T cell lymphoma is marked by the following cellular abnormality:
:::, a Sezary cell
b Plasmacytoid lymphocyte
c Mantle cell
d Reed Sternberg cell

Leukocytes: Laboratory Determinations


246. An oncology patient has the following results:

Day 1 Day 3
WBC 0.0. 103/µl 2.0• 103/~L
(8.0 • 109/L) (2.0• 109/l)
RBC 3.50 • 1<>6/µL 3.45 • 1~µ1.
(3.50• 101211.) (3.45. 101111.)

Hgb 10.0 g/dl (100 g/L) 9.9 gldl (99 g/l)


Hct 29.8% 29.5%
Pit 180 • 103/µL 150• 103~L
(180• 10 911.) (150 • 1091L)

The most probable explanation is:


a chemotherapy
b cold antibody
c dotted specimen
d inadequate mixi ng

CNnlcal Laboratory Certification Examinations 257


Olo\af\Sep ISSN 978-08918~
L cukocytos: Laboratory DctermJ . ~
4: Hematology . ""tio11~
'ff rentlal on a 40-year-old Caucasian man revealed: --......::.
247. A leukocyte count an d d 1 e
-. WBC: 5.4 • 103/µL (5.4 x 10"/L)
Olfforcntlal
Soos: 20%
Lymphs: 58%
Monos: 20%
Eos: 2%

This data represents:


a absolute lymphocytosis
b relative neutrophilia
c absolute neutropenia
)
d leukopenia
. 248. A leukocyte count and d ifferential on a 40-year-old Caucasian man revea led:
WBC: 5.4 x 103/µL (5.4 • 109/L) •
Differential
Segs: 20%
Lymph s: 58%
Monos: 20%
Eos: 2%

This represents:
a relative lymphocytosis
b absolute lymphocytosis
c relative neutrophilia
d leukopenia
249. Given the following data:
WBC: 8.5 x 103/µ L (8.5 x 109/L)
Diffe rential
Segs: 56%
Bands: 2%
Lymphs: 30%
Monos: 6%
Eos: 6%
What is the absolute lymphocyte coun·! ?
a 170/ µL (0.17 x 109/L)
b 510/µl (0.51 x 109/L)
c 2 ,550/µl (2.55 x 109/L)
d 4 .760/µL (4. 76 x 109/L)
250. Which or the following is the formula to calculate a manual white cell count?
a (number of cells counted x dilution x 10)/number or squares counted
b (number of cells counted x dilution)/10 x number of squares counted
c number of cells counted x dilution
d number o f cells counted x number of squares counted
251. If a WBC count is performed on a 1:1 00 cilution and the number of cells counted in 8
squares is 50, the total WBC count is:
a 5,000/µl (5.0 x 109/L)
b 6,250/µl (6.25 x 109/l)
c 50,000/µL (50.0 x 109/L)
d 62,500/µL (62.5 x 109/L)

258 The Board o f CerUficaUon Study Gulde 6e ISBN 978-089169-6609 CWtSASCI'


~ : Hemato log y
4 Loukocytos: Laboratory Ootormim1tlorrs
2 A totalleukocyte count Is 10.0 x 1QJ/ L ( 1 ,1
25 · ieukocytes on the d1fferent1al Whal ~ th O.O x 10 IL) and 25 NRBCs are seen per 100
1 e corrected leukocyte count?
9
3
2.000/µL (2.0 x 10 .'L) ·
b 8.000/µL (8.0 x 109 /L\
c 10.000/µL (10.0 x 10~/L)
d 12.000/µL (12.0 x 109/L)

a polychromatophllic normoblast (rubncy1e)


b mature lymphoey1e
c plasma cell
d lymphoblast
254. The cells seen in the image below are most consistent with:
...,,.v

a chronic myelogenous leukemia


b Infectious mononucleosis
c acute lymphocytic leukemia
d S~zary syndrome

Cllnlcal Laboratory Certification Examinations 259


"2018 ASCP ISSN 978--089189-6609
4 : Hemato logy
Leukocytes: Laboratory Dctermfn
255 atfo11i
· Cell description .
••to 12 to 16 µm
nucinua
OVf'I nolched folded ovu• to horsn1hoo sn11pc
Chrom1111n
fine lacy sta ns lf~ht pu•plu-ptnk
nUCleolo
none present
cy toplasm abundant. slate gray w ith m11ny r1ne l1lac-co1ored granu es

This cell 1s a :
a promyelocyte
b lymphocyte
c neutrophil
d monocy1e
256, The large cell in the center of the image would be best described as a(n):

a neutrophil
b basophll
c eosinophil
d myelocy te
257. The large cell indicated by the arrow in image below is a:

a myeloblast
b pro myelocyte
c m yclocyte
d metamyelocyle

260 The B o ard o r Cortlficotion Study Gulde 6e ISBN 976-089189~ C2()18ASCP


............
4: .,.,... -.--t· ~ ~d-. d Lc11kocy1 . L •
58 A pat1 en ts iagnose as having bacte . es. alJorntory Determinations .,
2 · describe the expected change in 11. naJ septicemia Wh· 1 •
. is Peripheral blood? ic 1 or the rollowmg would best
a granulocytic leukemoid reaction ·
b Jymphocyhc leukemoid reaction
c neutropema
d eosinophilia

259 · Characteristic morphologic features o f reactive


.
lym h .
, a small size, clumped nuclear ch . P ocytes include:
· • romatm scant b
b smaJI size. nuc1eoil, granular cytopJas~ Y asophilic cytoplasm
c large cell , oval nucleus. scant bl
d large size, irregularly shaped ~uc~e-gray cytoplasm
eus, abundant blu
2 60. A bone marrow shows foam cells ran . f e cytoplasm
· · h. · ging rorn 20 100 · ·
"" containing sp mgomyehn and is faintly PAS T . · µm in size, vacuolated cytoplasm
'''" a Gaucher disease - ~ · his cell type is
most charactensllc
. . of:
b myeloma with Russell bodies
c DiGuglielmo disease
d Niemann-Pick disease
261. Bone marrow examination reveals a hyperc 11 1ar marrow consisting of probable
c.6 lymphobl~sts. The cell s are positive for TdTecu
°"'' are negahve for surface immunoglobulins C0~93 ~~ CD7, however, the lymphoblasts
receptors. The most likely diagnosis is: ' • 10 (CALLA), Fe, and complement

a large granular lymphocytosis


b chronic tymphocylic leukemia (CLL)
c T cell leukemia (T-ALL)
d hairy cell leukemia
//
V~2. Which cell type shows the most intense staining with myeloperoxidase?
""'' a neutrophil
b basophil
c lymphocyte
d monocyte
, 263. Which substrate is used for the detection of specific esterase?
v ...,
""' a acetate ,, - 1~

b chloroacetate
c pararosanilin acetate
d phenylene diacetate
.; J4:'A useful chemical test for the diagnosis of hairy cell leukemia is the:
"'" a peroxidase test
b Sudan black test
c periodic acid-Sch iff test
d' tartrate-resistant acid phosphatase test
J.6(Cytochemical stains were performed on bone marrow smears from an acute leukemia
~' Patient. The majority of the blasts showed varying amounts of myetoperoxidase positivity.
Some of the blasts stained positive for chloroacetate esterase, some were positive for
alpha-naphthyl acetate esterase, and some blasts stained positive for both esterases.
What type of leukemia is indicated?
a lymphocytic
b myelogenous
c myelomonocytic
d erythroleukemia

020l8ASCP ISBN978-089189·6609 CUnlcal Laboratory Certification Examin.ation s 261


4 : H enla t ology Leukocytes: Lo/jo ralory o~ .
1""llo
II
266. Whicll or 111e following stains Is closely ossocln led with the lysosomal onzymo In P•lrn . ~
11

(azurophilic) gmnules? ary

a myeloperoxidase
b 11onspocific ester<Jse
c methylene blue
d Prussian blue
267. A 30-year-old woman was admitted lo the hospital for easy bruising and menorrhagl
Laboratory findings included the following: a.
WBC: 3.5 • 103/µL (3.5 • •O'IL)
RBC: 2.48 x 10G/µL (2.48 x 1012/L)
Pit: JO • 1O'lµL (30.0 • 109/L)
Hgb: 8 .6 g/dL (86 gll.)
Hct: 25.0%
MCV: 100.7 µm3 (100.7 IL)
MCH: 34.7 pg
M CHC : 34.3%
PT: 34.0 sec
aPTT: 62.5 sec
TT: 15.0 sec
FOP: > 40 µg/mL (>40 mg/L)
fibrinogon: 3 15 mg/dL (3.15 g/L) (conlro l 200 · 400 mg/dL (2.0-4.0 !)/LI)
D l lferontial
Polys: 3%
Lympns: 1%
Monos: 2%
Myelos: 4%
Abnormal fmmature: 5 8%
!!lasts: 31%
nRBC: 1%
Auer rods , 1• macrocytes. 1+ polychromasia

The cells identified as "abnormal immature" were described as having lobulated nuclei
with prominent nucleoli; the cytoplasm had intense azurophilic granulation over the
nucleus, with some cells containing 1-20 Auer rods, frequently grouped in bundles.
A t(1 S;17) chromosomal translocation was noted. Cells were myeloperoxidase positive.
Which of the following types of acute leukemia is most likely?
a myeloblastic
b promyelocytic
c myelomonocytic
d m onocytic
268. Chronic lymphocylic leukemia cells are most likely to express which of the following cell
~tv surface m arkers?
a CD3, CD?, CD19, CD20
b CD4, CDS, CD19, CD20
c CDS. CD19, CD20, CD21
d CD13, CD33, CD107
269. Which of the following markers, typically detected in normal myeloid cells, are expressed
.....
()NlY
on the surface of hairy cell leukemia lymphocytes?
a CD3
b CDS
c CD10
d CD11c

.2 62 The Board o f Ccrtlficotion Study Guide Ge


~gy Hematology Laboratory Operations
~II surface marker t~iat is expressed on neoplastic plasma cells and is helpful in the
210. . nosis of myeloma is:
... d139
,,., n CD19
i, co20
c CD38
d co1Ja
The t(8;14) chromosomal translocation brings which o f the following 2 genes in close
211. ·mity?
"' pro)(l .
,,.., core binding foctor alpha and the retinoic acid receptor
3
b the Abelson tyrosine kinase ~nd breakpoint cluster region
c c-myc .and the 1mmunoglobulin heavy ~hain
d core binding factor beta and the myosm hea•1y chain

212
. Which of the following markers are usually negative in hairy cell leukemia?
~' a CD5
b CD11c
c C025
d CD103
I
•'

273. Which of the following markers is expressed in most cases of AML?


~' a CD2
b CD10
c CD11b
d C01 17

274. A nag o f immature granulocytes (IG) is reported from a hematology analyzer. The next
, step is to perform a(an):
a auto verification
b smear review
c manual differential
d pathology review

Hematology Laboratory Operations


275. In an automated hematology analyzer, the WBC printed result is +++. The next step is to:
a repeat after warming the sample to 37'C
b make an appropriate dilution of the sample
c recalibrate the machine from pooled samples
,.i: d request a new sample immediately
1;,2(A specimen analyzed on an automated hematclogy instrument has a platelet count of
19 • 103/µ L (19 x 109/L). The first procedure to follow is:
a report the count after the batch run is completed
b request a new specimen
c review the stained b lood smear
d notify the laboratory manager
in. The electrical resistance method of cell counting requires:
a equal-sized particles
b a conductive liquid
c 2 internal electrodes for current
d 3 apertures for counting

alnlc•I Laboratory Certification Examinations 263


-.. ne1na t o l ogy Hema tology L~b
• oratory Op
278. An anemic patient has an RBC of 2.70 • 100/µL (2.7 " 1 0 1 ~/L) and a horn
0
. '""orit
~~. 13.5 !;!ldL.(135 g/L) as delern:ilned by an automated hematology analyzer~~bin or
following 1s lhe best e xplanation for these results? · •ch or 1110
a electrical interference
b specimen that is lipemic
c high anticoagulant to blood ratio
d a high coincidence rate
2 79. T he following results were obtained on an automated hematology analyzer:


....$
WBC:
RBC:
Hgb:
6.5 x 103 fµL (6.5 x 10 9 /l)
4.55 x 106fµL (4.55 x 1012/L)
18.0 gldL (180 g/l)
Hct: 41.5%
MCV: 90.1 µm 3 (90.1 IL)
MCH: 39.6 pg
MCHC: 43.4%

The first step in obtaining valid results is to:


a perform a microhematocrit
b correct the hemoglobin for lipemia
c dilute the b lood
d replace the lysing agent
280. On an automa ted hematology analyzer. if the RBC is erroneously increased. how will oth
parameters be affected? er
a increased MCHC
b increased hemoglobin
c decreased MCH .
d increased MCV
281 . On initial start-up of the automated hematology analyzer, one of the controls is slightly
'./' below the range for the MCV. Which of the following Is indicated?
a ca ll for service
b adjust the MCV up slightly
c shut down the instrument
d repeat the control
282. In an electronic or laser particle cell counler. clumped platelets may interfere with which of
1LS the following parameters?
./ OM.Y
a white blood cell count
b red blood cell count
c hemoglobi n
d hematocrit
283. Which of the following will cause erroneous results when using a phase optical system for
enumerating platelets?
a lysed RBCs
b aggregated platelets
c diluted sample
d intact leukocytes

~18..so'
ISSN 978-089189~
264 T he Board of Certification Study Gulde 6o
~tofogy
4· Hen Homotofogy Laborntory Oporolfons
...
1
~ calculated erythrocyte indices on an ad 11
=
is•. TheKC 38%. The calculations have bee~ ~ man are M CV = 69 fl . MCH 29 pg anc.J =
"'• MC d smear appear normocytic and normo hche~ke~; erythrocytes on the peripheral
,_. 0100. c rom1c with no abnormal forms. The next step
iS 10.
report the results
~ examine another smear
c repeat the hemoglobin and hematocrit
d repe<it the erythrocyte count and hematocril
The following results were obtained on an eleclfoni'c p rt' 1
2S5· a 1c e counter:
"' wee: s1 .J • 1oltµL (61 .J • 10•1L1
,,._, RSC: 1.19 x 105/µL (1,19 x 1012/L)
Hgb: 9.9 g/dl (99 g/L)
Hct: 21%
MCV: 125 µm3 (1?5 fl)
MCHC: 54.1%

What action should be taken to obtain accurate results?


a dilute the specimen and recount
b warm the specimen and recount
c check the tube for clots
d clean the aperture tubes and recount

2s6. Refer to lhe following illustration:

REL A B
NO ""' C

50 100 ?.00 300 400


.,.D

50 100 200 300

2 10 20 '
Which area in the histogram for an automated hematology an alyzer represents the RSC
distribution curve?
a A
b B
cc
d D
287. Hemoglobins are read on a photoelectric colorimeter Jn the laboratory. While reading the
~~' hemoglobins, a problem of drifting is encountered. To assess the problem, lhe fi rst thing
to do is:
a recalibrate the instrument
b check the filter
c set up new hemoglobin samples
d check the light source

llli>1!>.SCP ISBN 9~91119-6!!09 Clinical Lot>oratoty CMiflcatlon Examinations 265


4 · Hematology

Hematology Laborato~ PC/rat;0
288. When evaluating a peripheral blood smear ror aTrehliculocyJek c ounl, the red blood ceii ~i
observed to overlap throughout lhe enlire slide. e most 11 e 1y explanation is: s a,0
a grease on Iha slide prevented even s~reading
b improper proportions of blood and stain were used
c Iha slide was dried too quickly
d the drop used for the slide preparation was loo large
289. If a blood smear is dried 100 slowly, the red blood cells are orten:
a clumped
b crenated
c lysed
d destroyed
290. Blood collected in EDTA undergoes which of the following changes if kept at rooni
temperature for 6-24 hours?
a increased hematocril and MCV
b increased ESR and MCV
c increased MCHC and MCV
d decreased reliculocyte count and hemalocrit
291. The specimen of choice for preparation of blood films ror manual differential leukocyte
,,. counts is whole blood collected in:
a EDTA
b oxalate
c citrate
d heparin
292. A platelel determination was performed on an automated instrument and a very low value
~ was obtained. The platelels appeared adequate when estimated from lhe stained blood
film. The best explanation for lhis discrepancy is:
a many platelets are abnormally large
b blood sample is hemolyzed
c white cell fragments are present in the blood
d red cell fragments are present in th e blood
293. On a smear made directly from a linger slick, no platelets were found in the counting area.
'/ The first thing to do is:
a examine the slide for clumping
b obtain another smear
c perform a total platelet count
d request another finger stick
29!1.· Which of the following is the standard ca'ibration method for hematology instrumentation
~v against which other methods must be verified?
a latex particles of known d imension
b stabilized red cell suspensions
c stabilized 7 parameter reference controls
d normal whole blood

266 Tho Board o f Certification Study Guido 60 ISBN 978-06918!>-6609 C2i)IB ,o.scP
4: Hematology
Hematology L11/1oml ory Operations
Refer to the following illustration:
295.

~c
50 100 200 300 400
.,.D

50 100 200 300

2 10 20 m'
Which area in the histogram for an autom t d h
distribution curve? a e· ematology analyzer represents the platelet
a A
b B
c c
d E
296. The following CBC results were obtained from an automated hematology analyzer on a
" ' patient sample with lipem1c plasma:
_, "' Ot«.Y
WBC: 7.2 x 10'/µL (7.2 x 109/L)
RBC: 3.50 x 106/µL (3.50 x 1O' ZJL)
Hgb: 13.8 gldl (138 g/L)
Hct: 33.5%
MCV: 92 µm3 (92 fl)
MCH: 39.4 pg
MCHC: 41 .0%

Which of the following tests would probably be in error?


a WBC . RSC , MCV
b RSC, Hct, MCV
c RSC, Hgb, Hct
d Hgb, MCH, MCHC
297. On Monday a patient's hemoglobin determination was 11 .3 g/dl (113 g/L}, and on Tuesday
:,, it measured 11.8 gldl (118 g/L). The standard deviation of the method used is ±0.2 g/dl
(2 g/L). Which of the following can be concluded about the hemoglobin values given?
a one value probably resulted from laboratory error
b there is poor precision; daily quality control charts should be checked
c the second value is out of range and should be repeated
d there Is no significant change in the patient's hemoglobin concentration

~Ot8ASCP IS8N978-089189-6609 Cllnlcal Laboratory Certification Examinations 267


Platelets: Pllyslology
298. vWF antigen can be found in which of the following?
a myeloblast
b monoblast
c lymphoblast
d megakaryocytes
299. Which of the following is characteristic of cellular chang es as m egakaryoblasts rn
megakaryocytes within the bone marrow? ature into
a progressive decrease in overall cell size
b increasing basophilia of cytoplasm
c nuclear division without cytoplasmic division
d fusion of the nuclear lobes
300. Which of the following cells is the largest cell in the bone marrow:
v
a megakaryocyte
b histiocyte
c osteoblast
d mast cell
301. The life span of a platelet is:
(.,
a 5 days
b 10 days
c 20 days
d 30 days
302. Aspirin affects platelet function by interfering with platelets' meta bolism of:
Vtlls
ONl.1 a cyclooxygenase
b lipids
c carbohydrates
d nudeic acids
. 303. A lpha granules are found on the platelet in the:
v Ml.S
ot.tv a peripheral zone
b solgel zone
c organelle zone
d membranes
304. Which of the following best represents the 3 steps o f normal hemostasis (in order)?
a decreased heart rate, adhesion of platelets, plug formation
b platelet aggrega tion, formation of FXll l, fibrin plug
c vasoconstriction, platelet aggreg ation. fibrin formation
d vascular damage, stasis, endothelial injury
305. Which of the following platelet antigens is the receptor for collagen?
a GPlbN/IX complex
b GPllb/lll a complex
c GPla/lla complex
d GPIV/X complex

268 The Board o f Certification Study Gulde 6e


ISBN97&-0891~ o:ioiaASCI'
d
4; ti8fl1CUV •-::>J
Which platelet surface antigen acts Platolets: P/Jys/ology
306· as the rece 1
3
GPlbN/IX P or for fibrinogen?
b GPllb/llla
c GPla/lla
d GPlc/lla
How does GPlb become activated 1·n .
301 · ~~a~in~t
3
shear force, ristocetin ro, respecUvely?
b ristocetin, compression
c activation o f ADP receptor, ristocer
d binding vWF, epinephrine in

308. In patients "'.'ho present with bleeding disorder


• of bleeding is: caused by platelet, the most common type
"' a mucosal bleeding I1
b hemarthrosis
c delayed bleeding
d deep hematomas
I
309, The anticoagulant that is best directed against platelets is:
~' a argatroban
b hirudin
c tlrofiban
d dabigatran
310. How do ticlopidine and clopidogrel inhibit platelets?
a binding von Willebrand factor
b ADP mediated platelet aggregation
c inhibit GPllb/l lla
d depletion of platelet alpha granule content
311. Vasoconstriction is caused by several regulatory molecules which include:
/ a fibrinogen and vWF
b ADP and EPI
c thromboxane A2. and serotonin
d collagen and actomyosin
312. Von Willebrand factor mediates platelet adhesion by binding to platelet receptor:
a GP 1b/lla
b GP lb/GPIX/GPV
c GP lib/Ila
d GP lb/GPllla/GP X
313. A patient is placed on clopidogrel. The clinician wants to determine if the d?se is. sufficient
to impair platelet function. A platelet aggregation test is ordered. The agomst which would
result in a decreased aggregation pattern woi'd be:
a collagen
b epinephrine
c ristocetin
d adenosine disphosphate

Clinic•/ Laboratory certmcation Examln•tlons 269


C:1lli8ASCP ISBN97S.0891~9
4 · Hematology
• .
Platelot~ I0/09y
31 4 . A p<itient is on 100 mg of aspirin/day to pre ven tlthre fot~ma~1obn o f clots caused by Pla~t
T he mechanism In which aspirin impairs P1ale1e unc ion is y: s.
a inactivating cyclooxygenase which blocks thromboxane A2
b impai rs vWF by via Gp lb/IXN receptor
c decreased amounts of arachidonic acid
d inactivation of ADP and phospholipase A2

Platelets: Disease States


, 315. The disease state that presents with a quantitative platelet disorder is:
,..$
0 1" ' a von Willebrand disease
b hemophilia A
c Glanzmann thrombasthenia
d May-Hegglin anomaly
31 6. A patient presents with a very low platelet count and is diagnosed w ith acute idiopathic
~. thrombocytopenic purpura (I TP). which statement is associated with acute ITP.
a it is found primarily in adults
b spontaneous remission usually occurs within several weeks
c women are more commonly affected
d peripheral destruction of platelets is decreased
317. The most common cause or bleeding in patients is:
I ....
-" °"'·' a qualitative platelet defect
b qualitative abnormality of fibrinogen
c quantitative abnormality of fibrinogen
d quantitative abnormality of platelets
~318. A 53-year-old man was in recovery following a triple bypass operation. Oozing was noted
t.iv from his surgical wound . The following laboratory data were obtained:
Hemoglobln: 12.5 g/dl (125 g /l)
Hematocrlt: 37%
Prothrombin lime: 12.3 seconds
aPTT: 34 seconds
Platelet count: 40.0 • 10>/µL (40.0 • 10"/l)
Fibrinogen: 250 mg/dl (2.5 g/L)

The most likely cause of bleeding would be:


a dilution of coagu lation factors due to massive transfusion
b intravascular coagulation secondary to microaggregates
c hypofibrinogenemia
d dilutional thrombocytopenia
319. ADAMTS13 deficiency is responsible for thrombocytopen ia found in:
""
ONLY a TTP
b DIC
c HUS
d ITP
320. H~parin in~uced thrombocytopenia (HIT) is an immune mediated complication associated
MLS
Ct4!.Y with heparin therapy. Antibodies are produced against:
a ACLA
b PF4
c AT
d B2GP1

270 The Board o f Cortincallon Study Gulde 60 ••-· • --· ..... . ......,,,."" """''"AA.SCP
~matology Platelets: Physiology
.---: polycythemia vera. the platelet count is·
321. 1n ·
3 elevated
b normal
c decreased
d variable

322. A 60-year-oldd ~an has a painful right knee and a slightly enlarged spleen. Hematology
,,.,
'" results 1nclu e.
hemoglobin: 15 g/dl (15C•g/l)
ebsOl\Jte ncu1roph1I count:
l 0.0 • 10'lpl (10.0 • 10•1L)
platelet count: 900 • 10 3/µL (900 • 1o•tL)
uncorrected relic count 1%
normal red cell morphology and Indices
a slight increase In bands
rare metamyelocylc and myeloc yte
giant and bizarre·shoped plololots

This is most compatible with:


a congenital spherocytosis
b rheumatoid arthritis with reactive thrombocytosis
c myelofibrosis
d idiopathic thrombocythemia (essential or pr.mary)
323.,.-The type of bleeding that is the most characteristic in patients with platelet disorders is
/ a deep muscle hemorrhages
b retroperitoneal hemorrhages
c mucous membrane hemorrhages
d severely prolonged clotting times
324:'When reviewing platelet morphology on a blood smear, the presence of giant platelets is
/ noted. This may Indicate:
a Bernard-Soulier
b von Willebrand Disease
c Glanzmann thrombasthemia
d Ehlcr.Danlos
325. A patient with multimer pattern that has all bands present but in decreased concentrations
~r would be diagnosed with :
a hemophilia
b type 1 vWD
c type 2 vWD
d type 3 vWD
326. Which characteristic of Bernard-Soulier syndrome helps distinguish it from von Willebrand
, disease?
a concomitant storage pool defects in platelets
b giant platelets
c thrombocytosis
d thrombocytopenia

02013ASCP ISSN 978-069 189-6609 C'IJnkal Labor.Jtory Certification Examinations 271


4 ·• H ernatology Platelets: Phy ~
SIO/o
327. Which of the following characteristics are common between Hermansky-Pudlak and !Ir
Chediak-Higashi syndromes?
a giant inclusion granules in granulocytes
b alpha granule storage pool defects
c inclusions in macrophages
d oculocutaneous albinism
328. Patients with Wiscott-Aldrich syndrome present with:
a thrombocytopenia
b eczema
c small platelets
d all of the above
329. When evaluating a patient for von Willebrand disorder, the ABO blood type that has th
\., lowest level of vWF is: e
a 0
b A
c B
d AB
330. Which subtype of von Willebrand disease is the most common?
a type 1
b type 2A
c type 28
d type 3
331. Which of the following types of von Willebrand disease should not be treated with DDAVP?
a type 1
b type 2A
c type 2B
d type 3
332. TTP p resents with a pentad of symptoms which doesn't include:
a fever
b anemia
c thrombocytopenia
d liver failure
333. A 40-year-old female has pin point hemorrhages on,her legs. with an Hct of 43% and
(_./
a platelet count of 19 x 103/L (19 x 109/L). Steroids fail to increase platelet count. The
patient's diagnosis is:
a decreased platelet production
b suppressed pluripotent stem cells
c immune thrombocytopenic purpura
d defective platelet endothelial interaction

272 The Board of Certification Study Gulde Ge ISSN 978-089189-6609 C)20tSASCI'


.....
~ ....atology
4; He... Platelets: Pl1ys lology
- A 6()-year-old female presents with blurred · ·
3)4. count of 29 x 10JtµL (29 x 109/l) vision. confusion, and petechiae with a platelet
pr. 12.1 seconds (11.4-13.5 seeonds))
aPTT: 32.6 seconds (25-35 seconds)

The CBC RBC morphology shows schistocytes. Patient diagnosis is:


a ITP
b TTP
c HUC
d DIC

335. one of the main difference between ITP and HUS is


a neurological involvement
b kidney dysfunction
c thrombOCytopenia
d microangiopathic hemolytic anemia

336. several hours after birth3an infant develops petechiae, purpurlc hemorrhages and a
platelet count of 21 " 10 /µL (21 x 109/L). The most likely diagnosis is:
a drug induced immune thrombocytopenia
b thrombotic thrombocytopenlc purpura
c autoimmune neonatal thrombocytopenia
d neonatal idiopathic thrombocytopenia
337. In TTP, a deficiency of ADAMTS 13 causes clo1ing because of:
a increased vW factor
b inability to cleave ULVWM
c increased production of thrombin
d inability to antibod ies to prevent platelet aggregation
338. An orthopedic patient who is on heparin has a platelet count of 50 x 109/µL (50 x 109/L);
his platelet count the previous day was 120 x 103/µL (120 x 109/L). The patient is tested
for HIT and has a positive result. The first step in the treatment of HIT is:
a start LMWH
b stop heparin
c switch to warfarin
d give platelets
339. Arterial thrombosis is caused by
a RBC & platelets
b fibrin & WBC
c thrombin & FXlll
d WBC & platelets
340. In HELLP syndrome patients present with:
a normal liver enzymes
b elevated platelets
c high blood pressure
d all of the above
/341. Von Willebrand factor serves as a stabili~er for:
~ a platelets
b factor IX
c ristocetin
d factor VI 11

C>2o1a ASCP ISBN 978-089189-6009 Cllnlc•I L•boratory Certification Examinations 273


4 : He n1a to lo gy Plntclcts: Laboratory Date . ~
rrn1nai;
342. Th o platelet disorder in which the abnormality Is due to a defect in plalclet aggre . on,
"
b
c
von Willebrand disease
storage pool disease
.
Glanzmann thromba slhenia
Qalion·
··- ·

d Bernard-Soulier syndrome

Platelets: Laboratory Determinations


343. Blood is diluted 1:200, and a platelet count is performed. 180 platelets were counted.
center square millimeter on one side of the hemocytometer and 186 on the other sid~n.:::;
total platelet count is: · -
a 146,000/µL
b 183,000/µ L
c 366,000/µl
d 732,000/µ l
A phase-platelet count was performed and the total platelet count was 356,000/µL. Ten
high power (100><) fields on the stained blood smear were examined for platelets and the
results per field were:
16, 18, 15, 20. 19, 17, 19, 18, 20. 16

The next step would be to:


a report the phase-platelet count since it correlated well w ith the slide
b repeat the phase-platelet count on a recollected specimen and check for clumping
c check 1O additional fields on the blood sm ear
d repeat the platelet count using a different method
345. Which one of the following is a true statement about megakaryocytes in a bone marrow
UL$
OtllY
aspirate?
a an average of 2-1 O should be found in each low power field (10 x)
b the majority of forms are the MK 1 stage
c morphology must be determined from the biopsy section
d quantitative estimation is done using the 1OOx oil immersion lens
346. A platelet aggregation tra cing appears to confirm the diagnosis of Glanzmann
MLS thrombasthen ia in a patient presenting with a platelet disorder. How would these tracings
OH'..Y
look if they were performed using light transmittance optical density aggregation
a decreased platelet aggregation to e-istocelin
b Increased platelet aggregation to ristocetin
c normal response for all agonists
d markedly decreased aggregation to epinephrine. ADP and collagen
347. A patient has been taking aspirin regularly for arthritic pain. Which one o f the following
tests is most likely to be abnormal in this patient?
a platelet count
b PFA-100
c prothrombin time
d activated partial thromboplastin time
348. Which of the following p latele t responses is most likely associated with type lib von
w.s
ONLY
Wlllebrand disease?
a decreased platelet agg regation to low dose ristocetin
b normal platelet aggregation to low dose ristocelin
c increased aggregation to low dose ristocetin
d decreased aggregation to high dose ristocetin

2 74 Tho Board of Certification St udy Gulde 60 ISBN 978-089189·6609 @21J ISASCf


~
109Y . Pla lcl<Jt s: laborn1ory Dotormlnalions
..ie1113
10 . ptatelcl aggrcg:illon responses are 01051 l'k 1 . .
.i: r rotlO'"'"g ' c Yassoetaleu v11th:
fl\8
,it o I A • ADP
~· 10 • ......,""'""'!llllllfillllll\ B • EPt
v4) 20 D C • collagen
§ 30 c D • low doso ADP
~ 40
·e: 50
f/) 60 - B
§ 70
~ ao
~ 1~ ,L_,,.._,.1~.......-2-,.-'3"_4_
Time, Min
I
ge pool disorder
a st0 ra.mann thrombas1henia I
•I
1
b Gan•
irin ingestion I
as P
cd hemoph11a ·1· A
)51). piatelel aggregat1on will occur with the end production of:

•• cyctooxygenase
"" ~ arachidoni_c acid
c prostacychn
d thromboxane A2
The following platelet aggregation tracing represents:
)5 I.

a van Willebrand disease


b storage pool disease
~ Gla~~mann thrombasthenia
aspmn

Clioical L1boralory Certilicatiori EKamlriations 275


Platelets: Laboratory Dete
4 : Hcma to IogY rm1na 11
352. Which of the following can affec t the PFA 100 c losure time? ~
a asp1nn
b thromboeytopenia
c anemia
d all of the above
What does the secondary wave of plate et aggregation seen w ith the b 1phasic low d0
353.
ADP and epinephrine response represent? se
a increased binding to collagen
b release of platelet granules
c Increased activation by collagen
d forma tion of fibrin dimers
Which of lhe following tests provides an appropriate laboratory confirmation of Imm
354.
MLS media ted heparin Induced thrombocytoponla (type 2)? une
""'' a onti-PF3 antibody
b serotonin release assay
c nstocet1n induced platelet aggregation assay
d reptilase assay
355. The principle of platelet aggregation 1s based on:
a decreased light transmission
b increased light transmission
c decreased light absorbanoe
d increased light absorbance

I 356. Whole blood evaluation of blood clotting can be performed by using th


This methodology looks at :
b
rom oelastography.

a clot onset
b clot strength
c fibrinolysis
d all of the above
357. The following tracing represents a patient with :

....
low dose ristocetJn_

..
..."'
Joo
....
..•• ..
.....
..,
~ h.._h dose rlatclceu
~~_..!,..__--+-!-
•O

a type 1 vW disease
b type 2 vW disease
c type 3 vW disease
d no vW disease

276 The Board o f CortlOcatlon Study Gulde Ge


ISBN 978-0891~ 02Q!SASC1'
. Heniato •v!:JY
4
· jhe following tracing 1s from a cardiac sur llomost11sls
$SS· tracing shows the pal!ent during cardiac s~~~~~~~~nt using lhromboelastography This

·"--·+
patient "*•ult

3 under-anticoagulated
b sufficiently anl!coagulated
c over-anticoagulated
d normal

Hemostasis
359, Warfarin is classified as a v1tam1n K antagonist. The factors that are impacted by warfarin
therapy are:
a VIII, IX and X
b v
I, II. and VII
c II, VII, IX and X
d II, V and VII
360. When a patient is placed on warfarin therapy, the first factor that will be decreased is:
a factor II
b factor V
c factor VII
d factor VIII
361. A patient with a positive family history of bleeding presents to the ED with the following
results:
aPTT: '-/19.5 seconds (25-35 seconds)
PT. 19.2 seconds (10.5· 12 5 seconds)

The patient is not on any medication and so is I kely lo be deficient in factor:


a II
b v
c VII
d VIII
362. Hageman factor {XII ) is involved in each o f the following reactions except:
a activation of C1 to C1 esterase
b activation of plasminogen
c activation of factor X I
d transforma tion of fibrinogen to fibrin

:12oi8ASCp ISBN 97~918~ Ctittical Leboretory C• rtlfication &am/nations 217


~
4: He n,ato lo gy Horno 51
· . A t' t sample has been tested in the routine labora tory with .a PT and an aPn . as/3
363 cfu~~~~ter factor assays are ordered on the same sample. Which factor is tha rno. Six
11 St lab~c
a II ·?
b VII
c VIII
d x
364. The most potent plasminogen activa tor in the contact phase of coagulation is:
a kallikrein
b streptokinase
c HMWK
d fibrinogen
365. A patient presents with an aPTT.of 62.5.seconds (25-35 s~conds) and the only f:Jctor
is decreased is factor XII. What 1s the clinical picture for this patient? that
a negative bleeding history
b prolonged PFA
c decreased risk of thrombosis
d eplstaxis
366. The 2 factors that differentiate liver disease from vitamin K deficiency are:
L.,.
a II and Vil
b IX and Vil
c VIII and IX .•
d V and Vil
367. In a patient diagnosed with liver disease, which one of the following factors typically sh
an increase? ows

a factor V il
b factor V II I
c factor IX
d factor X
368. The results on a patient are as follows:
t est patient resulls roforonce r-ang e
PT: 18.5 seconds 11.0· 13.5 seconds
a PTT: 47.5 seconds 24·35 seconds
thrombln time: 14.0 seconds 12· 19 seconds
ATlll: 82% 7 0·130%
proleln C: 54% 77- 167%
proleln S: 48% 65- t40%
activated protein C resistance: 2.6 >2.1
These results reflect:
a thrombophilia
b factor IX deficiency
c heparin
d warfa ri n

278 T·h o B o ard of Cortlllca lion S tud y G ulde 6e


• 4: t-1 ernt""'~,,,
A4-year-old bo~ presents with chroni . Hemos tosis
369· iso presents with a bleeding diathe ~ ear 1nrections and
a P•tlo 1 sis. Factor assay 1s on prophylactic antibiotics. He
cost n results r 1 s reveal·
rae1or VIII: 100% • ••once range •
factor V· 75 "~ 50· 150%
focior lX: 36% ~~· 150%
factor 11: 22% • 150%
50 · 150%
possible causes are:
3
factor II deficiency
b 1upus anticoagulant
c hemophilia
d vitamin K deficiency
A hemophiliac male and a normal femal
370. . e can ~roduce a:
' female earner
' 3
b male carrier
c male hemophiliac
d normal female

371. Hemophilia B is a sex-linked recessive disorder that presents with a decrease in factor:
/
...
,,.., a VIII
b IX
c x
d XI

37 2·/ ro· distinguish between hemophilia and von Willebrand d'isea · 'th
b d ·11 t 'th h' se, a pa11en1w1 von
/~' Wille ran w1 presen w1 w 1ch of the following test results?
results aptt platelet scrocn ristocetin cofactor
resull A: abnormal normal normal
result B: normal abnormal normal
result C: abnormal abnormal abnormal
result D: normal normal abnormal
a resultA
b result B
c result C
d result D
373. A patient presents with bleeding 48 hours after tooth extraction. Results are as follows:
test patient results reference range
PT 11.5 seconds 10·13 teconds
aPTT 32.5 seconds 23·35 seconds
rtbrinogon 345 mgldl (3.45 gill 200·400 mg/dl (2.0·4.0 git)
plalelels 324 • t03/µL (324 x 10'/L) 150-450 x 103/µL (150·450 x 10' JL)

The cause or bleeding is most likely a deficiency in:


a plasminogen
b factor XII I
c alpha-2 antiplasmin
d factor XII

/ ..."'37("The activation of plasminogen to plasmin resulting in the degradation of fibrin occurs by


' a PAl-1
b alpha-2 antiplasmin
c IPA
d alpha-2 macroglobulin

Cllnical Laboratory Cerlific.ilion Examln•tlons 279


'Zl! "5CP ISSN978-06918M009
4 : H e1natolo gy ~
-;;;-"j\"::;:;;:::;:::::-;::;-::::;:;;:--;::-;:;~;:;:~:;.:;;;:t:;;tt7";:;;!;!;::i~;rt~r;:;~:;:;;:;--~~~~:
375. A deficiency or protein C Is ossociated w ith w h ich or tho rollowing? ~'~0~11~100 ~ 111~1,
t• ~

"''' n prolonged aPTT


b decreased fibrinogen level (<100 mg/dl (<1 .0 g/L])
c increased risk or thrombosis
d spontaneous hemorrhage
376. Biological a ssays for antlthrombin (AT) are b ased on the inh ibilion or:
a Iactor VII I
b h eparin
c serine p rotea ses
d anti-AT globulin
377. A patient presents with a low p rotein S activity and low total and free protein s ant
~v The C4b binding p ro tein is norm al. This is classified as: '9eris.
a no deficiency
b type I
c type II
d type Ill
378. APC re sistance is confirmed by the molecular test ror
1.~s
ONLY a PAI 1 4G/5G
b MTHFR
c FVL
d G2021 0A
379. Acute disseminated intravascular coagulation is characterized by:
a hypofibrinogenemia
b thrombocytosis
c neg ative 0-dimer
d shortened thrombin time
380. A p atient develops unexpected bleeding and th e following test results w ere obtained:
PT and aPTT: prolonged
fibrinogon: decreased
0-dlmer: increased
platelets: decreased

Wh at is the most probable cause of these results?


a familial afibrinogenemia
b primary fibrinolysis
c DIC
d liver disease
381 . The aPTT is a coagu lation screening tests th at is also u sed to:
a evaluated the extrinsic coagulation pathway
b monitors Coumadin® therapy
c requires tissue thrombopla stin
d m onitor unfractionated heparin therapy

g®t8ASCP
280 Tho B oar d of Certlncatlon Study Guido Ge ISBN 978.0S9 I89-6609
d
~gy Homostosls
,4: f'le . t presents with an aPTT of 49 seconds (25 35 · ·
Apauon "th results as follows· - seconds). A mixing study is
382· rtorrned wi ·
P8 .11111, = 39.8 seconds
~lad normal plasma = 32.0 seconds.
results indicate:
jhe . I
·~•la ting anticoagu ant
a c11~v fi .
factor de 1c1ency
~ decrease.d platelets
d fibrinolys1s
A patient is taking 10 mg p~r day of C?umadin'> {warfarin). The results o f which o f the
383' foilowlng laboratory tests will be most impacted?

3 protein C .
b antithromb1n
c factor V Leiden
d factor VII I
3S'· A patient presents with the following results:
thrombon time. 48 seconds (12-21 seconds)
replllase time: 38 seconds (14-22 seconds)

ihese results are characteristic of:


a dysfibrinogenemia
b increased D-dimer
c fibrin monome r-split product complexes
d therapeutic heparinizalion

385. The best test to determine if a sample contains residual heparin:


, a fibrinogen
b thrombin lime
c prothrombin time
d D-dimer
386. tn the Clauss fibrinogen method, the time to clot formation in plasma is measured after the
' . / addition of:
a calcium
b thrombin
c phospholipids
d kaolin
387. A patient has the following results:
PT: 12.1 seconds
aPTT! 52seconds
immediate 1:1 mixing study: patient s 50 seconds, poote-:1 normal plasma= 32.7 seconds

The ne)(t test that should be performed is:


a TCT
b ORVVT
c FVllt
d PFA 100
383. A patient is suspected of having a factor XIII deficiency, which test is most likely abnormal:
_,.
a prolonged activated partial thromboplastin time
b clot solubility In a 5 molar urea solution
c euglobulin dot lysis
d Prolonged prothrombin time

Clirc;cal Labo~tory C•rliflcation Examinations 281


4 : H emato logy
• .
~
31i
Ofllos1.,
389. A orthopedic patient is placed on l<;:>w molecul~r weight heparin ~Iler back surge .
lo determine if the level of LMWH 1s therapeutic the test to monitored levels wo 'Y 1
d· n orae
u 1 be; r
a anti-Xa assay
b aPTT
c PT
d anti-Ila assay
390. When a patient presents with an elevated factor VII I level what test will be impacted.
a prolong PT
b shorten PT
c prolong PTT
d shorten PTT
1
/ 391. Only an abnormal aPTT would be seen in the following disord er:
a deficiencies of factors X , V, and It
b d isseminated inlravascular coagulation
c liver disease
d anti-factor VIII antibodies
392. The most common coagulation factor in which a specific inhibitor can form is:
'·'1.S
ONLY a factor II
b factor VII I
c factor IX
d factor X
393. How does tissue factor pathway inhibitor inhibit coagulation?
a inhibition of tissue factor-factor VI ia-factor Xa complex
b conversion of thrombin to prothrombin
c uncoupling factor X III dependent crosslinking of fibrin
d binding and hiding tissue factor on the endothelial surface
394. What is usually the factor VIII level irt a hemophiliac patient with spontaneous bleeding?
a <1%
b 5-10%
C 20-30o/o
d 50-60°/o
395. What is.the most common presentation of factor XIII deficiency?
a c linically inapparent
b delayed bleeding tendency
c severe bleeding responsive to DDAVP
d severe bleeding not responsive to DDAVP
396. A patient is diagnosed with amyloidosis, they w ill be deficient in which of th e following
factors?
a factor II
b factor V
c factor VII
d factor X
397. Activated protein C (APC) resistance is associated with a mutation in:
a thrombin
b factor V
c factor VIII
d protein C

282 The Board of Certification Study Gu;do 6• ISBN 97a.-089189-6609


~ ~tolo9Y lltJlllQ!;/llSI.~
,...•• tJ611~· 111051common ct111so of lnhorlled lhrombop111.11 a 7
J! 1110
1 15
l'\11•~ 11011pid syndrome
11
i~· 11ti1>110~~in G20210A mulallon
3 ..r0th'°
U I'"-, r V LBJ
·den .
facto -ocysteinem1a
G yp<lrt10•"
foll~~~~~:~~~~~~~~~mbophilia
1
d :cl1 of lhB most often presents as lhrombotic
'lo'1~1 d~s with re '
,1!9. p1SO ~
l· e c
. deficiency
3
protein mbin deficiency
b anuthrombin G2021OA deficiency
ro111ro
c P tor Leiden
d faC
v . •
are patients with homozygous prolem C deficiency particularly at an increased risk
wtiat . 7
,~. f deve1op1nQ .
0
uransfusion purp.ura
a pOSrf rin skin necrosis
ll wa a .
urpura 1u1m1na ~s
c P mbocytoph1 1a 1
d thrO . . .
Which of the following proteins provides the binding site for heparin?
131.
8
prote!n C
b protein S q
c factor X .
d ant1thromb1n
. hibitor of plasmin activity is:
102. /lfl in
a tPA
b PAl-1 . .
c alpha·2. ant1plasm111
d plasminogen
The oRW screen test will be prolonged in a patient with lupus due to the reagent
1)3. • . •
c:onta1nif19.
8
decreased concentration of phospholipid
b increased concentration of phospholipid
, hexagonal phase phospholipids
d bilayer phospholipids
404, Which of the following tests is diagnostic in DIC?
a thrombin time
b 0-dimer
c fibrinogen
d none of the above
~s. Afactor X assay has the following results:
1:10 dilution: 50%
1·20 dilution: 77%
1:40 dilullon: 127%

These results Indicate the presence of:


a factor deficiency
b dotted sample
c inhibitor
d bad draw

Cl!l1aAScp Clinic;;/ Laboratory Corliflcatlon Exatninatlona 283


1S8N 978·0S918!H;609
~
4~_!:!~~~~~~-:-::--;:-~:::;::;::;-~~~~~~~~~~~~~~~~~~~~-H-e_11:.1os1
~ : Hematology :~
,:
.. 406. Antithrombin inhibits factors
a Ila and Xa
b Va and VIiia
c VIia and Xlla
d IXa and Va
. lant may bleed due to:
407. A patient that has a lupus an t icoagu
a factor VIII deficiency
b drugs .
c antibodies to prothro mbm
d infection
408. The measurement principle that uses factor Xa which is to a patient plasma and added 10
a substrate is a :
a factor assay
b chromogenic assay
c ELISA assay
d immunologic assay
409. Patients with factor XI II deficiency have:
a an abnormal aPTT
b delayed bleeding
c tissue damage
d joint bleeds
410. In hemolytic disease of the newborn, babies present with:
/
a hyperbilirubin emia
b thrombocytosis
c hyperalbuminemia
d leukocytosis
411 . Occasional spontaneous bleeding may occur in a hemophiliac who is classified as:
a severe
b mild
c moderate
d all of the above

-
, 412. In liver disease patients present with decreased synthesis of:
a
b
c
common pathway factors
intrinsic pathway factors
fibrinogen pathway factors
d vitamin K dependent factors
41 3. Alloantibodies are found in:
a congenital hemophilia
b acquired hemophilia
c congenital v on Willebrand disease
d acquired van Willebrand disease
414. When there is a disparity between the results of the fibrinogen antigen a nd the activity. the
,, most likely diagnosis i s:
a dysfibrinogenemia
b hypofibrlnogenemia
c hyperfibrinogenemia
d afibrlnogenemia

284 The Boord o f Certlncallon Study Gulde 6e ISSN 97a.-0891&g..6609 C2!)IS AW


~ . ~emato logY .
Homostas/s
A. e 1651 used lo quant1tale a rector inhibitor . lh
tS. fll IS e:
1 ractor iissay
3
r11ullimer test
b 0510cetin cofactor
~ Bethesda assay
leeding doesn't correlate well with ractor level . .
B
116. s in a deficiency or:
3
riictor VIII
b ractor IX
c ractor XI
d ractorVl l
excess O-dimers indicate that clots have been:
111.
3
converted lo fibrin monomers
b released into the circulation
c stimulated to activate platelets
d formed and are being excessively lysed

its. 1n the Clauss librinogen assay the time for clot formation in plasma is measured after the
addition of:
a calcium
b thrombin
c phospholipids
d fibrin

119. o-dimers are produced from:


a crosslinked and stabilized fibrin clots
b decreased librinogen and platelets
c plasminogen being converted to plasmin
d generation of thrombin from endothelial cells
420. A patient presents with an aPTT of 24 seconds (25-35 seconds) which is below the normal
range. A possible explanation for this result is the presence of a(n):
a ovr
b lupus anticoagulant
c acute phase reactant
d inhibitor
421. A newborn baby boy is known to have a homozygous protein C deficiency. This puts him
at a risk for:
a ovr
b warfarin induced skin necrosis
c increased risk of thrombosis
• d purpura fulminans
422. Protein S forms a reversible complex with:
a C4b binding protein
b protein C
c total protein S
d Fe receptors

C'lOl8ASCP ISBN978-089189-6609 Cllnlcal Loborotory Certification Examinations 285


......._
4 : Hematology
. ..
~ ornos1a
423. /\ 48-year-old male is screened pre-operatively. He has a positive family history f lla
bleeding. The patient is of Ashkenazi Jewish descent. His results are as follows· or
PT: 11.5 seconds (10.5-13. 5 seconds)
a PTT: 45. 1 seconds (25- 35 seconds)
1: 1 mixing sludy: pallent : 26.1 seconds. poolod normal plasma =29 seconds
Based on this history and the results of these tests, this patient's m ost likely d iagn .
. .
d e f 1c1ency .
in facto r:
os1s is a
a VIII
b IX
c XI
d XII
424. A pediatric inpatient presents with an a PTT of 54.8 seconds. With an abnormal thro b'
time of 26 seconds (12 - 19 seconds). a reptilase time is performed and is normal at rn •n
15 seconds (12 - 19 seconds). Based o n these results , what disorder has been ruled
0Ut?
a lupus anticoagulant
b fibrinogen deficiency
c heparin contamination
d presence or a DTI
425. A patient has their coagulation blood sample drawn from a line. This is the only option
for obta ining the sample. T he PT is normal at 11 .5 seconds, the aPTT is p rolonged at
67 seconds, and the thrombin time is prol,:>nged at 30 seconds. Ba sed on this information
how should testing proceed?
a factor assays testing should be performed
b a mixing study should be done
c a heparin neutralization should be done
d an inhibitor assay should be pe rformed
Unregulated and excessive formation of thrombin and plasmin is seen in:
--·426.
a liver disease
b vitamin K deficiency
c hemolytk: d isease of the newborn
d disseminated intravascular coagulation
. 427. Large blue inclusions bodies in WBC are found in patients with :
../'
a Wiskott-Aldrich s yndrome
b May-Hegglin Anomaly
c Ehlers-Danlos syndrome
d Hermansky-Pudlak syndrome
428. A patient presents with a factor VIII level or 2 o/o. The vWF activity (rislocetin cofactor) is
< 1% with a vWF antigen of 3%. The most likely diagnosis is:
a hemophilia A
b hemophilia B
c type 11 vWD
d type Ill vWD
429. The propagation phase of the in vivo coagulation model includes
a a final burst or thrombin
b activation or plate lets
c feedback m echa nism to thrombln
d activatio n of fa ctor Va and factor VIiia

286 Tho Board of Certlncatlon Study Guido Ge ISBN 978 089189-6609 ·m<l' 3ASCI'
~gy Hcmos tosis
4: tfC cell·based model or coagulation the intrinsic pathway operates on the:
tn tfUl
,;o. ctiV<iled platelet .surface to.P~?duce the burst or thrombin
a ~ e ractor bear.mg eel! to 1nibate and amplify coagulation
b (ISS~()lytic digesbon ol f1bnnogen
c P'°. ation or the contact pathway
d acuv
tient presents to t~e coumadin clinic with an INR or 3.1. He has a mechanical heart
131. Apa The tevel or anticoagulant should be:
varve.
decreased
3
b increased
c stopped
d not adjusted
A new PT reagen.t is being set up in the coagulation laboratory. The ISi of the new reagent
l32. is .o
1 vs the previous reagent with an ISi of 2.1. The new reagent is said to be:
a more sensitive
b ress sens111ve
c insensitive
d no change
Anew oral anticoag~lant a~ix~b~n has been given to a patient who was previously on
433• warfarin. This drug directly 1nh1b1ts:

a vitamin K dependent factors


b factor Ila
c factor Xa
d both factor Ila and Xa

4:14. The anticoagulant that directly inhibits thrombin is:


a LMWH
b argatroban
c warfarin
d rivorxaban
435. A patient who presents with renal impairment is being started on oral anticoagulant
therapy. The DOAC that should be avoided would be:
a dabigatran
b apixaban
c rivaroxaban
d warfarin
436. In secondary hemostasis, coagulation pro teins become activated to form a fibrin clot. Prior
/ to these proteins being activated, their inactive proenzymes are known as:
a serine proteases
b cofactors
c :zymogens
d substrates

C'.Qll!.SCP ISSN 978-089189-6609 Clinical L•bon1toty Certlflc.ation Examinations 287


~4~:~H
~e~n~1'..'.a~t~o~l~o~g~y~-:--:-:-=:;::::;-:::;,hjh,;;~i;;;';;;:;-;;-;;;;,~t;;;;:;;~-------H-o..".:.::
: 'Os1os15 J
437. A 42-year-old male presented with the fol'owing symptoms: ~
rn1lguo. difficulty breathing. tingling In tho hand s and reel
PT: 25.6 seconds (11.5- 13.9 secon ds)
aPTT: 77.5 seconds (25.0- 35.7 second s) s
clinician quesUon ed 1he rc sulls. patient had no bleeding sympl om
Hgb: 23.5 g/dL
Hct: 76.2%
This result could be caused by:
a aplastic anemia
b polycythemia vera
c chronic lymphocytic leukemia
d sickle cell d isorder
• 438. Patient wilh a clot presents with the fo llowing res ults:
~
PT: 15.5 second s (10.5 - 12.5 second s)
a PTT: 50.3 seconds (24 -34.0 seconds)
D -dlmer: 1.62 mg/L (<1.10 mgl L)
fibrinogen: 35 mg/dL (180-400 mgldL)
fibrlnogen antigen: 268 mg/ L (150-350)

The mos t likely diagnosis is:


a DIC
b hypofibrinogenemia
c dysfibrinogenemia
d afibrinogenemia
439. A sample is sent to the laboratory for an anti-Xa assay. The result of the PTT is 65.7
seconds. The result of the anti-Xa assay is 0.9 U/ml of heparin. T he patient is on
Lovenox®. T heir anti-Xa level is:
a subtherapeutic
b the rapeu tic
c supratherapeutic
d prophylactic
440. The following results are obtained on a pregnant w oma n who is short o f breath.
proced ure resul t reference interval
protein C activity 62% 70- 120%
p rotein S activity 50% 62 - 145%
vWF activity 199% >50%
vWF anligen 192% 50 - 150%
ractor VIII 189% 50- 150%

T he pa tie nt's obstetrician calls because o f concern a bout the p atient's clotting risk. Based
on this pregna nc y s tatus of this patient, the resu lts appear
a elevated
b that the patie nt is at risk for bleed ing
c that the patient is at risk for clotting
d normal

288 The Board o f Certification Stu dy Gulde 60 ISBN 978-089189-6609 c20ts.ASCI'


....
Hemostasis
fllatolo9Y . . .
J: tfe ,.old male with metastaticfancreat1c carcinoma shows elevated PT and aPTT.
A65·Yea uni 15 ~ 103/µL ( 15" 10 IL) and elevated 0-dimer. On a blood smea r you would
j.11· piat~kll co
see· veu-Jolly bodies
.
a tlO'ctO-°valocytes
b ~islocYles
c 1arge1 cells
d ar-otd male with celiac disease presents with occult positive stools. What vitamin
••2. p.25:Ye,.,, should you consider?
.. deflcsen~,
vitamin A deficiency
o
3 vitamin deOc!ency
~ vitamin E deficiency
d vilamin K deficiency
aPTT result from the main laboratory is 67 seconds, and Is frozen to be saved for
Ill. :d'fonal lupus testing in the special coagulation laboratory. The aPTT repeated the next
• daY';~ 37 seconds. This can be caused by:
wrong sample saved
: faclors were activated
platelets contaminated sample
~ different reagent sensitivity used

t'ti111.Scp
~ IS13N 978-089189-6609 Clln/CJll Laboratory Cattil/calion Examinations 289
4: Hernatology
~'
H c 1t1os1
1. b 57. c 113. d 169. c 225. c 281. d as;s
337. b
2. ii 58. c 114. d 170. b 226. a 282. a 393
338. b
3. b 59. b 115. d 171. d 227. a 283. b 339.
394'
. aa
4. b d
60. c 116. c 172. a 228. d 284. c 340. c 39s. ti
5. d 61. c 117. a 173. b 229. a 285. b 341 . 395. d
6. c d
62. b 118. b 174. a 230. a 286. d 342. a 397. b
7. d 63. d 119. a 175. b 231 . a 287. d 39a. c
343. c
8. d 64. c 120. c 176. c 232. b 288. d 344. a 399. b
9. d 65. d 121 . d 177. c 233. b 289. b 345. 400. c
10. d
a 401, d
66. b 122. d 178. b 234. b 290. a 346. d
11. b 67. a 123. a 179. b 235. b 291. a 347. 402. c
b
12. d 68. a 124. a 180. b 236. c 292. a 348. 403. a
13.
c 404. d
d 69. a 125. b 181. d 237. d 293. a 349. d
14. a 70. b 126. c 182. d 238. d 294. d 350. d
405. c
15. b 71. a 127. d 183. d 239. d 295. d 406, a
351. c 407. c
16. c 72 . c 128. d 184. a 240. d 296. d 352. d
17. a 73. c 129. c 185. d 241 . a 297. d 408. b
353. b 409, b
18. c 74. b 130. b 186. b 242. b 298. d 354. b
19. b 75. b 131. a 187. b 243. a 299. c 410. a
355. b 411. c
20. d 76. c 132. c 188. d 244. c 300. a 356. d
21 . c a 412. d
77. 133. d 189. c 245. a 301. b 357. d 413. a
22. a 78 . c 134. b 190. d 246. a 302. a 358. b
a 414. a
23. 79. d 135. c 191. c 247. c 303. c 359. c 415. d
24. b 80. b 136. d 192. d 248. a 304. c 360. c 416. c
25. a 81. d 137. d 193. a 249. c 305. c 361. c 417. d
26. a 82. c 138. b 194. d 250. a 306. b 362. d 418. b
27. b 83. a 139. d 195. b 251 . b 307. a 363. c 419. a
28. b 84. c 140. c 196. d 252. b 308. a 364. a 420. c
29. b 85. a 141. d 197. b 253. b 309. c 365. a 421. d
30. b 86. b 142. e 198. a 254. b 310. b 366. d 422. a
31 . b 87. d 143. a 199. d 255. d 311 . c 367. b 423. c
32. b 88. a 144. c 200. d 256. b 312. b 368. d 424. b
33. c 89. b 145. b 201. c 257. c 313. d 369. d 425. c
34. d 90. d 146. d 202. c 258. a 314. a 370. a 426. d
35. d 91. a 147. d 203. c 259. d 315. d 371. b 427. b
36. b 92. a 148. b 204. a 260. d 316. b 372. c 428. d
37. c 93. d 149. b 205. d 261 . c 317. d 373. b 429. a
38. c 94. d 150. c 206. c 262. a 318. d 374. c 430. a
39. b 95. b 151. c 207. b 263. b 319. a 375. c 431. d
40. b 96. c 152. a 208. a 264. d 320. b 376. c 432. a
41 . d 97. a 153. d 209. b 265. c 321. a 377. b 433. c
42. a 98. c 154. d 210. d 266. a 322. d 378. c 434. b
43. d 99. a 155. c 211. a 267. b 323. c 379. a 435. a
44. a 100. b 156. a 212. a 268. c 324. a 380. c 436. c
45. b 101 . b 157. d 213. d 269. c 325. b 381. d 437. b
46. c 102. b 158. d 214. b 270. d 326. b 382. a 438. c
47. a 103. a 159. a 215. a 271. c 327. d 383. a 439. b
48. d 104. d 160. b 216. d 272. a 328. d 384. a 440. d
49. a 105. c 161. a 217. b 273. d 329. a 385. b 441. c
386. b 442. d
50. b 106. a 162. b 218. a 274. c 330. a 443. c
51. a 107. c 163. d 219. b 275. b 331. c 387. b
52. d 108. b 164. c 220. c 276. c 332. d 388. b
53. d 109. a 165. a 221. a 277. b 333. c 389. a
54. d 110. d 166. d 2.22. d 278. b 334. b 390. d
55. c 111. c 167. b 223. b 279. b 335. b 391. d
56. b 112. b 168. c 224. d 280. c 336. c 392. b

"2\'.)18ASCP
ISBN 978-089189-661l9
290 T ho Board of Ccrtillcallon Study Guido 6e .........
Autoantibody Eval uation
, 0 109~Y------------------_:__ _ _ _
1
~unology
. s have been identified generally as appropriate for both entry love/ medical
,ot111g_ite~ts and medical laboratory technicians. Items 111at are appropriate for medical
1 110 scientl ts on/ are marked with an 'MLS ONLY."
fl~,;ol}'
SJ"""~i ~fl[J!E IS~='--"--'--­
~~~-
342 Answers with Explanations
auestl~~~y Evaluation 343 Autoanlibody Evaluation
Jt~f3 ~s Dis ease Serology 345 Infectious Disease Serology
323 In~ · Analysis 348 Protein Analysis
9 protein munily & Hislocompalibility 351 Cellular Immunity & Histocompatibility
32 clJ/Jutar 1m
Techniques
331 TodJlli'lUCS

antibody Evaluation
Auto . cl ar antibody tests a re performed to help diagnose:


Ant1nu e
acute leukemia
~ lupus erythemat.osus
hemolytic anerrna
I
~ crohn disease
In the anti·double-slranded DNA p rocedure, the antigen most commonly utilized is:
?.
:;-tr a rat stomach tissue
b mouse kidney tissue
c Crithidia /uciliae
d roxoplasma gondii

1 Reier to the following image.

Which of the ANA patterns shown above would b e associaled with high liters of a ntibodies
to the Sm antigen?
a image A
b image B
c lmagec
d image D

Cllnlt:al Laboratory Certifl~•t_lon Ex:imlnation• 313


5: Immunology Autoa11tiboc1
y €11,, ,11
4. Sera to be tested fo r tFA-ANA 6 days after drawing is be st stored at: «110~
a room temperature
b s·c ± 2•c
c - 70°C in a consta nt temperature freeze r
d - 20°C in a fros t-free self-defros ting freezer
5. Antibodies directed at native DNA are mos t frequently associated with which P
fluorescence in the IFA-ANA tes t? alter~ or
a rim
b diffuse
c speckled
d centromere
6. The technologist observes appare nt homog enou s staining of the nucleus or in ter h
cell s while performing an I FA-ANA, as well as s taining o f the ch romosomes in mif ase
This result is: Otic CeUs.
a indicative or 2 antibodies, which should be separately reported after titration
b expected for anti-DNA antibodies
c inconsistent; the test should be reported with new reagent
d expected for anti-centromere ant ibod es
7. The result of an a nti-nuclear antibody test wa s a titer of 1 :320 with a peripheral patte
Which of the following sets of resu lts best correlate with these results? m.
a anti-dsDNA titer 1 :BO. and a high titer of antibodies to Sm
b a nti-mitochondrial anti body tite r 1 :160, a nd an tibodies to RNP
c a nti-Sct-70, and antibodies to sing le-stra nded DNA
d high tilers or anti-SS-A and anti -SS-B
8. Systemic lupus erythematosus patients often have which of the following test results?
a high titers of DNA a ntibody
b decreased serum immunoglobutin levels
c high tilers of anti-smooth muscle antibodies
d high titers of anti-mitochondrial a ntibody
9. Systemic lupus erythematosus patients with active disease often have which of the
~~' following test results?
a high ti ters of anti-microsomal antibodies
b high liters of anti-smooth m uscle antibodies
c marked decrease in se rum CHso
d decreased serum immunoglobulin levels
10. Which of the following is decreased in serum during the active stages of systemic lupus
~~y erythematosus?
a anti-nuclear antibody
b immune complexes
c complement (C3)
d anti-DNA
11. A positive ANA with the pattern of anti-centromere antibodies is most frequently seen in
~' patients with:
a rheumatoid arthritis
b systemic lupus erythematosus
c CREST syndrome
d Sjogren syndrome

ISSN 97&-0891~ C20IS,J.SO'


31 4 Tho Board ol Certlncetion Study Gulde 6e
_.....
5·lrtl
·
rn1.inolo9Y
. 1t .
Autoanllbody Evaluation
the indirect n~ordesce'. anti-nuclear anllbody t9st, a hornogenous pattern inclicotos the
l
1 ,,rosenee of anllbO Y to.
7. IO
'" y
'"'' 3 RNP
b Sm
c RNA
d ONA
thB indirect fluorescent anti-nuclear antibody test. a speckled pattern may indicate the
t3· In ence of antibody to;
.,,, pres
""' 3 histone
b Sm
c RNA
d ONA
AutoantibOdies in the absence of Sm are found in patients with:
11 1

~~ mixed conneclive tissue disease


,.., ~ systemic lupus erythematosus
I~ I
c crohn disease t,
d multiple mye1oma •
tS. Rheumatoid factors are immunoglobulins with specificity for allotypic determinants located
on the:
3 Fe fragment of lgG
b Fab fragment of tgG
c J chain of lgM
d secrelory of component of lgA

16. Rheumatoid factor in a patient's serum may cause a false:


a positive test for the detection of lgM class antibodies
b negative test for the detection of lgM class antibodies
c positive test for the detection of lgG class antibodies
d negative test for the detection of lgG class antibodies
11. Rheumatoid factors are defined as:
a antigens found in the sera of patients wilh rheumatoid arthritis
b identical to the rheumatoid arthritis precipitin
c autoantibodies with specificity for the Fe portion of the immunoglobulin (lgG) molecule
d capable of forming circulating immune complexes only when lgM-type autoantibody is
present
18. Tissue injury in systemic rheumatic disorders such as systemic lupus erythematosus is
::, thought to be caused by:
a cytotoxic T cells
b lgE activity
c deposition of immune complexes
d cytotytic antibodies
t9. False-positive rheumatoid factor in agglutination and nephelometric methods can be due
::, to elevated levels of:
a cryoglobuiin
b hlstidine-rich-glycoproteln
c aspartame
d C1q

~!ASCp CUnical Laboratory Cert.lfic11tion Examinations 315


ISSN 978-069189-6609
5 : lrnmunology Autoanti~
20. . d"1es are most o ften found in:
High tilers of anti-microsomal anl1bo
10 "'" a,.
•o~

a rheumatoid arthritis
b systemic lupus erythematosus
c chronic hepatitis
d thyroid disease
21. Which of the following is an organ-specific autoimmune d isease?
Ml$
Olo.Y a myasthenia g ravis
b rheumatoid arthritis
c Addison di sease
d progressive systemic sclerosis
22. In chronic active hepatitis, high titers of which of the following antibodies are s een?
a anti-mitochondrial
b anti-smooth muscle
c anti-DNA
d anti-parietal cell

.,_.
23. In primary biliary cirrhosis, which of the following antibodies is seen in high titers?
au a anti-mitochondrial
b anti-smooth muscle
c anti-DNA
d a nti-parietal cell
24. Anti-RNA antibodies are often present in individuals having an anti-nuclear antibody
immunofluorescent pattern that is:
a speckled
b rim
c d iffuse
d nucleolar
25. Anti-extractable nuclear antigens are most likely associated with which of the following
anti-nuclear antibody immunofluorescenl patterns?
a speckled
b rim
c diffuse
d nucleolar
26. In an anti-nuclear antibody indirect immunofluorescence test, a sample of patient serum
shows a positive, speckled pattern. Which would be the most appropriate additional lest
to perform?
a a nti-mitochondrial antibody
b lmmunoglobulin quantitation
c screen for Sm and RNP antibodies
d anti-DNA antibody using Crithidia luciliae
27. Anti-glomerular basement membrane antibody is m ost o ften associated with this
..,.
~NlY'
condition:
a systemic lupus erythematosus
b celiac disease
c chronic active hepatitis
d Goodpasture synd rome

'"""' 4i2015 ,\SCP


ISBN 9 78-089189""""'
316 The Board ol CortlOcaUon S t udy Gulde 60 J
d
5: immuno logy ,
- A 25-year-old woman is seen by a physic' Autoantibody Evaluatlon
~.~· arthralgias and difficulty in swallowing T~=n because. of Raynaud phenomenon, myalgias,
er<• is 1:5120 .with. a sp~ckled pattern with ·mito~e IS no evo~ence of renal d~sease. An ANA titer
be found on this patient? c cells. Which of the following are also likely to
a high-level nONA antibody and a low CH
b high-level Sm antibody 50 1evel
c high-titer rheumatoid factor
d high-level ribonucleoprotein (RNP) antibody
29. In pernicious anemia, which of the foll owing
·
antibodies is characteristically detected?
~~v a anti -mitochondrial
b anti-smooth muscle
c anti-DNA
d anti-parietal cell

JO. Anti-phospholipid antibodies associated with autoimmune disorders tend to have


~. 1mmunoglobul1n (lgG) that belongs to which of the following subclasses?
a lgG 1 and lgG3
b lgG2 and lgG4
c lgG1 and lgG4
d lgG2 and lgG3
31. The llF staining pattern on ethanol-fixed leukccytes slides shows a perinuclear or nuclear
w staining pattern. This pattern is typically due to:
01.Y
a C-ANCA
b LKM
c P-ANCA
d GBM
32. The speciricity of an immunoassay is determined by the:
MIS
..... a label used on the antigen
b method used to separate the bound from free antigen
c antibody used in the assay
d concentration of unlabeled antigen
33. In assessing the usefulness of a new laboratOI)' test, sensitivity is defined as the
~. percentage of:
a positive specimens correctly identified
b false-positive specimens
c negative specimens correctly identified
d false-negative specimens
34. In the indirect lmmunofl uorescence method of antibody detection in patient serum, the
labeled antibody is:
a human anti-goat immunoglobulin
b rheumatoid factor
c goat anti-human immunoglobulin
d complement
35. A substrate is first exposed to a patient's serum, then after washing, a~ti-hu~ag i .
immunoglobulin labeled with a nuorochrome is added. The procedure escn e 5 ·
a fluorescent quenching
b direct fluorescence
c indirect fluorescence
d fluorescence inhibition

Cllnlcal Laboratory Certification Examirtaflonl 317


t"/.l1eAscp ISBN 978-089189-6609
5: Immuno logy
36. Avidity may be defined as the:
a degree of hemolysis
b tite r of an antigen
c dilution of an antibody
d strength or a reac ting antibody
37. Cholesterol is added to the antigen used in flocculation tests for syphilis to:
a destroy tissu e impurities present in the alcoholic beer heart extract
b sensitize the sheep RBCs
c decrease specificity of the antigen
d increase sensitivity of the antigen
38. The strength of a visible reaction is k nown as:
a prozone reaction
b absorption
c avidity
d elution
39. Which of the following describes an antigen-antibody precipitation reaction of .
.r nor1-1denr1
a precipitin lines cross. forming double s;:>urs 1y?
b precipitin lines fuse, forming a single spur
c no precipitin lines a re formed
d p recipitin lines fuse. forming a single a rc
-40. Which test has the greatest sensitivity for antigen detection?
a precipitin
b agglutination
c ELISA
d co mplement fixation
41. Excess antigen in precipitation gel reacticns will:
a have no effect on the precip itate reaction
b not dissolve precipitate after formation
c enhance the prec ipitate reaction
d dissolve the precipita te after format ion
42. Soluble immune complexes are formed u nder the condition of:
a antigen deficiency
b antigen e xcess
c antibody excess
d complement
43. The visible serological reaction between soluble antigen and its specific antibody is:
a sensitization
b precipitation
c agglutination
d opsonization

318 The Boa rd o f Certification Study Gulde 6e ISBN 978-0891~~


09 ¢>16,:P J
~ology
5: lrTinl . ll11t oa111ibocly Evn/1101/on
....--;; 9 curve below was obtamed by adding incre- ·
JI. Th es of monospecific anlisenmr csmg amounts or a soluble antigen lo fixed
volum ·

B
Increasing concentration
of antigen -- ·>

The area on the curve for equivalence precipitate is: I


a A
b B
c c
d D
45, The curve below was obtained by adding increasing amounts of a soluble antigen to fixed
volumes of monospecific antiserum.

-* c
:J
0 :
c
E 4>
"'
Cl~iii
c a.
·en ·o
oe _
"'
a.
Q)

.s: 0
B
Increasing concentration
of antigen - - ~

The area on the curve where no precipitate formed due to anligen excess is:
a A
b B
c c
d D

t'-"ls~ I.SCP ISSN 976-089169·6609 Ctinlcal Laboratory Certification Examinations 319


5 : Immuno logy
. .
A 11 t oant1bod
Y E~~
~
46. The curve below was obtain~d by a dding increa sing amounts or a solub le antigen t a110~
volu m es or monospecific ant iserum. o ri~
·~

Inc reasing concentration


of antigen - - ~

T he are a on the curve for p rozone is:


a A
b B
c c
d D
47. The curve below was obtained by adding i ncreasing amounts or a soluble antigen 10 fi
volumes or monospecific antiserum. xe.:1

-5 :c .-... c
E
Ill coa>
Ol"'"
.E a.
l{l ·~
Q) ~

u _a.
~

..E 0
B
Inc reasing concentration
of antigen - - ·>

The area on the curve where soluble antig en-antibody complexes h ave begun to form is:
a A
b B
c c
d D

cq:!18 ¢
3 20 Tho B oard o f Certificat io n S tudy Gui de 6e
ISBN 978-0891~---· d
gY Autoanllbody Eva/ualion
..,f11i.111°10
lo\\/ \\las obtained by adding Increasing amounts ol a soluble antigen lo fixed
<: 11•· rl6 bC specific antiserum.
' fllo c~ of mono
~ ~~~s;:_:~----~~~~~~-,
c .

A B
increasing concentration
of antigen -· -> ·'
. which the addition of more antibody would result in the formation of additional
TJ!e area '~
precipitate IS.
aA
bB
c c
d0
Refer the following illustration.
10
is.

(al'\ IQ'\~~
\(CJ
figure
\V
Figure 2
I
\1_}
Figure 3
~
Figure 4

Which of the above figures demonstrates a reaction pattern of identity?


a Figure #1
b Figure #2
c Figure #3
d Figure #4
so. Reier lo the following illustration.

<¥)®©~
Figuie 1 F'rgure 2 Figure 3 Figure 4

Wl1ic:h 01 the above figures demonstrates a reaction pattern or non-identity?


a Figure #1
b Figu1e #2
c Figure #3
d Figure #4

c//nlcal Loboratory Ctrtlficalion Enmlnations 321


5: Immunology
51. Refer to the following illustration:

C2) w C2) c:i)


Figure 1 Figure 2 Figure 3 Figure 4

Which of the above figures demonstrates a reaction pattern showin g 2 different .


molecu lar species? antigenic
a Figure #1
b Figure #2
c Figure #3
d Figure #4
52. R e fer to the following illustration :
MLS
CM.V

®©c:i)
Figure 2 Figure 3 Figure 4

A nonspecific precipitin reaction is demonstrated in:


a Figure # 1
b Fig u re #2
c Figure #3
d Figure #4
53. A series of 8 tubes are set- up with 0. 79 ml of d iluent in each. A seria l dilution is performed
MLS
ONLY by adding 10 µL of serum to the first tube and then transferring 10 µL through each
remaining tube. What is the serum dilution of tube 7?
a 1:2.431 "10 11
b 1:2.621 )( 10 11
c 1:1.920 )( 1013
d 1 :2.097 x 1013
54. The enzyme control tube in an ASO hemolytic assay exhibi ts no cell lysis. What is the
most likely explanation for this?
a incorrect pH of buffer
b low ionic strength buffer
c oxidation of the enzyme
d reduction of the enzyme
55. The following pattern of agglutination was observed in an antibody titration.

Tube 1 2 3 4 5 6 7 8 9 10 11
1+ 2+ 4+ 4+ 3+ 3+ 2+ 1+ 1+ 0 0

This set of reactions most likely resulted from:


a faulty pipetting technique
b postzoning
c prozoning
d the prese nce of a high-titer, low-avidity antibody
e201sAsO'
322 The Board of Cortlflcntlon Study Guide Ge ISBN 978-089189-0609
d
s: 1rn muno1ogy
In a positive anti -streptolysin .. 0 • l11fcct1011s Dlseaso Sorotogy
56· enzyme inhlbitl t
a streptolysin ·o· enzyme in the t' on est, the patient's:
reagent, resulting in no hemof:S;;nt serum neutralizes the anti-streptolysin "O"
b red blood cells are hemolyzed b th
c anti-streptolysin ·o· neutralizes rh etstre~tolysin ·o· enzyme in the reagent
d anti-streptolysin ·o· inhibits the re:; r~ptolysin ·~· reagent, resulting in hemolysis
en streptolysin "O"s, resulting in no hemolysis

Infectious Disease Serology


57, Blood is drawn from a patient for serol .
0
and again 4 weeks later. The results ~~~~a; letsts for a vi~al disease at the time of onset
es s are considered diagnostic if the·
a first antibody titer is 2x the second ·
b first an~ second antibody titers are equal
c first antibody Is 4x the second
d second antibody titer is at least 4x the first
58. Which of the following is most useful in establish'ng
1 a d'iagnosis
· · h
phase of a viral infection? int e convalescence
a slide culture
b serological techniques
c shell vial
d culture on McCoy media
59, The best me~hod to .d etect .infections due to rubella, Epstein-Barr and human
immunodeficiency viruses 1s:
a antigen detection by EIA
b cell culture
c antigen detection by Western blot
d antibody detection by EIA
60. lmmunoassays are based on the principle of·
a separation of bound and free analyte
b antibody recognition of homologous antigen
c protein binding to isotopes
d production of antibodies against drugs
61. A DPT vaccination is an example of:
a active humoral-mediated immunity
b passive humeral-mediated immunity
c cell-mediated immunity
d immediate hypersensitivity
62. Cells known to be actively phagocytlc include:
a neutrophils, monocytes, basophils
b neutrophils, eosinophils, monocytes
c monocytes, lymphocytes, neutrophlls
d lymphocytes, eosinophils, monocytes
63. The presence of HBsAg, anti-HBC and often HBeAg is characteristic of:

a early acute phase HBV hepatitis ..


b early convalescent phase HBV hep~~lts
c recovery phase of acute HBV hepatitis
d past HBV infection

Cllnlc•I Laboratory Certification Examinations 323


('/.i:eAS<;p ISBN 978-089189-6609
5: ln1munology fnfacllous Dise<J ~~
so Soro10
64. Refer to the following data: !ly

HBsAg anti-HBC l gM antl-HAV lgM


patient 111 +
patient 112 + +
paUent#3 +

From the test results above, it can be concluded that patient #3 has:
a recent acute hepatitis A
b acute hepatitis B
c acute hepatitis C (non-Nnon-B hepatitis)
d chronic hepatitis B
65. The disappearance or HBsAg and HBeAg, the persistence of anti-HBc, the appea·
"'5
0"-Y
anti-HBs, and often of anti-HBe indicate: •ance or
a early acute HBV hepatitis
b early convalescent phase HBV hepatitis
c recovery phase of acute HBV hepatitis
d carrier state of acute HBV hepatitis
66. An example of an organ specific disease with autoimmune antibodies is:
MLS
o"'v a Wegener granulomatosus
b rheumatoid arthritis
c Hashimoto thyroiditis
d systemic lupus erythematosus
67. Normal serum constituents that can rapidly increase during infection, injury or trauma are
referred as?
a haplens
b acute phase reactants
c opsonins
d chemotaxins

..o•lv
68.
..,,
An example or a live attenuated vaccine used for human immunization is:
a rabies
b tetanus
c hepatitis B
d measles
69. The acute phase reactant that has the fastest response time and can rise 1oox is?
a alpha-1 antltrypsin
b haptoglobin
c c-reactive protein
d ceruloplasmin
70. What kind of antigen-antibody reaction would be expected if soluble antigen is added to
homologous antibody? ·
a precipitation
b agglutination
c complement fixation
d hemagglutinalion

ISBN 978-0891~ C!2()lS ¢


324 Tho Board o f Cortlficallon Study Gulde 60
~oofoQY /11/oclious O/so;1se Sorology
5: lrflrfl . d i'mmunity can be ch0rnclerizct1 as having wl i"l qu 1•1• 7
quire " 0 1 10s
71. p.c daptive and nonspeclrlc
n non·llat and extern al mech anisms
b intcn;ivilY and short acting
c senS!ficily and memory
d 5 pec1
talion tests for syphilis detect the presence of·
f toccu .
72· reagin antibody
~ antigen .
11ernolys1n .
~ forssman antigen
skin. tactic acid in sweat, pH balance, mucous. and the motion of cilia represent which
73. tYPe of immunity?

3
natural
b 3cqulred
c adaptive
d auto
Ftocculation tests for syphilis use antigen composed of:
74,
Treponema pallldum
3
b reagin . .
c cardiolipin and lecithin
d charcoal
The following cold agglutinin tiler results are o':>served:
75,
Tube# 1 2 3 4 5 6 7. 8 9 10
dilution 1:1 1:2 1:4 1:8 1:16 1:32 1:64 1: 128 1:256 1:512
4•C + + + + + + + + 0 0

37°C 0 0 0 0 0 0 0 0 0 0

The best interpretation is:


a positive, 1:128
b negative
c invalid because 37°C reading is neg ative
d repeat the 4°C readings
76. Substances that are antigenic only when couoled to a protein carrier are:
a opsonins
b haptens
c adjuvants
d allergens
77. A haplenic determinant will react with:
I'll
°'" a both T cells and antibody
b T cells but not antibody
c neither T cells nor antibody
d antibody but not T cells
7s. A serological test for syphilis that depends upon the detection of cardiollpin-lecilhin-
chotesterol antigen Is:
a FTA-ABS
b RPR
c MHA-TP
• d TPI

t".:lt! I.SCP ISSN 97&-089t8M6119 Clinlcal Laboratory Certific6tlon Examinations 325


5: ln1rnunology Infec tious 01lsoaso
~
Sn._. """"""
79· The serological test for syphilis recommended for detecting antibody in cerebro . e'01 of/~
8 P1na1 .
11uro
a non-treponemal antibody 1~.
b CSF-VDRL .
c FTA-ABS
d MHA-TP
80. In t~e direct fluorescent antibody test for primary syphilis, spirochetes are detect
I.OU
ONLY add1t1on of labeled antibody to? ed by
a Treponema pal/idum
b cardiolipin
c human immunoglobulin
d nonpathogenic treponemes

....81.
,,.,y
In the FTA--?-B~ test, the presence of a beaded pattern of fluorescence along the
treponeme mdrcates:
a positive identification of Treponema pallidum
b presumptive d iagnosis of active syphilis
c presence or non-treponemal antibody (NTA)
d false-positive reaction
82. A 16-ye ar-old boy with infectious mononucleosis has a cold agglu tinin titer o r 1·2ooo
tJt.S important consideration o r this antibody's clin ical re levance is the: · ·An
o-..v
a thermal range
b titer at 4 °c
c specificity
d light chain type
83. What assay w ould confirm the immune status to hepatitis B virus?
a HBsAg
b anti-HBs
c lgM anti-HBcAg
d hepatitis C Ag
84. The following p rocedure has been routinely used for de tection or hepatitis B surface
antigen (HBsAg) becau se o r its high level or sensitivity:
a hemagglutination
b counterimmunoelectrophoresis
c radial immunodiffu sion
d ELISA
85. In an in direct ELISA method designed to detect antibody to the rubella virus in patient
serum, the conjugate used should be:
a anti-human lgG conjugated to a n enzyme
b anti-rubell a an tibody conjugated to an enzyme
c rubella antigen conjugated to an enzyme
d anti-rubella antibody conjugated to a substrate
86. Which or the following is the best indicator of a n acute infection with the hepatitis A virvs?
a the presence of lgG antibodies to hei;atitis A virus
b the presence of lgM antibodies to hepatitis A virus
c a sharp decline in the level or lgG antibodies to hepatitis A virus
d a rise in both lgM and lgG levels of antibody to hepatitis A vi rus
87. Biological false-positive VD RL reactions are frequently encountered in patients with:
a lupus erythematosus
b acquired immune deficiency syndrome (AIDS)
c gonorrhea
d te rtiary syphilis
326 The Board ol Certllication Study Guido 6e ISBN 978-08918~ C2(1!&..s<1
d
1
unol09Y . IHfo c U011s Disoasc Scrolot1Y
<· 111111 rological nlPrker of HBV (hepalills B virus) inlccllo11 1 . •
•· ·c11se "'1•calcs recovery anti
1¥n• unity?
$!- ;11111'. I ONA polymerase
'"r8 31111ne11
o H08 •.,
D aoti·H0S
' iJasA9
d file thal matches lhe typical test profile for chronic acr h .. .
we pro,' .
15
ive epabhs due to hepalilos
S9' 0 virUS •
~1 HBsAg lg M anll-HBc ontl-HBc antl·HBs
profile
+

" B
+ +

c
+
0

3
profile A
b profile B
c profile C
d profile D
for diagnosis of late lalenl or tertiary syphilis. the most appropriats assay is:
90.
a RPR
b voRL
c FTA·ABS
d FTA·ABS lgM
A 25.year·o~d nurse developed fatigue: a lo~-grade f~ver, po.lyarthrilis and urticaria . Two
91· months earlier she had cared _for a patient with hepalolls. Which of the following findings
are likely to be observed in this nurse?

3
a negative hepatitis B surface antigen test
b elevated AST and ALT levels
c a positive rheumatoid factor
d a positive Monospol ™ test
92. The classic antibody response pattern following infection with hepatitis A is:
a increase in lgM antibody; decrease in lgM antit:ody; increase in lgG antibody
b delectable presence of lgG antibody only
c ·detectable presence of lgM antibody only
d decrease in lgM antibody; increase in lgG antibody of the lgG3 subtype
93. The 20 nm spheres and filamentous structures of HBV are:
"'.._, a infectious
b circulating aggregates of HBcAg
c circulating aggregates of HBsAg
d highly infectious when present in great abundance
94. The enzyme-linked immunosorbenl assay (ELISA) technique for lhe detection of HBsAg:
a requires radiolabeled Clq
b ts quantitated by degree of nuorescence
c uses anti·HBs linked to horseradish peroxidase
d uses beads coaled with HBsAg

Clinlcal L11bor<Jtory Certification Examinations 327


a
5: ln1rnunology ""'":"u"s u isoaso s fl to/ ...,.
011
95. The antigen marker most closely associated wi th tra nsmissibility of HBV infection is: 1'
a HBsAg
b HBeAg
c HBcAg
d H BV
96. Chronic ca rriers of HBV:
a have chronic symptoms of hepatitis
b continue to carry HBV
c do not tran smit infection
d carry HBV but a re not infectious
97. Hepatitis C differs from hepatitis A because it:
a has a highly stable incubation period
b is associated w ith a high incidence of icteric hepatitis
c is associated with a high incidence of the chronic carrier state
d is seldom implicated in cases of posttransfusion hepatitis
98. The initial immune response following fetal infection with rubella is the production or Wh'1
class(es) o f antibodies? ch
a lgG
b lgA
c lgM
d both lgG and lgA
99. The air temperature throughou t the serolo;iy laboratory is 20°C. How will this affect VDRL
and RPR test results?
a no effect- the acceptable test range is 20-24•c
b weaken reactions so that false-negatives occur
c s tre ngthen reactions so that positive titers appear e levated
d increase the number of false-positives from spontaneous clumping
100. Which laborato ry technique is most frequently used lo diagnose and follow the course of
therapy of a patient w ith secondary syphilis?
a flocculation
b precipitation
c complement fixation
d indirect immunofluorescence
• 101. The most commonly used serological indicator of recent s treptococcal infection is the
~ antibody to:
a s treptolysin O
b hyaluronidase
c NADase
d DNA
102. A heterophile antigen is best described as?
a a n auto-antigen
b existing in an unrela ted animal
c resulting from an amnestic response
d a n adjuva nt to increase immune response

328 Tho Board of Cer11ncatlon Study Guido Ge ISBN 978·08918~609 C'20tSASCI'


~
5: 1111 01
unoloQY
:.:.--;Analysis
Protein Analysis
l
protei ~...<;cs composed of lgG immunoglobulin:
JIOlliJUU
t03· ccur during the primary response to antigen
a ore larger molecules than lgM antibodies
b 8 n cross the placenta from mother to fetus
~ ~:n be detected in saline crossmatches
asurement o f serum levels or which of the following lmmunoglobulins can serve as a
104· N1ereening test for multiple allergies?
sc
./
a lgA
b lgE
c lgG
d lgM

10S. C
ells that are precursors or plasma cells and also produce immunoglobulins are: \
'./
3
macrophages
b B lymphocyte s
c T lymphocyte s
d monocytes
• lgM antibodies are frequently hemolytic because of:
106
./
3
their dimeric structure
b the molecule's 5 antigen-binding sites
c their sedimentation coefficient or 7-15 S
d their efficient ability to fix complement

107. To which of the following classes do the antibodies that cause hemolytic disease of the
i/ newborn belong?
a lgA
b tgE
c lgG
d tgD
tGS. 11 is important to note that when an infant is borl\ levels of specific antibody of the
/ following dass are used to indicate neonatal infection:
a lgA
b lgG
c lgM
d lgD
109. The immu noglobulin classes most commonly found on the surface of circulating B
~·~ lymphocytes in the peripheral blood of normal persons are:
a lgM, lgA
b lgM, lgG
c lgM, lgO
d lgM, lgE
ttO. Anlibodies are p roduced by:
./
a killer cells
b marrow stem cells
c mast cells
d B cells

C<111a.&SCP
ISBN 978-069189-6609 Cllnlcal Laboratory Certifica Uon Examlnnllons 329
5 : Immu nology 1
Pro tf!iri 4,
. d b . h . a/Y~/
111 . Antibody class and antibody su~class ":Ire de_termtne Y maior P ysiochemica1 tliff '
0
and antigenic variation found pnmanly in the. •en~
a constant region of heavy chain
b constant region of light chain
c variab le regions of heavy and light chains
d constant regions of heavy and light chains
112. The ratio of kappa to lambda light chain-producing cells In normal individuals is:
.....
""1.V a 1 '. 1
b 2:1
c 3:1
d 4:1
\ j 13. Which of the following imrnunoglobulin classes is associated with a secretory compone
(transport piece)? nt
a lgA
b lgD
c lgE
d lgG
114. The irnrnunoglobulin class typically found to be present in saliva, tears and other
,., secretions is:
a lgG
b lgA
c lgM
d lgD
115. Treatment of lgG with papain results in how many fragm ents from each irnmunoglobulin
molecule?
a 2
b 3
c 4
d 5
116. The irnrnunoglobulin class associated with immediate hypersensitivity or atopic reactions
'.,,...r is:
a lgA
b lgM
c lgD
d lgE
c;1. Which of the following lrnrnunoglobu lins is the most efficient at agglutina tion?
a lgG
b lgA
c lgM
d lgE
118. Polyclonal B cell activation:
.....
""'-' a inhibits antibody production
b requires the participation of T helper cells
c results from the activation of suppressor T cells
d can induce autoanlibody production

:\:'\0 Th A a ... _ ,...,. .. • - - ..... •• - · •


~ ,,...unology .
5· lfll Protc/11 Analysis
.:.:--::;;.IJO(lies
011
10 which of the following imrnuno lob . .
t t!I. >'
anaP
hytactic reactions following blOOd lran fg • uhns is known lo have produced
s usion?
...~1' lgA
8
b 190
c 19E
d lgG
,.he key structural difference that distinguishes imrnunogl b • bcl . h
t20· o u11n su asses is l e:
number of domains
: stereometry of the hypervariable region
tile sequence of the constant regions
~ covalent linkage of the light chains
1. imrnunoglobulin idiotypic diversity is best expla ned by the theory of:
12
"' 3 somatic mutation. .
""' b germ lin? reco.mbinahon
c antigen 1nduct1on
d clonal selection :I

122• Which of the following are true statements about selective lgA deficiency?
~ a associated with.a decreased incidence of allergic manifestations
b high c_oncent'.allon ?f secretory component in the saliva
c associated w 11h an increased incidence of autoimmune diseases
d found in approximately 1 out of every 50 persons

123 Which class of immunoglobulin is thought lo function as an antigenic receptor site on the
"' · surface of immature B lymphocytes?
'"" a lgD
b lgE
c lgA
d lgG
124. The lgM molecule is a:
/ a dimer
b trimer
c tetramer
d pentarner
125. Which of the following immunoglobulins is present in the highest concentration in normal
1
human serum?
a lgM
b lgG
c lgA
d lgE
126. Which of the following statements about immunoglobulins is true?
' a immunoglobulins are produced ·by T lymphocytes
b lgA class is determined by the gamma heavy chain
c lgA class exists as serum and secretory molecules
d there are only 2 subclasses of lgG
~1. Membrane-bound immunoglobulin molecules:
°'' a have an additional amino-terminal sequence of about 40 residues
b are not anchored in a transmembrane configuration
c are anchored by a hydrophobic sequence of about 26 residues
d are anchored by a hydrophilic region

Cllnical Labof1110ry Ccttificalion Ex•minalions 331


Cli>llAS<;p ISSN 978-069189<;609 ......
5: lrnmunology Pro~
128, The area or lhe immunoglobulin molecule referred lo as the hinge region is loc<1t naiy~lt
~~v between which domains? ed
a VH and VL
b CH1 and CH2
c CH2 and CH3
d CH3 and VL
129. Antibody idiotype is dictated by the:
a constant region of heavy chain
b constant region of light chain
c variable regions of heavy and light chains
d constant regions of heavy and lighl chains
130. Antibody allotype is determined by the:
a constant region of heavy chain
b constant region of light chain
c variable regions of heavy and light chains
d constant regions of heavy and light chains
131. Which lgG subclass is most efficient at crossing the placenta?
a tgG1
b lgG2
c lgG3
d lgG4
132. The J-chain is associated with which of the following immunoglobulins?

' a lgA
b lgG
c lgE
d lgD
133. The assembly or the complement "membrane attack unit" is initiated with the binding or:
, .....
00t.r a C1
b C3
c C4
d cs
....<Rr
134. Macrophages are characterized by:
a surface receptors for C3b complement
b surface CD3 e xpression
c in vitro synthesis of immunoglobulin
d large amounts of rough endoplasmic reticulum
.135. Macrophage phagocY1osis of bacteria is enhanced by which of the following :
' ./
a opsonin
b antigen
c hapten
d secretory piece
136. Which of the following is most likely to activate the alternative pathway of complemenl
" activation?
a tipopotysaccharides
b glycoproteins
c haptens
d lgG complexed with antigen

('2018ASC!'
332 The Bou rd o f Cortmcatlon Study Gulde Go ISBN 978-089189~09
r 111
1ur1v•-D,,
Protein An01/ysls
r 5: 111 • 1 of the following is the larger residual 5 lit
~,1111c ' P Portion or C3?
,,1. c3a
3 c;3b
b C4
~Clq 11 . t' 't' .
wnich of the fO owing ac IVI 1es IS associated With C b?
3
138. opsonlzation
/ a anaphylax1s .
cb va soconstnct1
.
on
d chernotax1s
a penicillin injection. a patient rapidly devel . .
g, A.tter This reaction is primarily mediated by· ops respiratory distress, vomiting and
13 h1"6 5· '
a 19G
b lgA
c 1gM
d lgE
t4ll· Which of the following is the ·recognilion unit" in the classical complement pathway?
V 8 C1 q
b C3a
c C4
d cs
immunofixation electrophoresis:
t41. In
8
the antibod~ reacts with the antigen and then the complex is electrophoresed
b the ant!gen ~s electrophoresed into an antibody containing gel
c the antigen is electrophoresed and then monospecific anlisera is reacted with it
d the antigen is electr?phoresed, transferred to nitrocellulose and then antibody reacts
with II and an EIA is performed

112. Which or the following is the "membrane attack :ornplex• of complement activation?
a C1
b C3
c C4, C2, C3
d C5b. C6. C7. CS. C9
Ill. Which of the following releases histamine and other mediators from basophils?
.
J a C3a
b properdin factor B
c Clq
d C4
144. Which of the following is not a mechanism Involved in the complement cascade?
a apoptosis
b opsonlzation
c inflammation
d cytolysis
145, The serum hemolytic complement level (CHso):
1U
"" a is a measure of total complement activity
b provides the same information as a serum factor B level .
c is detectable when any component of the classical system is congenitally absent
d can be calculated from the serum concentrations of the individual components

C,n/cal ubof1110ty Ctrtilicalion Examinations 333


5 : lmrn unology P1 0 11J;,,
t I . I I' . th
146. A single, reliable screening test for delecling neona a m ec ion m e absence
1\ 11; 1
' ~ 111
01
<:lr~ic:;,
signs is:
1
a serum immunoelectrophoresis
b differential leukocyte count
c CD4 cell counts
d quantitative serum lgM determinalion
147. Bence Jones proteins are:
, a immunoglobufin catabofic fragments in the urine
b monoclonal light chains
c whole immunoglobulins in the urine
d Fab fragments of a monoclonal protein
148. Which 2 immunoglobulin classes have a •J" chain associated w ith the structure?
a lgG, lgA
b lgA, lgE
c lgG, lgM
d lgM, lgA
,149. Humoral antibodies are produced by which cells?
,/

a m acrophages
b T lymphocytes
c B lymphocytes
d neutrophlls
150. Initiation of the activation m echanism of the alternative compl ement pathway differs 1
:/ that of the classical pathway in that: rom
a antigen-antibody complexes containirg lgM or lgG are required
b endotoxin alone cannot initiate activation
c C1 component of complement is involved
d antigen-antibody complexes containing lgA or lgE may initiate activation
1§1. Wh ich o f the following is cleaved as a result o f activation of th e classical complement
pathway?
a properdin factor B
b Clq
c C4
d C3b
152. The component associated only with the alternative pathway of complement activation is:
a C4
b Clq
c properdin factor B
d C3a
153. Which of the following complement components is a strong chemotactic factor as well as a
~v strong anaphylatoxin?
-' a C3a
b C3b
c csa
d C4a
154. The C3b component of complement:
~,., a
- · is undetectable in pathological sera
b is a component of the C3 cleaving enzym e of the classical pathway
c is cleaved by C3 inactivator into C3c and C3d
d is not part of the alternative pathway

334 Tho Board of Cortificallon Study G uido 6e ISBN 978--0891~ caoisA..<Cl'


u no l oQY
s: 1111rt1 nents of the complement system most lik
con•P0 ely to coat a cell are:
P rotein /1110 /ysis

tSS· CI and C2
"" ~ C3 and C4
.!' ll cs and C7
c cs and C9
d rological test that can be mOdified to sele .
111e seted serum is: cbvely detect only specific lgM antibody in
tS6· ~ntrea
~" chterlony
"' a ou e immunoassay
tl enz~glutinalion inhibition
c hem 've hemagglutlnation
d pas 5i . .
. l's serum 1s being analyzed in a sandwich . .
. A pa~~nal antibody therapy, and shows a false- as.st.ay. This Pat1~nt has received mouse
t51 nion"".' d e to: posi ive reaction in the sandwich assay.
,Jt _.r,icll 15 U
"" ' he mouse antibody in the patient's serum reacting 10 th .
a t presence of human anti-mouse antibody activil e antigen
b th~ibOdY to a mouse virus Y
c an d lion of a monoclonal gammopathy of k . ..
cl pro uc un nown s1gmf1cance after the antibody
treatment
- -•anal spike of lgG. Bence Jones prote· · .
A monuv iruna · and bone pain are usually associated
1SS· \•/I'th'•
\f,.S
,,.., eur1<itt 1ymphoma
a eruton disease . . .
b severe combined 1mmunodefic1ency disease
~ multiple myeioma
fhe hyperviscosity syndro~e is most likely to be seen in monoclonal disease of which of
~9- the following 1mmunoglobulin classes?
,..,
a lgA
b igM
c lgG
d lgD
patients suffering from Walden~trCim macroglobulinemia demonstrate excessively
·~,60· increased concentrations of which of the following?
t<'
a lgG
b lgA
c lgM
d lgD
t6t. Which of the following is the mos t common humoral immune deficiency disease?
;:t, a Bruton agammaglobulinemia
b lgG deficiency
c selective lgA deficiency
d Wiskott-Aldrich synd rome
162. Which of the following is a true statemenl about Bruton agammaglobulinemia?
"'"" a it is found only in females
b there are normal numbers of circulating B eels
c there are decreased to absent concentrations of immunoglobuiins
d the disease presents with pyogenic infections 1 week after birth

c /'nlcal Laboratory C~rtification Examin•lions 335


:>: l n11nunol ogy
• Pro1
163. lrnmunodeficiency with thromboc yloperna and eczema Is orton rolerr 'd QI,,'\
t'l.S
0
a DiGeorge syndrome
"'•
b Bruton agammaglobulinemia
O l() '1R·
·· r"
'1oll

c ataxia telangiectasia
d Wiskott-Aldrich syndrome
164. In hybridoma technology, the desirable fused cell is lhe:
.....
ONLY a rnyeloma-myeloma hybrid
b myeloma-lymphocyte hybrid
c lymphocyte-lymphocyte hybrid
d lyrnph ocyle-granulocyle hybrid
165. Potent chemotac tic activity is associated with which of th e following corn
v complement system: Ponents 01the
a C1q
b C5a
c C3b
d lgG
166. Hereditary angioedema is characterized by:
a decreased activity of C3
b decreased activity of C1 esterase inhibitor
c increased activity or C 1 esterase inhibitor
d increased activity of C2
167. Which of the following has been associated with patients who have hornozygo
deficiency? us C3
a undetectable hemolytic complement activity in the serum
b s ystemic lupus erythematosus
\ c no detectable disease
d a lifelong history of life-threatening infections
168. Hereditary deficiency of early complement components (C1, C4 and C2) is associateo
with :
a pneumococcal septicemia
b small bowel obstruction
c lupus erythematosus like syndrom e
d gonococcemia
169. Hereditary deficiency of late complement components (C5, C6, C7 or CS) can be
associated with which of the following condi tions?
a pneumococcal septicemia
b small bowel obstruction
c systemic lupus erythematosus
d a systemic gonococcal infection if exposed
170. Combined immunodeficiency disease with loss of muscle coordination is referred to as:
MUI
°"" a DiGeorge syndrome
b B ruton agammaglobullnemia
c a taxia telangiectasia
d Wiskott-Aldrich syndrome
171. In skin tests, a wheal and flare development is indicative of:
°""
Ml.S
a immediate hypersensitivity
b delayed hypersensitivity
c anergy
d Arthus reaction
.........
336 The Board of CertlficaUon Study Gulde RP
,,..-: • 1111niunoloQY Cellular lmm1m11 &
5. h irrununologic mechanism is usu . Y Hlstoco111patlbility Toclrnlquos
WJ11c a11Y 1nvo1v~d in b
t1 2• .,,,niediale hypersensilivity ronchlai asthma?
~ ~nlibOdY mediated cytotoxicity
·,,,rnune complex
~ delayed hypersensilivily
tlhistamines like Benadryl®:
173· An
depress lgE production
~ block ~nlig~n binding to surface lgE
c bind h1stamrne
d block H1 histamine receptors

Cellular Immunity & Histocompatibility Techniques r


11•·
, Which test is used to evaluate the cellular immune s 1 .
a skin te~t for commonly encounlered antigens
.
ys em 1n a pahenl? I
b delermrna!lon of 1sohemagglutinin titer
c ifllmunoelectrophoresis of serum
d measurement of anti-HBsAg afler immunization

175. T cells are incapable of:


, a collaborating with B cells in antibody responses
b secretion of immunoglobulins
c secretion of cytokines
d producing posilive skin lesls
l76. T lymphocytes are incapable o f functioning as:
'
a cytoloxic cells
b helper cells
c phagocytic cells
d regulatory oells
1n. Nonspecific killing of tumor cells is carried out by:
"' a cytotoxic T cells
b helper T cells
c natural killer cells
d antibody and complement
'. 178. When a natural killer cell comes in contact with a cell expressing MHC class I proteins
what action will result?
I
a perforins released
b granzymes released
c inhibition of kil ling
d cytotoxic mechanisms
[.)19. ' A patient with a B cell deficiency will most likely exhibit:
a decreased phagocytosis
b Increased bacterial infections
c decreased complement levels
d increased complement levels

~1!1o.SCP Clinical Laboratory Certification Examinations 337


ISBN978-069189-6609
-'· 11n111uno lo g y Co/lular ln111111f1ity & His toc
. ompalibllit
180· MHC class 111 genes code for?
a antigens
b antibodies
c lymphocytes
d complement
181 . Which of the following is an important cellular mediator o f immune com •
1
a mast cell P ex hssue "'io0
b neutrophil ll
c basophil
d eosinophil
~2. Which o f the following mediators is released during T cell activation?
a immun oglobulins
b thymosin
c serotonin
d cyt okines
183. MHC Class I includes which peptides/proteins?
a complement
b HLAA, B , C
c cytokines
d HLA DP, DQ, DR

....184.
ONlY
Incompatibility by which o f the following procedure s is an absolute contraindicatio
allotransp lantatlon? n lo
a MLC (mixed lymphocyte culture)
b HLAtyping
c Rh typing
d ABO grouping
185. W hich is a recognized theory of the origin o f autoi mmunity?
a enhanced regulatory T cell fun ction
b diminished helper T cell activity
c production o f antibodies that cross-react with tissue components
d deficient B cell activation
186.' C 3b and Fe receptors are present on:
u4,sV
010.V a B lymphocytes
b monocytes
c B lympho cytes and monocytes
d n eith er B lymphocytes and monocytes
T lymphocytes th at possess the CDS surface marker mediate which or the following Teel
functions?
a d elayed type hypersensitivity
b regulatory
c cytoto xic
d helper
188. Delayed hypersensitivity m ay b e induced by:
a contact sensitivity to inorganic chemicals
b transfusion reaction
c anaphylactic reaction
d bacterial septicemia

338 The Bo;srd of Certlftc;stlon Study Guido Ge


~ogy
. 1111 • • •
Cc/111/ar 11111111111//y & Hls toco111patib/lily Toc/mic/tlOS
• .
5• 111051 rapid 1mmcd1ate hyperscns1Uv1ty reaction is associated with·
. r 11e ·
169
/ transfuslo~
a naphylax1s
b ~ntact dermatitis
~ serum sickness
he normal controls for a quantitative B tymphocite assay should have a value o f what
190. rpercentage o f total lymphocytes counted?
"'"''' 3 21%
b 48%
c 76%
d 89%
An immunofluoresc_ence test using reag~nt antibody directed again st the CD3 surface
191. marker would 1denhfy which of the foltow1ng cell rypes in a sample of human peripheral
brood?
all mature T lymphocytes
3
-r
b helper lymphocytes only
c cytotoxic T lymphocytes only
d T regulatory cells only
Refer to the following flow cytometric data.
192.
absolute wee: s .930
"'
( fll'f
total lymphocytes: 30%
a lymphocytes: 40%
T 1ymPhOcy1es: 58%

calculate the absolute count for B lymphocytes.


a 1,072
b 2,679
c 3,572
d 6,251
193. In flow cytometry, labeled cells:
a scatter the light and absorb fluorescence
b absorb fluorescence and emit electronic impulses
c scatter the light and emit fluorescence
d absorb both fluorescence and light
194. A marked decrease in the CD4 lymphocytes anc decrease in the C04/CD8 ratio :
'\...; a is diagnostic for bacterial septicemia
b may be seen in most hereditary immunodeficiency disorders
c is associated with a viral induced immunodeficiency
d Is only seen in patients w ith advanced disseminated cancer
t9S. Refer to the following data from a peripheral blood sample:
IU
a,, total WBC: 10.0 • 103/µL
differential
newophils. 68%
lymphocytes: 25% (40% T cell s)
monocytes: 4%
eosinophlls: 2%
basopltils: 1%

The expected total number of T cells is:


a 200
b 1,000
c 2,000
d 2.500

CIQieASCp ISai.t 91Ua9189-6609 Ct:nlcal Laboratory Certification Examinations 339


5 : hnn1unology Cellular /111nw11/ly & /-lis tocompalibility ~
~~ · 6
A periphera l blood total leukocyte count Is 10.0 " 103/µL ( 10.0 " 109/l). Tho d·rr"chniq 11o4
o.,.. re~eols 55"/o neutrophils. 2•1. eosinophils, 40°10 lymphocytes and 3% monocyt~ erenlia1
:3 d~n~phocyte recovery of 85-95%, what 1s the expected number of T colls in a s. Assuni;.,
111 1v1dual? norlllal · ·~
a 750/µL
b 2,500/µL
c 4,000/µL
d 8,000/µL
197.
..... In la~e r now cytometry, applying a voltage potential to sample droplets as they str
ONLY th e lt ght beam and using charged deflector plates results in: earrt Past
a an emission of red fluorescence from cells labeled with fluorescein isothiocyanat
b an emission of green fluorescence from cells labeled with rhodamine e
c a 90° light scatter related to cell size
d the separation of cells into subpopulations based o n their charge
198. Wh at is the immunologic method utilized in the flow cytometer?
a la tex agglutinatio n
b e nzyme lin ked immunoa ssay
c immunofluorescence
d radioimmunoassay
199. Given the following data:
wee: 5.0 • 103/µL (5.0 x 109/L)
lymp hs: 15%
C04: 8%
Calculate the absolute CD4:
a 40
b 60
c 400
d 750
200. Given this hematologic data:
wee: 5.0 • 103/µL (5.0 • 109/L)
lymphs: 15%
C04: 8%
Whic h of the following is the correct interpretation?
a CD4 % and absolute CD4 count a re normal
b consistent with an intact Immune system
c consistent with a vira l infection such as HIV
d technica l error
201 . Bone marrow transplant donors and their recipients must be matched for which antigen
1.U
OM.Y
system(s)?
a ABO-Rh
b HLA
c C04/COB
d p1a1
202. A 28-year-old man is see n by a physician because of severa l month s of Intermittent lo~
.....,.
b ack pain. The patient's symptoms are suggestive of ankylosing spondylilis. Which oft e
ONLY
following laboratory studies would su pport this diagnosis?
a a decreased synovial fluid CHso level
b low serum CH 50 level
c positive HLA-B27 antigen test. .
d rheumatoid factor in the synov1al fluid

340 The B o a rd of Certification S tudy Gu l de 6e ISBN 978-089 189~ ::.....JI


0201sAScP
5•. ln11 ""~· - .,,
11 Ccl/u/ar Im
cells from a pa I°1ent with
· hairy cell leuk . munfty & HI t
s ocomp atiblllty Tcclmiquos
ioJ. em1a have im .
"'
3
mast cells and B lymphocytes munolog1c and functional features of:
••'" b a lymphocytes and T lymphocytes
c granulocytes and monocytes
d a
lymphocytes and monocytes
Which T cell malig nancy may retain "hel er" . .
4
~ . synthesis by B cells? P activity With regard to immunoglobulin
""'
3
Hodgkin lymphoma
b acute tymphocyt1c leukemia (ALL)
c sezary syndrome
d chronic lymphocytic leukemia (CLL)

5 A patient's abnormal lymphocytes are posT 1


20 · negative for surface immunoglobulin. This'~~~ bor CD2 antigen. lack C3 receptors. and are
e classified as a disorder or:
3 T cells
b B cells ,•
c rnonocytes II
d natural killer cells

206. HLA typing of a family yields the following results:


.,. locus A locus B
(1>1T
father (8.12) (17,22)
molher (7,12) (13,27)

On the basis of th~se ~enotypes, predict the possibility of ankylosing spondylilis in this
percentage of their c hildren.
a 25% of their children
b 50% of their children
c 75% or their children
d 100% of their children
207. HLA-88 antigen has been associated with which of the following pairs of diseases?
~T a ankylosing spondylitis and myasthenia gravi.s
b celiac disease and ankylosing spondylitis
c myasthenia gravis and celiac disease
d Reiler disease and multiple sclerosis
208. Which or the following Is an important marker for the presence of immature B cells in
"' patients with acute lymphocytic leukemia (ALL)?
Oil
a terminal deoxynucleotidyl transferase (TdT)
b adenosine deaminase
c G6PD
d purine nucleoside phosphorylase

C tfj(i~tlon Ex•minanons 341


Clinical Laboratory e
5: lrnrnuno logy Cc//u/nr /11111111111ty & l1istoco111palibllit~
1. b 45. b 89. a 133. d 177. cc ''''"''o
~
2. c 46. a 90. c 134. a 178. c
3. c 47. d 91 . b 135. a 179. b
4. b 48. b 92. a 136. c 180. d
5. a 49. a 93. c 137. b 181. b
6. b 50. b 94. c 138. a 182. d
7. a 51. c 95. b 139. d 183. b
8. a 52. d 96. b 140. a 184. d
9. c 53. d 97. c 141. c 185. c
10. c 54. d 98. c 142. d 186. c
11. c 55. c 99. b 143. a 187. c
12. d 56. c 100. a 144. a 188. a
13. b 57. d 101 . a 145. a 189. b
14. a 58. b 102. b 146. d 190. a
15. a 59. d 103. c 147. b 191. a
16. a 60. b 104. b 148. d 192. a
17. c 61. a 105. b 149. c 193. c
18. c 62 . b 106. d 150. d 194. c
19. d 63. a 107. c 151. c 195. b
20. d 64. b 108. c 152. c 196. b
21 . c 65. c 109. c 153. c 197. d
22. b 66. c 110. d 154. c 198. c
23. a 67. b 111. a 155. b 199. b
24. d 68. d 112. b 156. b 200. c
25. a 69. c 113. a 157. b 201 . b
26. c 70. a 114. b 158. d 202. c
27. d 71. d 115. b 159. b 203. d
28. d 72. a 116. d 160. c 204. c
29. d 73. a 117. c 161. c 205. a
30. b 74. c 118. d 162. c 206. b
31. c 75. a 119. a 163. d 207. c
32. c 76. b 120. c 164. b 208. a
33. a 77. d 121 . b 165. b
34. c 78. b 122. c 166. b
35. c 79. b 123. a 167. d
36. d 80. a 124. d 168. c
37. d 81. d 125. b 169. d
38. c 82. a 126. c 170. c
39. a 83. b 127. c 171. a
40. c 84. d 128. b 172. a
41 . d 85. a 129. c 173. d
42. b 86. b 130. d 174. a
43. b 87. a 131. a 175. b
44. c 88. c 132. a 176. c

342 The Board or Certlncalion Study Guide 6e


ISBN 978-089189-6609 C20l SASCI'
Preanalytical & Susceptlblllty Testing
robiolo9Y
1

1ifcrobiology
. 1.1 ms have been identified generol/y as appropriate for both entry level medical
me 1o11owin9. ~tists and medical laboratory technicians. Items that are appropriate for medical
i·boratol'Y sc'.e ntists only are marked with an 'MLS ONLY.·
• •orato(J' sc1e
~
422 Answers with Explanations
Questions . . . 7i .
3S5 lytical & suscept1b1f1ty estmg 423 Preanalytical & Susceptibility Testing
355 Preana .. c · 429 Aerobic Gram-Positive Cocci
• ·r Gram-Po51t1ve occi
358 AelOv'• .. 432 Gram-Negative Bacilli
Gram-Negative Bac1/h .
375 erobic Gram-Negatl•1e Cocci . . .. 437 Aerobic Gram·Negative Cocci
389 A b·c or Fncullative Gram-Pos1trve Bac1/h 438 Aerobic or Facuttative Gram-Positive Bacilli
392 Aero 1 '
439 Anaerobes
394 Anaerobes 440 Fungi
398 Fungi .
443 Myccbacteria
405 MycoMcterm . .
445 Viruses & Other Microorganisms
409 Viruses & Otiler M1croorganrsms
413 parasites 441 Parasites

Preana/ytical & Susceptibility Testing


1. Which of the 2 different antimicrobial agents listeo below are commonly used and may
w1 rcsull in synergistic action in the treatment of endocarditis caused by E'nterococcus
"'' taecalis?
a an aminoglycoside and a macrolide
b a penicillin derivative and an aminoglycoside
c a cell membrane active agent and natidixic acid
d a macrolide and a penicillin derivative
2. A bronchoscopy sample with the request for culture of Legionelfa is sent to the laboratory.:
The correct plating protocol is:
a culture on thiosulfate citrate bile salt media
b incubate the culture media anaerobically
c reject the specimen and reqU"est a sputum sample
d culture on buffered charcoal yeast extract agar with antibiotics
3. A community hospital microbiology laboratory is processing significant numbers o f stool
cultures because of an outbreak of diarrhea following heavy rains and flooding in the
county. A media that should be incorporated in the plating protocol is:
a colistin nalidixic acid for Listeria
~ Macc_onkey agar with sorbitot for Campylobacter
d m~nnotol salt agar for Enterococcus species .l
thiosullate citrate bile salts sucrose for Vibrio species
4.
A male urethral discharge specimen submitted for culture should be inoculated to:
~ !heep blood and phenylethyl alcohol agars
c thosm -111t; lhylene blue and sheep blood agars
d c~oglycoll ate broth and chocolate agar
ocol.ite and modified Thayer-Martin agars

•'3918!}.6609 Cl/nlca/ Labol'Blory Cettlf/callon Exam lnalions 355


6 : M icrobiology Preanalytfcal & Susce~
5. The lowest concentration of antibiotic that inhibits growth of a test organism is the: ~
a minimum bactericidal concentration
b minimum inhibitory concentration
c serum bactericidal concentration
d serum inhibitory concentration
6. The steam autoclave method of sterilization req uires:
a 15 lb of pressure for 15 minutes
b dry hea t for 20 minutes
c a maximum temperature of 1oo·c
d a source of ethylene oxide
7. The proper blood-to-broth ratio fo r blood cultures to reduce the antibacterial effect of
~v serum in adults is:
a 1:2
b 1:3
c 1:10
d 1:30
8. Penicillin resistance In Neisseria gonorrhoeae ca n be d ue to the organism producing:
a beta-D-galactosidase
b beta-lactamase
c butyrate esterase
d DNase
9. Which selective medium is used for the isolation of Gram-positive microorganisms?

I a
b
c
Columbia CNA with 5o/o sheep blood
trypticase soy agar with 5% sheep blood
eosin methylene blue
d mod ified Thayer-Martin
10. The most sensitive substrate for the deteclion of beta-lactamases is:
a penicillin
b ampic illin
c cefoxitin
d n itrocefin

....11.
<""-Y
A Staphylococcus aureus isolate has an MIC of 4 µg/ml to oxacillin. There is uncertainty
as to whethe r this represents a n o xacillin (he te roresistant) resistant strain or a
hyperp roducer of beta lactam ase.
Strain ox..clllln Amox lclllln-el avulanlc acid
strain A susceptible susceptible
strain B susceptible resistanl
strain C resistant susceptiblo
strain O reslsl enl reslslanl

Based on the above results for oxacillin and amoxicillin-clavulanic acid, which strain is
heteroreslstant?
a stra in A
b strain 8
c s tra in C
d strain D
12. An Enterococcus iso lated from multiple b lood c ultures in a patient endocardilis should be:
a screened for high level aminoglycoside resistance
b c hecked for tolerance

__.
c assayed for serum antimicrobial actlvity
d tested for beta-lactamase production
ISBN 978-08918~ C1JOISJ.SO'
3 56 The Board o f Certification St udy Gulde 6e
~Microbiology Preana/ytical & Susceptibility Testing
~ procedure th~t assures the ~os_t accurate detection of mecA-mediated oxacillin
t3. ::s~stance in routine broth m1crod1luhon susce ptibility testing against s aureus is:
addition of 4% NaCl
~ incubation at 30"C
c incubatio!" for 48 ~_ours
d testing with cefox1tin
susceptibility testing p~rformed on quality control organisms using a new media lot
14. mber yielded zone sizes that were too large for all antibiotics tested. The testing
~~s repeated using media from a pre~iously used lot number, and all zone sizes were
acceptable . T he unacceptable zone sizes are best explained by the:

3
antibiotic disks were not stored with the proper desiccant
b depth of the media was too thick
c depth of the media was too thin
d antibiotic disk s were not properly applied to the media
campytobacter jejuni isolation requires the fecal specimen be:
15.
3 inoculated onto selective plating media and incubated in reduced oxygen with added
co 2 at 4 2°C
b stored in tryptic soy broth before plating to ensure growth of the organism
c inoculated onto selective plating media and incubated at both 35•c and at room
temperature
d incubated at 35°C for 2 hours in Cary-Blair media before inoculating onto selective
plating media
16. An expectorated sputum is sent to the laboratory for culture from a patient with respiratory
distress. The direct specimen Gram stain shows many squamous epithelial cells (>25/lpf)
and rare neutrophils. The microscopic appearance of the organisms present include:
moderate Gram-positi ve cocci in chains and diplococcl
moderate Gram·negal.ive diplococci
modoroto palisading Gram· positive bacilli all In moderate amounts

This G ram stain is most indicative of:


a a pneumococcat pneumonia
b an anaerobic infection
c a Haemophilus pneumonia
d oropharyngeat Hora
17. In disk diffusion susceptibility testing, as an antimicrobial agent diffuses away from the
disk, the concentration of antibiotic is:
a increased
b decreased
c unchanged
d inoculum dependent
18. The most appropriate method for collecting a urine specimen from a patient with an
~v indwelling catheter Is:
a remove the catheter, cut the tip , and submit it for culture .
b disconnect the catheter from the bag, and aseptically collect urine from the terminal end
of the catheter
c aseptically collect urine directly from the drainage bag
d aspirate urine aseptically from the catheter tubing
19. Which of the following specimen requests Is acceptable?
a feces submitted for anaerobic cu lture
b Foley catheter tip submitted for aerobic culture
c rectal swab submitted for direct smear for gonococci
d urine for culture of acid-fast bacilli
C//nlcal Laboratory CertificaUon &amlnallon• 357
6 : Microbiology Preanalytical & Susceptlbl/it .,.
Y •es11n
20. Which of the followin g groups of specimens would be acceptable for anaerobic CUit ~
Ure?
a vaginal swab. eye swab
b intraoral surface swab, leg tissue
c pleural fluid, bra in abscess fluid
d urine, sputum
21 . A liquid fecal specimen from a 3 -month-old infant Is submitted for culture. T he stool
should detect Salmonella, Shigella and: CUiture
a Campylobacter sp.
b Closlridium botulin um
c Entamoeba hartmanni
d enterotoxigenic Escheric/1ia coli
22. A cerebrospinal fluid specimen containing only 2 drops o f CSF was collected by a lumba
punctu re from a febrile 25-yea r-old male and was submitted for a stat Gram stain and r
culture. The direct specimen G ram sta in was reported as many neutrophils and no
microorganisms seen . The remaining drop of CSF sho uld be inoculated to:
a blood agar
b CNA agar
c chocolate agar
d Thaye r-Martin agar
23. A diabetic foot swab from an 82-year-old woman with recurrent infections is submitted for
culture. T he Gram s tain reveals:
many neulrophits. no squamous epithelial ceJls
many G ram-negative bacilli
mony Gram-positive cocci in chains

The physician requests th at all pathogens be worked up. In addition to the sheep blood,
chocolate and MacConkey agar plates routinely u sed for wound cultures, the technologist
m ight also process a(n):
a anaerobic blood agar plate
b BCYE agar plate
c CNA agar plate
d XLD agar plate
24. Which of the following is the most appropriate specimen source and primary media
selection?
a CSF Columbia CNA, MacConkey
b endocervical chocolate , Ma rtin Lewis
c sputum sheep blood , Thayer-M artin, KV-laked blood
d urine sheep blood, chocolate, Columbia CNA
25. Which of the following is the most appropriate organism and media combination?
a Vibrio spe cies-Sk irrow
b Enterohemorrhagic E co/i-phenyfethyl alcohol (PEA)
c Campylobacter species-charcoal yeast extract
d Yersinia enlerocolitica-cefsulodin-irgasan-novobiocin (CIN)

• .,,. C2018 AsO'


358 The Board or Cortincallon Study Guido 60 ISBN 978-089189-ovv•
d
~iology Preanalytical & Susceptibility Testing
_.A Gram stain from a swab of a hand wound reveals:
26• moderate neulrophils
"° squamous ep1lhelial cells
moderale Gram-posilive eoecl In cluslers
moderate large Gram-negallve bacilli

Select the appropriate media that will selectively isolate each organism.
a KV-laked agar. Thayer-Martin
b sheep blood. MacConkey
c Columbia CNA , chocolate
d Columbia CNA, MacConkey
l)pon review <?fa sputum Gram stain, the technician notes that all the neutrophil nuclei in
27· the smea r stained dark blue. The best explanation for this finding is the:
a iodine was omitted from the staining procedure
b slide was inadequately decolorized with acetone/alcohol
c sputu m smear was prepared too thin
d cellular components have stained as e~pected

28 . Sodium ,:>0lyanetholsulfonate (SPS) is used as an anticoagulant for blood cultures


..., because 11:
.....
a inactivates penicillin and cephalospori ns
b prevents clumping of red cells
c inactivates neutrophils and components of serum complement
d facilitates growth of anaerobes
29, An antimicrobial suscep~ibility method recommended lo detect vancomycin-inlermediate
Staphylococcus aureus 1s:
a M IC b roth dilution
b agar dilution
c kinetic diffusion
d d isk diffusion
30. When performing a disk diffus ion susceptibility test , the antibiotic disks are placed on the
agar 30 minutes after organism inoculation and then incubated w ithin 15 minutes of the
disk placement This procedure will result in:
a the antibiotic not diffusing into the medium, resulting in no zo ne
b decreased zo ne diameters
c increased zone diameters
d no effect on the final zone diameter
31. An Enterobacteriaceae organism will appear to be more resistant on a disk diffusion
susceptibility test if the
a depth of the agar is too thin
b inoculum is too concentrated
c the antimicrobial agent in disk is too concentrat~d . . _
d test interpretation occurs after 12 hours o f ambient air incubation
32. When performing antimic robial susceptibllily testing on Enterobacteriacaae, flrst-
generatlon cephalosporin s can be adequately represented by:
a cefuroxime
b ceftriaxone
c cefazolin
d ccfonicid

Clln/cal Laboratory Citrtlflc•tion Eltomlnatlons 359


°'°' :• ISBN 97&-089189-6609
~::~M~i~c~r~o~b~io~~lo~g~y~-::::-:-;:;::-:::;;-::;;tt;;;;;;~~-;;~~~~~~~~~~~~...::....:.:1un0
Preanalytlcal & Susceptlbll/ty Te
33. A n antibiotic that inhibits cell wall synthesis is: ~
a chloramphenicol
b colistin
c penicillin
d sulfamethox azole . .. .
34. . i does not require suscepl1b1hty testing to the
Which one_ of.the following organ, stedmsfrom a clinically significant source?
antimicrobial indicated when iso a
a Staphylococcus aureus c!ir:idamycin
b Proteus mirabilis gentam1c1n ...
c Streptococcus pyogenes p_enic1lhn
d Escherichia coll levofloxac1n
35. Antibiotics routi neIy t es t e d and reported for Pseudomonas aeruglnosa Isolates include·.
a cllndamycin
b erythromycin
c gentamicin
d penicillin
36. An organism that must be incubated in a microaerophilic environment for optimal recovery
is:
a Campylobacter jejuni
b ~scherichia coli
c Pseudomonas aeruginosa
d Proteus mirabilis
37. Vibrio parahaemolyticus is best isola ted from feces on:
a eosin methylene blue (EMS) agar
b Hektoen enteric (HE) agar
c Salmonella Shigella (SS) agar
d thiosulfate citrate b ile salts (TCBS) agar
38. Tests for Haemophilus influenzae beta-lactamase production:
a are not commercially available
b include tests that measure a change to an alkaline pH
c should be performed on all blood and CSF isolates
d are not valid for any other bacterial species

39. Media used to support growth of Legionel/a pneumophila should contain the additives:
a X and V factors
b hemin and vitamin K
c charcoal and yeast extract
d dextrose and laked blood
40.
The best medium for culture of Borde/el/a pertussis is:
a phenylethyl alcohol aga r
b potassium tellurite blood agar
c Regan-Lowe agar
d linsdale agar
41 .
A medium that can be used to recovor Francisel/a tularensis is:
a Bordet-Gongou aoar
b Buffered chorconl yo nst ex tract agar
c Loomer aga1
d MacConkey agor

360 The B oard o f c .,rtincstron Study Gulde e.,


ISBN 978-0891 8~ 02QlSASCI'
6: Microb io logy Preanalytica/ & SuscepUblllty 11'9flfqj
,2. The optJmal collection or a wound specirrien for culture or anaerobic organisms is a:
a swab of lesion obtained before administration or antibiotics
b swab of lesion ~bta1ned after administration of antibiotics
c syringe filled ~th pus. obta~ned before administration of antibiotics
d syringe filled with pus, obtained after administration o f antibiotics
43 A 21-year-old patient presents with pharyngitis. A throat swab is collected and submitted
• ror anaerobic culture. This specimen should be:
3 set up immediately
b rejected as unacceptable
c inoculated into thioglycollate broth
d sent to a reference laboratory
44• An antibiotic used to suppress or kill contaminating fungi in media is:
a amphotericin B penicillin
b chloramphenicol
c cycloheximide
d streptomycin
4s. A sputum specimen is received for culture and Gram stain. The Gram-stained smear from
._. this specimen is seen In the image (Iota magnification 100x):

..-
"'' ~ •
-~
. '
, • • •••

• •
- .
,
~,.,

; r ·~
- ··
•.

. .;,
•... .. ,!a
~
.
• t ·
... ..-.
'
. . ·1' ~
.-\ . .. ' ·•
.. . , .
. , -.., ' ' .. .... . ., .
.. -..
p- •
• ..• .

# -•. . -· " .....


.' '
The technologist's best course of actior would be to:
a Inoculate appropriate media and incubate anaer?bically
b inoculate appropriate media and incubate aerob1ca11r • . .
c call the physician and notify him of this "life-threatening situation
d call the patient care area and request a new specimen
46. Refer to the following illustration:
...""', Control
64 32 16 8 & 2 g:mL

fam.cin 000000 •
pret the gentamIc1.n MIC shown in 1hls broth microdllution susceptibility 1est.
64 µg/mL
2 µg/ml
µglmL
µglmL

>C ISBN 978-089199-$509


h
47. Who11 using a control strain or Stopllylococcus ourous, the technologist no~
the zone around the ce foxltin disk is too small. Which of the following Is the most ,at
explanation? lkeiy
a inoculation of the plates 10 minutes after preparing the inoculum
b incubation of the M ueller-Hinton plates at 35°C
c use or a 0.25 McFarland standard to prepare inoculum
d use or outdated ceroxitin disks
48. In the disk diffusion method or determining antibiotic susceptibility, the size of the inh"-' .
•~s
CNlV
zone used to indicate susceptibility has been determined by: •utt1on
a testing 30 strains or 1 genu.s of ?~cteria . . . .
b correlating the zone size with m1n1mum 1nh1b1tory concentrations
c correlating the zone size with minimum bactericidal concentrations
d correlating the zone size with the antibiotic content of the disk
49. An organism that may be mistaken for Neisseria gonorrhoeae in Gram-stained smears Of
uterine cervix exudates is:
a Lactobacillus species
b Streptococcus aga/acliae
c Psa udomo11as aerugi11osa
d Moraxella ostoensis
50. An antimicrobial that is inappropriate to report on an Escherichia coli isolated from a
w ound culture is:
a ampicillin
b cefazolin
c gen tamicin
d nitrofurantoin
51. The susceptibility results below are reported on an Enterococcus foecalls isolated from
peritoneal fluid.
ampicillin: susceptible
vancomycin: resistant
clindamycin: susceptible
levonoxacin: resistant
linezolid: suscep1iblo

The physician calls questioning the results. Which of the following should have been done
before the report was released?
a the clindamycin result should have been removed from the report since it Is inactive
against Enterococcus
b the ampicilli!"' res':'lt should have been changed to resistant since the isolate is
vancomyc1n resistant
c the linezolid result should have been removed from the report since it Is inactive againsl
Enterococcus
d ciprofloxacin should have been added to the report since levofloxacin was resistant

362 The Bou'd of Certification Studv Gulde k


. f.11Cr
. obiOI09Y . Preanalytlcal & Susceptibility Tes ting
~
,, AD
test is perfo.rmed o.n an isolate of Siaphytococcus aureus to determine inducible
ycin resistance.
,.. cJlndam
,..••

Based on tho result seen in the image how should the erythromycin and clindamycin be
reported?
a erythromyc1n: resistant; ciindamycin: resistant
b erythromyc!n: resistant; ciind.amycin: _susceptiole
c eiythromycin: susceptible: chndamycm: resistant
d erythromycm: susceptible; clindamycin: susceptible
Sl An antimicrobial combination that is useful for confirming the presence of extended
spe<:trum beta-lactamases in E coll is:
a ampicillin + cefepime
b cefoxitin + penicillin
c ceftazidime + clavulanic acid
d cefpodoxime + cefotaxime

54. Enzymatic drug modification is a mechanism of resistance for which antimicrobial?


"'
"'-'a levofloxacin
b sulfamethoxazole
c vancomycin
d gentamlcin

SS. The most imporuint variable in the recovery of organisms in adult patients with bacteremla
{bacterial sepsis) 1s:
a subculture of all bottles at day 5 of incubation
b the recommended volume of blood cultured
c collection of daily blood culture sets for 3 consecutive days
d collection of multiple blood culture sets from a single venipuncture
6: Microbiology Preonalytical & Suscepl/b/llty 7: .~
. es1m9
56. A 24-year--Old man presents with pain on urination and urethral discharge. A Grarn sta·-
... the d ischarge 1s seen in the Image: in Of

.,•

What is the most likely identification of this organism?


a Acmetobacter baumannii

'
b Nelsseria gonorrhoeae
c Heemophilus ducreyi
d Escherichia coli
57. A 1 O-year--Old child with cystic fibrosis presents wilh cough and shortness of breath. Her
sputum Gram stain is seen m the image:
••


••

Based on the Gram stain the best medium and Incubation condition to optimize recovery
of the organism seen is:
a MacConkey agar incubated in C02
b Tinsdale agar incubated in ambient air
c chocolate agar incubated in C02
d CNA agar incubated in ambient air
. robiol09Y Preana/ytical & Suscoptlb lllty Testing
6·· ~tC ·
age depicts ' (fimal magnification
a G ram stain · 1,000>< ) o f a knee fluid from a patient
•• r11e im s recently undergone knee replacement surgery:
,.. ,vhO ha

The best interpretation of this Gram stain is:


a Gram-posi t!ve coc?i. suggesti~e of Staphylococcus
b Gram-pos1t~ve bac1llt sugg est~ve o f Cor;nebacterium
c Gram-pos1t1ve bacilli suggestive o f Listeria
d Gram-positive cocci suggestive of Streptococcus
S9 A vag1nalfrec1al swat;> is .collected from a pregnan.t patient to screen for group B
streptococcus colornzatton. The best medium to inoculate the specimen to is:
a CNAagar
b UM broth
c sheep blood agar
d thioglycollate b roth
60. Susceptibility testing is performed on a Staphylococcus aureus isolate from a blood culture
with the following results:
oxaclllin: resistan t
cefazohn susceptible
cl1n<lamyc1n· susceptible
erythromycm: susceptible
tnmethoprim/sulfamethoxazote: susceptible
vancomycin: suscepbble

What should the technologist do next?


a ceftriaxone should be reported instead of cefazolin
b clindamycin should be tested for inducible resistance prior to reporting
c the trimethoprim/sulfamethoxazole result should be removed since all S aureus are
resistant
d the cefazoli n result should be changed to resistant since the oxacillin result is resistant
61. A Quality control procedure on a new batch of Mueller-Hinton agar plates using a
Staphylococcus aureus ATCC strain and ncubalion in ambient air resulted In all the disk
zone sizes measuring too small. The most likely reason for this is that the:
a Mueller-Hinton plates were poured too thin
b potency o f the antibiotic disks is too high
c bacterial su spension w as not diluted to the proper concentraUon
d in ubalion should have been in a 5 - 10% C~atmoaphere

N 978-089189.eeo&
6 : M ic ro b iology Preanalytical & Susceptib .1.
I 1ty 7',
62. What organism combination is appropriate to quality control the listed lest 0 esu119
characteristic? r organism
a beta-hemo lysis: Staphylococcus aureus and Streptococcus pyogenes
b catalase: Staphylococcus aureus and Staphylococcus epidermidis
c H2S production: Proteus mirabilis and Salmonella sp
d indole: Escherichia coli and Proteus m"abilis
63. The antimicrobial susceptibility test for an Escherichia coli isolated from a perito
.....
o-av had the following results: near Ouid
Antib iotic Susceptibility Interpretati on
amikacin rcsis1ant
ampiclllin susceptible
cefazolin (1st generation) suscep~ble
cefoxitin (2nd generation} susceptible
gentamicln susoeplible
tobramycin susceptible

The next best step is to:


a report out the antimicrobial susceptibilily results wilhout further investigation
b verify the results as it is unusual for amikacin to be resistant with gentamicin and
tobramycin testing susceptible
c verify the results as it Is unusual for Escherichia coli to be susceptible to ampicillin
d verify the results as it is unusual for Escherichia coli to be susceptible to cefazolin
64. The antimicrobial susceptibility test for a Klebsiel/a pneumoniae isolated from a pleural
~. fluid had lhe following results:
Antibiotic Susceptiblllt y Interpretation
amikacln susceptible
ampicillin susceptible
eerazolln (1 sl generation) susceptible
ceroxilin (2nd generation) susceptible
gentamiein susceptible
lobramyc1n susceptible

The next best step is to:


a report out the antimicrobial susceptlbilily results wilhout further investigation
b .verify the results as it is unusual for Klebsiella pneumoniae to be susceptible to
ampicillin
c ve.r ify the results as it is unusual for Klebsiel/a pneumoniae to be susceptible lo
cefazolin
d verify the results as it is unusual for Klebsiella pneumoniae to be susceptible to both
ampicilli n and cefazolin
65. A K/ebslella pneumonias isolate is known to produce carbapenemase; therefore, It
~. therapeutically will not respond to the antibiotics:
a amoxicillin, cefazolin and imipenem
b colistin and rifampin
c sulfamelhoxazole and trimethoprim-sulfamethoxazole
d tetracycline and doxycycllne
66. When processing a patient specimen for Gram stain and culture, the proper use of a
biological safety cabinet Includes:
a bringing into the cabinet all required media and equipment just prior kl setting up each
individual specimen
b keeping the ultraviolet fight on for the first 30 minutes of working in the cabinet
c not using any heat generating equipment such as open flames or microbumer/
incinerators cted
d not disrupting the air curtain barrier by keeping air flow and exhaust grills unobstru

366 The Board of Cet1111catlon Study Gulde Ge ISBN 97~18M6ll--cao


-ta•¢
....
. oblOIOY Y Preana/ytlca/ & Su . •
6: filter inal fluid test results that are most consist t . . s ceptib//1ty Testing
11• cerebrosP ased protein . IeveI en With viral meningitis inclUde··
a decse sed glucose level
b itteleased lactate level
c ittelea sed mononuclear leukocyte count
d 1ncrea fbT t f
fmicrobial
1 suscep 1 11ty est or an Escherichia coll.
,."' rM an ronowing results:
had 1he
1501ated from a peritoneal fluid
•' tlbiotic Susceptibility Interpretation
"'' An k! cln
am1 susceptible
n,pici11n . susceptible
e I zoin (1st generaioon) susceptJble
ce a . )
cttOli in ( nd generation.
. 2 rcs1113nt
'
...e'<>Ul•iln• (3rd generatJon) suscepUble
~lf\llllT\IC1n SUS<:epbble
neJCt best step is to:
Tile port out the antim icrobial susceptibility results without furthe · . .
a re 1 ·1 1
·s unusua1 for ce.o
, x1.t1n
. to be resistant with
r 1nves1
verify the resu ts as 1 r 1gat1on
r .
b susceptible . . 0 e azo on 1estong
·-rify the results as 111s unusual for Escherichia coli to be suscept.ble t
c ·~
. . .
11 ·1 · 1 r ,.. · . . 1 o amptcolhn
d verify the resu s as 1 is unusua or cschench1a colt to be susceptible to cerazolin

U. Recommended
indudeS: quality control surveillance o f commonly used microbiology equipment

a iocUbator: temperature reco.r ding on a daily basis


b centrifuge: checmg revolutions w ith tachometer on a yearly basis
c 1errigerator: temperature recording on a weekly basis
d ro1a1or: revolutions per minute on a m onthly b asis
11. A wound speccime
k n grew 2 cholonybt yp eds on sheep blood agar and a single, clear colony
lype on Mac on ey agar. 5 eep 1oo agar growth was documented as:
colony t)'pe #1 swarming over entire plate. Gram stain: Gram-negative ro<J
colony t)'pe #2 white colony, Gram stain: Gram-positive cow in dusters

The best way to isolate colony type #2 from colony type #1 is to subculture:
a colony #1 to sheep blood and chocolate agars
b colony #1 to sheep blood and/or MacConkey agar
c colony #2 to sheep blood and chocolate agars
d colony #2 to CNA and/or PEA agar

It. A differential medium tha t can be used as a primary isolation agar producing predictable
colored colonies that can b e distinguished from other organism colony types describes:
a buffered charcoal yeast extract agar
b blOOd phenylethyl alcohol agar
c campylobacter blood agar
d ctoromagar

n. Anticoagulants acceptable for use with blood, bone marrow and synovial fluid specimens
that are to be cultured include:
:I
a EDTA and sodium citra te
I
b heparin and sodium citrate !
c sodium polyanethol sulfonate (SPS) and hepari~ ;' .
d sodium polyanethol sulfonate (SPS) and EDTA

" I.
!
''
Clinic•/ Labonlory Cortiflcallon Exami nations 367
b
6 : Microbiology A erob ic G ra 111 .p0
. . Slllve C0
73. Representative beta-lactam antibiotics and their mechanism of action include: cc1
a ampicillin, cefazolin <md imipenem: and inhibition of cell wall synthesis
b ampicillin. cefazolin and imipenem: and inhibition o f p NA replicatio n
c cipronoxacin. levonoxacin a nd aztreonam; and 1nh1b1t1on of cell wall synthe .
. h.b. . f 0 SIS
d ciprofloxacin, levonoxacin and aztreonam: an d 111 1 1hon o NA replication

A erobic Gram-Positive Cocci


74. The colony coun t from a suprapubic urine culture g rowing 10 colo nies of Staphyl
saprophyticus is: ococcus
a OCFU/ml
b 100 CFU/ml
c 1.000 C FU/ml
d 100,000 CFU/ml
75. Three sets of blood cu ltures were obtained from an adult patient w ith fever and sus
endoca~ditis . The aerobic bottle of o ne set had growth of Staphylococcus epiderm;~:~ted
days of mcubahon. This md1cates that: t5
a there was low-grade bacteremia
b the organ ism is most likely a contaminant
c the patient has a line infection
d the blood culture bottles are defective
76. A p regnant patien t is screened at 36 weeks' gestation for g roup B Streptococcus (GBS). A
...... vagin al swab 1s collected a nd cultured 111 Tedd-Hewitt bro th with 8 µg gentamicin/ml and
°""'-'' 15 µg nalidixic acid/ml. The broth is subcultured onto sheep blood agar after 24 hours or
incubation. No GBS are seen on the s ubculture and the results are re ported as negative.
The patient later goes on to deliver a n infant with early onset GBS disease. What is the
most likely reason for the negative GBS c ulture?
a the patient was screened too early since screening afte r 38 weeks is re commended
b a vaginal swab was collected instead of a vaginal/rectal swab
c the Todd-Hewitt broth used was inhibitor{ to the organ ism
d the selective broth was in cuba ted only 24 hours before subculture
77. A u rine isolate G ram stain shows Gram-positive cocci in clusters. The o rganism tested
catalase positive. To identify this o rganism from culture, the technician should perform a
coagulase test and a/an:
a polymyxin B susceptibility
b novobiocin suscepti bility
c oxidase
d beta -tactamase
78. T he G ram stain from a blood culture shows Gram -posi tive cocci in chains. T he
subcultured plates from the blood c ullure bottle show no growth Additional testing should
be done (O detect the presence of:
a Staphylococcus sapropliylicus
b Aerococcus urinae
c Abiotropilia defecliva
d Streptococcus pneumoniae

~018.ASCl'
368 Tho Board of Cortificnti()rl S ludy Guld~ Go ISBN 978-089189-66-0
9
d
6: Microbiol ogy Aero bic Gram-Positive Cocci
Jg. Viridans streptococci can be differentiated rrom Streptococcus pneumoniae by:
a alpha hemolysis
b colony morphology
c catalase reaction result
d bile solubility

80. A reliable test for distinguishing Staphylococcus aureus from other staphylococci is:
a oxidase
b coagulase
c catatase
d optochin suscepti bility

81 . The optochin disk is used for the identification of:


a Haemophllus influenzae
b group A beta-hemolytic streptococci
c Streptococcus pneumoniae
d Enterococcus
82. In the _bacitraci ~ su~ceptibi~ity test, if there is a zone of Inhibition surrounding the disk
following overnight incubation at 37°C , the colony most likely consists of:
a Staphylococcus aureus
b Group A Streptococcus
c Streptococcus pneumoniae
d Group B Streptococcus
83. Which 2 diseases are usually preceded by Infection with beta-hemolytic streptococci?
a rheumatic fever, undulant fever
b glomerulonephritis, rheumatic fever
c rheumatic rever. tularemia
d glomerulonephritis, undulant fever
84. The enterotoxin produced by certain strains of hemoly1ic, coagutase positive
Staphylococcus aureus:
a is the primary cause of subacute endocarditis
b creates a biofilm on indwelling catheters
c causes a rapidly occurring (2 - 6 hours after ingestion ) food poisoning
d is of extremely low virulence
85. A gamma-hemolytic Streptococcus that blackens bile esculln agar but does not grow in
6.5% NaCl broth is most likely:
a group B Streptococcus
b Enterococcus
c group D Streptococcus (Streptococcus bovis group]
d Streptococcus pneumoniae
86. Gram stain examination from a blood culture bottle shows dark blue, spherical organisms
in clusters. Growth on sheep blood agar shows small, round, pale yellow colonies. Further
tests should include:
a catalase production and agglutination test for Protein A
b bacitracln susceptibility and latex grouping
c oxidase and indole reactions
d Voges-Proskauer and methyl red reactions

C2018ASCO ISBN 978-089189-6609 Clinical Laboratory Certlnc• tlon Ex•mln•llons 369


6 : Microbiology Aerob ic Gram-Positive Co
CC/
87. Gram-positive cocci in chains are seen on a Gram stain from a blood culture. The -
organism grows as a beta-hemolytic colony. Further tests that could be performed inciUde:
a PYR, bacitracin, and hippurate
b catalase and agglutination test for Protein A
c oxidase and mass spectrometry
d Voges-Proskauer and methyl red
88. "Nutritionally variant• streptococci are:
a enterococci
b group D enterococci
c beta hemolytic streptococci
d in the genera Granulicatella and Abiotrophia
89. After 24 hours a blood culture from a newborn grows catalase-negative. Gram-positive
cocci. The bacterial colonies are small, translucent and beta-hemolytic on a blood agar
plate. Biochemical test results of a pure culture are:
bacltracin: resistant
CAMP reaction: positive
bile eseulin: not hydrolyzed
65% NaCl broth: no growl h

Assuming that all controls react properly and reactions are verified , the next step would be
to:
a perform a Streptococcus group typing
I b report the organism as Streptococcus pneumoniae
c report the organism as Staphylococcus aureus
d report the organism as Staphylococcus epidermidis
90. A nonhemolytic streptococcus that has been isolated from an ear culture grows up to the
edge of a 0.04 unit bacitracln disk. Which of the following tests would help to determine if
the organism is Enterococcus?
a hydrolysis of PYR
b growth in the presence of penicillin
c optochin susceptibility
d fermentation of mannitol
91. The organism most commonly associated v.ith neonatal purulent meningitis is:
a Neissaria meningitidis
b Streptococcus pneumonias
c group B streptococci
d Haamophi/us influenzae
92. A common cause o f acute exudative pharyngitis is:
a Staphylococcus aureus (beta-hemolytic]
b Streptococcus pneumoniae
c Streptococcus agalacllae
d Streptococcus pyogenes
93. The m ost fr~quent cause of prosthetic heart va lve infections occurring within 2-3 months
after surgery 1s:
a Streptococcus pneumon/09
b StrrJptococcus pyogenes
I c Staphylococcus a11reus
d Staphylococcus epido•mict1s

370 Tho 0oou l


' ISBN 978-069189·6609 O:l()tS ASCP
A erobic Gram-Positive Cocci
. Microbiolo gy
6· theterized urine is inoculated onto blood and MacConkey. agar us_ing a 0.01 ml loop.
94. A car 24 hours. 68 colonies of a small translucent nonhemolytic organism greYf on blo~d
Aft!r but not MacConkey. Testing reveals small Gram-positive, catalase-negallve cocci.
~e preliminary report and follow up testing would be:
a growth of 680 CFU/ml of Gram-positive cocci, optochin and bacitracin susceptibility
tests to follow
rowth or 6,800 CFU/ml of a Staphylococcus species, latex agglutination test to follow
~ ~rowth of 6,800 CFU/ml of a Streptococcus species. esculin hydrolysis and 6.5% NaCl
growth test to follow
d growth or 6,800 CFU/ml of a Streptococcus species. no further testing

95, Children who have infections with beta-hemolytic streptococci can develop:
3 acute pyelonephrilis
b acute glomerulonephritis
c chronic glomerulonephritis
d nephrosls

96• A Gram-positive coccus isolated from a blood culture has the following characteristics:
oplOchin susceptibility: negative
bac•lracln (0.04 U) susceptibility: negative
bde esculln hydrolysis: negative
hoppurale hydrolysis: positive
catalase: negative

This organism is most likely:


a Staphylococcus aureus
b Streptococcus pneumoniae
c Streptococcus pyogenes
d Streptococcus agalactiae
97. A beta-hemolytic streptococci that is bacitracin-sensitive and CAMP-negative is:
a group B
b group A
c beta-hemoyltic, not group A , B , or D
d group D
98. A beta-hemolytic Streptococcus that is badtracin-resistant and CAMP-positive Is:
a groupAorB
b group A
c group B
d group D
99. Group B, beta-hemolytic streptococci may be distinguished from o ther hemolytic
streptococci by which of the following procedures?
a latex antigen grouping
b growth In 6.5% NaCl broth
c growth on bile esculin medium
d bacitracln susceptibility
100. It is important to differentiate between Enterococcus and group D streptococci because:
a viridans streptococci are often confused with enterococci
b several enterococci cause severe puerperal sepsis
c group D streptococci are avirulent
d enterococci often show more antibiotic resistance than group 0 streptococci

Cllrrlcal L aboraloty Certification Exam /no tions 3 71


6 : Micro bio logy Aoroo/c Gr;1111-Pos/1Jvo
. Cocci
101. Stf'O/ltococcus p11ewno11iae can be dirrerentitiled llesl from lho v1rldons group or --.....
slrep1ococci by:
a Gram slain
b the lypo of homolysis
c colonlal morphology
d bile solubility
102. Characterlslically, enlerococci are:
a unable 10 grow in G.5% NaCl
b relatively resistant 10 penicillin
c sodium hippurale positive
d bile esculin negative
103. Which of lhe following would best dlfferontiale Streptococcus agalocliae from
Streptococcus pyogenes ?
a ability to grow in sodium azide brolh
b a positive bile-esculin reaction
c hydrolysis of sodium hippurate
d bela-hemolysis on sheep blood agar
104. Which of the following organisms is, lo dale, considered universally susceptible 10
penicillin:
a Haemophilus iofluenzae
b Neisserie gonorrhoeee
c Streptococcus pyogenes
d Corynebeclerium diphlheriae
105. A bela·hemolylic Gram-positive coccus was isolated rrom lhe cerebrosplnal rl\Jld of a
2-day-old infant with signs of meningitis. The !sol~te ~row on.sheep blood a~ar under
aerobic conditions and was resistant to a bac1trac1n disc. Which of the following should be
performed for the identification of tho organism?
a oxldase production
b catalase formation
c latex antigen grouping
d esculin hydrolysis
106. During the previous month. Staphylococcus epidermidis has been isolated from blood
cultures at 2-3 times the rate from the previous year. The most loglcal explanation for the
increase in these isolates Is that:
a the blood culture media are contaminated with this organism
b the hospital ventilation system is contaminated with Stapliylococcus epidermidis
c there has been a break in proper skin preparation before drawing blood for culture
d a relatively virulen t isolate is being spread from patient to patient
107. A yellow colony lr?m a wound culture tested catalase-positive and coagulase-negative.
The organism ~tamed as Gram-po sitive cocci in clus ters. Which of the following tests
would d1fferenhate between a coagulase-negative Staphylococcus and Micrococcus?
a novobiocin susceptibility
b leucine aminopeptidase (LAP) p roduction
c furazolidone (100 µg/disk) susceptibility
d hydrolysis of bile esculin

372 Th e Bonrd o l CortlflcaUon Study Guid o Ge


ISBN 978.0B9189-E609 02()t&ASCI'
jiP
: Micro bio logy Aoro blc Gro111-Poslllv o Cocci
6
- A light yellow colony ~rem a skin les~on grew aerobically and losled os calalaso posllive
tOS. 11nd coagutase negative. The organism Gram slalned as posltlve cocci In cluslors. The
:!t. organism w as mo~ifled ~xldase p~slllve, b_acltracin (0.04U) susceptible and reslstanl to
lysostaphin. What is the 1denllfical1on of lh1s organism?
a Stapliylococcus oureus
b Micrococcus tuleus
c Staphylococcus epidermidis
d Peptostreptococcus anaerobius

9 An isolate of an un~nown be~a-hemolylic Streptococcus Is streaked perpendicular to


IO • a streak of bel a-lysm-producing Staphylococcus oureus. Aller incubation a zone of
arrowhead hemolysis is noled at lhe interface of the 2 streaks. What Is the name of the
test and the presumptive 1dentif1cation of the unknown Streptococcus?
a hippurate hydrolysis and S agalactiae
b CAMP test and S pyogenes
c hippurate hydrolysis and S pyogenes
d CAMP test and S agalactiae

110. Which of the following may be used as a positive quality control organism for the bile
esculin test?
a Stapliylococcus epidermidis
b Staphylococcus aureus
c Streptococcus pyogenes
d Enterococcus faecalis
111. Which test is used to differentiate viridans streptococci from Streptococcus pneumonlae?
a CAMP test I
b bacitracin disk test
c hippurate hydrolysis test
d optochin test
112. A gray, nonhemolytic, catalase-negative colony grows on a CNA plate. The following
biochemical results are obtained:
65% NaCl nogative
bite esculin positive
PYR neg alive
bacltracln resistant
hippurate hydrolysis negative
CAMP Test negative

The most likely identification is:


a Enterococcus faecalis
b Streptococcus bovis
c Streptococcus viridans
d Streptococcus pneumonia

••• 08918!Hi009 Cllnlca l Labo,.,lotY Certincallon Exemlnatlon$ 373


b
6: Mic ro bio logy Aer obic Gram-Pos/t1 ...
ve Cocci
113. A 56-year-old male has 2 sets o f blood cultures that grow Gram -positiv e cocci after . -
admitted to the hospital with an oozing leg wound pos t opera tio n. The infected area ~e111g
swollen, and warm to the touch. A red line has appeared at the sight of the wound a ~Cd,
15

beginning to travel up the patienrs leg. Biochemicals perfo!'"'led from the beta hem07d .'S
colonies on the sheep blood agar pla te revealed the following: Yl•c
CAMP test negalivc
hippurete hydrolysis negativft
PYR positive
bacitrocin sensiljve
65% NaCf no gro\.vlh
bile esculin negalivo

The most likely identification is:


a Streptococcus pyogenes
b Streptococcus aga/actiee
c Staphylococcus aureus
d Enterococcus faecalis
114. A 55-year-old man presents to the emergency room with chest pain and is round to have
suffered a heart attack. He has a past history of hypertension. and high cholesterol. The
patient is admitted and scheduled for a triple bypass procedu~e. D uring recovery, he
becomes septic, developing a high grade fever and pneumonia. Gra m-positive cocci in
clusters, isolated from both his lungs via an nduced sputum specimen and the surgical
incision, produce beta-hemolytic, catalase-p::>sitive colonies on sheep blood agar.

Identify the organism most likely isolated and the biochemical test pe rformed to confirm
the identification.
a Staphylococcus aureus and latex agglulir ation
b Streptococcus pyogenes and PYR
c Streptococcus agalactiae and latex agglu:ination
d Enterococcus faecium and PYR
115. A young boy who routinely bites is finger nails develops a wound on his right pointer finger.
A culture reveals alpha hemolytic, dry colonies on the blood and CNA plates that are
catalase negative. resistant to optochin. and 6.5% NaCl negative. G ram stain of the colony
is Gram-positive cocci in chains. The organism most likely isolated is:
a Enterococcus faecium
b Enterococcus faecalis
c Streptococcus viridans
d Streptococcus agalactiae
116. A patient with a prosthetic heart valve visits the dentist for her y early checkup. Two weeks
.....,later. she presents to her primary care physician with a high fever, chills, and shortness of
C>U
brea th and receives a diagn osis of subacute endocarditis. Multiple blood culture sets are
drawn.on the patient.and sent to the lab. Twi;nty-four hours later th e bottles are positive for
bactenal growth. _Which organism would you expect to grow?
a Staphylococcus lugdenensis
b Staphylococcus saprophyticus
c Streptococcus viridans
d Streptococcus egalacliae

374 The Board of Certmea llon Study Guido Ge j


ISBN 978-48918U609
...............
02()!&¢
~ Microbiology Gram-Negative Bacilli
1t7."A 1s-year-old is admitted to the ER with severe sinusitis. Aspi ration specimens from the
nasal pa~sage rev_eal a pure culture o f alpha-hemoly1ic, depressed center colonies w ith
a d1stinct1ve mucoid appearance on a blooo agar plate. Gram stains of the colonies are
stiown below:

'

Which of the following could aid in the identification of the organism recovered?
a bile solubility, optoch1n sensitivity
b hippurate hydrolysis, bile esculin
c bacitracin sensitivity, Lancefield grouping
d PYR positive. catalase negative
118. A 6-year-old m ale presents to his pediatrician with a severe case of tonsillitis. The
,., physician collects a throat swab specimen and orders a GAS (Group A Streptococcus)
. , probe test. The following day the probe comes back negative. A culture is requested. The
following results are obtained:
cata1ase negative
bacatraon disk sens1t1ve
hippurate hydrolysis neg olive
CAMP lest negative
PYR negative
Gram stain Gra1n-posiOvo cocci Jn chains

Which of the following organisms is most I kely causing the tonsillitis?


a group A beta-hemolytic streptococci
b group B beta-hemolytic streptococci
c group C beta-hemoly1ic streptococci
d group 0 beta-hemolytic streptococci

Gram-Negative Bacilli
119. A clean catch urine culture (obtained with a 0.01 mL calibrated loop) grows 60 colonies of
Escher hia coll Which of the following represents the final colony count in CFU/mL?

a 60 ~UlmL
b 61'.' :FUlmL
c 6£' CFUlmL
d 6 CF U/mL

Cllnlc•l L.,,otalOly C - - EA....,._ 375


6: Microbiology Gr .........._
am-N egau """"'
""20--\iii;;:;;;~~;:;;:;:;;:;:;-;;;;:;--;:;--;;:~-;::-:i;:-;;:-;;--;:;;~~;;;;::-;;;;;;:;;;;;;-;;:r;;:;:;--:;:;;;:;:::-;:;::;::-..:.:.:::'..:~v~a~,aa
120. When performing a stool culture. a colony type typical of an enteric pathogen . cI///
subcultured on a blood agar plate. The resulting pure culture is screened with rs
to obtain the following results: several lasts
TSI: acid bull, alkaline slanl, no gas. no H 2S
phenylalanine deaminasa: negallve
mofflity: positive
serological typing: ShigeHa fle•neri (Shigella subgroup 8)

The serological typing is verified with new kit and controls. The best course or acuon
would be to:
a report th e organism as Shigella flexneri without further testing
b verify reactivity of motility medium with positive and negative controls
c verify reactivity of the TSI slants with positive and negative controls for H2 s PrOd .
d verify reactivity of phenylalanine deaminase with positive and negative controls Uchon
121. When performing a Kovac indole test, the substrate must contain:
a indole
b tryptophan
c ornilhine
d paradimethylaminobenzaldehyde
122. The ONPG test allows organisms to be classified as a lactose fermenter by testing for
which of the following?
a permease
b beta-galactosidase
c beta-lactamase
d phosphatase
123. The most rapid method for detection of Francise/fa tularensis is:
'4.$
outv a serological slide agglutination utilizing specific antiserum
b dye stained clinical specimens
c lluorescent antibody staining techniques on clinical specimens
d polymerase chain reaction
124. Infection of the urinary tract is most frequently associated with:
a Staphylococcus aureus
b Escherichia coli
c Enterococcus faeca/is
d Serratia marcescens
125. MacConkey media for screening suspected cases or hemormagic E coli 0157:H7 must
contain:
a indole
b citrate
c sorbitol
d lactose
126. Members of the family Enterobacteriaceae sllare which one of the following
characteristics?
a produce cytochrome oxldase
b ferment lactose
c produce beta-hemolysis
d reduce ni trate to nitrite

. . '
: Microbiology Gram-N egative Baell/I
6
- Which one of the following genera is among the least biochemically reactive members of
127· the Enterobacteriaceae?
3 Proteus
b pseudomonas
c Citrobacter
d Shigella

t28. Which one of the following Gram-negative bacilli ferments glucose?


3 Alcaligenes faecalis
b Burl<holderia cepacia
c Acinetobacter fwoffli
d Yersinia enterocolitice

129. A sputum culture from an alcoholic seen in the ER grows gray mucoid stringy colonies on
sheep blo~d agar. The i~olat~ grows readi yon MacConkey agar and f~rms mucoid, dark
pink colonies. The colonies yield the following test results:
ONPG: +
indolo:
glucose: +
oxiclase:
citrate: +
VP: +

The organism is most likely:


a Edwardsielfa tarda
b Klebsiella pneumoniae
c Escherichia coli
d Proteus vufgaris
130. An organism was inoculated to a TSI tube and gave the following reactions:
alkaline slant/acid bull, H2S. gas produced

This organism most likely is:


a Klebslella pneumoniae
b Shigella dysenteriae
c Salmonella typhimurium
d Escherichia coli
131. An isolate from a stool culture gives the following growth characteristics and biochemical
reactions:
MacConkey agar: colorless colonies
Hekloen agar: yellow-orange colonies
TSI: acid slant/acid bull. no gas. no H~
urea: positive

These screening reactions are consistent with which of the following enteric pathogens?
a Yerslnla enterocolltica
b Sh/gel/a sonnei
c Vibrio parahaemolyticus
d Campylobacter jejuni
132. A TSI tube inoculated with an organism gave the following reactions:
alkaline slant. acid bull; no H 2S, no gas produced

This organism is most likely:


a Yersinia enterocolitica
b Salmonella typhi
c Solmonel/a enteritldis
d Sl>•gclla dysenterlae

Clinical Laboratory Cer1lflc11tlon Exam/nal/ons 377


6 : M icroo10 1ogy
133. A n orga nism g ave the following reactions:
TSI: acid slant. acid butt: no H, S gas produced
indole: positive
molihty: positive
citrate: negative
lysine docarboxyiase: positive
ure3: negative
VP. negative

This organism m ost likely is:


a K lebsiella pneumoniae
b Shigella dysenteriae
c Escherichia coli
d Enterobacteria cloacae
134. Which of th e following organisms can gro w in th e small bowel and caused'
children, traveler's diarrhea, or a severe cholera-like syndrome through th~~;:a i~
entero toxins? "Ctioiict
a Yersinia enterocolitica
b Escherichia coli
c Salmonella typhi
d Shigella dysenteriae
135. Shigella species characteristically are:
a urease positive
b nonmotile
c oxidase positive
d lactose fermenters
136. A Gram-negative bacillus has b een isolated from feces. and the con firmed biochemical
reactions fit those o f Shigella. T he organism d oes not agglu tin ate in Shigella antisera.
What should b e done next?
a test the organ ism w ith a new lot of antisera
b test with Vi antigen
c repeat the b iochemical tests
d boil the organism and retest with the antisera
137. Biochemical reactions o f an organism are consistent w ith S higella. A suspension is
tested in antiserum w ithout resulting agglutination. How ever, a fter 15 minutes of b00i'19.
a gglutination occurs in group-D antisera. T he Shigel/a species is:
a dysenteriae
b nexneri
c boydii
d sonnei
138. >100.000 C FU/ml of a Gram-negative bacilli w ere isolated o n M acConkey rrom a urine
specimen. Biochemical resu lts are as follow s:
glucose: acid, gas produced
indofe: neg alive
urea: posiUve
TOA: positive
H,S: positive

The organism is mos t likely:


a Morganella morganii
b Proteus mirabilis
c Proteus vulgaris
d Providencia stuartii

378 T h o Boa rd or Cortlflcatlon Stu dy Gu ido 6e


1 0
.; ~ology Gra111-Negotlvo Bacl/J

. e culture had the following culture results:


Aunn
t39· P blood. swarming
E-hee
co111nibin CNA: no growth
~•~cConkny: >100,000 CFU/mL nonlactose .fermenter
>100,000 CFU/mL nonlactoso-lermentor with red pigment

The isolates from MacConkey agar had the following biochemical reactions:
t••• lsolato 1 lsolalo 2
rst atk/acid atk/acld
urea positive negative
TOA positive negall11e
H s posilive negallve
2
The organisms are most likely:
a proteus vulgaris and Enterobacter cloacae
b Proteus mirabilis an~ Serratia "!arcescens
c Morganella morganu and Klebs1ella pneumoniae
d Providencia stuartii and Serratia liquefaciens
An s-year-old girl was a?.m itted to the hospital with a 3-day history of fever, abdominal
!!0· pain. diarrhea. and vomiting. A stool culture grew many lactose-negative colonies that
•'·' yielded the following test results:
0J1Cidase: negative
TSI: acid slan t/acid bull
indo!e: negative
urease: posillve
ornithine docarboxylase: poslt111e
sucrose: positive
H,S: negative
motility al 25' C : positive

The most probable identification of this organism is:


a Escherichia coli
b Providencia stuartii
c Yersinia enterocofitica
d EEdwardsiella tarda
t41. A fecal specimen, inoculated to xylose lysine deoxycholate (XLD) and Hektoen enteric
...,
.u (HE) produced colonies with black centers. Additional testing results are as follows:

Biochemical screen Result Serologl cal tes t Res ult


Glucose rermentation posHive polyvalent no agglutinalion
H,S poslll11e group A no agglutination

lysine decartloxylase posillve group B 1 no agglutination

urea negative groupC no agglulinatlon


ONPG negative group 0 no agglutination

indole positive group V1 no agglutination

The most probable identification is:


a Salmonella enrerica
b Edwardsiella tarda
c Proteus mirabifis
d Shiga/la sonnei

Cl/nlcal Laboratory Certlncotlon Examinations 379


6: Microbiology Gram·N ogatlve Bac/IJ/
142. A 10-year-old boy was admitted to the emergency r~om with lower right quadrant Pain;;;;;;
:=:v tenderness which mimicked appendicitis. The following laboratory results were obtained:

Patient value Normal range


16 -60%
% sogmented neutrophils 75%

wee count 200 • 10>/µL (200 • 109/L) 13.0 • 1o'/µL (13.0 • 10'IL)

The admitting diagnosis was appendicitis. OurXig surgery the appendix appeared normal·
an enlarged node was removed and cultured. Small Gram-n_egative bacilli were Isolated '
from the room temperature plate. The organism m ost likely ts:
a Prevotella melaninogenica
b Shigella sonnei
c Listeria monocytogenes
d Yersinia enterocolitica
143. A 25-year-old man who had recen tly worked as a steward on a transoceanic grain ship
presented to the emergency room with high fever, diarrhea and prostration. Axillary lymph
nodes were hemorrhagic and enlarged . A \'Vright-Glemsa stain of the aspirate showed
bacilli that were bipolar, resembling safety pins. The m ost likely identification of this
organism is:
a Bruce/la melitensis
b Streptobacillus monilifonnis
c Spiril/um minus
d Yersinia pes/is
144. Biochemical reactions of an organis m are consistent with Salmonella. A suspension is
tested in polyvalent antiserum A through G and V i antiserum. There is agglutination in the
'"
""" Vi antiserum only. What should be done next?
a boil suspens0on of the organism for 10 minutes to inactivate the Vi antigen
b test organism with individual anlisera for agglutination
c report "no Salmonella Isolated"
d repeat biochemical identification of the crganism
145. A clean catch urine sample from a nursing home patient is cultured using a 0.001 ml loop.
It grows 67 colonies of a lactose fermenter that has the following biochemical reactions:
TSI: aclCl/ncid
oxldase: negative
motility: posltlvo
fndolo: nogelive
ell rote: positive
VP: positive
lysine decarboxytase: negaUve
omilhine decarboxylase: positive
uroa: negative

What should the microbiologist report?


a 670 CFU/ml Serratia marsecens
b 6,700 CFU/ml Providencia stuartil
c 67,000 CFU/ml Entarobactar cloacae
d 67,000 CFU/mL Klebslella oxytoca
146. Pleslomonas shigelloidas Is a member of the f • tic
that differentiates Pleisiomonas from other E am 11by Enterobacteriaceee. The charactens
n 1aro acterlaeae is:
a positive oxidase
b glucose fermentation
c reduc!ion of nitrates to nitrites
d growth on MacConkey agar

380 Tho Board of Certlllcallon Study Gulde 6e


ISBN 978--089189~609 020~
G: r,1(cro blology Gram-Nag;itlve B aell/I
- A blood culture from a 64-year-old male with lymphoma was poslllve blood culture at
1•7• 1s 11ours incubation. The organism Is a nonlactose fermenting Gram-negative bacillus on
~· MacConkey agar. Further testing gives the following reactions:
oxid~so: nogellve
TSI: alkaline/acid. no hydrogen sutr.de
n10111ity: positive
indolo: posillve
eitrato: poslllve
omithino decarboxy1ase: negalive
uro:i: positive
phenylalanine deamlnase: positive
VP: negative
The genus is:
a Morganella
b Proteus
c Providencia
d Serralia

14s. The stock cultures needed for quality control testing or motility are:
a Salmonella typhimuriurn-Escherichia coli
b Escherichia coli--Pseudomonas aerug inosa
c Serralia marcescens-Escherichia coli
d Klebsiella pneumonias-Escherichia coli
149. The stock cultures needed for quality control testing or oxidase production are:
a Escherichia coli--Klebsiella pneumoniae
b Salmonella typhimurium-Escherichia coli
c Escherichia coli--Pseudomonas aeruginosa
d Proteus mirabilis-Escherichia coll
\50. The stock cultures needed for quality control testing or deaminalion activity are:
a Escherichia coli-Klebsiella pneumonias
b Salmonella typhimurium- Escherichia coli
c Escherichia coli-Pseudomonas aeruginosa
d Proteus mirabilis-Escherichia coli
151. The stock cultures needed for quality control testing of deoxyribonuclease (DNase)
production are:
a Salmonella typhimuriurn-Escherichia coli
b Escherichia coll-Pseudomonas aeruginosa
c Proteus mirabllis- Escherichia coli
d Serralia marcascens- Esc/1erlchfa coli
152. Quality control o f the spot lndole test requires the use of ATCC cultures of:
a Pseudomonas aeruginosa-Protaus mirabi/ls
b Salmonella typhi -Shigefle sonnei
c Escherichia coli-Proteus vulgaris
d Escherichia coli-Enterobacter cloacae
153. An organism that exhibits the satellite phenomenon around colonies of Staphylococcus
aureus is:
a Hoemophilus influenzee
b Neisseria meningilidis
c Ncisseria gonorrhoeae
d Klebsiella pneumonioe

Clin;cal L.a bonttory Certincation Examinations 381


6 : Mic robiology Gram-Neg atlvo
8 8 C/f//
~
,
154. An organism isolated from the surface of a ~kin burn 1s fou~d to produce a diffusible ---
pigment on a blood agar plate. Further studies of the Ofganism would m ost likely sh 9•een
organism to be: ow the
a Stapliylococcus aureus
b Serratia marcescens
c Flavobacterium meningosepticum
d Pseudomonas aeruginosa
155. A nonfem1enting Gram-negative bacillus is isolated from a wound. The nitrate and O>(d
1
are strongly positive. The growth on Mueller-H inton agar produces pyoverdin. The ase
organism is:
a Burl<holderia cepacia
b Moraxella lacunata
c Chryseobacterium (Flavobacterium) meningoseptlcum
d Pseudomonas aeruginosa
156. A small, pleomorphic Gram-negative _bacillus is isolated from an ~ye ~ulture. It grows onl
on chocolate agar and is oxidase-vanable. The m ost likely organism 1s: Y
a Acinetobacler /woffii
b Haemophilus influenzae
c Stenotrophomonas maltophilia
d Pseudomonas aeruginosa
157. A blood culture bottle with macroscopic signs of growth ts Gram stained and the technician
notes small, curved Gram-negative bacilli resembling •gull wings.· It is subcultured to
blood and chocolate agar, and incubated aerobically and anaerobically. After 24 hours, no
growth is apparent. The next step should be lo:
a subculture the bottle, and islcubate in microaerophilic conditions
b assume the organism is nonviable. and ask for repeat specimen
c utiliZe the oxidase and indole test to detect Aeromonas
d subculture the bottle to a medium containing X a$ld V factors
158. The optimal incubator temperature for Isolation of the Campylobacter jejunilcoli group is:
a 4"C
b 2o·c
c 2s·c
d 42°C
159.
....
....,
A patient with a nosocomial pneumonia has a sputum Gram stain that shows many
neutrophils and numerous small Gram-negative coccobacilli. The organism grew in
24 hours as a mucoid, hemoly1ic colony on blood agar and a colorless colony 001 a
MacConkey agar. The organism had the follo'Ning characteristics:
oxidase: negative
catalase: POSiOve
nlttate: negative
O NPG: negative
ornithine decarboxylase: nogative
lysine decarboxyfase: negative
The organism is:
a Stenotrophomonas maltophilia
b A lca/igenes faecalis
c Moraxella lacunata
d Acinetobacter baumannil

382 The Board ol Certilic•llon Study Guide Se


~Microbiology Gram-Neg ative Bae/Ill
r ~ A gastroenterologist submits a gastric biopsy from a patient with a peptic ulcer. To obtain
· presumptive evidence of Helicobacter pylori. a portion of the specimen should be added to
which media?
a urea broth
b tetrathionate
c selenite
d tryptophan
l61. Acinetobacter twoffii differs from Nelsseria gonorrhoeae in that Acinetobacter.
~. a exhibits a Gram-negative staining reaclion
b will grow on MacConkey and EMB media
c is oxidase-positive
d produces hydrogen sulfide on a TSI slant
162. A 4-year-~ld is admitted with symptoms of meningitis, and a Gram stain of the
,..., cerebrosp1nal fluid reveals small, pleomorphic, Gram-negative coccobacilli. After 24 hours
.,..., incubation at 35•c. small, moist, gray colonies. which are oxidase variable, are found on
the chocolate agar plate only. Which of the following biochemical data would be consistent
with this isolate?
a CTA dextrose: positive
CTA maltose: positive
ONPG: negative
b sodium hippurate hydrolysis: positive
A disc: negative
CAMP test: positive
c X factor: no growth
V factor: no growth
XV factor: growth
horse blood: no hemolysis
d catalase:
esculin hydrolysis:
positive
positive
I
methyl red: positive
"umbrella" motility at room temperature
163. A Gram s tain of a touch pre p from a gastric biopsy shows Gram-negative bacilli that are
slender and curved. The most likely pathogen is:
a Burkholder/a cepacia
b Corynebacterwm urealyticum
c Helicobacter pylori
d Pasteurella mullocida
164. A characteristic that is helpful in separating Pseudomonas aeruginosa from other
members of the Pseudomonas family is:
a a positive test for cytochrome oxidase
b oxidative metabolism in the OF lest
c production of fluorescein pigment
d growth at 42"C
165. A very bloody stool is received by the laboratory. The following day a pathogenic strain
of E coli is isolated . Which sugar should this isolate be tested against to begin the
identi fication process?
a mannitol
b sorbitol
c lactose
d arabinose

• '-SCP ISBN978•089189·6609 Clinical ~•boraroiy Certlf/cell°" Exam/nalion• 383


6: M icro bio logy ___,,,,,.._
166 Wt11. h Gron1-Nogat1ve a """11111
• c one of the folio I ac1111
. w ng results is typical of Campylobocter fejuni:
a op~imal growth at 42•c
b oxidase negative
c catalase negative
d nonmotile
167.
~ptimutm growth of Campylobacter jejuni is obtained on suitable media incubated at
1 an a mosphere containing: 42·c
a 6"/~ 02. 10-15% C0 2 • 85-90% nitroge11
b 10 Yo H2, 5°/o C0 2. 85% n itrogen
c 10% H2. 10% C0 2. 80% nitrogen
d 25% 02. 5% C02 , 70% nitrogen
168. The porphyrin test was devised to detect strains of Haemophilus capable of:
.....
ONLY
a ampicillin degradation
b capsule production
c hemin synthesis
d chloramphenicol resistance
169. Haemophilus influenzae is most likely considered normal indigenous flora in the:
a oropharynx
b female genital tract
c large intestine
d small intestine
170. Haemophilus influenzae becomes resistant to ampicillin when the organism p roduces :
a a capsule of polysaccharide material
b NAO
I c
d
porphobilinogen
the beta-lactamase enzyme
171. An isolate on chocolate agar from a patien t w ith epiglottitis was suggestive of
Heemophilus species. Additional testing showed that the isolate required NAO ror growth
and was nonhemolytic. Th e organism is most likely Haemophilus:
a haemolyticus
b ducreyi
c influenzae
d parainfluenzae
172. Which or the following specimens is considered to be the most sensitive for the recovery
of Bruce/la in cases of chronic infection?
a blood
b urine
c bone marrow
d lymph n ode

384 The Board of Certification Study Guido 60


6: Microbiology Gram·Nogatlve Bacilli
7J A genus that is found i~ soil and water and causes Infections in immunocompromised
1 · patients has the following characteris1ics:
....,_., sh<ICP blood agar: violot plg mont
/\lncConkoy agar: growth
42•c incubation: growth
oxld.se: positive
OF glucose: fermenter
lndolo: negative

The genus is:


a Campylobacter
b Chromobacterium
c A eromonas
d Serratia

174. Which one of the following results is typical of Campylobacter fetus subspecies fetus?
;;;t, a optimal growth at 42°C
b oxldase negative
c growth at 35.37•c
d catalase negative
175. A Gra':"·nega1ive bacillus with bipo!ar staining w.as Isolated from a wound Infection caused
;-:, by a bite from a pet cat. The following characteristic reactions were seen:
ox.idase: positive
glucose OF: fermentative
motility: negative
MecConkey agar: no growth

Wllich of the following is the mo st likely organism?


a Pseudomonas aeruginosa
b Pesteurefle multocida
c Aeromonas hydrophila
d Vibrio choleras

...""'176. A 3-year-old unimmunized fe male presented in the ER with a severe cough, fever, and
nulike symptoms. The pare nts reported that the child had vomited a few times due to the
severe coughing. A nasopharyngeal swab was used to collect the specimen and planted
on a c hocolate, Bordet-Gengou, and Regan Lowe media. After 5 days of incubation,
colonies grew on all of the media w ith the growth on Bordet-Gengou described as "drops
of mercury." The Gram stain showed minute coccobacilli that were calalase and oxidase
positive. T he most like ly identification of this isolate Is:
a Bordetefla paraperlussis
b Yersinia pestis
c Francisel/a tularensis
d Bordetefla pertussis
177. While swimming in a lake near his home, a young boy cu! his f<>?t· and a~ ~nfection
developed. The culture grew a nonfastidious Gram-negative, ox1dase positive , beta-
hemotytic, motile bacilli that produced deoxyribonuclease (DNAse). The mo st likely
identification is:
a Enterobacter cloacae
b Serratia marcescens
c Aeromonas hydrophila
d Escherichia coli

1'.;!l:~ 976-069169-6609 CllnlciJI Laboratory Certification E xaminations 385


6 : Mic robiology
Gram-Negative e
178.

a antibody tiler represents an earlier infection


b posil~ve antigen test is a false-positive
c specimen was cultured on the wrong media
d culture was not Incubated long enough
179. Which characteristic best differentiates Acinetobacter species from Moraxe/la species?
a production of oxidase
b g rowth on MacConkey agar
c motility
d susceptibility to penicillin
180. An organism has been identified as a member of the nuorescent group of Pseudomonas
Which or the following sets of tests should be used to determine the species or the ·
organism?
a growth at 42•c. pyocyanin production, gelatinase production
b pyocyan in production, gelalinase production. OF glucose
c growth at 37°C, pyocyanin production, OF glucose
d gelatinase production, growth at s2•c, H2S
181. Appropriate culture requirements for a specimen from a patient suspected of having
tularemia include:
a a media with cysteine such as buffered charcoal yeast extract agar
b colistin nalidixic acid agar
I c
d
Mueller-Hinton agar with 5% sheep blood agar
Regan-lowe media
182. A child was bitten on the arm by her sibling and the resulting wound grew a slender Gram-
~. negative bacilli that has the following characteristics:
g rowth o n SBA: colonies Jhat "pit" the agar
colonies odor. like blaaoh
catatase: negative
oxidase: positive
TSI: no growth

The identification of this organism is:


a Moraxe/la cata"halis
b Eikenella co"odens
c Kingella kingae
d Legionella pneumophlla

...
183. Characteristics of the genus Capnocytophaga include:
o•<T a grows in ambient air
b colonies are large and spreading after 2-4 days
c considered "nonfermenter"
d Gram-positive bacillus

386 The Board of Certification Study Gulde 6e ISBN 97&-089169-6609 CQOJS/\SCP

d
Gram-Negative Ba e/If/

6: tudllnt attended a beach party where raw oysters and ot er s e Gram-


" · A collegods The next day, he had symptoms of septicemia. The blood cultures grew
1... coosumll · · c h aracteris
· t'1cs:
•·' bacilli with the following
·
9
·""' noga1111
'cl so·.
())(J a
positive
~~3ceookey agar. pink co'?nios
01129 (ISO µg): susceptible

fhe rnost likely organism is:


Aeromonas liydro~hila
3
b pseudomonas put1da
serratia marcescens
cd Vibrio vulm'IiICUS
Oifferenlialing tests that will separate Burkholderia from Stenotrophomonas include:

3
Gra m stain reaction
b growth on MacCo~key agar
c glucose fermentation
d oxidase
A 17.year-old female with cystic fibrosis is diagnosed with pneumonia. A sputum
~~6• sample grew G ram-negative bacilli with yellow, smooth colonies th at have the following
""' biochemical reactions:
oxtdase: positive
TSI: alk/alk
gtucoso: oxidized
fluorescence: negative
lysine decarboxylase: positive

The most likely organism is:


a Burl<holderia cepacia
b Klebsiella pneumoniae
c Shewanel/a putrefaciens
d Stenotrophomonas mattophilia
187. Characteristics of the HACEK group of bacteria include:
...,
...,, a association with urinary tract infections
b Gram stain of pleomorphic Gram-positive bacilli
c requirement of 5-10% C02 for growth
d requirement of 42•c for growth
188. The labora1ory receives a blood culture from a veterinarian who has been ill fo r many
weeks with revers in the afternoon and evenings, arthritis. and fatigue. T he blood culture is
positive after 5 days, and the organism has the following characteristics:
Gram stain: small, Gram-negative coccobacilll
sheep blood agar: growth arter 48 hours wi th small , smooth, rai sed coloni es

What should the microbiologist do next?


a consider the growth contamination a nd perform another Gram stain
b perform biochemical identification for HACEK organisms
c perform identification and susceptibility testing using an automated system
d take extra safety precautions for possible Bruce/la
189. What are the most appropriate screening tests to presumptively differe ntiate and identify
~~v the nonfermentative Gram-negative bacilli from the Enterobacteriaceae?
a catatase. decarboxylation of arginine, growth on blood agar
b motility, urease, morphology o n blood agar
c oxidase, nitrate reduc tion . growth on MacConkey agar
d oxidase, indole, and growth on blood agar

Clinical Laboratory Certlficatlon Examlnatlnn.c ~A.7


G: Microbiology Gram-Negative~
190, Which g d . ? ac1111
'""' enera a re positive for phenylalanine deaminase (PAD} pro uction -
°"'' a Klebsiella, Serratia. Enterobacter
b Proteu~. Providencia, Morganel/a
c Eschench1a, Edwardsiella Salmonella
d Citrobacter, Klebsiella, Pa'n toea
191. A ~pecimen from a foot ulcer of a 52-year-old male diabetic patient. was sent to the
~' M1crob1ology laboratory for culture. The following results were obtained from a clear COio
growing on the MacConkey: rry
oxidase posilive
catalase positive
OF tubes oxidation positive
pigment production blue/green
growth at 42'C positive
The results indicate which of the following organisms has been isola ted from the culture?
a Acinetobacter baumannii
b Serralia marcescens
c Stenotrophomonas maltop/1ilia
d Pseudomonas aeruginosa
192. A 26-year-old female presents to her doctor with several lacerations on her right hand
MLS
OOLY
and a swollen knuckle. The patient tells the clinician she is a boxer and received the
lacerations 2 days prior while sparring w ithout gloves when she accidentally caught her
partner if! the mouth. At 48 hours a culture reveals
colonies on the blood p late and chocolate plate with no growth on the MacConkey agar plate.
Gram stain of 1he colonies reveals small. slender. G ram·negalive rods
oxidase reaclion is positive
indole reaction is negative
reduces nitrate to nitrite
does not require X and V fac1ors
catalase is negative

What is the most likely identification of this organism?


a Pasture/la multicoda
b Eikenel/a corrodens
c Pseudomonas aeruginosa
d Escherichia coli
193. A young girl cuts her foot on a rock while swimming at the ocean. Her foot begins to
show si9ns of infection ~nd her parents take her to the ER. A culture grows a non-lactose
fermenting Gram-.n.egative rod .that produces copious amounts of hydrogen sulfide and
gas, 1s indole positive and motile. The organism most likely isolated is:
a Shigella spp
b Escherchia spp
c Edwardsiella spp
d Klebsiella spp
194. A. medical technologist is worki~g on a .stool culture from a patient with severe, bloody
d iarrhea. ?he wants to set up b1ochem1cals to differentiate Shigella and E coli. Which of
the following tests would be the most appropriate?
a hydrogen sulfide, ONPG, motility, urease
b lactose, indole, ONPG, motility
c urease, citrate, VP, hydrogen sulfide
d gas, MR, urease, c itrate

~1sASCP
388 The Board of Certification Study Guido Se ISBN 978-089t 6g.6609 d
. Microbiology Aerobic Gram·Nogatlve Cocci
~ Foley catheter urine specimen from an 88-year-old male patient is received by the
195· ~icroblology Laboratory
for culture. At 24 hours, the culture is growing 100,000 c fu/ml
~' colonies or a no~·lactose fermenting Gram-negative rods . The isolate also tested posilive
for u1dole. orrnthine decarboxylase, urease, motility, and phenylalanine deam1nase, and
negative for hydrogen sulfide production. The organism isolated is most likely:

3
E<Jwardslella spp
b Morganella spp
c Hafnia spp
d Shigella spp
A college stud.e n! w~o rece.nll y ~tudi ed a semester abroad in Southeast Asia is admitted to
~~6· the 11osp1tal w ith a d1agnos1s of glande~s·like" infection (melioidosis). A sputum specimen
,,., grows a small, Gram-negative rod that 1s positive for oxidase, reduces nitrate to gas, and
oxidizes glucose, lactose, and manrntol. What is the most likely organism?
a Stenotrophomans mattophilia
b Burkholderia pseudoma/lei
c Pseudomonas aeruginosa
d Acinetobacter baumannii

197 . A nonfermenting organism that g~ows on MacConkey, is nonmotife, oxidase negative, and
,.... has th e characteristi c coccobac1lh Gram stain seen below is:
"'' ' . . ,,,; ...
.·.·. . .. ' . . -;,,-~ --.- .. . ,< , ; ' ...
..\'\"''..." ...:1'
,( '". ,

- .. .. \ •. ~..,,,,,. .. - ..... ...."- .., ..,


\., ~. l
' • •
.' . .. "' ~ . ~. , ,~ ' .-.,....... . .. . '" .. .. '
t I' .
'
.... , .,. - . .t..
, ("' .. • ,,

' .. .
~
...
....
.•..
. ' ,>,_,..
-., o .. ,... _, •' .. ' ( ....
o y)•
\
~

_.l
• ....
"' . . . . · ,

~
...


t

_,

....

..
..: -
~
• 4-

\'"
••

'S • .., '7 .


#
• .. ,
..
..
s,
,

I • • .a.
("" -'! J • •~ ~
., I ; "'J
'

~.
· ~
...
t
"I •
t ..;..
'•, •

... ... • <.;J ., ·


. ...
I

••••
',
• r I
"":'-

...• ..,
I.



, ,

'"'--.
'
• , ;.:

.·,.,,. "'
'\;

... ,.
..
.#

~
l

· •. ~··
. ··
- ~-..

.- _,. . ...
"
,/ ..,... ..,
. • • -

... .

\ \. ~ ' /;•
..

a Pseudomonas aeruginosa
b Stenotrophomonas maltophila
c Proteus mirabilis
d Acinetobacter baumannii

Aerobic Gram-Negat ive Cocci


198. A technologist is reading a Gram stain from a CSF and observe sFmany intra~~lular ~~~~jy
negative diplococci. Which ~el of che~istf)'. and hematology C 5 resu 1ts wou mos 1
be seen in someone with this type of infection?
CSF results WBC glucose protein
A increased increased increased
B decreased decreased decreased
c increased decreased increased
D decreased Increased decreased

a result A
b result B
c result C
d result D
Clinical L.ebor"atory Certlfl<;atlon Examination• 389
el2018 ASCP ISBN 97&-089189-6609
i : Microbiology Aerobic Gr am-N egative Cocci
99. An 18-year-old boy is admilled to the hospila l w ith suspected meningitis. He is r.ethargic
and presents w ith a rigid neck. He has not haj most of the recommer:ded va ccines frorn
ch ildhood to now. Gram stain of his spinal Oui:l shows many PMNS w ith rnlra and extra
cellular Gram-negative d iplococci. The suspected pathogen Is:
a Listeria monocytogenes
b Haemopllilus influenzae
c Streptococcus aga/actiae
d Nelsseria meningilidis
200. The prim ary Isolation of Neisseria gonorrhoeae requires:
a anaerobic condilions
b starch media
c carbon dioxide
d blood agar
201. A urethral swab obtained from a man wilh a urethral exudate was plated direcUy on
chocolate agar and modified Thayer-Martin agar, and a Gram stain was made. The Gram
slain showed Gram-negative diplococci. The cullure plates were incubated at 35•c in
ambient air, but had no growth at 48 hours. The most likely fa ilure for o rganism growth is
that the:
a wrong media were used
b anaerobic chocolate agar plate not sel up
c organism only grows at room temperatu re
d organism requires C02 for growth
202. A Gram st~in pertorr:ned. on a ;;!nus aspirate revealed Gram-negative diplococci with in
PMN s. Ox1dase _testing 1s pos1t1ve and carbohydrate degradation tests are inert. The
organism most lrkely rs:
a Neisseria lactamica
b Moraxella cata"halis
c Neisseria meningitidis
d Neisseria slcca
203. Clinical resistance t0 · ·ir
in: penici rn correlates most frequently with beta-lactamase production

a Chlamydia trachomatis
b Neisserla gonormoeae
c Neisseria meningitidis
d Treponema pallldum
204. All specie s of the genus Neisser/a have lh
e enzyme to oxidize·
a naphthylamine ·
b dimethylaminobenzaldehyde
c glucopyranoside
d tetramethyl-phenylenediamine
205. Which nonculture method is best for the dia n .
female? 9 osrs of Neisseria gonorrhoeae in an adult
a clinical history
b Gram stain of cervical secretions
c MALDJ-TOF MS
d NAAT

390 Th o Board of Certmcatlon Sl udy Gul de Go


ISBN 978-08918~9
........-
6: Microbiology Aerobic Gram-Negative Cocci
206 • The laboratory aid prepared and performed a Gram stain of a vaginal smear for Neisseria
,,,.,
"' gonorrhoeae, as requested by a resident. The findings on the stain were:
many \vhite blood cells
rew epithelial cells
many Gram- positive bacilli
rew Gram-negative diplococci
few Gram-positive cocci In chains

The technologist should:


a report out smear positive for gonorrhea
b report out smear negative for gonorrhea
c request a new specimen due to number of wh ite blood cells
d not read or report a Gram stain on a vaginal specimen
201. Which of the following is the most reliable test to differentiate Neisseria factamica from
Neisseria menlngilidis?
a growth on a modified Thayer-Martin agar
b nitrite reduction to nitrogen gas
c rapid ONPG
d utilization of maltose
208. A method for the definitive identification of Neisseria gonorrhoeae is:
a degradation of amino acids
b EIA
c utilization of carbohydrates
d resistance to penicillins and cephalosporins
2-09. A Gram-negative diplococcus that grows on modified Thayer-Martin medium can be
further confirmed as Neisseria gonorrhoeae if it is:
a oxidase positive, glucose positive, and maltose positive
b oxidase positive and glucose positive, maltose negative
c oxidase positive, maltose positive, and glucose negative
d glucose positive, ox id ase negative and maltose negative
210. An organism previousiy thought to be nonpathogenic, Moraxella catarrhalis, is now known
to be associated with opportunistic respiratory infection and nosocomial transmission.
Characteristic identification criteria include:
a beta-lactamase negative
b butyrate esterase positive
c Gram-negative bacilli
d oxidase negative
211. Which ONA+ organism's colony is not easily broken up and therefore displays the "hockey
puck" characteristic where it can be pushed across the plate of medium?
a Moraxel/a calarrhalis
b Neisseria gonorrhoeae
c Neisseria meningitidis
d Vibrio cholera

•5BN 978-089189-6609 C//nlcal Laboratory CenmcaUon Examinations 391


6 : M icrobiology Aerobic or Facultatlve Gram-Positive Bac//Jf

Aerobic or Facultative Gram-Positive Baci/U


212. A Gr_am stain of organisms on Loeffler agar showed pleomorphic Gram-posllive bacilli. A
medium that will select for this organism is:
a blood
b chocolate
c MacConkey
d potassium tellurite
213. Which organism commonly causes food poisoning by cons~m.ftion or foods containing
excessive populations of organisms or preformed enterotoxon ·
a Salmonella enteritidis
b Shigella sonnei
c Bacillus cereus
d Escherichia coli
214. An organism recovered from a sputum has the following characteristics:
cullure: growth et 6 days on buffered charco&I y<1ast extract (BCYE) agar, incubated
under aorobfc cond1t1ons \vilh C02 at 35•c
Gram stain: delicate branching Gram·posllive bacilli
modlned acld-fasr sraln: branching, ftlamenrous. ·partially" acid-fast bacterium

These results are consistent with which or the following genera?


a Nocardia
b Mycobacterium
c A c/inomyces
d Streptomyces
215. The best test to differentiate Listeria monocytogenes from Corynebactarium species Is:
a catalase
b motility at 2s·c
c motility at 35•c
d Gram stain
216. Establishing the pathogenicity of a microorganism isola ted from a child's throat and
identified as Corynebacterium diphtheria would depend upon:
a the morphological appearance as revealed by Gram slain
b the type of hemolysis on blood agar
c a positive toxigenlcity test
d the appearance of growth on Tinsdale tellurite agar
217. Which feature distinguishes Erysipelothrix rhusiopathiae from other clinically significant
non spore-forming, Gram-positive, facultatively anaerobic bacilli?
a •tumbling· mot/lily
b beta-hemolysis
c more pronounced motility at 25°C than 37•c
d H2S production
218· Listeria can be confused with some Streptococcaceae because it Is beta hemolytic and:
a nonmotile
b catalase negative
c oxidase positive
d esculin positive

'1Q' The Board of Certification S tudy Guido 6e


ISBN 978-08918~
r5: f,1 ic roOIUIV~,
'd from a cutaneous black lesion was
A erobic or Fa
b . cultallvc Gram-Positive Baell/I
it9· ~~:1ours of incubation at 35•c l~ere wass~o ~;~~~or routine bacterial culture. After
shceP blood. agar. The colonies were nonh <;>n MacConkey agar, but J+ r wt
O::d off-white with a ground glass appearance e~olyt1c, nonmolile 4.5 mm in dia~~te
h
~on1111a-shaped.oulgrowths that stood up like :.b ach colony had ~n
irregular edge wil~
vilh an inoculaling needle. A Gram slain of a 1 eaten egg whites" when gently lifted
~cctangular bacilli. The organism is most likel::p1cal colony showed large, Gram-positive
c1ostridium perfringens
~ Aeromonas hyd~ophila
c aacillus anthrac1s
d Mycobacterium marinum
A branching Gram-positive, partially acid-fast 0 . .
120 · washing on a 63-year-old woman receiving ch rgarnsm is isolated from a bronchial
hydrolyze casein, tyrosine or xanlhine. The m~~~.~~rapy. The org~nlsm does not
1 e Y genus
. d causing the Infection is·
a Actrnoma ura ··
b Erysepelothrix
c Nocardia
d Streptomyces

2.21. Which one of the following Gram-positive bacilli is associated ·ih · f .


pregnant women, elderly adults, and outbreaks of food cont VI!' t;n ectlons 1n neonates.
sheep blood agar is shown in the image. amma on? Its growth on

a. Clostridium perfringens
a Corynebacterium jeikeium
b Erysipetothrix musiopathiae
c Usteria monocytogenes
222. A wet mount of vaginal fluid is examined microscopically and large squamous epithelial
~. cells are seen with gram-variable bacilli clustered on the cell edges. The pH of the fluid is
5.0. The most likely pathogen is:
a Escherichia coli
b Arcanobacterlum haemolytlcum
c Gardnerella vaginalis
d lactobacillus

,BN97~89189·6609
Clinlcal L•boratory Certification Examln•tio.n s 393
6 : Mic rob io lo gy q
Anaerobes
223. An aspirate of a dee p wound was plated on blood agar p la tes and incubated aerobically
-
and anaerobically. At 24 hours there was growth on both plates. This indicates that the
o rganism is a{n):
a nonfermenter
b obligate anaerobe
c aerobe
d facultative anaerobe
224. Th e c haracteristic that is most commonly associated w ith .the presenc:;e of stri~t anaerobic
bacteria and can be taken as presumptive evide nce of their presence in a chmcal
specimen is the:
a presence o f a single bacterial species
b production of gas in a thioglycollate brot'l culture . .
c growth on a blood agar plate incubated in an anaerobic 1ar
d presence of a foul. putrid odor from tissue specimens and cultures
225. Gram stain of a thigh wound showed many Gram-positive spore-forming baciUi. The
specimen was placed on brain heart infusion blood agar and incubated aerobically at
35•c for 3 days. At the end of that time, the plates showed n? growth . The most likely
explanation is that some o f the specimen should have been incubated:
a on chocolate agar
b for 5 days
c under 5% C02
d anaerobically
226. An aspirate of a deep wound was plated on blood agar plates a erobically and anaerobically.
At 48 hours there was growth on the anaerobic plate only. The next step in the evaluation
of this culture is to:
a reincubate both plates for another 24 hours
b Gram s ta in and begin organism identification
c call physician and request blood culture
d set up a Bauer-Kirby susceptibility test
227. In general, anaerobic infections differ from a erobic infections in which one of the following?
a they usually respond favorably to aminoglycoside therapy
b they usually arise from exogenous sources
c they are usually polymlc robic
d Gram stains of specimens are less helpful in diagnosis
228. The following growth results were observed on media inoculated with a foot abscess
""'
ONLY aspirate and incubated in 3-5% co2.
SBA: 2+ large gray colonies
PEA: no growth
chocolato: 3+ large gray colonies
M acConkey: 3+ lactose lermenters
lhioglycollate broth: Gram-negative bacllli ard Gram-positive bacllll

Biochemicals were set up on the colonies from the MacConkey agar plate. Whal should
the microbiologist do next?
a set up bioch emica ls on the colonies from SBA
b Gram stain colonies on SBA
c subculture thioglycollate broth to SBA aerobic and SBA anaerobic
d test colonies on chocolate agar with hemIn and NAO

.66()9 c201sASCI~
394 T t>e Board of Certification Study Guld• 6e ISBN 978.()89 189-_ _ _ . . . . . . . _
~obiology
6
Anaer obes
·
--::;\\'llich of the io II owmg
. pairs
• o I organisms
• usually grow on kanamycin, vancomycin, laked
l•·· o!<)OCI agar?
sacteroides and Prevotella
3
b Mobifuncus and Gardnerella
c p0 rphyromonas and Enterococcus
d Veil/one/la and Capnocytophaga
. Acceptable specimen sources for cultu re of anaerobic bacteria includes:
230
3 abscess
b stool
c clean catch urine
d vaginal

2)1. propionibacterium acnes is most often associated with:


3 food poisoning
b post-antibiotic diarrhea
c tooth decay
d blood culture contamination

232. The etiologic agent of botulism is:


a highly motile
b non spore-forming
c a glucose fermenter
d an exotoxin producer
233. A strict anaerobe that produces terminal spores is:
a Clostridium tetani
b Corynebacterium diphtheriae
c Bacillus anthracis
d Propionibacterium acnes
234. An anaerobic. box-car shaped, beta hemolytic Gram-positive bacillus isolated from a foot
wound is m ost likely;
a Actinomyces israelii
b Clostridium perfringens
c Bacillus subti/is
d Eubacterium /entum
235. The laboratory is considering adoption of a rapid and sensitive "stand alone" method
~' that detects C/ostridium difficile toxins A and B. Which one of the following testing
methodologies wlll provide this?
a cell culture cytotoxin assay
b latex agglutination
c lecithinase production
d NAAT
236. The reverse CAMP test, leclthlnase production, double zone hemolysls, and Gram stain
morphology are all useful criteria in the identification of:
a Clostridium perfringens
b Streptococcus agalactlae
c Propionibacterlum acnes
d Baell/us anthracis

Cllnlcat Laboratory Cortlncatlon E111.tmln•tlons 395


6: Microbiology

...
237.
("!~~y

238. Clostridium difficile can be detected by:


"-$
..._, a fluorescent staining
b glutamate dehydrogenase
c growth on LKV media
d high-pres.sure liquid chromatography
239. Anaerobic Gram-positive bacilli with subterminal spores were isolated from a P8riton
abscess. The colony has a swarming appearance. The m ost likely identification of th~ai
. .
organism 1s:
~

a Bacillus cereus
b C/ostridium septicum
c Eubacterium lentum
d Bifidobacterium dentium
240. If a stool sample is sent to the laboratory to rule out Clostridium difficile, what medium
should the microbiologist use and what is the appearance of this organism on this
medium?
a BBE: colonies turn black
b Bruce/la agar: red pigmented colonies
c CCFA: yellow. ground glass colonies
d CNA: double zone hemolytic colonies
241. A Gram stain of a peritoneal fluid showed large Gram-positive bacilli. There was 3+ growth
on anaerobic media only, with colonies producing a double zone of hemolysis. To assist
with the classic identification of the organism, the microbiologist could:
a determine if the organism ferments glucose
b perform the oxidase test
c set up egg yolk agar plate
d test for bile tolerance
2.42. A patient has a suspected diagnosis of subacute bacterial endocarditis. His blood cultures
.....
OH.•
grow non spore-forming pleomorphic Gram-positive bacilli only in the anaerobic bottle.
What test(s) will give a presumptive identification of this microorganism?
a beta-hemolysis and oxidase
b catalase and spot lndole
c esculin hydrolysis
d hydrolysis of gelatin
243. The Gr~ stain of drainage from a pulmonary sinus tract shows many wees and 3+
.....
OH.Y branching Gram-positive bacilli. Colonies grow only on anaerobic media after 3 days
incuba~on. They are yellow-tan and have a molar tooth appearance. T he most likely
genus rs:
a Aclinomyces
b Bacteroldes
c FusobacJerium
d Nocardia

0201&~
3 96 The Board of CerllllcaUon Study Gulde 611 ISBN 978-089189~809 1
~icrobiology Anaerobes
,...--Which organism Is the most common anaerobic bacteria Isolated from lnlecllous
2''· processes ol soft tissue and anaerobic bacleremla?
·"
.... aacreroides trogi/is
~ F;1sobacterit1m nucleatum
porpliyromonas asaccharo/ylica
~ C/ostridium perfnngens
An organism from a peritoneal abscess is isolated on kanamycin-vancomycin laked blood
2'5· agar and gr~ws black colonies o!' BBE agar. It is nonpigmented, catalase positive, and
~· indole negative. The genus of this organism is:
a Acidomli:iococcus
b aactero1des
c porphyromonas
d prevotella

246. Which of the following sets of organisms may exhibit a brick red fluorescence?
~~' a porphyromonas asaccharolytica and Clostridium ramosum
b C/ostrldium difficile and Fusobacterium sp
c Prevotella melaninogenica and Porphyromonas asaccharolytica
d Fusobacterium sp and Bacteroides tragilis

247. A 1-2 mm translucent, nonpigmented colony, isolated from an anaerobic culture of a tung
'"' abscess after 72 ~ou rs, was found to fluoresce brick-red under ultraviolet tight. A Gram
'''" stain of the organism revealed a coccobacllius that had the following characteristics:
growlh in bile: inhibited
vancomycln: resistant
kanamycin: resistant
colistin: susceptible
calalase: negative
esculi n hydrolysis: negative
indole: negative

The identification of this isolate is:


a Bacteroides ovatus
b Propionibacterium acnes
c Prevotella me/aninogenica
d Porphyromonas asaccharolytlca
248. A thin, Gram-negative bacillus with tapered ends isolated from an empyema specimen
iu grew only on anaerobic sheep blood agar. It was found to be indole positive, lipaso
'''" negative, and was inhibited by 20o/o bile. The most probable identification of this isolate
would be:
a Bacteroides distasonis
b Prevotel/a melaninogenica
c Fusobacterium nucleatum
d Clostridium septicum

...249..
CH,,
The presence of 20% bile in agar will ollow g rowth of:
a Fusobacterlum necrophorum
b Bacteroides fragi/is
c Prevotel/a me/aninogenica
d Porphyromonas asaccharolytica
2SO. A control strain of Clostridium should be used in an anaerobe jar to assure that:
a the plate media is working
b an anaerobic environment is achieved
c lhe jar is filled with a sufficient number of plates
d the indicator strip is checked

C!2018 ASC? ISSN 978-089169-6609 C/inlcal t.aboratoty CettlflcaUon ExaminaUons 397


6: Microbiology
Fung/

....251.
00<.V
Whic~1 one of the following organisms could be used as the positive quality control test for-
lec1thinase on egg yolk agar?
a Bacteroides fragilis
b Fusobacterium necrophorum
c Clostridium petfringens
d Clostridium sporogenes

Fungi
252. The major features by which molds are routinely categorized are:
a macroscopic growth characteristics and m croscopic morphology
b biochemical reactions and microscopic morphology
c macroscopic characteristics and selective media
d specialized sexual reproductive cells and phialides
253. A sputum specimen from a patient with a known Klebsiella pneumoniee infe?lion is
received in the laboratory for t ung us culture. The proper procedure for handling this
specimen is to:
a reject the current specimen and request a ·epeat culture when the ba cterial organism is
no longer present
b incubate culture tubes at room temperature in order to inhibit the bacterial organism
c include media that have cycloheximide and chloramphenicol added to inhibit bacterial
organisms and saprophytic fungi
d perform a direct PAS stain; if no fungal organisms are seen, reject the specimen
254. Many fungal infections are transmitted to man via inhalation of infectious structures. Which
of the following is usually contracted in this manner?
a Sporolhnx schenckii
b Trichophyton rubrum
c Malassezla furfur
d Histoplesma capsu/alum
255. Using a fluorescent microscope, a wet preparation of skin tissue reveals fluorescent
septate hyphae. The smear was prepared using:
a acridine orange
b caloofluor white
c Gomori methanamine silver
d periodic acid-Schiff
256. The formation of germ tubes presumptively Identifies:
a Candida lropicalis
b Candida parapsilosis
c Candids glabrsta
d Candids albicans
257. An HIV-positive patient began lo show .
cultured for bacteria and fungus Ab d~~gns of meningitis. A spinal fluid was collected and
organism is consistent with thi' s :n, u ing, encapsulated yeast was recovered Which
1 1ormation? ·
a Candida glabrata
b Cryptococcus neoformans
c Paracoccidioides brsziliensls
d Sporothrix schenckii

398 The Board of Certlllealfon Study Gulde 6e


IS8N97~189-0609 C!2018ASCP
G: r111 crobiology F1111yi
_.-c :irlclc F1cid disks i11ocul11to1I with n yea st isoltite show brown pigment prod uctio n in a !ew
...
15S· timt1!' This is useful in lhe ide11tilicnlion o l
~' n C11ndkfR Rlbicans
i,CRnrlidR gtabmta
c sncctmromyces cerevisi.ie
d C1yptococcus 11eofo1111011s

259. The one characteristic by \~hich an unknown Cryptococcus s pecies ca n be idenlilied as


,.. Cryptococcus neoformans 1s:
f'.'
1111ppeara nce of yellow colonies
b positive urease test
c presence of a capsule
d positive phenol oxidase test
2f(I. A urine c~lt~re from a patient w ith a urinary tract infection yields a yeast with the following
... characteristics:
~· reiluro to produce germ 1ubos
h yphAO not formed on cornmoal ogAr
urooso·negative
nsslmllalcs trchalose

The most likely identification is:


a Geotrichum candidum
b Cryptococcus lauren/ii
c Candida tropicalis
d Candida glabrata
261. The r~covery of some molds and yeasts may be compromised if the isolation media
contains:
a cycloheximide
b gentamicin
c chloramphenicol
d penicillin
262. A neo natal blood culture collected through a catheter grows a small yeast. Microscopically,
....
"' the y east appears round at one end, with a budlike s tructure on a broad base at the other
end . Growth is enhanced around olive oil-saturated discs. The organism isolated is:
a Candida tropicalis
b Ma/assezia furfur
c Candida llpolytica
d Cryptococcus gattii
263. Which medium could the technologist use to verify that a yeast isolate from a blood culture
~. . is pure?
a Sabouraud dextrose agar
b potato dextrose agar
c cornmeal agar
d CHROMagarm
264. Quality control testing o f CHROMagar'" Candido medium shows very weakly colored
~~' colonies after incubation at 2s• c , ambient atmosphere ror 46 hours. Tho technologist
should :
a repeat the quality control tests to verify results
b repeat the quality contro l tests but incubate at 35•c . .
c repeat the quality co ntro l tests using new subcultures of the quality control organisms
d discard this lot of CHROMagar' " and repeat the quality control tests o n a new lot
number

020l8 ASCP ISRll 978-089189-6609 Cllnlcnl Laboratory Cortlf/cor/on Exam/notions 399


Fung/
6 : Microbiology . lasrna are shown in the
iss. . . of a yeast grown 1n rabbit P
The morphological charactensbcs
"• 1ma9e:
""'
,
~~
.. 0 •
f - \')

-·~~
, ... 0
0

The most likely identification or this yeast is:


a Candida tropical1s
b Candida krusei
c Candida albicans
d Candida glabrata . ococcal disease is:
266. The most sensitive test for the initial diagnosis of crypt
a India ink
b Gram stain
c cryptococcal antigen
d Giemsa stain .
267. Which of the following procedures is recommended to confirm that an unknown mold is
one of the pathogenic dimorphic fungi?
a animal inoculation
b conversion from yeast to mold form .
c demonstration of sexual and asexual reproduct10n
d molecular testing
268. Laboratory workers should always work under a biological safety hood when working With
cultures of:
a Streptococcus pyogenes
b Slephyfococcus aureus
c Candida albicans
d Coccidlo ides immitis

.._.
269. Structures important in the microscopic identification of Coccidioides immitls are:
..... a irregular staining, barrel-shaped arthrospores
b tubercutate, thick-walled macroconidia
c thlek-walled sporangia containing sporangiospores
d small pynform microcomd1a
270.
....,
'<$ Which of the following Is the most useful ITIOl'J>hological feature in identifying the mycelial
phase or Histoplasma capsulatum?
a arthrospores every other cell
b 2-5 µm microspores
c 8-14 µm tuberculate m&croconiclia
d 5- 7 IJITl nonseptate mac:roconldla
p
: M icrobiology Fung i
6
- A mold grown at 2s•c exhibited delicate. septate, hyaline hyphae and many conidiophores
2n. 8 ,tend1ng al nght angles from the hyphae. Oval, 2-5 µm conid1a were formed at the end of
the comd1ophores g iving a Oowerhke appearance. In some areas "sleeves- of spores could
be found along the hyphae as well. A 37°C culture of this organism produced small, cigar-
s.haped yeast cells. This organism is most likely:

3
H1stoplasms capsulatum
b sporotl1rix schenckii
c Blastomyces dermatitidis
d Acremonium falciforme
~ Wh ich of the following is a dimorphic fungus?
2«·
a 8/astomyces dermat1tidis
b Candida albicans
c Cryptococcus neoformsns
d Asperg1//us fumigatus

273. Examination of a fungal culture from a bronchial washing reveals while, cottony aerial
rnycelium. A tease prepara tion in lactophenol cotton blue shows the structures shown in
the image:

. _,
\
'I,
-
.I ' : ·'-
/~. -~
, . .) Ii~
I . . --;::·
....

The m ost rapid test for definitive identification is:


a Maldi-Tof mass spectrometry
b animal inoculation
c exoantigen test
d slide cultu re
274. A mold is isolated from a bone marrow culture of a patient who trav.elled to southeast .
~. Asia . The isolate produced a red pigment that diffu sed Into the medium. The technologist
should suspect
a Blastomyces dermatitid/s
b Aspergillus niger
c Penicillium marneffei
d Rh1zopus species
275. The micro scopic structures that a re most useful in the identification of dermatophytes are:
•u
.,., a septate and branching hyphae
b racquet and pectinate hyphae
c chlamydospores a nd mlcroconidia
d macroconidia and microconidia

Cllnlcel l.•boratory Certification E••mln•tlona 401


'le~ 181H609
6 : Mic ro biology ~
276, Which or tho following is the best aid In the Identification of Epldermophylon f/occosCJ u~
"' ' macroconidia? m
""'' a parallel side walls with at least 10 cells
b spindle-shaped spore with thin walls
c spindlo·shaped spore. thick walls and distinct terminal knob with echinulations
d smooth walls, club-shaped
277. Culture of a strand of hair, that fluoresced yellow·g.reen when ex~mlned with a WOOds
lamp, produced a slow-growing, Rat gray colony with a salmon-pink reverse. Microscop·
examination demonstrated racquet hyphae, pectinate bodies, chlamydospores, and a f~
~bortive or bizarre-shaped macroconidia. The most probable Identification of this isolate
1s:
a Epidermophyton f/occosum
b Microsporum canis
c Microsporum audouinii
d Trichophyton rubrum
278. On day 3 of a fungal culture, a dense grayish cottony growth is observed. It fills the
container and looks like "cotton candy". The most likely mold isolated is a:
a dermatophyte
b dimorphic mold
c zygomycete
d dematiaceous mold
279. Penicillium species can be separated from Aspergillus fumigatus by:
.....
°''" a production of conidia on phialids
b optimum growth temperature
c presence of rhizoids
d lack of vesicle
280. A fungus superficially resembles Penicillium species but may be differentiated because
.....
<><.Y
its phialides are long and tapering and bend away from the central axis. The m ost likely
Identification is:
a Exophia/a
b Acremonium
c C/adosporium
d Paecilomyces
281. An isolate from a cornea infection had the following culture results:
J.C.$
Oo<.Y Sabouraud dexlrose: while & collony at 2 days. violet color at 6 days
slide culture: slender sickle shape macroconidla

The most likely organism is:


a Acremonium
b Aspergil/us
c Fusarium
d Geotrichum
282. In the USA, a common cause of eumycotic mycetoma is:
Ml$
Oo<.Y a Pseudallescheria boydll
b Nocardia brasiliensls
c Coccidioides Imm/Ifs
d Aspergillus fumigatus

402 The Board of Cer11flcaUon Study Guld<I 6e


~ Fungi
,.. . Microbiology
5
· st from a cauliflower-like lesion on the hand exhibi ted brown spherical bodies
2s3. C~~ 2 m 1n diameter when examined microscopically. A slow-growing b~ack mo!d grew on
11
6 t>ouraud dextrose agar. Microscopic e xamination revealed cladosponum. ph1a!ophora
~~d ronsecaea types of sporulation. The probable identification of this organism is:
Fonsecaea pedrosoi
~ p 5 eudallescheria boydu
c p11ialopl1ora verruc.osa.
d Cladosponum camonu
Which of the following scenarios presents the greatest risk for coccidioidomycosis?
2s4.
,
3
Missouri catUe rancher
b Wyoming rabbit rancher
c Central Valley California migrant worker
d pregnant woman with several housecats

285. Which one of the following media is most helpful in distinguishing the morphology of
yeasts?
a cornmeal aga r with Tween 80
b brain-heart infusion medium
c potato dextrose agar
d urea agar
286. The Aspergillus species in this image is c haracterized by blue-green colonies with a white
...
..,, apron .

"


The identification is:
a A terreus
b A niger
c A fumlgatus
d A flavus
287. Which of the following dermatophytes can be identified with its clavate or peg-shaped
~ microconidia that resemble "birds on a wire"?

a Microsporum canis
b Trichophyton rubrum
c Epidermophyton floccosum
d Trichophyton tonsurans
288. Which fungal organism Is urease+ and phenol oxidase+?
us
"" a Cryptococcus neoformans
b Ma/assezia furfur
c Rhodotorula
d Tnr• p hyton mentagrophytes

Clinicel LaborelOIY C - l l o n . , _ _ 403


b
6: M icro biology ,
289 Wh . h Fung/
iu • IC of the following zygomycetes has sporangiophores that arise between rhizoic:Js? -
"'-• a AbsKf1a (Uchthe1m1a) ·
b Cunnmghamel/a
c Fusarium
d Rhizopus
290, A dema tiaceous mold grew in 4 days on Sabouraud agar, isolated from a r~t wound. lhe
image shows chaining macroconidia with longitudinal and horizontal septat1ons.

The most likely identification is:


a Altemaria
b Curvularia
c Paecilomyces
d Scopulariopsis
291 . This fungus is encapsulated and produces by budding. It is associated with the Eucalyptus
tree and is usually acquired from the environment. The o rganism uses glycine as a
sole carbon and nitrogen source in the presence of canava nine (ca navanine glycine
bromothymol blue agar). The organism is:
a Candida krusei
b Cryptococcus gattii
c Cryptococcus neoformans
d Rhodotorula spp
292. Which testing platform meets this description? An isolated colony is irrad iated by a lazer,
which ionizes the biomolecules and ca uses them to become accelerated in an electric
field. The ionized biomolecules then ente r a flight tube where they are separated by their
mass-to-<:harge ratio.
a MALOJ-TOF
b multiplex PCR
c pulsed-field gel electrophoresis
d sequencing

404 Th<I &o.rd ol Ce1llfl11llon Sludy Gulde =.,________ ___l!!l~I97&09


=.a. !?!!!!~tl~
l-~'....:-
::"":~-::lll'
~-_.
Mycobactorla

Mycobacteria
A rirst morning sputum is received for culture of mycobaclerla. It is digested and
zg3. 00110entrated by the N -acetyl-L-cysteine alkali method. Two Lowenstein-Jensen slants are
incubated in the dark at 35°C with 5 - 1Oo/o C02. The smears reveal acid -fast bacilli, and
arter 7 days no growth appears on the slants. The best explanation is:
a improper specimen submitted
b incorrect concentration procedure
c exposure to C02 prevents growth
d cultures held for insufficient length of time
A first morning spu tum specimen is received for acid-fast culture. The specimen is
centrifuged, and the sediment is inoculated on 2 Lowenstein-Jensen slants which are
incubated at 35°C !n
5-10o/o C02. After 1 week, the slants show abundant growth over the
entire surface. Stains reveal Gram-negative bacilli. To avoid this problem:
a utilize a medium that inhibits bacterial growth
b add sodium hypochlorile to the sed iment before inoculation
c incubate the lubes at room temperature lo retard bacterial growth
d decontaminate the specimen with sodium hydroxide

295. A first morning sputum is received for acid-iast bacilli culture. It is digested and
decontaminated by the N-acetyl-L-cysleine alkali method. Two Sabouraud dextrose slants
are incubated in the dark al 35°C with 5-10% C02. The smears reveal acid-fast bacilli, but
the slants show no growth after 8 weeks. Tne explanation is:
a improper media used
b incorrect decontamination procedure used
c improper specimen submitted
d Improper incubation temperature and atmosphere
296. In reviewing the number of Mycobacterium isolates for the current year. it was noted that
u.s there were 76°/o fewer isolates than the previous year (115 vs 28). The technologist in
""' charge of the area has documented that the quality control of media, reagents and stains
has been a cceptable and there has been no gross contamination of the cultures noted.
The most appropriate next course of actlon1s to:
a stop the use of com m ercial media and produce in-house
b change to different formulations of egg and agar based media
c invest in an updated Bactec'"' system for isolation of Mycobacterium
d review the digestion and decontamination procedure
297. A mucolytic, alkaline reagent for digestion and decontamination of a sputum for
mycobacterial culture is:
a N-acetyl-L-cystlne and NaOH
b NaOH alone
c zephiran-trisodium phosphate
d oxalic acid
298. The function of N -acetyl-L-cysteine in the reagent for acid-fast digestion-decontamination
procedure is lo:
a inhibit growth of normal respiratory flora
b inhibit growth of fungi
c neutralize the sodium hydroxide
d liquefy the mucus

t"/. ·~ J ' /,P ··.;;' ' WS-089189-6609 Clinical Laboratory Certlnc.• tlon Ex•min•tlons 405
M ycobacler/a
6: Microbiology . the dark. and incubated in the
. d H media must be refrigera.ted iny prove to>dC for mycobacteria
299. M iddlebrook 7H1 0 an 7 11 ot met the media ma
dark as \\/ell. If these conditions are n •
because:
a carbon dioxide will be released
b gro\111h factors will be bro~en dO\~nte
c light destroys the ammonium sul a . .
d formaldehyde may be produced . ulture contaminated with
. for mycobactena1c
300. The be
st method to process specimens
Pseudomonas is:
a N-aoetyl-L-cystine and NaOH
b NaOH
c zephiran-trisodium phosphale
d oxalic acid 'II' at 1 week while the agar slant shows
·rve for acid-fast baco '
301. An AFB broth culture i_sk~oso ~planation for this is:
no grow1h. The most h e Ye
a the organism is a contami~an~ ·
b AFB grow more rapidly in llquod media
c PANTA was added to the broth 5% CO
d the agar slant was incubated on o 21 . f t'on of laboratory personnel when
h Id be taken to preven 1n ec 1
302. ~~~~~C:~~~~i~e~~ for mycobacterial culture? .
dd NALC in the ratio of 1 part NA~C to .1 part specimen
a
a · s under ultraviolet light
process all spe~omen only after the addition of preservative
b
centrifuge specimens
c . . ood
d
process all specimens in a b1olog1cal safety h . . . .
. . M b · the primary stain used in the acid-fast staining process 1s.
303. When slammg yco ac1en 8 •
a 1% acid fuchsin
b carbol fuchsin
c crystal violet
d methylene blue
304. A positive niacin test is characteristic of Mycobacterlum:
....
""v a avium complex
b fortuitum
c kansasii
d tuberculosis
305. Charae1eristics necessary for lhe definitive identification of Mycobacterium tuberculosis
"-' are:
°'"' a buff color, slow grow1h at 37°C, niacin production-posilive, nitrate reduction-negative
b rough colony, slow growlh al 37°C, nonpigmented
c rough, nonpigmented colony, cording positive, niacin production-negative, catalase·
negative at pH 7/68"C
d rough, nonpigmented colony, slow grow1h at 37°C, niacin p roduction-positive, nitrate
reduclion-positive
306. The disease-producing capacity of Mycobacterium tuberculosis d epends primarily upon:
a production of exotoxin
b produclion of endotoxin
c capacity to withstand intracellular digestion by macrophages
d lack of susceptibility lo the myeloperoxidase system

406 The Board of Certification Study Gulde 6e ISSN 978-089169-6609 C2()tBASCI'


p 6: Microbiology Mycobacteri a
_.. Whot Mycobacteriwn species includes a BCG strain used for vaccination against
307. . ?
tuberculosis .
a txJviS
b forlwwmlc/1elonae complex
c kansasii
d wberculosis
30s. Mycobacterium ~uberculosls complex can be identified directly in A FB smear positive
respiratory specimens the same day the smear was read by:
a cording seen on the AFB smear
b molecular testing
c QuantiFERON®-TB test
d MALDI TOF testing
J09. A primary drug used for the treatment of Mycobacterium tuberculosis is:
a ethionamide
b kanamycin
c rilabutin
d rllampin
310. An unu~ual num ber of Mycobacterium gordonae have been isolated. The most likely
...
..,, source 1s:
a an outbreak o f infections due to Mycobacterium gordonae
b contamination by water organisms
c contamination of commercial Lowenstein-Jensen tubes
d contamination of the specimen collection containers
311. When grown in the dark, yellow to o ra nge pig mentation of the colonies is usually
~~' demonstrated by:
a Mycobacterium tuberculosis
b Mycobacterium kansasii
c Mycobacterium fortuitum group
d Mycobacterium scrofulaceum
312. The mycobacteria that produce a deep yell:iw or orange pigment both in the dark and light
are:
a nonchromogens
b photochromogens
c rapid growers nonchromogens
d scotochromogens
313. Mycobacteria that produce pigment only after exposure to light are classified as:
a nonchromogens
b photochromogens
c rapid growers
d scotochromogens
314. In a suspected case of Hansen d isease (leprosy), a presumptive diagnosis is established
by:
a isolation of o rganisms o n Lowenste in-Jensen medium
b detection of weakly acid-fast bacilli in infected tissue
c Isolation of organisms In a cell culture
d detection of niacin produc1ion by the isolated bacterium

Cllnlcal La b o rato ry Cenlflca lfon Examinations 407


6: Microbiology Mycobacteria

315. Media used to culture Mycobacterium tuberculosis include:


a Bordet-Gengou agar and Middlebrook 7H10 agar
b Loeffler medium and PANTA medium
c Lowenstein-Jensen agar and Middlebrook 7H11 agar
d PANTA medium and cystlne blood agar
316. A 27-year-old scuba dover has an abrasion on his. left t~igh. A culture of this wound grew
an acid fast organism at 30'C. This isolate most likely is.
a Mycobacterium chelonae
b Mycobacterium marinum
c Mycobacterium tuberculosis
d Mycobacterium xenopi
'ff · ·
317. Do erent1at1on o
· m avlum from Mycobacterium intracellulare can be
f Mycobact erw
accomplished by:
a nitrate reduction test
b Tween• hydrolysis test . 'd (TCHJ
c resistance to 10 µg thiophene-2-carboxylic acid hydraz1 e
d molecular testing
318. What Mycobacterium does not usually fluoresce when stained with a fluorochrome stain?
a Mycobacterium bovis
b Mycobecterium fortuitum
c Mycobecterium tuberculosis
d Mycobacterium ulcerans
A thoracic surgical wound specimen grew visible white colonies on sheep blood agar at
....319. 48 hours in 5-10% C0 incubation. When the isolated organism was Gram stained, faint
2
"'"' staining Gram-positive rods were observed. The most probable organism and appropriate
stain to better view organism morphology is:
a Corynebacterium jeikeium and acridine orange sta in
b Mycobacterium fortuiltJm and acid-fast sta in
c Mycobacterium tuberculosis and Aura mine stain
d Mycoptasma pneumoniae and Giemsa stain
320. When compared _to direct speclmei:i carbolfuchsin stains used for detecting mycobacleria.
~v fluorochrome slams are fess sensitive for detecting mycobacteria classified as:
a Mycobacterium tuberculosis complex
b nonphotochromogens
c photochromogens
d rapid growers
321. The principle difference between th z· hi N .
Klnyoun acld-rast stain technique iseth~~ - eelsen acid-fast stain technique and the
a type or dyes used
~ ~feen~~m~f':~~=~~?z~~ interpret sta ined smears
d use or heat to allow the dye to penetrate organism
322. Wha_t slain is most often used as a . .
respiratory specimen? screening stain when mycobacteria is suspected in a
a acridine orange
b auramine o
c calcofluor white
d Gram slain with carbolfuchsin substituted a th
s e counter stain

408 The Board of Certlncalion Study Guido 6e


ISBN 978-089189-6609 02()18ASCI'
: Microbiology Viruses & Other Microorganisms
6
...--Mycot>ncterium tuberculosis is initially irolated from the sputum or a 35-year-o'd ma'e.
;13 Ant1microbial suscepllbohty testing on this isolate is:

3
routinely performed
b only performed if the isola te is from an irrmunosuppresse<l indr1idual
c only performed if the isolate is recovered from a sterile body site
d not routinely performed as no standa rdized method is available
organisms ~art of the Myc<;Jbacterium tuberculosis complex (MTBC) include
Mycobactenum tuberculos1s;
,.
••
a M africanum and M bovis
b M africanum and M kansasli
c M avium and 1"1 bovis
d I.A bominissuis

325. Specimen;; for mycobacte.rial culture that routinely undergo the digestion and
decontam1nat1on process include:
a blood and pleural fluid
b bone marrow and tissue biopsies
c bronchial washings and sputum
d cerebrospinal fluid and pleural fluid
326. The recommended medium/media to inoculate for the primary isolation or U ycobacteria
include:
a a broth-based medium only
b a solid-based medium only
c a broth-based and a solid-based medium
d solid-based media

Viruses & Other Microorganisms


327. Virus transport medium should contain agents that:
a enable rapid viral growth during the transport time
b inhibit bacterial and fungal growth
c destroy nonpathogenic viruses
d inhibit complement-fixing antibodies
323. Which one of the following provides a presumptive Identification of a viral infection?
a cytopathic effect on cell cultures
b intranuclear inclusions in RBCs
c cell lysis or sheep red blood cells
d presence of mononuclear inflammatory cells
329. The specimen of choice for detection of RSV is:
a nasopharyngeal aspirate
b cough plate
c expectorated sputum
d throat swab
330. The genus of virus associated with anogenital warts , cervical dysplasia and neoplasia is:
a herpes simplex virus
b human papillomavirus
c cytomegalovirus
d coxsackievirus

Cllnlcol l.oborotory Certmcatlon Examinations 409


6: Microbiology 4iii
Viruses & Otllor M icroorgan/s
331. Encephal'r .
1 is is ms
most commonly associated with which of the following viruses? -
a Epstein-Barr
b herpes simplex virus
c coxsack ie B
d varicella zoster virus
332. Colds and other acute respiratory diseases are most often associated with;
a Epstein-Barr virus
b adenovirus
c coxsackle B
d reovirus
JJJ.. The Epstein-Barr virus is associated with which or the following?
U LS
~' a chickenpox
b Hodgkin lymphoma
c Burkitt lymphoma
d smallpox

...
334. Which organism fails to grow on artificia l media or In cell cultures?
~' a Chlamydia trachomatis
b Neisseria gonorrhoeae
c Treponema pallidum
d herpes simplex virus
335. Oarkfield microscopy is used to visualize:
1.tlS
ex• a Borre/is recurrentis
b Mycop/asma pneumoniae
c Treponema pallidum
d Legionella pneumophila
336. Microorganisms resembling Mycoplasma pneumoniae have been isolated from the blood
~' or patients treated with antibiotics that:
a complex with nagellar protein
b interfere with cell membrane function
c inhibit protein synthesis
d interfere with cell wall synthesis
337. Relapsing fever in humans is caused by:
a Borrelia recurrentis
b Bruce/la abortus
c Leptospfra interrogans
d Spirillum minus
338. Psittacosis is transmissible to man via contact with:
a insects
b birds
c cattle
d dogs
339. Chlamydia trachomatis infections have been implicated In:
a LGV and conjunctivitis
b gastroenteritis and urethritis
c neonatal pneumonia and gastroenteritis
d neonatal meningitis and conjunctivitis

A10 Tl>• Soard of Certification Study Guide e.. ISBN978-089189·6&19 020IS¢


p r
: Microbio o gy Viruses & Oll1or Micr oorganisms
6
-
340. M"coplasmas
I
ditrer from bacteria in that they:
3 do not cause disease in humans
b cannot grow in artificial Inanimate media
c 1ack cell walls
d are not serologically antigenic

341 . A jaundiced 7-yearb-old boy, with a history or playing in a pond in a rat-infested area, has a
"' urine specimen su molted for a direct darkfield examination. Several spiral organisms are
,,.. seen. Which or the following organisms wou ld most likely be responsible for the patient's
condition?
a Cardiobacterium hominis
b Streptobacillus moniliformis
c Listeria m onocytogenes
d Leptospira interrogans

342. What sample material is recommended to prepare slides for direct smear examination for
.., virus detection by special stains o r FA technique?
o<<V
a vesicular fluid
b leukocytes from the edge of the lesion
c hair samples
d specimen with epithelial cells
343, A 29-year-old man is seen for recurrence of a purulent urethral discharge 1O days after the
"" successful treatment of culture p roven gonorrhea. The most likely etiology of his urethritis
(MY j5 :

a Mycoplasma hominis
b Chlamydia trachomatis
c Trichomonas vagina/is
d Neisseria gonorrhoeae
344, Ureap/asma uraalyticum is difficult to grow in the laboratory on routine media because of
,... its requirement for:
"""' a sterols
b horse blood
c ferric pyrophosph ate
d surfactant such as Tween® 80
345. Which of the following agents is the most common cause of pediatric viral gastroenteritis?
0.\ $
,_. a adenovirus, serotypes 40 and 41
b Norwalk virus
c coronavirus
d rotavirus
346. Hanta or Sin Nombre virus is a Bunyavirus found in the 4 Corners area of the US (Arl:i:ona,
New Mexico, Nevada, Colorado). What is the vector?
a deermouse
b Norwegian rat
c domestic ca nine
d lxodes tick
347. Which type of virus causes severe acute respiratory syndrome?
a paramyxovirus
b enterovirus
c rhinovirus
d coronavirus

~18ASCP ISBll 978-089189·66!19 Cllnlc•I 1.•boretory Certification Examinations 411


q
6: Mlcroblology Viruses & Other Micr o organisms

348. What Is the animal reservoir of West Nile virus?


a mice
b rats
c domestic cats
d birds
349. Which of the following organisms is the causative agent of hand, fool and mouth disease?
....
.,.... a Adenovirus
b coxsackie A
c coxsackie B
d human herpes virus 6
350. Which one of the following viruses is responsible for the most common congenital Infection
,....
o...v
In the United States?
a VZV
b CMV
c EBV
d adenovirus
351. Which one of the following clinical syndromes Is associated with VZV infection .
a infectious mononucleosis
b shingles
c primary CNS lymphoma
d Burkitt lymphoma

...
352. Which of the following clinical presentations is associated with HHV8?
-· a Kaposi sarcoma
b Duncan disease
c fifth disease
d exanthem infectiosum
353. In a person vaccinated against hepatitis B virus several years prior, which serological
marker would be expected?
a HBsAg
b HBeAg
c anli-HBs
d anti-HBC
354. The persistence of which marker is the best evidence or chronic HBV infection?
a HBeAg
b HBsAg
c anti-HBe
d anti-HBs
355. Which of the following statements regarding antigenic shift or drift in influeollza is correct?
....o..v a an tigenic drift is due to point mutations in the H&N genes
b antigenic drift Is responsible for pandemics or influenza
c antigenic shirt is responsible for seasonable epidemics of influenza
d local annual outbreaks of influenza are often due to antigenic shift
356. This virus is responsible for nearly all cases of Qrifantile respiratory bfonchiolilis:
a parainfluenza virus
b metapneumo virus
c coxsackie A virus
d respiratory syncytial virus

41 2 The Board o r Ce rtlfic1llon Study Gulde 6e ISBN 978-089189-6609 020 18 ,SCf'


d
p . rJl icrobio I ogy Parasites
6. . .
----Which test is the pnmary screening
V?
test for HI .
;57.
a serum enzyme linked immunosorbent assay
b western blot
c quanlitalive HIV RNA
d C04 count
;ss. Which of the following HIV tests is the assay for determining response lo anti-retrovirals?
~' a serum ELISA
b quantitative HIV RNA
c CD4 count
d p24 antigen detection

359. Which one of the following is a characteristic of hepatitis B virus?


a hepatitis B infects CD4+T lymphocytes.
b it can be reactivated, causing "shingles".
c it is an enveloped DN A virus that is primarily a blood-borne pathogen.
d this RNA virus is almost a lways transmitted by the fecal-oral route .

Parasites
360. Artifacts found in a stool specimen that can be confused with ova or cysts are:
a partially digested meat fibers
b degenerated cells from the gastrointestinal mucosa
c dried chemical crystals
d pollen grains
361. Polyvinyl alcohol used in the preparation of permanently stained smears of fecal material:
.....
""" a concentrates eggs
b dissolves artifacts
c serves as an adhesive
d enhances stain penetration
362. A method to culture A canthamoeba sp from corneal ulcer scrapings is lo inoculate
'"'
""'' a McCoy cells
b Novy, MacNeal and Nicolle (NNN) medium
c an agar plate overlaid with Escherichia coli
d Regan-Lowe medium
Primary amoebic encephalitis Is caused by:
....363.
""'' a Entamoeba coli
b Dientamoeba fragilis
c Endolimax nana
d Naegleria fowleri
364. A formed stool is received in the laboratory at 10:30 pm for ova and parasite exam.
~' The night shift technologist Is certain that the workload will prevent examination of the
specimen until 7 am when the next shift arrives. T he technologist should:
a request that a new specimen be collected after 7 am
b hold the specimen at room temperature
c examine a direct prep for lrophozoites and freeze the remaining specimen
d preserve the specimen in formalin until It can be examined

Clinical Laboratory Cartlflcatlon Exam/nal/ons 413


6: M icrobiology Parasites
365. The advantage of thick blood smears for ma arial parasites is to:
~

""' a tmprove staining of the organisms


b improve detection of the organisms
c remove RSC artifacts
d remove platelets
366. Multifocal brain lesions in AIDS patienls are commonly caused by:
a Toxoplasma gondil
b Pneumocystis 1iroveci
c Cryptosporidium paNum
d Taenia solium
367. A 44-year-old man was admitted to the hosp Lal following a 2-week history of low-
grade fever, malaise and anorexia. Examination of a Giemsa stain revealed many
intraerythrocytic parasites. Further history revealed frequent camping trips near Martha·s
Vineyard and Nantucket Island, but no travel outside the continental United States. This
parasite could easily be confused with:
a Trypanosoma cruzi
b Trypanosoma modesiense/gambiense
c Plasmodlum fa/ciparum
d Leishmania donovam
368. A patient is suspected of having amebic dysentery. Upon microscopic e xamination of a
fresh fecal specimen for ova and parasites, the following data were obtained:
a 11ophozot10 of 25 µm
progressive unidirectional crawt
evenly d1stt1buted peripheral chromatin
f111ely granular cytoplasm

This information indicates:


a Entamoeba coli
b Entamoeba histolytica
c Endolimax nana
d lodamoeba bOtschlii
369. Refer to the following image:

Trophozoites or the cyst shown abo


a contain red blood cells ve are likely to:
b have dear. pointed pseudopodia
c contain few, 1f any, vacuoles
d have slow. undefined motility

41 4 The Board or Certlficauon Study Gulde 6e


Parasites
11 MIC• ohlology
)I~ f;o1f1H h> ltlll fUllllWllllJ 111111011
"

Upon finding the above on a fecal concentrate, the technologist should:


:i 1mmodl:itoly telephone the report of this pathogen to the physician
b revlow the fecal concentration carefu lly for the presence or other microorganisms ihat
may be pathogenic
c look for motole trophozoites
d request a new specimen because or the presence of excessive pollen grains
371. Refer to lhe following image:

\

Which organism below can be confused with the image?


a Enramoeba /1istoly/ica
b Oien/amoeba fragilis
c G1ardia lamblia
d Tnchomonas vagina/is
q
6: Mic ro biology Par as/to&

....,,.,
372. Refer to the following image:

This structure depicts a:


a cyst or a nonpathogenic amoeba
b trophozoite of a nonpathogenic amoeba
c cyst of a pathogenic amoeba
d trophozoite of a pathogenic amoeba
373. Refer to the following image:

The organism depicted is a(n):


a amoeba
b flagellate
c filaria
d sporozoan

416 The Board o f Certlftcatlon Study Gulde 6e


Parasites

A 24-yea r-old woman , who_just returned from vacationing in Russia, became ill with
diarrhea. The above organism was found in her stool. The patient most likely is suffering
from:
a giardiasis
b amebiasis
c ascariasis
d balantid ia sis
375. A liquid stool speci~en is collected at. 10:00 pm and brought to the laboratory for c ulture
and ova and parasite examination. It 1_s refrigerated until 10:10 am the next day, when the
phys1c1an requests that the technologist lock for amoebic trophozoites. The best course of
action wou ld be to:
a request a fresh specimen
b perform a concentration on the original specimen
c perform a trichrome stain on the original specimen
d perform a saline wet mount o n the original specimen
376. Small protozoan cysts are found in a wet mount of sed iment from ethyl-acetate
.., concentra ted material. Each cyst has 4 nucle i that do not have peripheral chromati n, and
"'·' each nucleus has a large karyosome, which appears as a refractive dot. These oval cysts
are most likely:
a Endolimax nana
b Chilomastix mesnili
c Entamoeba histolytica
d En/amoeba hartmanni
377. The term "in ternal autoinfection" is generally used in referring to infections with:
....
00<v a A scaris /umbricoides
b Necator americanus
c Trichuris trichiura
d Strongy/oides stercoralis
378. Proper collection of a sample for recovery of Enterobius vermicularis Includes collecting:
a a 24-hour urine collection
b a first mornin g stool collection with proper preservative
c a scotch tape preparation from the penanal region
d peripheral blood from a finger

a.q89189-6609
C/lnlc•l l.•bor•tory Certification &em/nation• 417
6: M ic rob iology Peras/tes l
379. A fibrous skin nodule is removed from lhe back of a patienl from Cenlral America. A -
·~, m1croftlana seen upon microscopic exam of the nodule is most likely:
a Wuchorena bancroft1
b Brugia ma/ay1
c Onchocerca vo/vulus
d Loa loa
380. Refer 10 the following image:

The egg depicled abOve is m o st likely to be round 1n children suffering from:


a diarrhea
b conslipation
c penanai itching
d stomach pain
381. Refer 10 lho following illus1ra1ion:

Tho specimen or c hoice for finding the above parasite is:


a stool
b duodenal washing
c rectal swab
d scotch tape preparalion

..
382. Human races is nol a recommended specimen In the detection of:
"' a Strongylot<los stercora/1s
b Entamoeba h1stolytica
c Echmococcus gronulosus
d Ancytostoma duodena/a

<4 18 Tho Bonr<I o f Conlflca rlo n S1udy Gui do 6o


~Microbiology Parasltos
-3- The causative agent of cysticercosis is:
38 .
n Taenia solium
b
raenia saginata
c Ascaris lumbricoides
d Tricfwn·s trichiura

384. Organisms .that can be easily identified to the species level from the ova In fecal
specimens inc Iu d e:
a Metartonlfr!US yokogawai, Heterophyes heterophyes
b Taema solwm, Taema sag111ata
c Necator americanus, Ancy/ostoma duodena/a
d Paragonimus westermani, Hymenolepis nana
385. The preferred specimen for the diagnosis of paragonimiasis is:
~. a bile drainage
b blood smear
c skin snips
d sputum
J86. A stool specimen for ova and parasite examination contained numerous rhabdlliform
larvae. Which factor(s) aid in the identification of larvae?
a Jarva tail nuclei and presence of sheath
b length of the buccal cavity and appearance of the genital p rimordium
c presence of hydatid cysts
d prominen t kinetoplasts in trypomastlgote
387. Which one of the following routine tests for Entamoeba histolytica has the highest
....
.... sensitivity and specificity?
a colonic ulcer biopsy
b stool microscopy
c stool EIA
d urine PCR
388. What is the principal means of distinguishing Entamoeba histotytlca from Entamoeba
hartmanni by light microscopy?
a size of trophozoite
b appearance of karyosome
c appearance of nuclear chromatin
d number of nuclei in cyst form
389. Which characteristic will identify lodamoeba bOtschlii?
a nuclei in mature cyst
b small (5-10 µm) size
c prominent vacuole in the cyst form
d presence of up to 8 nuclei in the cyst form
390. The only medically significant ciliate organism is:
a Acanthamoeba
b Balantidium coli
c Cryptosporidium parvum
d Chi/omastix mesnili
391. Where do Plasmodium sporozoites proliferate?
a bone m arrow
b liver
c red blood cells
d nucleated erythrocyte precursors

Cllnlcal Labol'8tory Cerllflc;tlion Examinations 419


6: Microbiology Parasites

...392.
. against
O't..Y
Individuals who lack the Duffy a ntigen on the surface of their red blood cells aro protected
which species or Plasmodium?
a Pvivax
b P falciparum
c Pma/ariae
d Pova/e
393. Wh ich nematode produces eggs with characteristic hyallne polar plugs at each end?
a Ascaris /umbricoides
b Necalor americanus
c Strongyloides stercorslis
d Trichuris trichiura
394. Which nematode has a characteristic mammillated bile stained egg?
a Ascaris
b Necator
c Strongyloides
d Trlchuris
395. Which organism ls predominantly responsible for visceral larva migrans?
....
c-..v a Ancylostoma braziliensis
b Onchocerca volvulus
c Toxocara canis
d Trypanosoma brucel
396. Which organism has the largest egg?
a Clonorchis
b Diphyl/obothrium
c Fasciola
d Paragon/mus
397. The eggs of which species of Schistosoma can be isolated from urine?
a S haematobium
b S japonicum
c Smansonl
d S stercoralis
398. Which one of the following features of Taenia saginata helps distinguish it from T solium?
a egg with a radially striated wall
b pork tapeworm
c proglottid with <13 uterine branches
d unarmed rostellum
399. Infection by this organism can cause o f 3 12 ceficiency:
a Diphyl/obothrium /alum
b Echinococcus granulosus
c Taenia solium
d Schistosoma mansonl
400. This organism is responsible for hydatid cysts of the fiver:
a Diphyllobothrium
b Echinococcus
c Hymenolapis
d Trichomonas

420 Tho Boa rd ot Cortlrl~tlon Study Guido 6e ISBN 97S.089189-Wl9


p
G: Microbiology Paraslt•

401. How is Tricllomonas vagina/is transmitted?


" ingestion of cyst stage
b ingestion of lrophozoite stage
c laNal stage burrows through the skin
d sexual transmission

402. This parasite has the following characteristics:


banana shaped gamelocytes in RBCs
multiple ring forms In RBCs
all RBCs are Infected

The Identification of the parasite is:


a Plasmodium falciparum
b Plasmodium malariae
c Plasmodium ova/a
d Plasmodium vivax
403. This parasite has the following characteristics:
stains red with acid fast stain
zoonotic transfer to humans
spread by fecal-oral route

The most likely organism is:


a Cryptosporidium
b Giardia
c Naegleria
d Necator

Cf2018ASC? 1$!)' 1918-089 189 6609 Cllnlcal Laboratory Certmcatlon Exam i n ations 421
M olecular Biology
The follow1i1g ilerns have been identified genernlly as approp · "
1
laboratory scientists and medical laboratory l!Jchnicians. Item~~: °r both entry_ level medical
81
laboratory scientists only are marked with an 'MLS ONLY.. ara appropna/e for medical

451 Questions 4 60 Answer-s with Explanations


451 Molecular Science 461 Molecular Science
453 Molecular Techniques 461 Molecular Techniques
456 Applications ofMolecular Tasting 463 Applic8/ions Of Molecular Testing

Molecular Science
I. Which nucleotide has a high frequency of mutation in human chromosomal DNA?
IU
- a adenine
b thymine
c cytosine
d guanine
2. Intervening sequences are found in:
a precursor mRNA
b mature mRNA
c ribosomal RNA
d transfer RNA
3
· Which is considered an epigenetic modification of DNA?
a transversion of an A nucleotide to a Tin an exon of a gene
b transition of an A to a G in an intron of a gene . ne
c !f!eth~ation of CpG islands in the prc;imoter.region of ~~e
d insertion of a nucleotide into the cadmg region of a ge
<. Reverse transcriptase would best be described as a:
~ gzA-dependent DNA polymerase
c RN A-dependent RNA polymerase
d RN'A-(fependent DNA polymerase
s. ,. ._1 A-<Jependent RNA polymerase allele Is x. What
••
:;:.
" e 2 alt 1
is the
x
nd x The mu tan1 h disease
e es for a sex-linked recessive disease are a d · be affected by I e
from ,J:'centage of mate offspring that would be expecte to
1
rents vtho have the following genotypes?
"<>llior;
'•lhor. XK
a 0 KY
b 25
c so
d 10()
7: Molecular Biology
M olecular Science
6. The term that best descr'b
1
a het es males regarding X-linked genes is:
erozygous
b homozygous
c haplozygous
d hemizygous
7.
The mode of inheritance of mitochondrial DNA is:
a dominant
b recessive
c codominant
d maternal
8. Which structure is most resistant to denaturation?
Ml$
Of4LY a RNA:RNA duplexes
b RNA:DNA duplexes
c DNA:DNA duplexes
d all are equally resistant to denaturation
9. What chemistry joins successive nucleotides of DNA?
a glycosidic bond
b ionic bond
c hydrogen bond
d phosphodiester bond
10. Which of the following nucleotides is a purine?
a adenine
b cytosine
c thymine
d uracil
1. Which sequence represents the replicate ONA strand of
5'-GGG ATC GAT GCC CCT TAA AGA GTT TAC-3'?
a 5'-GGG ATC GAT GCC CCT TAA AGA GTT TAC-3'
b 5'-CCC TAG CTA CGG GGA ATT TCT CAA ATG-3'
c 3'-GGG ATC GAT GCC CCT TAA AGA GTT TAC-5'
d 3'-CCC TAG CTA CGG GGA ATT TCT CAA ATG-5'
12. Whicil of the following best depicts the general route of production in a human cell ?
a mRNA _, ONA - protein
b protein - mRNA- DNA
c ONA - mRNA - • protein
d ONA- protein - mRNA
13. There is a bulge in the DNA double helix at position 292 of a gene. This bulge is most
likely due to which type of base m ispairing?
a purine:purine
b purine:pyrimidine
c pyrimidine:pyrimldlne
d adenine:thymine
14. Which of the following is a known limitation of the uracil-N-glycosylase (UNG) chemistry
~Y commonly used for PCR contamination containment?
a Taqman® probes are not compatible
b UNG will re~ai~ ac_tivity if temperatures drop below ss•c
c dUTP substitution increases affinity of probes
d UNG Inactivates GC residues of naturally occurring DNA

452 The Board of Certlncatlon Study Gulde 6e ISBN 978-089189.0009 e2Q!SASCP


p
1: Mokcular Biology Molecular Techniques

Molecular Techniques
s. Which condition has the highest stringency ror ONA probe hybridiz f ?
1
a low temperature. low salt concentration a ion
b high temperature. low salt concentration
c high temperature. high salt concentration
d low temperature, high salt concentration
16. The Kienow fragment of Escherichia coli DNA polymerase 1 is used:
....
x• a to make cDNA from an RNA template
b to label DNA probes by random primer labeling
c in quantitative real lime PCR
d to label a DNA probe by nick translation
17. How many volumes of ethanol are added to 1 volume of a ONA:salt solution to cause the
~' DNA to precipitate?
a 1
b 1.5
c 2
d 2.5
ts. When quantifying the amount of genomic ONA in a sample by spectrophotometry. an
OD 260of1 .0 corresponds to what concentration of ONA?

a 10 µg/ml
b 20 µg/mL
c 50 µg/ml
~· d 100 µg/mL 1 40
A RNA sample is isolated from peripheral blood cells of a patient. When perform;i .1
fi;8~
>v spectrophotometric analysis to determine the yield of RNA in the sample you
dilution of the 0.5 ml sample gives an OD 260 reading of 0.03125 and an O~
e ci·ng
rea
0.01760. What is the total amount of RNA contained Jn the 0.5 mL sample.
01
a 50 µg
b 25µg
c 12.5 µg

~;
d 5 µg
Which statement is true concerning the analysis of short tandem repeats (STR)?
a STR
b STR are amplified
. by reverse transcriptase PCR
c STR code for small cytokines
d STR =~e ° n~lyzed by capillary electrophoresis
~,
11. Whi
P ch reage t
1
a ysis requires high molecular weight DNA
f 1
native real time
CR reaction; generates a signal during the anneallng stage o a quan

a SYBR®
bH Green
c Ydrolysls (1:
d Mo1ecu1ar B aqMan®) probes
"..,. Seorp·ions ' 11ea con

llenat pnmers
~ liydroge
uratfon of DNA dunng . a PCR reaction refers to breaking:

n~c
1
Piles n bondsboe
c CovafhOdiester b tween nitrogenous bases in base-paired Ieo fdeS
cl tides
d Pe .ent bond nds between nitrogenous bases Jn base-paired nu eo
Ptlde bonds sb~~~ween nitrogenous bases Jn base-paired nucleotides
een nitrogenous bases in base-paired nucleotides
7 : M o Iecu Iar B .10 Iogy Molecular Techniques
23. Whal is the most critical step in determining the specificity of a PCR reaction?
a denaluration temperature
b annealing temperature
c extension temperature
d number of cycles in the PCR reaction
24. A PCR reaction in which 4 different sets of primers are used to simultaneously amplify 4
distinct loci in the same reaction tube is known as a:
a multiplex PCR reaction
b heteroplex PCR reaction
c polyplex PCR reaction
d quadraplex PCR reaction
25. Probes are often used during real time PCR to quantitate the formatio n of specific
IALS
~lLY
amplicons during the reaction. Which system involves the use of 2 distinct probes to
generate a fluorescent signal?
a Hydrolysis (TaqMan®) probes
b Molecular Beacon
c FRET
d Scorpions n• primers
26. Refer to the figure. An amplicon for gene X is 176 base pairs (bp) in size. A variant allele
llLS
OllLY
x, gives the same size amplicon but has an Alwl restriction enzyme recognition site that is
not present in the amplicon arising from allele X . A PCR reaction is run on a DNA sample
isolated from 3 patients. The sample is digested with Alwl after the PCR is complete and
products of the digestion are resolved on a 4.0°/o agarose gel .

.. - 1 176 bp

118 bp
+ + +
#1 #2 #3

- lnolcales no A~vl digestion; + indicates Alwl dige$led

Assuming all appropriate controls have worked correctly, what is the interpretation of the
genotype of patient #3 with regards to alleles X and x?
a patient is homozygous for X
b patient is heterozygous for X and x
c patient is homozygous for x
d patient's genotype cannot be determined
27. Methylation of cytosine !esidues is Involved in the alteration of gene expression in a .
14$
ONLY number .0 f cancers. Which sequencing technique is best suited to determine if methylation
of cytosine residues has taken place in the promoter region of a given gene?
a Maxam/Gilbert chemical cleavage
b Sanger dideoxyterminator
c bisulfile sequencing
d pyrosequencing

454 Tho Board of Certification Study Guide 6e


- 7. ecular Biology
-....
28 · " the purpose o f the enzyme uracif-N-gly . Molecular Technique$
.... ratory? CO1ase rn a molecv1ar diagnostics
a lower the melting temperature of RNA·ONA h . .
r0 actions · Ybrids on reverse tran""""ta
· I - .... sePCR
b , -emove any previous y generated amplicons co · .
c to remove contaminating genomic DNA from a rev::ndUTP from PCR reactions
d to melt double-stranded DNA to single-stranded DNA . PCscn R ptase PCR react10ns
on reacuons
29. Whal is the best method to rule out the possibility of genom DNA
resul! in a reverse transcnprase (RT)PCR reaction? ic 9Mng a false-pcsrtive
a use of upstream and downstream primers that span an exon-intro .
target n-exon regron of the
b treatment of RNA that wilt be used rn the RT reaction with RNase
c addition of uracil-N-gtycolase to the RT reaction
d omission of Taq DNA polymerase from the PCR reaction
30. What is the conformation of DNA at g5 •c7
a double-s tranded
b fragmented
c hairpin loop
d single-stranded
31. What type of enzyme can be used to cut a chromosome into smaller fragments of DNA
wrthout removing a ny nucleotides o r functional groups?
a endonuciease
b exonuclease
c tigase
d phosphatase
32. Which method of tissue prese rvation 1s inapprop ria te for molecular testing?
a snap freeze
b ethanol
c formadehyde
d mercuric chlonde
lJ. Wh
..... at Plocess does the fluorography rep re se nt?
7 : Mo lecular Biology ~ppu~auums or M o1ecu/
ar resll
The real-time PCR curves represent dHutions of a single sample. Assuming PCR ~
34.
;... of 100%, what dilution is represented between aliquot A and aliquot B? 'Cleticy 0

••
...
,.
••
••
••
...
... A:Ct=16
- -

"'° "1 ll f 'e t11Jlf" '' l' lJH lt ,. l l N J1 11' • •1 ct""

Cydff

a 2-fold
b 5-fold
c 10-fold
d SO-fold

Applications of Molecular Testing


35. A rommon use for pulsed-field gel electrophoresis is:
a DNA fingerprinting
b mitochondrial DNA typing
c epidemiological typing of bacterial strains
d tumor cell phenotyping
36. Molecular-based testing was performed on a nasopharyngeal swab sample taken from a
....
...... patient. Results are reported as positive for 15481 and negative for 151000. These results
indicate the patient's nasopharynx is colcnized with:
a Bordetella pertussis
b Borde/el/a holmesii
c Borde/el/a parapertussis
d Bordetella spp
37. DNA-based testing for the presence of MRSA uses PCR with primers specific for which
gene?
a vanA
b lnhA
c mecA
d rpoB

4 56 The Board of CerttficatJon Study Guide Ge


olecular Biology
· · . Appl/cations of M
A translocatron rn whrch chromosome pa·r olecutor Testing
associated w ith chronic myeloid leukemi~?creates a BCRIABL 1 fusion gene product
a 11;18
b 14 ;18
c 9;22
d 9 ;14

39. The molec ular-b_ased diagnostic test for Mycobacterium tuberculosis uses ribo
~v as the target. Primers are made to be specific for which RNA subunit? somat RNA
a 5.8S
b 16S
c 18S
d 28S
40. A PCR-based assay is performed to dete·mine the cionality of B cells in a patient using
::v a lorward primer specific for the innermost framework region and a reverse primer
complimentary to the joining region (ol the immunoglobulin heavy chain). The resulting
electropherogram was obtained. Assuming all controls worked properly for the assay, what
is your interpretation of the results for this patient?

-•
........=i
....
....
:i
3
-_,
-!
I

~_,
......:
::!
..:

~----1----ln~.~--~.:.:=~IA. . ._ ____.

: Patient is normal
Patient has mononucleosis
c Patient has a polyclonal population of B cells
d Patient has a monoclonal population o f B cells

-~-~~----~------"----~- __ ,,.,,.-"!!!!.
. ,. '51
--~
7: Molecular Biology
App/fC<Jtion
41. The results o r str 1yp1ng o r a child, the child's mother a d
01
s lrfotec11 1,, r.
are listed n the table below n 3 allegea fathers (AF •s~~
) Of the c:t
Locus Child Mother AF1 AF2 ~
name genotype genotype genotype genotype AFJ
10.12 12 12 11. 12 eenotype
CSFlPO 10 12
12 13
TPOX 9.9 9, 11 8 ,9 9, 11
911
TH01 7,9 7.93 7.8 7,9
7.93
F13A01 6.9 9.9 6.9 6.6 9.9
FESFPS 10.11 · 1. 13 11. 14 7, 10 10.9
..WA 14 18 18 19 13.18 13 14 15 18

Which or the statemen ts below is correct concerning the alleged fathers?


a AF1 .s most likely the rather of the child.
b AF2 is mos t likely the rather or the child.
c AF3 is most likely the fa ther of the child.
d none o f the alleged fa thers is likely the father o f the c hild
42. DNA sequencing of hypervariable regiors 1 and 2 (HV1 and HV2) is used to:
·~·
""'-' a detect polymorphisms in huma n TP53
b genotype strains o f Mycoplasma
c to monitor chemotherapy for 8 cell lymphoma
d detection or polymorphisms in human mitochondrial DNA
43. Which locus is used lo identify the gender of the ind ividual from whom a DNA sample 1s
...
ON<•
obtained?
a amelogenin
b CSF1PO
c T POX
d vWA
44. Which method for quantifying ONA would be most appropria te when testing small FFPE
....
,,,... tissue sections by next generation sequencing?
a agarose gel
b in tercalating dyes
c real-time PCR
d UV adsorption
45. What type o f short read files with quality scores are used for storing next generation
""'
0'0.Y sequencing read data?
a FASTA
b FASTQ
c RRBS
d WGBS

....
46.
..... ~:~:~~~ cha!7 termination sequencing (Sanger method), what does a heterozygous
P<>sit on look hke on an electropherogram?
: ~ peak twi~ the height or those a round ii
peaks In the same pos·u .
c 2 peaks of equal he' h 1 on, one twice the height o f the other
d 3 peaks of equal helg1g ht at the same position
I at lhe same position

458 Tho Board of C•rtlfleauon S tudy Guid• Ge


7: Molecular Biology
47. Monitoring the load of HIV circulating in a patient's pl
Applications of Molecul8 r
Tes ting
,
using w hich m ethod? asma WOUid best be accomp rshea
a ELISA
b DNA sequencing
c northern blot
d real-time nucleic acid amplrfication

48. A patien t sample contains 500 particles/ml. 200 µL of sample is extracted, and DNA is
eluted into 200 µL . 5 µl of DNA is added per PCR reaction. Calculate the average number
of viral particles added per PCR reaction, 2ssuming 100% extraction efficiency.
a 2.5
b 25
c 250
d 2500
What is the sequence of the DNA shown on this pyrogram?
.49.
.,
:»<•

ISOf l

:,, =!.~t~J~::;::~~j~~~~G=;.J~.._
t= SGATACGTGAG C
I
a allele 1: ATACGTGCC alfete 2: ATACGTGCC
b allele 1: ATACGTGCC alfele 2: ATACGTACC
c allele 1: ATACGTGAGC allele 2: ATACGTGAGC
d allele 1: ATACGTGACC allele 2: ATACGTGACC .
so·
... What term best describes the relatedness of sequences, the percent identity or
'"" COnservarion?
a domain
b homology
c motif
d similanty

~~ - ··-- ----- - - - - - - - - - - --
4S(:p It•.
- - Clll•!l !!~~'~£'*~l
.. .. ~1 ::·........
~
7: Molecular Biology
_::.::.:,~---=-=----:-:----=----:;:;-:::----....:.:.::.:~a~
r Te111
mo1ocu1
J</JJJll<..OUVH'> VI
---:-_____
1. c 11. d 21. c ~~· ~ 41. b ~
2. a 12. c 22. a 33·· c 42. d
23. b 43. a
3. C 13. a 24. a M· c 44. b
45.. Cc 14. b
15. b 25. c
26. b
35
36.
.
~ 45.
46.
b
c
.
6 d 16. b 37. c 47. d
. 27. c c a
7. d 17. C 28. b
38. 48.
8 a 18. C 29. ad
39. b 49. b
. 19. b 40. d 50. d
9 d
10. a 20. c 3~.

460 Tho Bo•rd of Certiflc::etion Study Gulde 60


S: Laboratory Operations
- Quality Assessment

Laboratory Operations
The fol/owing items have been identified generally as appropriate for both entry level m d:
1
laboratory scientists and medical laboratory technicians. Items that are appropriate for ::,::cal
1
18tioratory scientists only are marked with an 'MLS ONLY.·

465 Questions 500 A nswers with Explanations


465 Quality Assessment 501 Quality Assessment
470 Safety 503 Safety
479 Management 508 Management
483 Laboratory Mathematics 510 Laboratory Mathematics
490 Instrumentation & Genaro/ Laboratory 513 Instrumentation & General LaboratDf)'
Principles Principles
495 Education & Communication 515 £ducation & Communication
498 Lallotatory Information Systems 516 Laboratory Information Systems
Quality Assessment
1. The laboratory manager receives a complaint from the ICU about turnaround times for
~' coagulation tests. The first step in problem solving should be:
a gather data on current times by shift
b ta:~ to staff about various solutions
c perform root cause analysis
d draw a process map to send to the ICU explaining why it takes so long
2. Which action by the phlebotomist will comply with the College of American Pat~ologists
(CAP) Patient Safety Goal "to improve patient and sample ldentification_at the time of
5 PeC1men collection" and The Joint Commission P-atient Safety Goal to improve the
accuracy of patient ldentifteatlon"?
a match the name and room number on the patient's ID bracelet to the name and room
number on the preprinted collection label
b match the name and medical record number on the patient's ID bracelet to the name
c and medical record number on the preprinted collection label . .
d ~enty Patient in~ormation by stating the patient's name wh7n approaC!1rng the patient
bel the collection tub(ls prior to the blood draw at the patient s bedside
3.
Preana1yi· 1 •
•ca vanables in laboratory testing include:
a result accuracy
b rePort d r
c test tu e rvery to the ordering physician
d s . maround time
4. Pecimen acceptability
lhe first pr ·
Pararnet ~ure to be followed If the blood gas instrument 1s out-0 -con
r trol for all
ers is:
a recar.brat th
b repeat e, en repeat control
c replaceco~trol on the next shift
d rePort P!t~ctrOdes, then repeat control
nt results after duplicate testing

n on EJC11m/na tlons 465


r11nlr•l I nhoratorv Cetf/ ca I/
S: La boratory Operations
Quality A ssessment
'· Refer to the following graph:
.....
u

o--~-~---~--~--'
0 10 20 30 40 so 60
Old method

A new methodology for amylase has been developed and compared with the existing
method as illustrated in the graph shown a bove. The new method can be descnbed as:
a poor correlation with constant bias
b good correlatio n with constant bias
c poor correlation with no bias
d good correlatlon with no bias
6. Refer to the following illustration:

..~so
•• so · ·x··---~-------·-·····
......
_,so ~~~-
-z so
2 3 4 S 6 7 8 9 10 11 12 13
Doy

Shown above is a Levy-Jennings q uality control chart, which represents control values for
13 consecutive analyses for a particular serum constituent. If the 14th value is below the
-2 SD limit. which of the following should be done?
a control should be repeated to see if ii will fall within the established interval
b analysts system should be checked for a deteriorating component
c analysis system should be checked for a change in reagent lot number
d no action is needed

....7.
OH.Y a
The precision of an instrument is validated by:
running the same sample multiple times
b performing serial dilutions
c processing unknown specimens
d monitoring normal and abnormal controls
8. The mean value of a series or hemoglobin cor trols was found to be 15 2 g/dl and the
~tandard deviation. ~s calculated at 0.20. Acxeptable control range fo; th e laboratory
•s ±.<!?standard dev1atrons. Which of the following represents the allowable limits for the
control?
a 14.5-15.5 g/dl
b 15.0-15.4 g/dl
c 15.2 - 15.6 g/dl
d 14.8-15.6 g/dl

466 The Board ol Certlncotlon Study Gulde 6e


ISBN 978-08918!Mi609 02018AS<:P
b 0 ratory Operations Qual/ty Assessment
s:
_ La d d · rf th ·
.t control ±2 standar ev1a ions rom . e mean includes what percentage of the
9 In qua11 Y .• ?
...
.;,. sample population .

a 50%
b 75%
c 95%
d 98%
Upon completion of a run of chol~sterol tests, the technician recognizes that the controls
io. are not within the 2 standard dev1at1ons confidence range . What is the appropriate course
of action?
a report the results without any other action
b run a new set of controls
c run a new set of controls and repeat specimens
d recalibrate instrument and run controls
II. The following data were calculated on a series of 30 determinations or serum uric acid
= =
control: mean 5.8 mg/dL, 1 standard deviation 0.15 mg/dL. If confidence limits are set
at :t2 standard deviations, which of the following represents the allowable limits for the
control?
a 5.65-5.95 mg/dL
b 5.35-6.25 mg/dL
c 5.50-6.10 mg/dL
d 5.70-5.90 mg/dL
12. An index of precision is statistically known as the:
a median
b mean
c standard deviation
d coefficient of variation
13. The term used to describe reproducibility is:
a sensitivity
b specificity
c accuracy
d precision
t4 Th .. ( ) 't I ·ms to measure is called:
· e ab1hty or a procedure to measure only the component s 1 c ai
a specificity
b sensitivity
c Precision
d reproducibility
ts. Th
e extent to which measurements agree with the true va1ue
of the quantity being
measured is known as:
a reliability
b accuracy
~ reproducibility
Precision
16. D
iagrioslic specificity is defined as the percentage of individuals:
a with . . .. by 8 given test
b with a give~ disease who have a positive r~su11 It by a given test
c w· out a given disease who have a negative resu a iven test
d ..,:th a give~ disease who have a negall".~ result ~{by~ given test
lhout a given disease who have a positive resu

,..-"!".:
8: L aboratory Operations Quality Assessment --
17.
....
....$
If lhe correlation coefficient (r) of 2 variables is 0:
a lhere is complele correlalion belween the variables
-
b lhere is an absence of correlation
c as one variable increases, the other increases
d as one variable decreases, the other increases
18. Employees are guaranteed the right 10 engage in self-organizalion and collective
bargaining through representatives of their choice, or to refrain from these activities by
which of the following?
a Civil Rights Act
b Freedom of lnformalion Act
c Clinical Laboratory Improvements Acl (CLIA)
d National Labor Relations Act
19. Which of the following organizalions was formed lo encourage the volunlary altainmenl of
uniformly high standards in institutional medical care?
a Centers for Disease Control {CDC)
b Heallh Care Finance Administration {HCFA)
c The Joint Commission {JCAHO)
d Federal Drug Administralion (FDA)
20. The process by which an agency or organization uses predetermined standards to
evaluate and recognize a program of study in an Institution is called:
a regulation
b licensure
c accredilation
d credentialing
21. CLIA was established to provide oversight to:
a research labs
b w aived point-of-care lesting by nonlaboratory personnel
c CAP-accredited labs
d any lab performing patienl testing
22. According to CUA who is responsible for classifying lab test complexity?
a the medical director whose name is on lhe CUA certificate
b FDA (Food and Drug Administration)
c CMS {Centers for Medicare and Medicaid Services)
d AMA {American Medical Association)
23. Which statement about proficiency testing (PT) is true?
~·,
.... a results can be compared to another hospital prior to submission if that hospilal is .in
your system
b results belween 2 technologists can be averaged
c CAP requires duplicate testing lo ensure good instrument performance
d it is necessary to assess results even if a Pi challenge is ungraded
24. Which of lhe following Is part of The Joint Commission's Nallonal Patient Safety Goals?
f4S
..._. a communication of critical results
b documentation of lab QC
c trending of Instrument problems
d reconciliation of lab orders and results In the medical record

468 Th• B o•rd of CorttncaUon Study Guk19 6o ~a,,s<J'


ISBN 978-089189-li609
8: Labo ra tory Opera ti ons
25. If your lab performs a test for which thoro 5no comm . Oual/ty Assossmont
KS proficiency test material. which of the follo.ving is ercia/ly available conr
(JM,r accuracy? acceplable for docvmenr;,?1or
JOn of test
a perform the test In duplicate
b you do n~t have to do anything if there is nothing available
c make an rnternal lab control from a pre~ious negative ..
d have 2 technologists perform the test independently and posilive

26. CAP requires refrigerator temperatures to be recorded:


a daily
b weekly
c monthly
d periodically
27. A paper or electronic report o f lab results must include:
a the name of the person who collected the specimen
b the test price
c a pathologist's signature
d the name and address of the testing laboratory
28. CAP requires that glassware cleaning practices include periodic testing for:
....
Gitt a chemical residues
b silicates
c detergents
d heavy metals
29. Prior lo implementing a new lab lest. the analytical measurement range (AMR) must be
::.', verified. This is to verify a value that can be:
a directly measured on a specimen w ithout any dilution or concentration
b reported after specimen pretreatment
c reported up to a 1: 100 dilution
d sent out to a reference lab for verification
34• HIPAA is a federal law that requires:

~ ;=onfid.entiality of patients' health care information between 2 organizations


eporting of errors in laboratory results
~ access to patient records when there is a lawsuit
unannounced inspections by acc red itation agencies
~~~,, ..
Violate? ca s and asks you to access his test results. Which of the fo/lowrng does this
a CAP
b TheJ ·
c HIPAAOint Commission
d CltA

°''~· An IC0.10
• Pau .
COde is related to··
b '1" em ~arges
G

c ~llos1s
d test accreditation
33. methOdolog
4n ino· . y
"'ldua/lzed .
a risk Qua/ity control I
b reva~SS~sment P an (IQCP) does not require:
c qi.la/it dation of tesr
Cl qua111yy eon1ro1 Plan Performance
asse
' ssmenr
~ ~~'1~~
, 8: Laboratory Operations
34. Which or the following is NOT a potential source of postanalytical errors?
a excessive delay in reporting or retrieving a test result
-
Safety

b interpretation of result
c verbal notification of test result
d labeling the specimen at the nurse's station
35. A preanalytical error can be introduced by
a drawing a coagulation tube before an EDTA tube
b mixing an EDTA tube 8-10 times
c transporting the specimen in a biohazard bag
d vigorously shaking of blood tube to prevent clotting
36. The most important diagnosis and therapeutic management decision tool used to interJllel
test results is:
a statistical analysis
b reference intervals
c specimen acceptability
d the age of a patient
37. A quality management system (QMS) is a syslem for designing, implementing,
maintaining, and managing quality in a laboratory. A fundamental element of a OMS
is training and competence assessment of testing personnel. Which of the following
statements is false? Training is required:
a ror newly hired, transferred or promoted personnel
b when new testing platforms are implemented
c when there is a change in the medical director of the lab
d when an employee demonstrates repeated performance issues
38. What are the 3 steps of an individualized quality control plan (IQCP)?
a inventory management, instrument selection, quality control plan
b assessment of preanalytic, analytic. and postanalytic errors
c quality control plan, quality assessment. and quality Improvement
d risk assessment, quality control plan, quality assessment

Safety
39. A technician is asked to clean out the chemical reagent storeroom and discard any
reagents not used In the past 5 years. How should the technician proceed?
a discard chemicals into biohazard containers where they will later be autoclaved
b pour reagents down the drain, followed by flushing of water
c consult SOS sheets for proper disposal
d pack all chemicals for incineration
40 • Using a common labeling system for hazardous material identification such as HMl.s01 or
NFPA 704, the top red quadrant represents which hazard?
a reactivity
b special reactivity
c health
d flammability

470 Tho B°"rd of Cortlncatlon St udy Gulde 6e


8: Laboratory Operations ~
41. If the HMIS® or NFPA 704 hazardous material Ide . S;,fety
lefl blue quadrant, it represents a: n1111 cation system has a numbe •
r 4 tn the
a high health hazard
b low hearth hazard
c high reactivity hazard
d low reactivity hazard
42. Which chemical is a potential carcinogen?
a potassium chloride
b fonnaldehyde
c mercury
d picric acid
43. Compressed gas cylinders should:
a be stored with nammable materials
b be transported by rolling or dragging
c have safety covers removed when pressure regulators are unattached
d be secured upright to the wall or other s table source
44. A chemical that is extremely volatfle, nammable, a nd capable of forming explosive
:;:, peroxides upon long-term contact with atmospheric oxygen. is:
a ethyl alcohol
b ethyl acetate
c diethyl ether
d xylene
45. The HMI~ or NFPA 704 hazardous material identification system rating ror a slightly toxic
chemical would be:
a l in the ye/low quadrant
b 4 in the blue quadrant
c l in the blue quadrant
d 4 in the yellow quadrant
~chemical that causes immediate visible destruction or irreversible alterations of human
46
'
tissue at the contact site is best classified as:
a carcinogenic
b toxic
c ignitable
d corrosive
47. La
... .1 bels on sh· . · · 'red to
.,, nclude · , 'P~d Chemicals from manufacturers, importers or d1stnbutors are requi
in,ormation on·
a .
Physical P~ .
b accident . operties of the chemical
~ appropriat~structions
4a. 9
)(posure lim~~Z.Srd warnings
Wnen h
COntai aZardous ch 1 · d
transi ner(s) to a sed:,m
Cals are transferred from the original appropnately labe!e h
a er, it: ndary container for immediate use by the person performing t 8
b f'llust be I h-
lllus1 b a..,.,iea with a
~ fllusr b! labeled With thn emergency response phone number(s)
'10e8 n labeled With h e Identity or contents of the hazardous chemical(s)
ot require labe/l~rd warnings related to the effect on involved target organs

C/if1ical Laboratory Ctu1lflcatlon Exom fnatfons 4 71


8: Laboratory Oper ation s
Satory
49. Which hazardous chemical combinations are incompatible and should not be stor d -
.....
~ together? e
a acetone and xylene
b chlorine and ammonia
c ethanol and acetone
d sodium and potassium
50. A technologist spilled 10 gallons of fomlaldehyde on the floor. After determining the
chemical poses a significant health hazard. the first action step would be to:
a notify emergency assistance
b control the spill with appropriate absorbent material
c evacuate the area

l 51.
.....
""-'
d

a
b
don appropriate personal protective equipment
A gallon of xylene waste should be:
flushed down the sink
allowed to evaporate in an open room
c disposed of with nonincinerated regulated medical waste
d disposed of as an EPA hazardous waste through a licensed waste hauler
A tech nologist, who has been routinely working with hazardous chemicals, begins to
...52..
0!4.Y
notice symptoms of persistent headaches after exposure to these chemicals. What is the
first action the technologist should take?
a seek independent medical consuliation and evaluation
b continue to perform work assignment to see if symptoms persist
c acquire involved MSDS to investigate signs and symptoms
d report situation to supervisor
53. When an employee reports signs and symptoms of a chemical exposure, the employer
MLS
ON.Y
should suggest a medical consultation and evaluation, which Is paid by the:
a employee using the employee's personal benefit time
b employer using the employee's personal benefit time
c employer on work time without loss o r pay
d employee on work time without toss of pay
54. When initial or baseline chemical exoosure monitoring required by OSH~ ro.r sub~~~_;
~v like formatdellyde or xylene is performed and the results are within perrrnsstble e,...,...--
limits, repeat monitoring should be i:erformed:
a when procedures or equipment surrounding use of the specific chemical change
b annually
c twice a year
d every 2 years . ts ;s:
. h deus chemie3
An example of personal protective equipment (PPE) for handhng azar
.55.
...
0N.Y
a eyewash or safety shower
b fume hood
c latex or vinyl gloves
d neoprene or nitrile gloves
. I hygiene plan1
56. Which of the following require laboratories to establish a cl\emica
a CAP (College of American Pathologists)
b ACS (American Chemical Society)
c OSHA (Occupational Safety and Health Act)
d CUA (Clinical Laboratory Improvements Act)

472 The Board of CertmcaUon Study Gulde 6e


~ Laboratory Operations Sofoty
57• Ono of rho elomonrs of il writlo n laboratory chomlC<JI hyglono plnn is lo:
3 require employees who handle chemicals lo have onnuof rnodfcaf ov 1 • r
b prohibil use of carcinogens a Ud ions
c designate a foboralory chemical hygiene officer
d perform chemical monlloring every 6 months for OSHA regulated substances
The purpose of the OS~A Hazar.d Communication, Ge~eraf Industry Standard, 29 CFR,
58· Subpart z. 1910.1200, 1st~ requ1re ~mployers lo establish a program ensuring personnel
are provided with information regarding the workplace dangers of:
a bfoodborne pathogens
b environmental hazards
c general safety hazards
d hazardous chemicals
Refer 10 the following illustration:
59.

This symbol indicates which of the following hazards?


a nammabfe
b efectrlcaf
c radiation
d biohazard
60. When working with sharp equipment and o bjects, use a:
a double-glove technique with specimen handling gloves
b mechanical device
c paper towel or gauze as a barrier
d 2-handed technique
6
1. For safe operation of a centrifuge:
a clean with soap/detergent when maintenance is perform~ or spills occur
b open the centrifuge cover when it is in the process C?' slo~ing down
c leave liquid specimen tubes uncovered during centnfugation
d ensure proper balance is maintained
52. F
~ or safe use and handling o f liquid nitrogen:
a use che ·
b shield ~•~11y resistant gloves
c stor a1 skin and use a face shield
d stor! ~:nders away from ventilation
61. t . cy nders in a horizontal position in a oool dry place
nt1dent repo
8 . rts for occuparionaf injury or illness should:
b Include lnror
be flied Onl r;iauon. on the employee's pa st medical history
~ be fifed for ~ ~' Incidents involving serious injury or Illness
11
riot be reta;ne:incidents including near miss Incidents
Cl!i ~ after review by a safety commiUee or officer

ISSl1 97~1~ Cllnlc•I Labon1tory Cert/flea


II0 n Examinations 473
.J
8: L aboratory Operations Safety
...
64. According to OSHA. what type of warning sign should be posted in an area where an
immediate hazard exists and where special precautions are necessary?
a red, black and white "Danger" sign
b yellow and black ·caution· sign
c green and white ·safety Instruction· sign
d orange and black "Biohazard" sign
65. All laboratory instruments should:
a have repairs conducted while connected to facility wiring
b be grounded or double insulated
c have safety checks performed initially and then every 6 months
d be connected to multiple outlet adapters
66. If areas of the laboratory are designated as "clean" or ·contaminated." it is appropriate for
a technologist to:
a clean technical area bench tops after spills and on a weekly basis
b wear a lab coal in the break or lunch room
c apply lip balm in a contaminated area
d touch a ·contaminated" area phone wi:h ungloved hands if hands are washed afterward
67. For fire safety and prevention:
a fire drills should be announced and practiced in advance
b hallways and corridors should be clear and free of obstruction al all times
c only one exit Is necessary in laboratories that contain an explosion hazard
d hazard evaluations only need to be done prior to initiation of clinical operations
68. What type of identification system does this symbol represent?

a transmission-based precautions
b physical environmental hazards
c chemical hazardous materials
d radiation hazards
69. Flammable and combustible liquids in containers 2:5 gallons should be stored in:
a a Rammable safety cabinet vented to room air
b a nonexplosion proof refrigerator
c a fume hood
d an approved safety can
70. After receiving appropriate training, the first step in using a fire extinguisher is to:
a sweep the now of the hose from side to side
b pull the pin
c squeeze the top handle or lever
d aim the hose at the base of the fire

474 The Board of Certlncatlon Study Gui"- 6e


ISBN 978-0691~ (l201S"5Cf
8: Laboratory Operations
71. To help prevent eleclrical fires in healthcare facilities: Satoty
a
use mulliple oullet or gang plug adaprers
b
change circuit breakers annually
c
tape over worn wiring with certined electrical tape
use only UL or other safety-agency-rated electrical equ1pment
d ·
12. In addition to keeping the load close to your body and tight · .
when lifting heavy boxes of supplies, it is important to bende;:~~/:our abdominal muscles
a waist; lift with legs and buttocks
b knees and hips; lift with legs and buttocks
c knees and hips; lift with arms and back
d waist; twist your body when lifling

...
73.
<><r
The best way to prevent or relieve symptoms of carpal tunnel syndrome is to:
a raise arms and bend wrists downward
b redesign facilities
c bend back and neck slightly forward
d maintain wrists in a neutral position
74. A fire occurs in the laboratory. The first course of action is to:
a evacuate the entire area
b pull the fire alarm box
c remove persons from immediate danger
d contain !he fire by closing doors
75. An electrical equipment fire breaks out in th e laboratory. Personnel have been removed
from immediate danger, the alarm has been acllvated. What is the next aclion to be taken?
a evacuate the facility
b co~tain_ the fire by closing doors
c extinguish tire with type A extinguisher
d lock all windows and doors in the immediate area
76
· Class C fires involve:
a grease and oH
b xylene and alcohol
: ~r. Wood and plastics
n. ectnca1 equipment
.., A laboratory . . ft' movernenls
""' during h'1s/h employee identifies arm and neck pain after perfonmrng repe i ive
a er Work assignment What is the best first aclion to be taken?
b report to and d" . .
Continue t iscuss issue with supervisor
~ ffiaJ<e an a~~e~orm work assignment and see if it improves
11 _ Change or ad?•ntm~nt with his/her personal physician
A tech iust his/her workstation
llie r. noroglst sp/ h 1injur}(
1tsr action sh a~ ad a corrosive chemical in his/her eyes. To prevent permanen '
a band ou d be 10:
b llusti age the eyes
c use lh0 Yes With a J:nd .seek immediate emergency medical assistance
d seek 1 ~ eyewash s t:i~1 ca1 of opposite pH to neulralize the Injury
mediate eme ion to flush eyes with waler for 15 minutes
rgency medical assistance

475
Clfnl1:1tl Labo,--rory Corllflcatlon Examinations
8 : L ab orato ry Operations
Safety
79. A technologist spilled concentrated hydrochloric acid on his/her clothing and skin aff . -
a large portion of the body. After removing involved clothing, the next fi rst aid treatmeen~ing
step would be to:
a seek immediate emergency medical assistance
b use emergency safety shower and n~sh body with water
c apply burn ointment to affected skin
d pour baking soda on the skin and bandage
80. An example of personal protective equipment (PPE) is:
a a biological safety cabinet
b an emergency safety shower
c an eyewash station
d a lab coat
81. Gloves worn in the laboratory for specimen processing must be removed and hands
washed when:
a answering the telephone in the technical work area
b carrying a specimen outside the technical work area through "clean" areas
c answering the telephone in a designated "clean· area
d after handling specimens from known isolation precaution patients
82. Safety glasses, face shields or other eye and face protectors must be worn when:
a working with caustic or toxic materials
b present in technical work area
c viewing microbiology culture plates
d processing specimens using a splash barrier
83. To prevent injury, a safe lab work practice is to:
a secure long hair and jewelry
b store well-wrapped food in the supply refrigerator
c wear contact lenses for eye protection
d wear comfortable, rubber-bottomed. open-weaved shoes
84. Safe handling and disposal of laboratory generated infectious waste require:
a disinfection of all waste
b thorough mixing of infectious and noninfectious waste
c separation of infectious and noninfectious waste
d incineration of all waste
85. Which o f the following is the best choice for decontaminating bench tops contaminated by
the AIDS virus?
a sodium hypochlorite bleach
b fo rmalin
c a quaternary ammonium compound
d 100% alcohol
86. The safest method of disposing of hypcdermlc needles is:
a recap the needle with its protective sheath prior to discarding
b c~t the needle with a special device before disposal . Immediately
c discard the needle in an impermeable container without other handling
after use
d drop the needle in the waste basket immediately after use

476 Tho Board of Certification Study Guid a 6"


S: Laboratory Operatio ns
87. Precautions for health care worker d .
s ealrng With · s
a m~uth ~ipeUing when specimens la • Pa~ents or Patient spec; . afety
b re1ns~rtmg needles into their origina~ ~ Precautron· label 1
mens ncfUde:
c weanng a mask and disposable
9
d prompl deaning o f blOOd spills With
0W: eaths after draWing blOOd i
d10 _draw biood mm a patient
a Stnfectant solution
88. Infection rale is highest for laboratory p~0 fe . sueh as SOdium ~
containing: ssionals exposed to blood and""-~ .
vvuy nu1ds
a hepatiti s A
b hepatitis B
c CMV
d HIV
89. Which of the following forms of exposure places a teehnol · t . .
infection wilh human immunodeficiency virus (HIV)? ogis at the hrghest nsk for
a _aero~I inhalation (eg._AID:;> patient's sneeze)
b ingestion (eg, mouth pipetting of positive serum)
c needlestick (eg, from AIDS contaminated needle)
d splash (eg, infeclgd serum spill onto intact skin)
90. Which disinfectant inactivates HIV and HBV?
a alcohol
b iodine
c phenol I
d sodium hypochlorile
91. Filters generally used in biological safety cabinets to protect the laboratory worker from
II
partlculales and aerosols generated by microbiology manipulations are:
a fiberglass
b HEPA
c APTA
d charcoal
92. What is the single mos! effective method 10 prevent nosocomial spread of infection?

l
I
93.
a wear mask, gown and gloves
b require infectious patients to mask
c wear an N 95 respirator mask
d l>erfonn frequent and appropriate hand hygiene
contaminated needles and syringes w llhoul safety self-sheathing devtces
· should be··
~ Sheared by a needle c utter or bent
c ~~pped ~sing a 2-handed technique .
d 5Catded directly into an appropriate shaips container

l
I
14. removed from the syringe/needle holder
01
ll$e •standard" (universal) precautions mirimlzes exposure to:
a blOOdbor
•' b Ch . ne Pathogens
emtea1 haz rd

II
c radiau a s
d envi, on hazards
ts. <>nmenta1 hazards
Alier an aCci If n should be to:
1 dental needle stick with a contaminated needle, the first ac 0
b apPly anr;sep ·
.• c ~k immed'13~c ointment to the wound
d anciage the e medical assistance
I llior0ugh1y "-'lltJnd
wash the wound w ith soap and water
~l,fo. is~_, a...,_
8 : Laboratory Operations
Safety
<
96. Wha~ is t~e most likely mode or transmission for bloodborne pathogens in laborat -
acquired infections? ory
a parenteral inoculation of blood
b contact with intact skin
c airborne transmission
d fecal-oral transmission
97. Which infectious agent is considered to be the primary occupational health ha:c:ard
regarding transmission of bloodbome palhogens?
a human immunodeficiency virus
b hepatitis B
c tuberculosis
d methicillin-resistant Staphylococcus eureus
98. When processing specimens for mycobacterial testing, what specific engineering control
must be used?
a horizontal laminar now hood
b barrier protection only
c biological safety cabinet
d fume hood
99. Hepatitis B vaccine is:
a administered as a single 1-lime inject'on
b required for all healthcare employees
c must be provided by the employer free of charge
d recommended only when an exposure incident occurs
100. When cleaning up a small (5 ml) blood spill on the countertop, the first step after donning
appropriate personal protective equipment is to:
a flood the area with an appropriate intermediate to high-level disinfectant
b absorb the spill with disposable absorbent material
c evacuate the area for 30 minutes
d clean the area with an aqueous detergent solution
101. The most effective disinfectant recommended for bloodborne pathogens is:
a sodium hypochlorite
b isopropyl alcohol
c chlorhexidine gluconate
d povidone-iodine
102. Which of the following microbial agents do not respond to the general rules regarding
au
ONl.Y
microbial inactivation and decontamination?
a Mycobacterium tuberculosis
b transmissible spongiform encephalopathy agents (prions)
c agents of bioterrorism (smallpox, Bacl/lus anthracis)
d Coccidioides immitis
103. When processing patient blood specimens and handling other potentially Infectious
material, the best choice of gloves is:
a reusable utility gloves
b latex gloves only
c single use and disposable gloves
d cut-resistant gloves

478 The Board of CertJnc.Uon Study Gui do 8•


1 • bronchial wash specimen on his/her gloves. Thegfis ~pla~hes a few small drops
. . . rs achon should be to· o1a
a wash the gloves w1lh ant1sept1clsoap and water ·
b continue lo wear the gloves until grossly contaminat .
c wash the gloves with an appropriate disinfectant ed or leaving the area
d change gloves and wash hands with antiseptic/soap and water

10S• This symbol represents:

a a biohazard
b a radiation hazard
c a chemical hazard
d an environmental hazard
106. Regulated medical waste refers to:
IU
e>t• a chemical waste
b infeciious waste
c radioactive waste
d an waste from healthcare facilities
to7• Alaboratory safety program includes engineering controls such as:
a biosafety hoods, chemical fume hoods and approved safely policies
b appropriate gloves and gowns
~ sound dampening materials and radiation shielding
!100d h,and hygiene practices

Management
ICa A
"' · technotogf5t. . · · ed ears for acid-
"' fast bacilfi repeatedly misses tubercle bacilli when examining stain Sf!1 bf m?
a · What plan of action should the supervisor first take to correct this pro e
Issue a ·
b sene1 lh Wntten warning
~ review ~:~Ployee lo a.workshop to improve his/her kno~ledge
to, reassign em ~gnosuc criteria with the employee and monitor progress
~ · 'Nnico 01 P oyee to another part of the laboratory
"'
a ovetli mg is considered to be a variable cost In a c/inlcal laborst0 ry?
the follow· ·
b ~ealth~e Pay
c Ftc •nsura
11 A
Pension
nee Premiums
COntributions
I '

I
-
I
II!.
8: Lab oratory O perations
. • . . . Management
11 o. Direct, indirect and overhead costs incurred during the production of tests pe . -
.....
.... classified as: r unit lime are
a total costs
b actual costs
c standard costs
d controllable costs
111. An advantage of reagent lease/rental agreements is:
a less time spent by a laboratory manager justifying new instrumentation
b increased nexibifily lo adjust to changes in workload
c nexibility in reagent usage from 1 manufacturer to another
d fess expenditures over fife expectancy of instrument
112. The number of hours used to calculate the annual salary of a full-time employee is:
M.$
ONLY a 1,920
b 1,950
c 2,080
d 2,800
113. The overtime budget for the laboratory is $38,773, but $50,419 has already been spent.
.u What percent over budget does this rep·esent?
ONLY

a 30%
b 70%
c 77%
d 100%
114. Matching the content and requirements or the task with the skills, abilities and needs of the
1LS
ONLY
worker is a function of:
a leadership
b job design
c recruitment
d reward systems
115. The most important part of any effective behavior modification system is:
M.$
ONLY a feedback to employees
b salary structure
c job enrichment
d tactful discipline
116. Disciplinary policy is generally developed as a series of steps with each step more s!licl
M.$ than the previous. Normally, the first step in the process is to:
""" a send the employee a warning letter
b send the employee a counseling memo
c counsel the employee verbally
d dismiss fes.s serious in fractions
. 1· 'd when making
117. A supervisor notices that a technologist continues to mouth p1pet 1qu1 s
>LS
(H'( reagents. The supervisor's best course of action is to: · I""'
. . when dea ""'
a allow the technologist to continue this practice as long as 1t 1s not done
with specimens
b discuss this problem with the employee immediately
c order a mechanical device (bulb pipet) for employee to use
d compliment the employee on his rapid pipetting technique

480 The Board o f CertlRcatlon Study Gulde 6e


a: Laboratory Operauons
8 on repealed occasions, the day shirt superiiso Mon.1gomont r
~ • shift sleeping. Which of the following is the mo ~has observed a technol .1
"'' day supervisor? s appropriate in/Ual cour~ s t on .the night
e o action for the 1 I
a ignore the ri:p~aled inc!dents I
b discuss the incidents with the technologist's im d'
c notify the personnel department me iate supervisor I
d advise the laboratory director
A workload reporting system is an Importan t part of laborat
119• ory management becau · .
;::, a tells exactly how much should be charged per test so it.
b keeps personnel busy in their free time
c counts only tests done and specimens received in the laborat . . .
figures by a~ding in qual!ty control a~d ~t~ndardization etfort~ry without inflating these
d helps in planning, developing, and maintaining efficient laboratory se . .
administrative and budget controls rvioes with

120· Which one of the following questions can be legally asked on an employment app1.1cat1on?

a Are you eligible to work in the US?
b What Is your date of birth?
c fs your wife/husband employed full-time?
d Do you have any dependents?
121. Which of the following topic areas can be discussed with a prospective employee during a
:::, job interview?
a have you ever been arrested
b number of dependents
c previous employment that the applicant dis'iked
d if they are a US citizen

..
122. An effective program of continuing education for medical laboratory personnel should first:
""' a find a good speaker
b motivate employees to attend
c determine an adequate budget
d identify the needs

..,~ · tn general, 70% of the operating expenses of laboratories are:


3

a labor or labor related


~ reagents and supplies
d equipment replacement and maintenance
safety supplies and disposables
114. legal p
re-employment questions on an applica6on are:
a llled'
b Pl ica1 history of an employee
ace of birth
c felonies
d name unrelated to job requirements
I
12$, A . and address of person to notify in case of emergency I
"' requirement i · · h a current
'" ernp1oyee h or a JOb is to be able to stand for long periods of time. If you ave
do their jo;:d 0 comes to work with a doctor's le tier stating that they can no longer stan
a fine1 th ue to a disability, what is the manager's responsibitily?
d to
I
b """- e employee d
c .. ......, areas
.
a esk JOb
d ~u are not ronable accommodation for them 10 allemare sitting and standing 1 po
· ·r ssible I
t the emp10= ~~equlred to do anything
1

- ·· . ~,
~ •mlnatlons 481
, .. .._,,.,,,,,.., Cttrt/Ucatlon ~x
8: Laboratory Operations q
. . . Managem
....
126. Which of the following 1s a tool that can be used to follow the progres . ent
sron of a proJ'ect? -
"'" a
b
Pareto analysis
fishbone diagrams
c Gantt charts
d FIFO (first in, first out)
127. Which of the following is an indirect cost?
....
"'"'' a equipment rental
b office supplies
c PT test material
d ITsupport
128. Which of the following activities is not under the direction or control of the lab manager?
a number of employees
b direct test costs
c skill mix
d military leave
129. An advanced beneficiary notice (ABN) is required when:
....
ON.Y a a test may not be covered by insurance
b when there is no CPT code associated with a test
c when an HMO submits any lab test
d when there is automatic reflex testing for a screen
130. A new clinic in the area is sending a very large number of additional chemistry tests to the
M1S
CMY
laboratory. The existing chemistry instrument is only 2 years old and works well; however.
there is a need to acquire a high throughput instrument. Which one of the following is the
appropriate "justification category"?
a replacement
b volume increase
c reduction of FTEs
d new service
131. A general term for the formal recognition of professional or technical competence is:
a regulation
b licensure
c accreditation
d credentialing
132. Evaluating the performance of employees should be done:
....
()M.Y
a annually
b semiannually
c as needed in the judgment of ma~agement
d in the form of immediate feedback and at regular intervals nee of
the compete
133. Which of the following method Is NOT an acceptable means to assess
testing personnel?
• procedure
a a quiz to test the knowledge of background information or a given
b direct observation of patient testing and/or instrument maintenance
c acknowledgement that an Individual's training Is complete
d successful completion of a problem-solving exercise

.-o., T .. ~ o ... ,.,.... "f t'i•rtlflcatJon Stuctv Gulde 6e


Laborato M
Laboratory Mathematics ry athomatlcs
134. An automated CK assay gives a reading that ·
the serum sample is made by adding 1 ml of ':e~~v~ot~e limits of linearity. A dilution of
reads 350 U/l. The correct report on the undiluted ml of water. The instrument
serum should be· now
a 2,850 U/l ·
b 3, 150 U/l
c 3,500 U/l
d 3,850 U/l
135. The common unit of measure for a standard solution is:
a g/l
b %
c mg/%
d mg/ml
136. A glucose determination was read on a spectrophotometer. The absorbance reading of the
standard was 0.30. The absorbance reading or the unknown was 0.20. The value of the
unknown is:
a 213 of the standard
b 3/5 or the standard
c the same as the standard
d 1Yzx the standard
137. A technician is asked by the supervisor to prepare a standard solution from the stock
standard. What Is the glassware of choioe for this solution?
a graduated cylinder
b volumetric flask
c acid-washed beaker
d graduated flask
138· How many ml of red blood cells are to be used to make 25 ml of a 4 % red cell
suspension?
a 0.25 ml
b 0.5 ml
c 1 ml
d 2ml
139 Th 00 ml of 5% working
· e ~olume or 25% stock sulfosalicylic acid needed to prepare 1
SOiution is:
a 1.25 ml
b Srnl
c 20rnl
d so rnl
140 .,.
..., · •o Prepar 25 'd ·s needed?
"" e rnl or 3% acetic acid how much glacial acetic aci 1
~ 0.75 ml '
1.5 mL
c 3.0 ml
1 d 7.5 rnL
•1. H<>w many rs % normal saline?
a o. 9 ms of sodium chloride are needed l o prepare 1 Lor o.9
9
b 1.8
c 9.Q
d 18.Q

,,.,.,.,.,.1n1tJons
483
Lauorarory Math
142. To prepare 40 ml of a 3% working solution, a technician would us h oma11cs

143.

144. A new method is being evaluated. A recovery experiment is performed with the •
...s results: 1o11owing
°'1.Y
0.9 ml serum sample + 0.1 ml H;O 89mEq/L
0.9 ml scrum sample+ 0.1 ml analyte standard at 800 mEq/L 161 mEq/L
The percent recovery of the added analyte standard is:
a 55o/o
b 81%
c 90%
d 180%
145. Which of the following is the formula for standard deviation?
a square root of the mean
b square root of (sum of squared differences)/(N-1)
c square root of the variance
d square root of (mean)/(sum of squared differences)
146. The acceptable limit of error in the chemistry laboratory is 2 standard deviations. If you run
the normal control 100 times, how many of the values would be out of the control range
due to random error?
a 1
b 5
c 10
d 20
147. A mean value of 100 and a standard deviation of 1.8 mg/dl were o~tained !rom ad~e!,!ld
glucose measurements on a control solution. The 95% confidence interval in mgl
be:
a 94.6-105.4
b 96.4-103.6
c 97.3-102.7
d 98.2- 101.8
148. When 0.25 ml is diluted to 20 ml , the resulting dilution is:
a 1:20
b 1:40
c 1:60
d 1:80

484 The Boan:! of Cer1JncaUon Study Gulde tie


·--- 8: Laboratory Operations
149. When the exact concenrrarion of the solute of a s r . LaborDtory MDthomot/cs
the concenlralion of an unknown solution lhe k 0 1u IOn rs known and is Used
• nown solulion is· 1o evaluare
a standard ·
b nonnal
c conlrol
d baseline

150. A serum glucose sample was too high to read so a 1·5 d~ ti . •


was made. Dilution A was tested and was agafn 100 hfgh ; u ondusrng saline (dilution A)
was made using saline (dilution B). To cale1.late the result o~ad·i ~. further 1:2 dilution
1
multiplied by: • u IOn B value must be
a 5
b 8
c 10
d 20
fS1. In performing a Sf?lnal fluid P.rotein determination, lhe specimen is diluted 1 part spinal
fluid to 3 parts saline to obtarn a result low enough to measure. To calculale the proleln
concentralion, the result must be:
a multiplied by 3
b mulliplied by 4
c divided by 3
d divided by 4

...152. aHow0.1many ml of antl-0 reagent are needed


<H.Y
to prepare 5 ml of a 1:25 dilution?

b 0.2
c 0.25
d 0.5
153. To make 1 l of 1.0 N NaOH from a 1.025 N NaOH solution, how many ml of the NaOH
~ Should be used?
a 950.0
b 975.6
c 997.5
d 1,025.0
154
· !f 0.5 ml of a 1:300 dilution contains 1 antigenic unit, 2 antigenic units would be contained
111 0.5
ml of a dilution of:
a 1:150
b 1:450
c 1:500
d 1:600
tss. A2% sar 8
::_. 11'1... 01 iso'ton" .eryth.rocyte suspension contains how many ml of packed erythrocytes per 5
•c saline solution?
a 0.1
b 0.2
c 0.5
d 1.0
t~. Asoo I
~ ""''dlmg/dL glucose SOI . . .
..,.,, is:
a 2
b <!()
c 180
ution Is diluted 1:30. The concentration of the final solutron m
I
d 1,8Qo

~.'-b
~ ~~-- r:llnlr.al Lllbontwy Certffk:11tlon E.r11mln11tlons 485
8: Laboratory Operations Laboratory Mathemallcs
157. How many ml of 30% bovine albumin are needed to make 6 ml of a 10% albumin
'""' solution?
00<.Y

a 1
b 2
c 3
d 4
158. Which of the following is the formula for calculating the dilution of a solution? 01 =volume
C=concentration) '
a V1 + C1 = V2 + C2
b V1 + C2 = V2 + C1
c V1 x C1 = V2 x C2
d V1 x V2 = V1 x C2
159. A colorimetric method calls for the use of 0.1 ml of serum, 5 ml of reagent and 4.9 ml of
water. What is the dilution of the serum in the final solution?
a 1:5
b 1:1 0
c 1:50
d 1:100
160. Four ml of water are added to 1 ml or serum. This represents which of the following
serum dilutions?
a 1:3
b 1:4
c 1:5
d 1:6
161. Which of the following is the formula for calculating a percent (wfv) solution?
a grams or solutefvolume of solvent >< 100
b grams of solute >< volume of solvent >< 100
c volume of solvenllgrams of solute >< 100
d (grams or solute >< volume of solvent)f100
162. A solution contains 20 g of solute dissolved In 0.5 l of water. What is the percentage of
this solution?
a 2%
b 4%
c 6%
d 0%
163. How mony gmms of sulfosalicylic acid (MW = 254) are required to prepare 1 l of a 3%
(w/v) solution?
II 3
b 30
c 254
d 300
164. How mony ml of a 3% solution can be mode If 6 g of solute are available?
II 100 ml
b 200 ml
c 400 n1l
d 600 ml

4 86 lho Ooortl of Ctrllno-Uon Shady Guido 9o


...--- - 8: Laboratory Operations
f65. The following results were obtained from a set of Laboratory Mathematics
.,..,
.u performed on 40 samples: automated white blOOd cell coun•·
~

standard devialion: 153.2/µL


mean value: 12.450/µL

The coefficient of variation.


a 0.01%
b 1.2%
c 2.5%
d 8.1%
166. The following 5 sodium control values in unit (mEq/l) were obtained:

140, 135, 138, 140, 142


Calculate the coefficient of variation.
a 1.9%
b 2. 7%
c 5.6%
d 6.1%
167. The statistical term for the average value is !he:
a mode
b median
c mean
d coefficient of variation
168. The most frequent value in a collection of data is statistically known as:
a mode
b median
c mean
d standard deviation
169
• The middle value of a data set is statistically known as the:
a mean
b median
c mode
d standard deviation
110. Wtf
lch of the following is the formula for arithmetic mean?
: ~uare root of the sum of values
c n~m of Values >< number of values
d su~r of va!ues/sum of values
111. ~
0
values/number of values
the following values·
100 120 .
. . 150, 140, 130
Wtia1 is lh6
a fOo mean?
b 12a
c 130
d 640
8: Labo ratory Operations Laboratory Mathematics
172. Which of the following is the formula ror coefficient of variation?
a (standard deviation 100)/standard error
x
b (mean x 100)/standard deviation
c (standard deviation " 100)/mean
d (variance >< 100)/mean
173. A cholesterol QC chart has the following data for the normal control:
X (mean of data) = 137 mg/dL
#x • 1,918 mg/dl
2 SO = 6 mg/dL
N • 14

The coefficient of variation for this control is:


a 1.14%
b 2.19%
c 4.38%
d 9.49%
174. The sodium content (in grams) in 100 g of NaCl (atomic weights : Na = 23.0, Cl =35.5) Is
approximately:
a 10
b 20
c 40
d 60

..175.
. Given the following results, calculate the molar absorptivity:
ON.Y absorbance: 0.500
light path: 1.0 cm
concentration: 0.2 mol/L
a 0.4
b 0.7
c 1.6
d 2.5
176. Absorbance (A) of a solution may be converted to percent transmittance (%T) using the
"''
Otl.Y formula:
a 1 + logo/oT
b 2 + 1og%T
c 1-logo/oT
d 2 - log %T
177. Which of the rollowing is the formula for calculating the unknown concentratiOn based on
::O::v Beer's law? (A= absorbance, C = concentration)
a (A unknown/A standard) x C standard
b C standard ><A unknown
c A standard ><A unknown
d (C standard)/(A standard) x 100
8

I
178. Which of the following is the formula for calculating the gram equivalent weight of
chemical?
a MW " oxidation number
b MW/oxidation number
c MW + oxidation number
d MW - oxidation number

488 The Bo;ird of Certification Study Gulde 6e


p
B: LabOratory Operations
119• 80 g NaOH (t.WV 40) are how many m::>lgs?
I
a 1
I b 2
c 3
I
I tsa.
d4
A serum potassium (MW= 39) is 19.5m-,/ioo mL. T>.,_ •
I m "fl.__
e~n ? ••lb v.il!Je IS tl:??"""'31 to, ..fl:>IV ~f

I
I a 3.9
b 42
I c 5.0
I d 8.9

I f81. Which of the following is the formula for calrufa!:h; !hg ~ c: tnoes of a o'e.li::a?
a glG1M
I b g
c GMW/g
x Gt.fW

I
I
d (g x 100)/GMW
182. A 1 molal solution is equivalent to:
I a a solution containing 1 mole of solute per kg of solvent
b 1,000 mL of solution containi ng 1 mole of solute
I
I
c a solution containi ng 1 GEW of solute in 1 L of so.Won
d a 1 L solution containing 2 moles of sol ute
I f!l. Vlltlch of the following is the formula for calculating the mo!arily of a so.'u:ion?

I
1
a number of moles of solute/L. o f solution
b number of moles o f solute x 100

I
r c 1GEWofsolutex10
d 1 GEW of solute/L o f solution

,I ....!!'· water?is the molarity of a solution that contains


What 18.7 g of KCI (MW " 74.5) in SOOmL of

I a 0.1
I
b 0.5
I
J
c 1.0
d 5.0
I 125. 25 . of this solutioll if
~ an ~Na~H (MW =4 0) are added to 0.5 L of water. What is the molarity
droona1 0.25 L o f waler are added?
I
I
a 0.2S M
b 0.SO M
c 0.7S M
d 0.8J M
'
I
1<(
~ What is the . H (•iW =40) in 2.000 ml of
"' S<llutioO? normality o f a solution that contains 280 g NaO "
r
b5.S
3.S N
I ttr
~ 7.0 N
8.o N
N

·~ . fio.,,
•..., /Jlany g Of H 1
1 36 2S04 (MW= 98) are in 750 ml o f 3N H1S04
b 72 g
c 1109
d 146 g
g
8: L ab oratory Operations fnstrumontatlon & Genoral Laboratory Prlnc/p/os
188. How many ml or 0.25 N NaOH are needed to make 100 ml of a 0.05 N solution or NaOH?
--
a 5ml
b 10ml
c 15 ml
d 20 ml
189. A pH or 7.0 represents a H • concentration of:
...s
ON.Y a 70 mEq/L
b 10 µmolll
c 7 nmol/l
d 100 nmol/l
190. The predictive value of a positive test is defined as:
....
""' a (true-positives + true-negatives)ltrue-posllives x 100
b true-positives/(true-positives + false-positives) x 100
c (true-positives+ true-negatives)ltrue-negalives • 100
d lrue-negatives/(true-negatives + false-positives) " 100

Instrumentation & General Laboratory Principles


191. The reliability of a test to be positive in the presence of the disease it was designed to
detect is known as: ·
a accuracy
b sensitivity
c precision
d specificity
192. Which of the following parameters of a diagnostic test will vary with the prevalence of a
given disease in a population?
a precision
b sensitivity
c accuracy
d specificity
193. Package inserts may be used:
a Instead of a typed procedure
b as a reference in a procedure
c at the bench but not In the procedure manual
d if initialed and dated by the laboratory director
194. The following target shows a set of results that show a high degree of:
....
<»<•

a accuracy
b precision
c sensitivity
d specificity

4 9 0 The Board ol Certlncatlon Study Guide lie


ISBN978-089169-6609 0201 8,o\SCI'
8: Laboratory Operations
Instrumentation & G
. f f . . enera/ Laborat
195. Which o the ollow1ng 1s the advantage for add. . ory Prlnciplos

196.

197. The methodology based on the amount c f energy absorbed by a substance as a function
I
of its concentration and using a specific source of the same material as the substance
analyzed is: J
a flame emission photometry
b atomic absorption spectrophotometry
c emission spectrography
d x-ray fluorescence spectrometry
198. Which of the following wa velengths is within the ultraviolet range?
a 340 nm
b 450 nm - · y
c 540 nm
d690nm .
199. ~~e means of checking a spectrophotometer wavelength calibration in the visible range is
by using a:
a quartz filter
b diffraction grating
c quartz prism
d didymium filter ths is the:
, rrow band of wave 1eng
200. tn spectrophotometry, the device that allows 1or a na
a hollow cathode lamp
b monochromator
c refractometer
d photodetector
1
201. What is the first step in preparing a spectrophotometer for an assay
::., a adjust wavelength selector
b zero with deionized water
c read standard absorbance
d place a cuvette in the well
2
02. The nanometer is a measurement of:
~ wavelength of radiant energy
Specific gravity
c density
d Intensity of light
203. I 8
n double-beam photometer, the additional beam is used to:
. ~ ~mpensate for variation in wavelength
I
{
c correct for variations in light source Intensity
c1 rrect for changes in light path
compensate for variation in slit-widths
I
8: Laboratory Operations Instrumentation & General Laboratory Principles
-
204. The source of radiant energy in atomic absorption spectrophotometry is:
.....
00
' a hollow anode lamp
b hollow cathode lamp
c halogen vapor lamp
d deuterium lamp
205. A spectrophotometer is being considered for purchase by a small laboratory. Which of the
~r following specifications reflects the spectral purity of the Instrument?
a photomultiplier tube
b dark current
c band width
d galvanometer
206. A chemistry assay utilizes a blchromatic analysis. This means that absorbance readings
~r are taken at:
a 2 wavelengths so that 2 compounds can be measured at the same lime
b 2 wavelengths to correct for spectral interference from another compound
c the beginning and end of a time interval to measure the absorbance change
d 2 tlmes and then are averaged to obta in a more accurate result
207. Nephelometers measure light:
a scattered at a right angle to the light path
b absorbed by suspended particles
c transmitted by now-particulate mixtures
d reflected back to the source from opaque suspensions
208. A technologist is asked to write a procedure to measure the Evan blue concentration on a
spectrophotometer. The technologist is given 4 standard solutions of Evan blue:
Sid A" 0.8 mg/dL
Sid B = 1.6 mg/dL
Sid C • 2.4 mg/d ~
Sid D = 4.0 mg/dL

The first step is to:


a calculate the slope of the calibration curve
b determine the absorbance of the 4 standards
c find the wavelength of the greatest% transmittance for Evan blue
d find the wavelength of the greatest absorbance for Evan blue
209. Which or the following is used to verify wavelength settings for narrow bandwidth
spectrophotometers?
a didymium filter
b prisms
c holmium oxide glass
d diffraction gratings
210. Which of the following statements about fluorometry is true?
a a compound fluoresces when it absorbs light at 1 wavelength and emits light at a
second wavelength
b the detector_ in a nuoron:i7ter is positioned at 1ao• from the excitation source
c fluorometry is less sensitive than spectrophotometry
d an Incandescent lamp is commonly used In a nuorometer
211. The measurement of light scattered by particles in the sample Is the principle of:
a spectrophotometry
b fluorometry
c nephelometry
d atomic absorption

492 The Boord of Certifle11tlon Study Gulde ee


ISBll 978-0tl918U609 0'2018ASCP
8.• Laboratory v1.1c umenra11on & G
1 . .. , . , , '"' ,, ,,..,
onora/ Laboratory P f I
2. In a spectrophotometer. light of a specific wavelength Is isolated r . 'nc plos
21 the: rom the light source by
a double beam
b monochromator
c aperture
d slit
213. A mass spectrometer detects which property of ionized molecules?
a column retention time
b charge to mass ratio
c mass to charge ratio
d fluorescence
214. Chromatography is based on the principle of:
a differential solubility
b gravity
c vapor pressure
d temperature
215. Gel filtration chromatography is used to separate:
....
"" a polar and nonpolar compounds
b compounds on the basis of molecular weight and size
c isomers of the same compound
d compounds on the basis of different functional groups
216. An Rr value of 0.5 in thin-layer chromatography means:
....
"''"' a solute moves twice as far as solvent front
b solute moves half lhe distance of solvent front
c solute moves with solvent front
d solvent moves half the distance of solute
217• An HPLC operator notes that the column pressure is too high, is rising too rapidly and the
~. ~ecorder output is not producing normal peaks. The most probable cause of the problem
1s:
a not enough sample injected
b bad sample detector
~ 0 fl!ll'ent Oine obstructed
stnp char1 motor hanging up
chromatogra~hy a compound must:
218
' To be analyzed by gas liquid
~ be volatile or made volatile
not be volatile
~ be water-soluble
219 contain a nitrogen atom
· A true state . d' high performance
liquid anct ment about column chromatography methods, lncJu ing -
gas chromatography, is that it:
a all Utilizes f
b requires d a. lame Ionization detector

I c can be envatlon or nonvolatile compounds


d Can be ~~ed to separate gases, liquids or soluble solids .
I throrn led for adsorption partition ion-exchange and stenc-exclusion
a ography ' •

I
I
~.
8: Laboratory Operations lnstrument<Jtlon & General Laboratoryp
rlnc/pfe

=
(G 0.00001 >< radius In cm >< (RPM)2)
a 1.8 G
b 2,700 G
c 27,000G
d 90,000 G
226. In a centrifugal analyzer, centrifugal force is used to:
a add reagents to the rotor
b transfer liquids from the inner disc to the outer cuvette
c measure changes in optical density In the centrifugal force field
d counteract the tendency of precipitates to settle in the cuvette
227. Which of the following is the best guide to consistent centrifugation?
a potentiometer setting
b armature settings
c tachometer readings
d rheostat readings
228. A benefit or mlcroassays, such as point-or-care methods, Include:
a increased analytical rellablllty
b reduced sample volume
c increased diagnostic specfnclty
d reduced numbers of repealed tests

Th" Ao,.rd o f Cot11fleatlon Study O uSdo Oe CQ018p.SC1'


ISON 97$.()691~
A QA
Ill""" Laboratory Operation s Education & Comm
S: t t d h d caCl:z, the desicca Unlcatlon
z29 In the pro Per use of cobalt rea e an Y rous
.
t h
n s outd be:
· ed when it turns pink
a chanQged when it turns blue
b (han
kept in the dark
~ kept in the cold
E me-multiplied immunoassay techniques (EMIT) differ from all othe ty
230. ~:Unoassays in that: r l>es or en:zyme
tyso:zyme is the only enzyme used to label the hapten molecule
~ no separation of bound and f~e antigen !S required
c inhibition of the e_nzyme label is accomplished with polyethylene glycol
d antibody absori;>t1on to polystyrene tubes precludes competition to labeled and
unlabeled antigen

231. Which of the following statements about immunoassays using enzyme labeled antibodies
or antigens is correct?
a Inactivation of the enzyme is required
b the enzyme label is less stable than an isotopic label
c quantitation of the label can be carried out with a spectrophotometer
d the enzyme label is not an enzyme found naturally in serum
232. Which of the following Immunoassay labels offer the greatest detection limit?
a fluorescence
b electrochemiluminescence
c radioactivity
d chemiluminescence

Education & Communication


233. The objective. "The student will be able to perform daily maintenance on the Hematology
II
~ analyzer" is an example of which beh avioral domain?
a psychomotor
b affective
c intellectual
d cognitive

...~- To be effective, criticism should be:


a specific to the behavior
b related to general laboratory performance
~ ~ocused on the person, not the behavior
epeatedty discussed for reinforcement ntinuinQ
235. The fi . . . f a laboratory co
~' edu 1 ~1 step in the development of long-term ob1ect1ves or
cation program must include:
a total
b total cost of the program
c a list n~mb~r of hours in the program
d a sta~ topics to be covered
I•
I
ement of competencies to be achieved
Il
l
t
8: Laboratory Operations Education & Commu 1

...
236. Given the following objective:
O"<Y
n C~tlon
-
"After listening to the audioconference, the student will be able to describe the interact·
between T and B lymphocytes in the immune system, to the satisfaction of the instruct:.
Which of the following test questions reflects the intent of this objective?
a how are T and B lymphocytes separated in vitro?
b how many T lymphocytes does a normal person have in peripheral blood?
c what are the morphological characteristics of B lymphocytes?
d how are antibodies produced after a viral infection?
237. Higher levels of employee motivation occur when the supervisor:
.....
°"'' a collaborates to set goals to be accomplished
b provides all the details or the task
c constantly monitors progress
d immediately corrects every error
238. Several complaints have been received from parents or children in the pediatric wing
about the anxiety that venipuncture causes their children. An informal staff meeting with
the phlebotomists reveals that they feel both parents and pediatric nurses are less than
supportive and frequently make the task of venipuncture in ch ildren worse with their own
anxiety. The best course of action would be to: -
a have pediatric nurses do venipuncture on children as they are more familiar with the
children
b limit physicians to only one draw per day on children
c prepare written pamphlets for parents and in-service education for nursing personnel
d take no action as parents will always overreact where their children are concerned
239. A major laboratory policy change that will affect a significant portion of the laboratory
personnel is going to take place. In ord er to minimize the staffs resistance to this change,
the supervisor should:
a announce the policy change the day before it will become effective
b discuss the policy change in detail w th all personnel concerned, well in advance of
implementation
c announce only the positive aspects of the policy change in advance
d discuss only the positive aspects of the policy with those concerned
240. When employees are going to be responsible for implementing a change in procedure or
policy, the manager should:
a make the decision and direct the employees to implement it
b solicit the employee input but do what he/she thinks should be done
c ~nvolve the employees in the decision-making process from the very beginning
d involve only those employees in the decision-making process who would benefit from
the change

.241.
..
ONLY
The best way to motivate an ineffective employee would be to:
a confirm low performance with subjective data
b set short-term goals for the employee
c transfer the employee to another department
d ignore failure to meet goals
242. A tech.nologist has an idea that would possibly decrease the laboratory turnaround time fo<
~
Ol<LY reporting results. In order to begin Implementation of this idea, he/she should:
a encourage the staff to utilize the Idea
b discuss it with his/her immediate supervisor
c try out the Idea on himself/herself on an experimental basis
d present the idea to the laboratory d irector

L 496 The Board of CertJncallon Study Gulde 6o


, Laboratu• 1 - ,. - . . . - ---~ .. ~ """lln11nfr.• 1J,,,,
8· t ·mportant aspect of superv1s1on 1s:
- The mos i
!?· nclng the budget
'''" a b3 ~orming technical pro~~res
1
~ ~iting accurate job descnpbons
d dealing Wlth people
44. Acourse of instruction is b.ein~ plan 7ed ~? tea~h la~oratory employees lo reco;;nizf:
2 trOUbleshoOt and co_rrect s1mp eh~ah1unc 1ofnths infs e e<'.'ed laboratory instrumr:nt~. ln ~t(•.c'4
,..
,,,., the objectives for this course, w 1c one o e o11owing would be most appro;>(ia-H?
learn how to repair 9 of 10 simple instrument malfunctions
~ correcUy answer 9 or 1O test questions. deali~g with simple instrument malfuncil?rn
c recog~ize, detect and correct 9 of 10 simple inslrument malfunctions
d document corrective action procedure for 9 of 10 simple instrumenl malfunctions

245 . The ability to make good decisions often depends on the use of a logical sequsncr: c:f
steps that Include:
a defining problem, considering options, implementing decisions
b obtaining facts, considering alternatives, reviewing results
c defining problem, obtaining facts, considering options
d obtaining facts, defining problem, Implementing decision
246. In planning an instructional unit, the term •goal" has been defined as a:
a plan for reaching certain objectives
b set of specific tasks
c set of short- and long-term plans
d major purpose or final desired result

...247. Delegation is a process in which:


""' a interpersonal influence is redefined
b authority of manager ls surrendered
c power is given to others
d responsibility for specific tasks is given to others
~· What action should be taken when dealing w ith a long-term problem?
(lr(y'
a ignore the problem
b seek more information
~ base decision on available information
refer the problem to another level of management
249. Wh' h . . · s?
,.. ic of the following actions will facilitate group interactions at staff meeting
°'·' ab at dhering strictly to an agenda
realing every. probl em consistently
c enco .
d annouraqing input from all staff
250 uncing the assignments for upcoming projects .
"'* · As lnformar
¢0<, commu . ion is reported upward through an organization. t e am
h ount of detail
nicated Will generally·
ab decrease to f Tt ·
increas t aci 1ate the flow of information
c remain ~h~ allow consideration of all options .
d remain lh same lo ensure consistency in reporting
2St. To e same to ensure goal accomplishment
"' sustain th h· . Id indude:
°'<> a e ighly motivated employee :he evaluation shou
Perform
b retrain' ance feedback
c Qua1ity1ngf opportunities
d "~ o Perfonm ·
""mPetency b ance discussions
~ • ased tasks
•e~cp l~ a ... ,.,_,
c
8: Lab orato ry Operations Laboratory lnformM/011 SyMoms
252. Corrnnunlcalion Is enhanced by:
planned slrntegy lhat Includes listening skills and ensuring an understanding with
11 11
questions
b relying on e-mail. memos and voice mall lo communicate new information
c formal, hierarchical patlerns instead of nformal networking patlerns
d assumptions if there are questions aboul lhe intent of the message
253. In its guidelines. CAP stresses that communlcallon must be effective and efficient.
An effective component to enhance a mullifacoted communication plan in the clinical
laboratory is:
a posting department goals on communicalion bullelfn boards
b daily meetings with a question to rosier conversalion and focus the discussion
c wrillen e-mail to communicate all new changes in processes and procedures
d posting errors on communication bulletin boards lo prevenl repeal errors

Laboratory Information Systems


254. Identify the first step a laboratory manager must take in the selection of a laboratory
:::. information system.
a write a request for proposal (RFP)
b select a computer vendor
c select an LIS team
d decide on services needed

...
255. Laboratory results can be sent lo a hospital's electronic medical record by:
0 •tv a autofax
b HL-7 interface
c internet routing
d backup server
256. The use of securily systems such as firewalls and dala encryption for electronic
transmission of patient data from a laboratory information system to a remote location are
required for:
a LOINC
b HIPAA
c ICD-9
d CLIA
257. CODE 128, ISBT 128, CODE 39 and Interleaved 2 of 5 symbologies are used by
laboratory information systems to create which of the following?
a barcode labels
b workllsls
c instrument download files
d patient reports
258. ~standard electronic file format recommended for transmitting dale rrom the laboralory
information system to an eleclronic medical record is:
a Heallh Level 7
b ISBT 128
c FTP
d SNOMED

498 Th e Boord o f Certlflcallon Study Guido 6e


ISSN 978-089189·6609 CQ018ASCP
J s: Laboratort Operations L
11borllfory ,,,1,,,,,, ,.
1111
veritieation of test results requires nll or lho rollowlng to b0 "" "Y11 """•
159. ~::atorY except: OSlnbll~hod by 11111
patient results entered into the US via nn instrumont inlorfn
~ patient results !'.valuated bas~ on va~idated rules defined h~~ho LI
successful quality control ~estmg obtained prior 10 refensing · II 5
~ review of results by a qualified technologist or technician pa ont rosutts
Yalidalion of calculated test results perfom1ed by a laboratory inrorm ti
260. per<onned: a on systom must bo
a every 6 months
b annually
c biennially . .
d only upon initial U S mstallat1on

261 • The Hematology taborat~ry is evaluating new instru~ents r?r purchase. The supervisor
wants to ensure that the instrument they select has b1direct1onal interface capab'llt' Th
instrument speci'fi cation
. necessary t o mee t th"1s requirement
' is: I IOS, e
a 9 600 baud rate
b o~rd tes t selection menu
. c HL-7 file fonnat
• d host query mode
262. The Chemistry department has requested that a new test be defined in the US to run
on lhe existing analyzer. The new test set up is completed by the LIS coordinator. A few
days later. the accessioning department receives a request for the new test but an error is
displayed when they try to place the order. All other tests can be successfully ordered. The
most likely cause of the error is the:

.• a instrument interface for the Chemistry analyzer is down


b test was not defined on the Chemistry worklist
c database did not properly update with the new test infonnation
d ADT interface with the hospital system is down
2SJ. !he process of testing and documenting changes made to a laboratory inlormation system
IS known as:
a validation
b quality engineering
c Customization
d hazard analysis
264. ~~~ance of laboratory information system back-up procedures includes all or the
--""'ll except

~ ~~an exact copy of LIS data


c ~;:e storage or the data media
d COm ;19.dawn the LIS and bringing it back up
2S5. Pelion at regularly defined intervals
Alarge hos ·1a ·u· Ive processing
~ Silmp!es ~ Pl . I has.implemented an outreach program, which w1 inv? f with the
hospifars G8"'ed
With a variety of barcode labels that are not compatib e ·men received
from their~ The la~ratory does not have the staff to re-label ev~~~~st appropriale
far the us ea~ d1ents. Which or the following solutions would
COOrdinator to consider?
~ lllandate th · 'ble t>arcode rat>efs
b Purctiase a ~:e outreach dients modify their LIS to print ~m":;lfferent t>arcoda
c . l0trnats dleware product to manage samples conlaininQ
111
d Slc!n a new L . ch clients' systems
' fl'lanuauy p IS in the hospital that is compatible with the 0 .utrea
l'OCess all samples received rrom the outreach clients

8: Laboratory Operations Laboratory Information Systems

1. a 55. d 109. a 163. b 217. c


2. b 56. c 110. b 164. b 218. a
3. d 57. c 111. b 165. b 219. d
4. a 58. d 112. c 166. a 220. a
5. b 59. a 113. a 167. c 221. c
6. b 60. b 114. b 168. a 222. c
7. a 61. d 115. a 169. b 223. c
8. d 62. b 116. c 170. d 224. a
9. c 63. c 117. b 171. b 225. b
10. c 64. a 118. b 172. c 226. b
11. c 65. b 119. d 173. b 227. c
12. d 66. d 120. a 174. c 228. b
13. d 67. b 121. c 175. d 229. a
14. a 68. c 122. d 176. d 230. b
15. b 69. d 123. a 177. a 231. c
16. b 70. b 124. d 178. b 232. b
17. b 71. d 125. b 179. b 233. a
18. d 72. b 126. c 180. c 234. a
19. c 73: d 127. d 181. a 235. d
20. c 74. c 128. d 182. a 236. d
21. d 75. b 129. a 183. a 237. a
22. b 76. d 130. b 184. b 238. c
23. d 77. a 131. d 185. d 239. b
24. a 78. c 132. d 186. a 240. c
25. d 79. b 133. c 187. c 241 . b
26. a 80. d 134. c 188. d 242. b
27. _d 81. c 135. d 189. d 243. d
28. c 82. a 136. a 190. b 244. c
29. a 83. a 137. b 191. b 245. c
30. a 84. c 138. c 192. c 246. d
31. c 85. a 139. c 193. b 247. d
32. b 86. c 140. a 194. b 248. b
33. b 87. d 141. c 195. c 249. c
34. d 88. b 142. b 196. c 250. a
35. d 89. c 143. b 197. b 251 . a
36. b 90. d 144. c 198. a 252. a
37. c 91. b 145. b 199. d 253. b
38. d 92. d 146. b 200. b 254. c
39. c 93. c 147. b 201. a 255. b
40. d 94. a 148. d 202. a 256. b
41 . a 95. d 149. a 203. b 257. a
42. b 96. a 150. c 204. b 258. a
43. d 97. b 151. b 205. c 259. d
44. c 98. c 152. b 206. b 260. c
45. c 99. c 153. b 207. a 261. d
46. d 100. b 154. a 208. d 262. c
47. c 101. a 155. a 209. c 263. a
48. d 102. b 156. b 210. a 264. c
49. b 103. c 157. b 211. c 265. b
50. c 104. d 158. c 212. b
51. d 105. a 159. d 213. c
52. d 106. b 160. c 214. a
53. c 107. c 161. a 215. b
54. a 108. c 162. b 216. b

500 The Board of Certification Study Guido 6o ISBN 978·089189-6609 ®:!OISASCP

You might also like